You are on page 1of 194

实分析

高峰

May 23, 2020


目录

1 集合论 · · · · · · · · · · · · · · · · · 5
1.1 集合的基本概念 · · · · · · · · · · · · 5
1.2 关系:映射、函数 · · · · · · · · · · · · 11
1.3 基数 · · · · · · · · · · · · · · · 17
1.4 Cantor 集 · · · · · · · · · · · · · · 24

2 距离空间 · · · · · · · · · · · · · · · · 27
2.1 拓扑空间的基本概念 · · · · · · · · · · · 27
2.2 距离和距离空间 · · · · · · · · · · · · 28
2.3 Banach 空间和 Hilbert 空间 · · · · · · · · · · 30
2.4 距离空间的拓扑 · · · · · · · · · · · · 33
2.5 连续性和乘积拓扑 · · · · · · · · · · · · 40
2.6 拓扑空间与距离空间收敛的差异 · · · · · · · · 42

3 完备性与紧性 · · · · · · · · · · · · · · · 45
3.1 全有界 · · · · · · · · · · · · · · · 45
3.2 完备性 · · · · · · · · · · · · · · · 46
3.3 紧性 · · · · · · · · · · · · · · · 55

4 连续性和连续函数空间 · · · · · · · · · · · · 63
4.1 连续性 · · · · · · · · · · · · · · · 63
4.2 连通性 · · · · · · · · · · · · · · · 69
4.3 连续函数空间 · · · · · · · · · · · · · 71
4.4 连续函数的延拓 · · · · · · · · · · · · 79
4.5 Banach 空间上的线性泛函 · · · · · · · · · · 82

2
目录 3

5 测度 · · · · · · · · · · · · · · · · · 85
5.1 测度的定义 · · · · · · · · · · · · · · 85
5.2 R 空间的 Lebesgue 测度
n
· · · · · · · · · · 89
5.3 可测集的 Lipschitz 变换· · · · · · · · · · · 105
5.4 测度与集合列的极限 · · · · · · · · · · · 106
5.5 补充 · · · · · · · · · · · · · · · 107

6 可测函数 · · · · · · · · · · · · · · · · 109
6.1 映射的可测性 · · · · · · · · · · · · · 109
6.2 可测函数的性质 · · · · · · · · · · · · 113
6.3 可测函数的收敛 · · · · · · · · · · · · 119
6.4 应用:极大似然估计 · · · · · · · · · · · 125

7 积分与期望 · · · · · · · · · · · · · · · 127
7.1 积分的定义 · · · · · · · · · · · · · · 127
7.2 积分的几个收敛定理 · · · · · · · · · · · 131
7.3 可积函数空间的性质 · · · · · · · · · · · 135
7.4 对偶空间 · · · · · · · · · · · · · · 139
7.5 Riemann 积分与 Lebesgue 积分 · · · · · · · · · 141
7.6 Riesz-Markov 表示定理 · · · · · · · · · · · 145

8 乘积测度 · · · · · · · · · · · · · · · · 150
8.1 乘积测度的定义 · · · · · · · · · · · · 150
8.2 Fubini- Tonelli 定理 · · · · · · · · · · · · 155
8.3 高维 Riemann 积分 · · · · · · · · · · · · 157
8.4 概率与测度 · · · · · · · · · · · · · · 158

9 单调函数与可微性 · · · · · · · · · · · · · · 167
9.1 Borel-Kolmogorov 悖论 · · · · · · · · · · · 167
9.2 单调函数 · · · · · · · · · · · · · · 169
9.3 单调函数的可微性 · · · · · · · · · · · · 170
9.4 覆盖定理和极大函数 · · · · · · · · · · · 178

10 有界变差函数 · · · · · · · · · · · · · · · 180
10.1 有界变差函数 · · · · · · · · · · · · · 180
10.2 有界变差函数空间 · · · · · · · · · · · · 183
4 目录

10.3 绝对连续函数 · · · · · · · · · · · · · 190


第 1 章 集合论

1.1 集合的基本概念

1.1.1 集合的运算
子集,真子集,包含 ⊂(注意此处此符号允许两个集合相等)
,A ⊂ B, A ⊃ B ⇔ A = B,
属于 ∈
幂集,就是指一个集合所有子集形成的集合.记为 P ( A ),注意 ;, A ∈ P ( A ).
常见的集合的运算有交 ∩,并 ∪,补(余) A c ,差 A − B 或者 A \B,对称
差 ∆, A ∆B = ( A − B) ∪ (B − A )
交换律,结合律:任意多集合的交(并)的运算与运算顺序无关
S S T T
分配率: A ∩ ( Bi) = (A ∩ Bi) ,A ∪ ( Bi) = ( A ∪ B i ),对任意集合 I
i∈ I i∈ I i∈ I i∈ I
都成立
De
µ Morgan
¶ 法则,建立了求补集和并、交运算之间的关系.
µ ¶ c c
S T T S
Ai = A ci , Ai = A ci 对任意集合 I 都成立.
i∈ I i∈ I i∈ I i∈ I

1.1.2 集合类
下面介绍集合类的概念,也就是集合的集合。通常我们考虑的是给定一个全集之
后,其子集形成的集合 (类)。根据集合类是否对集合运算封闭,可以将其分为不
同的类型,设 C 是全集 X 的某些子集组成的集类(有限和可列的概念参见基数
部分)
1. 若 C 对有限交封闭,则 C 为 π 类
2. 若 ; ∈ C ,且 A ,B ∈ C ⇒ A ∩ B ∈ C , A \B 可表示成 C 中有限个互不相交
的集合的并.则 C 为半环.

5
6 CHAPTER 1. 集合论

3. 若 A, B ∈ C ⇒ A ∆B ∈ C , A ∩ B ∈ C ,则 C 为环。(可证明,必有 ;, A − B, A ∪ B ∈
C)
4. 如果 X ∈ C ,且 C 是半环,称 C 为半代数.
5. 如 X ∈ C ,; ∈ C ,且对有限交、取余集运算封闭,则 C 为代数.
6. 若 X ∈ C ,; ∈ C ,且对可列交、取余集运算封闭,则称 C 为 σ 代数(σ 域)

S T
7. 若 C 对单调集合序列求极限封闭 (即 A n ∈ C , A n ↑(或 ↓) A = An( A n ),
n n
则 A ∈ C ),则 C 为单调类.
8. 若 X ∈ C ; A ,B ∈ C ,B ⊂ A ⇒ A \B ∈ C ; A n ∈ C , A n ↑ A ⇒ A ∈ C ,则 C 为
λ类
) +
⊂ 代数 π类
显见:σ代数 ⇒ σ代数,代数 ⊂ 半环, ⇒ σ代数
⊂ 单调类 λ类

注意. 1. 若 C 为 σ 代数, X ∈ C ,称 X 为 C 可测的。而定义在 C 上的非负


可数可加集函数 (空集的函数值为 0) 就是测度。课程的后半部分主要就讨
论测度的有关内容。
2. 与一些代数概念的对比。
域:集合 K 上定义加法和乘法,加法满足交换律,结合律,存在零元和负元,
乘法满足交换律,结合律,存在单位元和逆元,加法和乘法还满足分配律
环:域的要求中除去逆元存在性
请问如何将集合运算和加法乘法对应来理解集合类中的环和 σ 域。
3. 给定集合 X , 有限域 F2 = {0, 1}, A 是 X 到 F2 上的映射全体,注意 A 中的函
数恰好就是 X 的子集的特征函数,由函数的加法和乘法可以知道对应的集
合的运算
1M · 1N = 1M∩N , 1M + 1N = 1M∆N

练习

1. S = {(a, b]} 是半环,但不是代数,是 π 类.S 0 = {(a, b)} 不是半环,S n =


{(a 1 , b 1 ] × · · · (a n , b n ]} 是 Rn 上半环
2. 若 C 同时为代数和单调类,或者同时为 π 或 λ 类,则 C 为 σ 代数.
3. 证明:对于 X 上的任一非空集类 C ,包含 C 的最小 σ 代数存在。
4. 证明:若 C 为代数,则 m (C ) = σ (C )。
1.1. 集合的基本概念 7

S
5. F 是 X 上的 σ 代数,{E n , n ∈ N} 是 F 中的递增集合列,并且 n∈N E n = X,
T S
定义 Fn = F E n = {E ∩ E n : E ∈ F },问是否有 n∈N F n =F

1.1.3 单调类定理
定义. 对于 X 上的任一非空集类 C ,存在(存在性的证明留作练习)包含 C 的
最小 σ 代数,最小 λ 类,最小单调类,称为由 C 生成的 σ 代数 σ (C ),生成的
λ 类 λ (C ),生成的单调类 m (C ).

T
(一般情况下并不关心具体如何生成,不过一定有 σ (C ) = C α ,其中任意
α
C α 是 σ 代数且 C ⊂ C α ,其它类似)。
总有 m (C ) ⊂ λ (C ) ⊂ σ (C )(为什么?),下面定理说明什么时候可取等号。

定理 1.1 (单调类定理). 1. 若 C 为 π 类,则 λ (C ) = σ (C )。2. 若 C 为代数,则


m (C ) = σ (C )。

:关键证 σ (C ) ⊂ λ (C )。只需要证 λ (C ) 是 σ 代
Proof.(只证 1,2 留给大家补全)
数,因为 σ (C ) 是最小的。
由于 λ (C ) 是 λ 类.
故只要证明 λ (C ) 是 π 类,即 ∀ A ,B ∈ λ (C ) 有 A ∩B ∈ λ (C )。
已知 C 为 π 类,故若 A ,B ∈ C ,则 A ∩ B ∈ λ (C ) 成立.要证明对任意 A 、
B ∈ λ (C ) A ∩ B ∈ λ (C ) 都成立,分两步证,即先考虑只有一个在 λ (C ),再考虑两
个都在的情形。
先是 B ∈ C , A ∈ λ (C ).定义 AB = { A : A ∩ B ∈ λ (C )},我们想证明 AB 就是
λ (C ).
显然 C ⊂ A B ,X ∈ AB .
若 A 1 ,A 2 ∈ AB .
A1 ⊂ A2.
则由 ( A 2 ∩ B) \ ( A 1 ∩ B) ∈ λ (C )
知 A 2 \ A 1 ∈ AB .若 A n ∈ AB ,A n ↑ A ,则由 A n ∩ B ↑ A ∩ B ∈ λ (C ) 知 A ∈ AB 。所
以 AB 是 λ 类.⇒ λ (C ) ⊂ AB .这就表明 ∀B ∈ C . A ∈ λ (C ) ⇒ A ∩ B ∈ λ (C ).
第二步:固定 A ∈ λ (C ),定义 B A = {B : A ∩ B ∈ λ (C )},要证明 B A 就是 λ (C ).
由前一步知 C ⊂ B A ,而且可证明 B A 也是 λ 类 ⇒ λ (C ) ⊂ B A ⇒ ∀B ∈ λ (C ),
A ∈ λ (C ) ⇒ A ∩ B ∈ λ (C ).

注意. 这个定理的用处在于说明某个 σ 代数具有某种性质,可以通过两步实现:


(1) 有一个生成该 σ 代数的 π 类 C ,其元素满足性质;(2) 有该性质的集合全体
构成 λ 类.
8 CHAPTER 1. 集合论

定理的证明方法也提示我们要证明满足一个集合类具有某种性质,可以先
把该集合类中满足性质的所有集合取出组成一个集类,再说明该集类就是原来
的集合类。在后面的课程,特别是涉及到测度论的部分,会多次碰到类似的证明
方法。

1.1.4 集合的极限:
S∞
如果 A 1 ⊂ A 2 ⊂ · · · ⊂ A n ⊂ · · · ,则可定义 limn→∞ A n = n=1 An
T∞
反之,若 A 1 ⊃ A 2 ⊃ · · · ⊃ A n ⊃ · · · ,limn→∞ A n =
An n=1
S T
对于任意给定的集合列 { A n },极限未必存在,但是 B n = ∞ A ,C n = ∞
i=n i
A
i=n i
的极限是存在的,它们的极限就对应 { A n } 的上下极限:
∞ S
T ∞ ∞ T
S ∞
lim sup A n = A i ,lim inf A n = A i(一个帮助记忆的方式是把 sup
n=1 i = n n=1 i = n
和 ∪ 对应,把 inf 与 ∩ 对应)。
根据定义,∀ x ∈ lim sup A n ,x ∈ B n ,故 ∀n,∃ j ≥ n,x ∈ A j ,表明 x 属于无
穷多个 A j 。另一方面,∀ x ∈ lim inf A n ,∃ n, x ∈ C n ,故 x ∈ A j , j ≥ n,表明最多
除了靠前的 n 个集合外, x 属于剩余的所有集合,显然也属于无穷多个 A j 。这
T S
说明 lim inf A n ⊂ lim sup A n 。(∀ 对应 ,∃ 对应 )
此外,如果将集合理解为事件,
lim A n = { A n 发生无穷多次},lim A n = { A n 最终总会发生}

练习:

1. { f n ( x)} 是一列实值函数,试证明:

[
∞ \
∞ [ 1
{ x : lim inf f n ( x) < a} = { x : f j ( x) < a − }
n→∞ n
n=1 k=1 j > k

T
∞ S

2. 若 f n ( x) = a − n1 ,则 { x : liminfn→∞ f n ( x) < a} = ; 6= { x : f j ( x) < a}
k=1 j > k
T
∞ S ∞ © ª
3. 若 f n ( x) = a + n1 ,则 { x : liminfn→∞ f n ( x) ≤ a} 6= ; = x : f j ( x) ≤ a
k=1 j > k
S S
4. 若 lim E n 和 lim F n 存在,证明 lim (E n F n ) = lim E n lim F n
n→∞ n→∞ n→∞ n→∞ n→∞
5. 若严格递增实数列 { xn , n ∈ N} 收敛于实数 a, 问 lim { xn } 是否等于 {a}.
n→∞
1.1. 集合的基本概念 9

集合与实数的类比关系,包含与大小比较,实数列极限.

再进一步学习度量空间的知识之前,还请大家再回顾一下与实数系有关的几个
重要定理,并思考哪些性质可以推广到 n 维欧式空间。
Cauchy 收敛定理:实数列收敛的充要条件是该序列为 Cauchy 列。
单调收敛定理:单调有界数列必定收敛。
区间套定理:I n = [a n , b n ],I 1 ⊃ I 2 ⊃ ... ⊃ I n ⊃ ...,limn→∞ (b n − a n ) = 0,则存
T

在唯一的实数 a ∈ I n ,a = lim a n = lim b n
n=1
确界存在定理:直线上非空有界点集必存在唯一的上确界(上确界可不在集
合中)。
Bolzano-Weierstrass 定理:有界数列必有收敛子序列。
Heine-Borel 有限覆盖定理:若开区间族 F 覆盖有界闭集 E ,则必可以从 F
中选出有限个开区间来覆盖闭集 E 。

• 实数的大小比较和集合的包含与否类似,但实数总可以进行比较,任两个集
合可能并不存在谁包含谁的关系,所以实数上的大小比较这种序关系是全
序关系,而集合包含与否是个半序关系.(一会儿具体定义序关系)
• 实数列中有 Bolzano-Weierstrass 定理,即有界序列总有收敛子序列.实数列
的上下极限实际上就给出了所有收敛子序列的极限的取值范围.即若 A 为
有界序列 { xn } 所有可能的收敛子列的极限.则 lim inf xn = min A ≤ max A =
lim sup xn .

Proof. 设 s = lim sup xn ,我们证 s 是最大的极限点.


令 m ∈ N ,ε > 0.
因 supk≥n xk &
故存在 n > m.
s. 使 s ≤ supk≥n xk < s+ε,由上确界的定义,可知存在某个 k ≥ n > m,
使 s − ε < xk < s + ε.由 ε 的任意性,可知有一列 { xk } 极限为 s.为证 s 为最大极限
点,令 x 为一个极限点.∀ε > 0,对任一 n ∈ N ,存在 m > n 使得 x − ε < xm < x + ε,
可知 x − ε < supk>n xk ,∀n ⇒ x − ε ≤ infn=1 supk>n xk = s,∀ε > 0.⇒ x ≤ s.

注意:要证明 x ≤ s,可以通过证明 ∀ε > 0, x < s + ε


对于集合列是否有类似结果,比如在其中是否能找到单调集合列?
关于 (任何有界) 数列 {a n } 中找一个单调序列的方法:
© ª
令 s = k ∈ N : 对所有m ≥ k, xk ≤ xm
© ª
1. 若 s 无限,则可以记 s = {k1 , k2 , · · ·},k1 < k2 < · · · ,则 xk i 即为所求单增
序列.
10 CHAPTER 1. 集合论

2.s 有限或空,记 k1 = 1 + max s(s = ; 时= 0),则 k ≥ k1 时,必有某个 m > k,


© ª
xm < xk 记之为 k 2 ,继续下去可找到 k 3 …使 xk n 是单调递减数列.

思考题:

对于任意集合列 { A n } 是否能找到一个单调集合列?为什么?对于集合极限存在
的 { A n } 是否能找到单调的集合列呢?

数列上下极限的一些性质:

{ xn }, { yn } 为两个实数列
1. 若 { yn } 收敛,则 lim( xn + yn ) = lim xn + lim yn , lim( xn + yn ) = lim xn + lim yn
2. lim( xn + yn ) ≤ lim xn + lim yn , lim( xn + yn ) ≥ lim xn + lim yn
3. xn ≤ yn ⇒ lim xn ≤ lim yn , lim xn ≤ lim yn
α > 0 ⇒ limα xn = αlim xn , limα xn = αlim xn
4.
β < 0 ⇒ limβ xn = βlim xn , limβ xn = βlim xn

数列上下极限与集合上下极限:

若 A n = { x: x ≥ xn }, B n = { x : x ≤ xn },则:

lim sup A n = ∩∞ ∞ ∞
k=1 ∪ n= k { x ≥ x n } = ∩1 { x ≥ inf x n } = { x ≥ sup inf x n }

lim inf A n = ∪ ∩ { x ≥ xn } = ∪{ x ≥ sup xn } = { x ≥ inf sup xn }

lim sup B n = ∩ ∪ { x ≤ xn } = ∩{ x ≤ sup xn } = { x ≤ inf sup xn }

基础逻辑与集合语言

如果 p 是个谓词(取值为真 =1 或伪 =0 的映射),则 p 是集合 { x, p( x)} 中那些


使 p( x) = 1 的子集的特征函数
判断 p 推出判断 q 对应于包含关系: p =⇒ q 当且仅当 { x, p( x) = 1} ⊂
{ x, q( x) = 1}
∀ 对应于交:{ x, ∀ i ∈ I, p i ( x) = 1} = ∩ i∈ I { x, p i ( x) = 1}
∃ 对应于并:{ x, ∃ i ∈ I, p i ( x) = 1} = ∪ i∈ I { x, p i ( x) = 1}
1.2. 关系:映射、函数 11

所以,R 上实值函数 f 的收敛点根据柯西判别法表示为

{ x ∈ R : ∀ j ∈ N, ∃ N ∈ N, ∀ n, p > N, | f n ( x) − f p ( x)| < 2− j }

同时也可以写成

∩ j∈N ∪ N ∈N ∩n,p> N { x : k f n ( x) − f p ( x)| < 2− j }

1.2 关系:映射、函数

1.2.1 关系的概念
定义 (关系). : 一个 (二元) 关系是指 X ×Y ( X 与 Y 的直积集即 {( x, y), x ∈ X , y ∈ Y })
中的有序元素对所成的任意一个集。
Q
定理 1.2 (选择公理). 如果 { A i } i∈ I 是非空集合族,则 i∈ I A 非空.

序关系:P 集合上的半序关系 (partially ordering) 指满足下列条件的关系


≤⊂ P × P
1. 自反性 x ≤ x
2. 反对称性 x ≤ y, y ≤ x ⇒ x = y
3. 传递性 x ≤ y, y ≤ z ⇒ x ≤ z。
若还有 4. x,y ∈ P 意味着 x ≤ y 或 y ≤ x 或既有 x ≤ y 又有 y ≤ x(完全的)(三
分法).则 ≤ 叫做 P 上的一个全序关系 (total ordering).
偏好关系:满足 1,3,4 的二元关系(思考:偏好关系和全序关系有何不同?).
良序关系 (well ordering):全序关系+任意非空集合都有最小元素.

例. 1. Q 是全序集 (按照普通的 ≤ 比较),请问是良序集吗?自然数集合呢?


R × R 上按照 x + y 排序,则是偏好关系.
2. 在经济和金融研究中经常会遇到对随机变量进行排序,其中常见的两个半
序关系如下:
(a) 一阶随机占优:
X ≺FSD Y ⇔ ∀ x, P ( X > x) ≤ P (Y > x) ⇔ ∀任何递增函数φ, E [φ( X )] ≤
E [φ(Y )]
12 CHAPTER 1. 集合论

(b) 单调似然比占优:
f (t)
X ≺ MLR Y ⇔ ∀ x, g(t) 是 t的减函数, f 和 g 分别为 X 和 Y 的密度函数
⇒ X ≺FSD Y
(c) 在统计决策理论,包括统计学习理论中,做各种统计推断,寻找合适的
模型,也可以看成是根据某种序关系寻找最优的过程。
一般化的分析框架是这样的,决策者在行为空间 ( A ) 里选择决策 a。考
察的对象本身潜在状态 θ 未知,决策后果 c(a, θ )。决策之前,可以观
察到与 θ 相关的一些实验结果 X = x。观测可以认为服从分布 P X (·|θ )。
如果假设决策后果产生的负效用或者损失函数是 l (a, θ ),我们的问题就
变成:观察到 X = x,选择 d ( x) ∈ ( A ),使得损失 l (d ( x), θ ) 尽可能小。
注意这里的 θ 是未知的,具体目标的设定还需进一步明确。如果认为
大家有关于 θ 的先验分布信息 Pθ (·),那就成了 Bayes 决策问题,目的
是选择
d ( x) = ar gmin a (E θ [ l (a, θ )| x])

定理. 对于组合选择问题 α∗i = arg maxα E [u( z+α X i )]。若 X 1 ≺MLR X 2 ,则 α∗1 ≤ α∗2 。
其中 u 递增且为凹函数(u0 > 0, u00 < 0)。

定义. 半序集 (P, <) 中的链是指满足如下条件的 P 的子集 C , ∀ x, y ∈ C , 或者


x < y, 或者 y < x
P 的子集 A 的上界定义为 u ∈ P , 如果 ∀ x ∈ A, x < u
P 的极大元定义为 m ∈ P , 如果 x ∈ P, m < x ⇒ x = m

定理 1.3 (Zorn 引理). 非空半序集 (P, <) 中的每个链都有上界,则 P 有极大元

定理 1.4 (Hausdorff 最大化原则). 任意半序集存在一个最大的全序子集。即存


在 E ⊂ X ,关于 ≤ 是全序的,而且不存在其他的全序子集包含 E

定理 1.5 (良序原则). 任意非空集合都可以良序化

定理 1.6. 选择公理,Zorn 引理,Hausdorff 最大化原则,良序原则等价。

Proof. 我们证明 Zorn 引理能够推出选择公理,反过来的证明相当困难。考虑非


空集合族 A i , i ∈ I 定义 F = { f : dom( f ) ⊂ I, ∀ i ∈ dom( f ), f ( i ) ∈ A i }
首先,F 非空,因为只在某个 i 0 上定义的 f 满足要求。其次,在 F 上定
义序关系 f ≤ g,如果 dom( f ) ⊂ dom( g) 且 g 是 f 的延拓
1.2. 关系:映射、函数 13

我们证明根据此序关系得到的每个链都有上界。设 C ⊂ F 为一个链,则存
在一个在 ∪ g∈C dom( g) 上定义的函数 h,满足 ∀ g ∈ C ,只要 i ∈ dom( g),都有
h( i ) = g( i ),即 h 是 g 的延拓,也是 C 的上界
根据 Zorn 引理,F 中的最大元就是选择函数。

定理 1.7 (超限归纳法). X 为良序集, A 是 X 的子集,满足:只要 { y ∈ X : y <


x} ⊂ A ,则 x ∈ A 。那么 A = X

Proof. 反证法,如果 X 6= A ,令 x = in f ( X − A ),注意 X 是良序集,最小元素 x


存在,而且 { y ∈ X : y < x} ⊂ A 但 x ∉ A 。矛盾

注意到任意集合都可以良序化,因此是可以对任意集合应用超限归纳法
以下两个证明实质上都用到了选择公理。
1. 证明函数连续性等价性,即 xn → a =⇒ f ( xn ) → f (a) 等价于 ∀ϵ > 0, ∃δ >
0, ∀ z, | z − a| < δ =⇒ | f ( z) − f (a)| < ϵ
反证法,假如后面的不成立,构造 A n = { z : | z − a| < 1/n, | f ( z) − f (a)| ≥ ϵ},从
每个 A n 中选取 z n ,可以验证 z n → a 但 f ( z n ) 并不收敛到 f (a)
2. 证明 f 是 X 到 Y 的满射等价于存在映射 h : X → Y 使得 f ◦ h 是 Y 到 Y
上的恒等映射
假定 f 为满射,对每个 y ∈ Y ,其原像集 F y 是 X 的非空子集,任取元素记
为 h( y) 就有 f (h( y)) = y

定义. 一个半序集 (P, ≤) 是格 (lattice ),如果 P 中任意两个元素,都可以找到他


们的最小上界和最大下界。
称格 (P, ≤) 是完备的,如果 P 的任意子集 B 都有最小上界 ∧B 和最大下界
∨B 。

例. 下面四个集合哪个是格,那个是完备的格?
• P 为正整数集合,m ≤ n 代表 m 整除 n
• P 为实数集,≤ 按通常意义理解
• P 为某个非空集合的幂集,≤ 相当于 ⊆
• P 为全体实函数, f ≤ g 表示 ∀ x, f ( x) ≤ g( x)

等价关系:满足 1,3 和 5(对称性 x ≤ y,则 y ≤ x)的二元关系.一般记作 ∼


给定 X 上的等价关系,可以定义等价类 [ x] = { y : y ∼ x}. x ∼ y ⇒ [ x] = [ y]. x ≁ y
则 [ x] ∩ [ y] = ;.故等价关系将 X 划分为互不相交的集合,
14 CHAPTER 1. 集合论

商集:设 ∼ 是 A 上的等价关系,Q 是 A 中等价类全体,称 Q 是 A 由等价


关系导出的商集,记为 A / ∼

例. 等价类的例子:
我们在以后的学习中会遇到要将所有几乎处处相等的函数划为一个等价类.
另外一个例子就是无差异曲线的每一条曲线就是一个等价类
反过来,若 X 可分为互不相交的集合,也可将同一集合内的元素视为等价
下面的例子利用等价类的思想给出了严格的正有理数构造过程:给定自然
a 1 2
数集合和乘法运算,将 b 理解为有序对 (a, b) ∈ N × N,考虑到 2 = 4 = ...,我们
可以定义等价类:[(a, b)] = {( c, d ) ∈ N × N : ad = cb},这样每个等价类 [(a, b)] 就能
够对应一个有理数。

练习:(偏好关系的函数表示)

1. 如果 X 为有限集或可列集,则 X 上的偏好关系总有一个效用函数表示即存
在函数 u: X → R.使 x ≥ y 等价于 u ( x) ≥ u ( y).(思考如何给出构造性的
证明)

1.2.2 映射和函数
定义. 映射(函数)关系:映射 f : X → Y 是从 X 到 Y 的二元关系 R ⊂ X × Y ,
使 ∀ x ∈ X ,∃ 唯一的 y ∈ Y ,使 ( x, y) ∈ R . x 称为 y 的一个原象, y 称为 x 在映
射下的像.记为 y = f ( x).
© ª © ª
定义域 dom( f ) = x : 存在 y使得( x, y) ∈ f .
值域 rng( f ) = y : 存在 x使( x, y) ∈ f
集值映射:如果二元关系 R ⊂ X × Y ,使 ∀ x ∈ X ,∃ 集合 Yx ∈ Y ,使 ∀ y ∈
Yx , ( x, y) ∈ R ,则称该二元关系定义的为集值映射。
满映射:若对每一个 y ∈ Y 都有 x ∈ X ,使 y = f ( x),则 f 为满映射
单射:如果 x1 ∈ X , x2 ∈ X , x1 6= x2 ,则 f ( x1 ) 6= f ( x2 )
一一映射:既是单射,又是满射
注意,定义映射时, f : X → Y 表明每个 X 都是有定义的,但 Y 不一定
被映满
复合映射: f : X → Y , g : Y → Z ,则 g ◦ f : X → Z 定义为 g ◦ f ( x) = g ( f ( x))
给定 D ⊂ X ,E ⊂ Y ,映射 f : X → Y .
D 的像集:f (D ) = { f ( x) : x ∈ D } ⊂ Y .E 的原像集:f −1 (E ) = { x : f ( x) ∈ E } ⊂ X
1.2. 关系:映射、函数 15

注: f 为一一映射,有逆映射 f −1 : Y → X , f −1 ( y) = x 其中 x 由 y = f ( x) 确

但当 f −1 (·) 的变量是集合时, f −1 指的是从 Y 的幂集到 X 的幂集的映射.
(
1 x∈E
特征函数,示性函数:X 为全集,E ⊂ X ,E 的示性函数,1E ( x) =
0 x ∈ X \E

练习

1. f 是 X 到 Y 的一个函数, g 是 Y 到 X 的一个函数,满足:对一切 y ∈ Y ,
有 f ◦ g ( y) = y,对一切 x ∈ X ,有 g ◦ f ( x) = x.证明:f 是 X 到 Y 上的一一
映射,且 g = f −1 .
2. 验 证: 若 f 为 函 数, 则 f −1 : P (Y ) → P ( X ) 可 与 并、 交、 补 运 算 交 换.
f : Pµ ( X ) →¶P (Y ) 可与并交换,但不能与交和补交换 µ ¶
−1 S S −1 −1 T T −1
f Bα = f ( B α ), f Bα = f (Bα ),
α∈ I α∈ I α∈ I α∈ I
¡ ¢c
f −1 (B c ) = f −1 (B)
µ ¶ µ ¶
S S T T
f Aα = f ( A α ), f Aα ⊂ f ( A α ), f ( A c ) 6= ( f ( A )) c
α∈ I α∈ I α∈ I α∈ I
µ ¶ µ ¶
S S T T
注意. 若 f 为集值映射,仍然有 f Aα = f ( A α ),f Aα ⊂ f ( A α ),但
α∈ I α∈ I α∈ I α∈ I
其余关系未必都成立。
例:定义 f : {0, 1} → {0, 1},f (0) = f (1) = 0.A = {0},B = {1},则 f { A ∩ B} = Φ,
f ( A ) ∩ f (B) = {0}, f ( A c ) = {0}, f ( A ) = {0},( f ( A )) c = {1}

1.2.3 Tarski 不动点定理及匹配问题


定理 1.8 (Tarski’s fixed point theorem). 如果 (P, ≤) 是完备的格,映射 f : P → P 是
单调增的,即 ∀ x, y ∈ P ,x ≤ y ⇒ f ( x) ≤ f ( y),则 f 的不动点集合 A = { x ∈ P : x = f ( x)}
非空,且 ( A, ≤) 为完备格。

Proof. 令 D = { x : x ≤ f ( x)},则 D 非空,因为 P 中最大下界必在其中。记 u = ∨D ,


则由 f 的单调性,总有 x ≤ f ( x) ≤ f (u),故 f (u) 是 D 的上界,有 u ≤ f (u),所
以 f (u) ≤ f ( f (u)) ⇒ f (u) ∈ D ⇒ f (u) ≤ u,故 u = f (u)。因为所有的不动点都在 D
中,所以 u 也是最大的不动点。同样方法,E = { x : f ( x) ≤ x},v = ∧E 是最小的不
动点。
16 CHAPTER 1. 集合论

下面证明所有不动点的集合 A 是完备的。设 W 为 A 的任意子集,记


w = ∨W , 记 1 = ∨P , 容易证明 [w, 1] ≡ { x : w ≤ x ≤ 1} 是完备格。为证 W 完备,需
证明 W 在 A 中有上确界。∀ x ∈ W ,f ( x) = x ≤ w,所以 x = f ( x) ≤ f (w) ⇒ w ≤ f (w)。
故 ∀ y ∈ [w, 1], f ([w, 1]) ⊂ [ f (w), 1] ⊂ [w, 1],这表明 f 是完备格 [w, 1] 上的递增映
射,存在最小的不动点 (不动点集合的下确界),该不动点也必然是 W 在 A 中的
上确界。

Gale 和 Shapley (College admissions and the stability of marriage. American


Mathematical Monthly, 1962) 中讨论了匹配问题在现实中有广泛应用,比如婚
姻、入学申请、就业等等。这里只讨论最简单的情形,对称的双边一对一匹配
问题。
设 H = { A i , i = 1, · · · , n}(男人) 和 F = {a i , i = 1, · · · , n}(女人) 为两个有 n 个元
素的集合。从 H 到 F 的双射就称为匹配。现在假定每一个 A 都对 F 中元素有
严格的偏好排序,每个 a 都对 H 中元素有严格的偏好排序。

定义. 称一个匹配是不稳定的,如果其中有未匹配的 A 和 a,更偏好对方的匹配


对象。即原匹配中有 ( A, b) 和 (B, a),但 A 更偏好 a 于 b,但 a 更偏好 A 于 B。
匹配是稳定的,如果前述情形不会发生。

定理 1.9 (稳定匹配的存在性). 存在从 H 到 F 的稳定匹配。

Proof. 这里我们利用 Tarski 不动点定理来证明存在性,Gale 和 Shapley 的原始


论文通过直接构造寻找稳定匹配的算法来证明 (尽管论文发表于 60 年代,但相
应的匹配算法已经在 40 年代的医学生实习市场得到应用)。
这里的关键是将匹配问题抽象成某个格上的映射。
对于每个个体,其匹配对象的空间是 H 或 F ,将所有人的匹配对象集合
起来可考虑乘积空间 V = H n × F n ,每一个匹配都会是 V 中的一个元素。对于
v ∈ V ,定义映射 T (v) = (T H (v), T F (v))

T H (v)( A i ) = ar gmax(a ∈ F : v(a) ≤ A i )

T F (v)(a i ) = ar gmax( A ∈ H : v( A ) ≤ a i )

即对于每个个体更换挑选对象时,并没有使对方任何人变差。
注意到 T 是完备格 V 上的递增映射,存在不动点,而这样的不动点刚好对
应稳定的匹配 (请验证)。得证。
1.3. 基数 17

1.3 基数
这一节主要讨论如何计算集合中的元素个数

定义. 如果存在 A 到 B 上的一一映射,则称 A 与 B 对等,记为 A ∼ B.

例. (−1, 1) ∼ R : f ( x) = 1−xx2

有些一一映射可以帮助我们将约束优化问题转化为无约束优化问题。另外,
这里对一一映射并没有连续性等要求,因为这里并没有引入拓扑的概念。
显然,A ∼ A ;A ∼ B ⇒ B ∼ A ;A ∼ B,B ∼ C ⇒ A ∼ C ,这表明 ∼ 是等价关系
基数 (势):若 A 、B 两个集合且 A ∼ B,则称 A 和 B 有相同的基数,记为
A = B.
引入 ≤ 来方便比较基数:设 u,v 为两个基数,U ,V 为两个集,U = u,
V = v,U ≤ V 表示 U 对等于 V 的某个子集.U < V 表示 U ≤ V 且 U 6= V

定理 1.10 (Cantor Bernstein 定理). :如果 X ≤ Y ,Y ≤ X ,则 X = Y

注:这一定理提供了判断两个集合对等的一种方法,即不用直接构构造 X
与 Y 之间的一一映射.比如要判断 [−1, 1] ∼ R ,只需考虑 R ∼ (−1, 1) ⊂ [−1, 1] ⊂ R

Proof. : X ≤ Y 也意味着存在 X 到 Y 的单射 f : X → Y ,Y ≤ X 意味着有 Y 到


X 的单射 g : Y → X .考虑 X 中的点 x.若 x ∈ g (Y ),则可找到 g−1 ( x) ∈ Y .若
¡ ¢
g−1 ( x) ∈ f ( X ),则有 f −1 g−1 ( x) ,如此继续可能出现三种情况:1. 无穷次进行
g−1 ,f −1 运算;2. 停在某个 X \ g (Y ) 的元素上,无法进行 g−1 ;3. 停在 Y \ f ( X )
上,无法进行 f −1 .这样,依照最终的情况可将初始的 X 分成互不相交的三部
分, X ∞ , X X , X Y .同样方法,Y 也可分成互不相交的,Y∞ ,YX ,YY .
若 x ∈ X X ,那么 f ( x) ∈ YX ,因为最终停在 X \ g (Y ) 上.如果 x ∈ X ∞ ,则
f ( x) ∈ Y∞ 。这说明 f : X X ∪ X ∞ → YX ∪ Y∞ 单射,下面证明在其上 f 还是满射,
即:对任意 y ∈ Y∞ , f −1 ( y) 肯定存在,
¡ ¡ ¢ ¢
对任意 y ∈ YX ,肯定有 f −1 · · · g−1 f −1 ( y) · · · ∈ X \ g (Y )
即存在某个 x 使 f ( x) = y ⇒ f : X X ∪ X ∞ → YX ∪ Y∞ 是满射.
对任意 y ∈ YY ,显然有 g ( y) ∈ X Y .
¡ ¡ ¢ ¢
对任意 x ∈ X Y ,则有 g−1 · · · g−1 ( x) · · · ∈ Y \ f ( X ) 表明存在 y ∈ YY 使 g ( y) =
x ⇒ g : YY → ZY 是一一映射
( ⇒ g−1 : X Y → YY 为一一映射.
f ( x) x ∈ X∞ ∪ X X
定义映射:h ( x) = −1
,则 h ( x) 是 X 到 Y 的一一映射
g ( x) x ∈ X Y
18 CHAPTER 1. 集合论

注意. 我们也可以利用 Tarski 不动点定理。注意到 (P ( X ), ⊆) 是完备格, i :


P ( X ) → P ( X ), i (S ) = X − g(Y − f (S )) 是单调增的,故存在不动点 C ⊆ X 。因
为 X − C = g(Y − f (C )),可定义双射: h( x) = f ( x) 如果 x ∈ C , h( x) = g−1 ( x) 如果
x ∈ X − C。

利用选择公理或 Zorn 引理,我们还可以证明


对于任意两个集合,不是 U ≤ V ,就有 U ≥ V

Proof. 取 x ∈ U , y ∈ V ,显然存在 { x} 到 { y} 的一一映射,考察集合

{ A × B : x ∈ A ⊆ U, y ∈ B ⊆ V , A = B}

按 照 集 合 包 含 关 系, 形 成 一 个 半 序 集, 而 且 每 个 链 C = { A i × B i } 都 有 上 界
(∪C = ∪ i A i × ∪ i B i ,请验证 ∪ i A i = ∪ i B i ),故存在极大元。显然这个极大元必须
为 U × Y 或 X × V ,否则可以找出更大的元。

下面的定理证明,不存在最大的基数.

定理 1.11 (Cantor). :U 为任意一个集,则 U < P (U )

Proof. :若 U = ;,则 P (U ) = 1 > 0 = ;.若 U 6= ;.由于 f ( x) = { x} 是一一的,


故 U ≤ P (U ).
若 U = P (U ),则存在一一映射 h : U → P (U ).令 S = { x ∈ U : x ∉ h ( x)}.
由于 S ⊂ U ;S ∈ P (U ).因为 h 是到 P (U ) 上的.故存在一个元素 a ∈ U .使
h (a) = S ,要么 a ∈ S 或者 a ∉ S .若 a ∈ S ,由定义,a ∉ h (a) = S ⇒ a ∉ S .矛盾.
但是 S = h (a),若 a ∉ S ,根据 S 的定义就有 a ∈ h (a) = S ,也矛盾,故
U 6= P (U ).

练习

1. U 非空,证明不存在 U 到 P (U ) 的满射
2. U 非空,A ⊂ P (U ),且 A 中任意有限个集合的交非空,证明存在包含 A
的最大的集合类 B ⊂ P (U ) 使得 B 中任意有限个集合的交非空
1.3. 基数 19

罗素悖论与 Cantor 对角线定理

上述定理的证明实际上也证明了如下的对角线定理:
Cantor 对角线定理:U 为一个集合, f : X → ( X ) 为一个映射,则不存在
x ∈ X 使得
f ( x) = { x ∈ X : x ∉ f ( x)}

罗素悖论质疑上述证明的有效性。比如设 S 是所有集合组成的集合,定义 S 上
的映射 ϕ( A ) = {B ∈ S : B ∈ A }。因为 ϕ( A ) 代表 A 中所有元素的集合,故 ϕ( A ) = A ,
表明 ϕ 为恒等映射,即所有集合都是某个元素在 ϕ 下的象。而由对角线定理,
{ A ∈ S : A ∉ ϕ( A )} 不可能是 ϕ 象中的元素,矛盾。
罗素悖论的出现在于前述 S 这个 “集合” 太大了。而另一方面,这个例子也
说明集合与集合的集合是不同层次的集合,当他们一同出现时,应当加以区分。

1.3.1 可数和不可数的实例
迄今为止,我们还都是在比较抽象的意义上谈元素多少,接下去给出一些具体的
集合的基数.
例:; 的基数为 0,{1, 2, · · · , n} 的基数为 n,若集合 S = ; 或与 {1, 2, · · · , n}
能建立一一对应,则称 S 是有限的,否则称 S 是无限的.
自然数集 N 的基数为 N0 (阿列夫零),若 S 与 N 可建立一一对应,称 S 是
可列的,S 是有限或可列.统称为可数.不是可数的集合是不可数的.元素序列
就是指指标集是自然数集合的子集的集合。
©m ª
例:Q 是可列的,注意 Q = n , n, m ∈ N互质
将 | n + m| = 0,| n + m| = 1,| n + m| = 2,……的各自找出来,记为 A k .
每个 A k 都是有限的,故 A 1 ∪ A 2 ∪ · · · ∪ A k ∪ · · · 是可列的.
例:R 是不可数的(R 的基数为阿烈夫一 N1 )
,实际上只需证 [0, 1] 是不可
数的.将所有数写成小数形式:
假设可列则有 a 1 = 0.a 11 a 12 a 13 ...,a 2 = 0.a 21 a 22 a 23 ...,a 3 = 0.a 31 a 32 a 33 ...
构造小数 β = 0.b1 b2 … b n …,其中 b n 6= a nn (a nn = 1,则 b n = 2;否则 b n = 1)
显然 β 不在 {a n } 中间,表明 [0, 1] 不可能可数.
连续统假设:在阿列夫零 N0 和阿列夫一 N1 之间没有别的基数。
可数集的例子:R 1 中互不相交的开区间族;R 1 上单调函数的间断点;(a, b)
上实值函数 f ,其右导数及左导数存在但不相等的点的集合
20 CHAPTER 1. 集合论

不可数集的例子:R ,平面上一切直线的集,一切单变量或多变量连续函
数的集

练习

1. 如果 x 是某一有理系数多项式的根,则 x 称为代数数.证明代数数全体可
数.并证明超越数 (不是代数数的数) 的存在.
注意. 历史上超越数的存在性是 Liouville 通过直接构造方法证明的。其中
的一个主要定理是:对任意阶数为 n >1 的代数数 z,存在正整数 M 使
得对于所有整数 p 和 q( q > 0),| z − qp | > 1
M qn . Liouville 给出的超越数称
为 Liouville 数,即无理数 z, 使得对每个正整数 n,存在整数 p 和 q 使得
p 1 P∞
1
|z − q | > q n , q >1。 10k!
就是一个 Liouville 数
1

1.3.2 关于有限、无限集合的简单性质
1. 集合 s 无限的充要条件是 s 与它的某个真子集等价

Proof. S 有限时显然不与真子集等价。S 无限时,按照下面的性质 2 可以


取出一个可列的子集 {s 1 , s 2 , ...},注意到 {s 1 , s 2 , s 3 ...} ∼ {s 1 , s 3 , s 5 ...},有:

S = (S \{ s 1 , s 2 , s 3 , ...}) ∪ { s 1 , s 2 , s 3 , ...} ∼ (S \{ s 1 , s 2 , s 3 , ...}) ∪ { s 1 , s 3 , s 5 , ...}

2. 无限集皆有可数无限子集 (证明可数无限是最小的无限)

Proof. A 为无限集,由归纳法,取 A 1 = { x1 },给定 A n 之后, A n = n,总有


x ∈ A ∩ A nc ,定义 A n+1 = A n ∪ { x} ⇒ A n+1 = n + 1, A n+1 ⊂ A ,得到 { A n }n∈ N
n−1
对每个 n ∈ N ,规定 B n = A 2n \ ∪ A 2k ,{B n } 互不相交,B n ≥ 1
k=0
用选择公理,从 {B n }n∈N 中的每一个集合选出一个元素,这是个可数无限
子集.

3. 可数集的任意子集也是可数的.
4. N × N 可数 (考虑一一映射 f (m, n) = 2m−1 (2n − 1))
1.3. 基数 21

S
5. 可数集的任意可数族的并也可数. i∈ N A i ∼ N ( A i ∼ N )(类似有理数)
a 11 → a 12 a 13 …
. %
a 21 a 22 a 23 . . .
↓ %
a 31 a 32 a 33
6. A 可数,则 A 的所有有限子集的集合是可数的.

练习

1. A 为任意无限集,F 是 A 的有限子集全体所成的族.试证:F = A (参考


Halmos,Naive Set Theory,section 24)
2. A ≤ [0, 1], A 为无限集,C 为 A 可数子集全体所成的族,证明:C = [0, 1]
an
3. (对角线方法) 称 {b n } 比 {a n } 增长快,如果 lim = 0.
n→∞ b n
若 Z 为序列集合,使得对任一序列 {a n } 存在 Z 中的 {b n } 比 {a n } 增长快.
则 Z 不可数.
4. 证明集合 X 是无穷集合的充要条件是:对于每个从 X 到其自身的映射 f ,
都存在 X 的非空子集 A ,使得 A 6= X ,且 f ( A ) ⊂ A 。

1.3.3 基数的运算
下面引入基数的算术运算,这将大大简化对无限基数的比较.

定义. a、b 是两个基数, A 、B 是两个集合,


µ ¶ A = a,B = b.若 A ∩ B = ;,规定
A ∪ B = a + b,ab = A × B,a b = A B ,其中 A B 代表所有从 B 到 A 的函数全体

性质. 若 u、v、w 为任意基数,则

1. u + v = v + u
2. u + (v + w) = (u + v) + w
3. u (v + w) = uv + uw
4. (uv) w = u (vw)
5. uv = vu u v u w = u(v+w)
22 CHAPTER 1. 集合论

6. u w vw = (uv)w
7. (u v )w = u vw
8. u ≤ v ⇒ u + w ≤ v + w, uw ≤ vw, u w ≤ vw ,w u ≤ wv

对无限基数还有:任意基数 a < 2a ( A = a,P ( A ) = 2a ).2N = R (二进制表示)


无限基数 a 满足 a + a = a,0 ≤ b ≤ a 则 a + b = a,ab = a,a2 = aa = a,

N N
µ ¶N
R=2 ≤N N
≤ R = 2N = 2N ×N = R

¡ ¢W
定理. (u v )w = u vw .即 U V ∼ U V ×W
¡ ¢W
Proof. 在 U V 上规定映射 φ:φ ( f ) = g ∈ U V ×W ,g ( y, z) = ( f ( z)) ( y) ∈ U ,( y, z) ∈
V × W 于是 φ 是到 U V ×W 上的.因为若 g ∈ U V ×W ,只要如下定 f : f 在 z ∈ W
处的值是 V 上的函数,对于每个 y ∈ V .这个值等于 g ( y, z) ∈ U .故 φ ( f ) = g.为
¡ ¢W
判断 φ 的 1 - 1 性,假定在 U V 内.f 1 6= f 2 .则有 z0 ∈ W .使 f 1 ( z0 ) 6= f 2 ( z0 ).
因在 V 上这两个函数不同,故必定有一个 y0 ∈ V .使 f 1 ( z0 ) ( y0 ) 6= f 2 ( z0 ) ( y0 ).于
£ ¤ £ ¤
是 φ ( f 1 ) ( y0 , z0 ) 6= φ ( f 2 ) ( y0 , z0 ),即 φ ( f 1 ) 和 φ ( f 2 ) 不同.得证.

注意. 定理的证明表明 M N 代表了从 N 到 M 的映射全体,比如


R 3 = R {1,2,3} ; f : {1, 2, 3} → R
R
R ; f : R → R 实函数全体
RN; { x1, x2 , · · · } 
a 11 · · · a 1n
 
R m× n ;  ··· ··· ··· 
 
a m1 · · · a mn
有时候需要把函数看成一个点,即把 M N 看成集合 V
回忆向量空间的定义,利用实数的加法和乘法 R A 可自然的定义为一个向量
空间。通常,我们关心向量空间 V 上的线性变换 T : T ( x + y) = T ( x) + T ( y), T ( cx) =
cT ( x); c ∈ R, x, y ∈ V
类似的,如果 W 是向量空间, A 是任意集合,W A 也是向量空间。如果 A
同时也是向量空间,那么我们可以对 W A 的子集:所有 A 到 W 的线性变换的集
合 Hom( A,W ) 进行研究,可以验证它是 W A 的线性子空间,对加法和数乘封闭,
有零元
1.3. 基数 23

练习

1. 证明:2 ≤ a ≤ R ⇒ a N = R ,aR = 2R

1.3.4 对偶性
考察二元函数 F : A × B → C
固定 x ∈ A , F ( x, y) 是 y 的函数 h x ( y) = F ( x, y), 可得到 x 到函数 h x ( y) 的映
射 x 7→ h x , 记为 ϕ : A → C B
固定 y ∈ B, 有 g y ( x) = F ( x, y), 以及映射 θ : B → C A
反过来,给定映射 ϕ : A → C B ,对于任意 x ∈ A , 有函数 h( x) : B → C , 可定义
F : A × B → C , F ( x, y) = h( x)( y)
这表明 ϕ : A → C B ,F : A × B → C ,θ : B → C A 是等价的,通常称 ϕ 和 θ 互
为对偶
比如优化问题

min f 0 ( x)
s.t. f i ( x) ≤ 0

可定义拉格朗日函数 L : R n × R m → R
X
m
L( x, λ) = f 0 ( x) + λ i f i ( x)
1

其中 λ 为对偶变量,原优化的对偶问题可表示为

min g(λ)
s.t. λ≥0

其中 g 为对偶函数,定义为 g(λ) = in f x L( x, λ)

1.3.5 可数在效用理论中的应用
首先由偏好关系 ≥ 可以定义  即 x 6 ≥ y 但 y 6= x ⇒ x  y.

定理. 对于任意集合 (可数或不可数) X 和严格的偏好关系 Â,那么存在效用函数


u : X → R ,使得 u ( x) > u ( y) ⇔ x  y 的充要条件是 X 有一个可数的子集 Z ,使
得对任意 x, y ∈ X 且满足 x  y 的,存在 z ∈ Z 满足 x ≥ z ≥ y.
24 CHAPTER 1. 集合论

Proof. 必要性留做练习.
充分性,假定有这样的可数 Â 稠密子集 Z = { z1 , · · ·}.令 r ( z n ) = 1/2n .
对每一个 x ∈ X ,记 Z̄ ( x) = { z ∈ Z, z  x}, Z ( x) = { z ∈ Z, z ≺ x}
¡ ¢ ¡ ¢
不难发现 x ≥ x0 ⇒ Z̄ ( x) ⊆ Z̄ x0 , Z ( x) ⊇ Z x0 (留作练习)
而且若 x  x0 ,前述包含关系是严格的.因为若 x  x0 ,则存在 z ∈ Z ,使
x ≥ z ≥ x0 或者 x  z ≥ x0 ,或者 x ≥ z  x0
P P
定义 u ( x) = r ( z) − r ( z) ∈ [−1, 1],{ r ( z n )} 是可和的,故 u ( x) 有意义.
z∈ Z(x) z∈ Z̄(x)
¡ ¢ ¡ ¢
显然,若 x ≥ x0 ,则 u ( x) ≥ u x0 .而若 x  x0 ,则 u ( x) > u x0 .

练习

1. X = [0, 1]×[0, 1],定义 ( x1 , x2 ) ≥ ( y1 , y2 ) 如果 x1 > y1 或者 x1 = y1 ,x2 ≥ y2 .


证明
≥ 是偏好关系,但不存在函数 u : X → R .使得 x,y ∈ X ,x ≥ y ⇔ u ( x) ≥ u ( y)

1.4 Cantor 集

nth: I n 由 I n−1 去掉 2n−1 互不相交的中间开区间,每个长度为 3−n .去掉


的集合为 Jn
I n 是 2n 个 I n−1 的闭子区间的并, I nc = J1 ∪ · · · ∪ Jn
T
Cantor 集 ∆ 为上述过程最终剩下的点集.∆ = ∞ n=1 I n .由于 I n 是闭集套,
故 ∆ 6= ;.无疑,∆ 至少有可数个点,即 I n 的端点,比如 0,1,1/3,2/3,1/9,
a
2/9,….注意,∆ 的端点总可表示为 3n ,n,a 为整数.下面我们证明 ∆ 不可数
且与 [0, 1] 等势.
1.4. CANTOR 集 25

方法 1:将构建过程看成从 I 0 到 ∆ 中某一条道路 LRLR…(向左或向右),这


个道路实际上相当于二进制表示.故 ∆ = 2N = [0, 1]
方法 2:[0, 1] 间任何数可表示为三进制小数. x = 0.a 1 a 2 a 3 . . .,a n = 0, 1或2.
£2 ¤
其中 a 1 说明了数是处于 [0, 31 ],[ 13 , 23 ] 还是 .故实际上 x ∈ ∆ 当且仅当 x 能 3,1
P∞
¶n=1 a n /3 ,a n = 0 或 2.于是我们可以定义 Cantor 函数 f : ∆ → [0, 1],
n
表示成
µ
P

2b n P

bn
f 3 n = 2n ( b n = 0,1)
n=1 n=1
f 显然是满射,这就说明 ∆ 是不可数的.
但 f 不是一一映射,因为 f (1/3) = f (0.0222 · · ·) = 0.0111 · · · = 0.1 = f (0.2) =
f (2/3)
还能看出 f 是递增的,可将 f 延拓至整个 [0, 1] 区间.
即 f ( x) = sup { f ( y) : y ∈ ∆, y ≤ x},∀ x ∈ [0, 1] \∆
这个 f 仍是递增的,而且连续. f 0 = 0 若 x ∈ [0, 1] \∆.称为 Cantor 函数。

(定理:若 f : [a, b] → [ c, d ] 单调且是满射,则 f 连续)


Cantor 集的构造也提供了另一种证明 [0, 1] 不可数的思路.

Proof. 假定可列,则可表示成 {a n },我们证明 [0, 1] 中有一 p,不等于所有的 a n .


即在 I 1 所对应的三个区间中找一个不包含 a 1 的,然后三等分,再找一个
不含 a 2 的,继续下去.可找到一系列闭区间,不包含 {a n },但由闭区间套定理,
交集非空,且异于 {a n }

P
∞ ∞ ¡ ¢ n−1
P
2n−1
(Cantor 集) c 的长度= 3n = 13 2
3 = 1 ⇒Cantor 集长度为 0
n=1 n=1
26 CHAPTER 1. 集合论

思考:可以仿照 Cantor 集,构造出一个集合,它与 Cantor 集等势但长度


>0?
p 进制小数:设 x = 0.a 1 a 2 · · · a k · · · 是任意一个 p 进制的小数。 p 是自然数,
是 0 与 p − 1 之间的任意整数,则可表示为十进制的 x = ap1 + ap22 + · · · + apkk + · · ·
第 2 章 距离空间

在这部分里我们将以 R n 为主要研究对象,介绍拓扑空间,距离空间,向量空间,
Banach 空间,Hilbert 空间的概念。主要涉及的是紧性 (一种拓扑性质),完备性,
以及连续函数。这一部分概念很多,但引入概念的目的是为了叙述的方便,而大
部分定理都是比较直接的,希望大家尽可能的花时间理解这些概念的含义.

2.1 拓扑空间的基本概念
我们这里从分析的角度来讨论拓扑。分析中最关心的是收敛与极限的概念,比如
给定映射 f : X → Y ,需要赋予 X 与 Y 怎样的结构,等式 limx→a f ( x) = b 才有
意义呢?
在 X 与 Y 都是实数空间时,等式可以描述为“当 x 充分靠近 a 时,f ( x) 充
分接近 b”,这里的接近指的是,比如, x 与 a 之间的距离小,或者指 x 处于以
a 为中心的一个开区间内部。
用前者的方式描述对应距离空间的概念,后者则对应拓扑空间。
拓扑空间的定义:拓扑空间是指满足如下条件的集合 X 与其上的子集类 O
即 ( X , O ),O 中集合称为开集,O 称为拓扑空间上的拓扑。通常也直接称 X 为
拓扑空间。[比较可测空间的定义]
1) 任意多开集的并是开集;
2) 任两个开集的交集是开集;
3) ; 与 X 是开集。
注意,X 上可以有不同的拓扑,很多时候需要指明在哪个拓扑底下进行讨
论。如果 T 和 O 都是 X 上的拓扑,且 T ⊂ O ,称 T 弱于 O
定义:若 X 是拓扑空间,则
1) A ⊂ X 是闭集如果 X \ A 是开集

27
28 CHAPTER 2. 距离空间

2) U ⊂ X 称为 x ∈ X 的邻域,如果存在开集 V 使得 x ∈ V ⊂ U
3) 点 x ∈ X 称为集合 B ⊂ X 的内点(外部点)如果 B( X \B)是 x 的一个
邻域; x 是 B 的边界点,如果 B 和 X \B 都不是 x 的一个邻域
4) B 的所有内点组成的集合 B o 称为 B 的内部
5) X 中不是 B 的外部点所组成的集合 B̄ 称为 B 的闭包
6) x 是 A 的极限点,如果对于 x 的任意邻域 U ,(U \ x) ∩ A 6= ;

注意. 对于拓扑空间 ( X , O ),若 Y ⊂ X ,则 (Y , OY ) 也是一个拓扑空间 (子拓扑空


间,继承了原空间的拓扑),其中 OY = {U ∩ Y : U ∈ O }。

2.2 距离和距离空间

2.2.1 概念
现在从距离的角度来理解接近的概念,这个更加直观。有了距离之后,定义拓扑
也很容易。
定义:设 E 是集合,E 上的距离指 E × E 到实数集 R 的函数 d ,满足:
1) ∀ x, y ∈ E , d ( x, y) ≥ 0
2) d ( x, y) = 0 ⇔ x = y
3) d ( x, y) = d ( y, x)
4) d ( x, z) ≤ d ( x, y) + d ( y, z)(三角不等式)
若不满足 2,则称为 E 上的伪距离 (pseudometric)(即只有 x = y ⇒ d ( x, y) = 0)
(E ,d ) 即集合及其上定义的距离函数共同组成距离空间.
:(E, d ) 为距离空间。V ⊂ E 成为开集,如果对任意
定义(距离空间的拓扑)
的 x ∈ V ,∃ε > 0 使得 { y ∈ E : d ( x, y) < ε} ⊂ V 。(E, d ) 中的所有开集就是 E 上的
拓扑。

注意. 距离空间中并没有涉及 +、数乘等运算.此外,根据距离可以自然地定义


拓扑,但给定一个拓扑空间,并不一定能找到一个距离函数能够诱导出原来的拓
扑。否则称原来的拓扑空间可距离化。[关于什么样的拓扑空间可距离化,可参
见 Royden 的 real analysis P179 的定理 9]
和子拓扑空间类似,距离空间的子空间可以继承原空间的距离函数,成为一
个子距离空间。
2.2. 距离和距离空间 29

2.2.2 距离的例子
1. 任意集合 E ,定义离散距离 d ( x, y) = 1, x 6= y;d ( x, x) = 0.
p | a− b |
2. 实数集 R ,定义 d (a, b) = |a − b|,或者 |a − b|, (1+| |
a− b|) ,min {|a − b , 1}

3. 任意距离空间 (E, d ),若 M 为 E 的子集,它可继承 E 的距离,故 ( M, d ) 也


是距离空间
qP P

1 | xn − yn |
4. R n ,定义 d ( x, y) = n
x − yi )2 ( n → ∞, d ( x, y) =
i =1 ( i n! 1+| xn − yn | )。
n=1
P
∞ qP
l2 ⊂ R∞, x2i < ∞. d ( x, y) = ∞
x − yi )2
i =1 ( i
i =1
5. [a, b] 上的连续函数 C [a, b], d1 ( f , g) = maxa≤t≤b | f ( t) − g ( t)|(可留作练习)
s
Z b
d 2 ( f , g) = ( f ( t) − g ( t))2 dt
a

6. R 上的随机变量 X 与 Y ,各自的分布函数分别为 F 和 G ,定义

λ( X , Y ) = inf{ε > 0 : ∀ x, F ( x − ε) − ε ≤ G ( x) ≤ F ( x + ε) + ε}

该距离函数称为 Levy’s metric,其对应的收敛就是随机变量的依分布收敛。


若定义 α( X , Y ) = inf{ε ≥ 0 : P (| X − Y | > ε) ≤ ε},称为 Ky Fan metric,对应于
随机变量的依概率收敛。
注意:距离函数不同,可能会导致点列的收敛性质不同.
定义:距离空间 E 的子集 A 的直径,定义为 sup x,y∈ A d ( x, y). A 的直径有限则
称 A 为有界。子集 A 、B 的距离定义为 inf d ( x, y),点到集合的距离定义为
x∈ A,y∈B
d ( x, B) = d ({ x} , B).

练习

1. 举例说明前述定义的集合的距离并不是 E 的幂集空间上的距离函数,也不
是拟距离.
2. 对于距离空间 E 上的非空闭集全体,验证如下定义的函数是其上的距离:

h(K, L) = max{sup d ( x, L), sup d ( x, K )}


x∈ K x∈ L

称为 Hausdorff 距离(可用于定义集值映射的连续性)。
30 CHAPTER 2. 距离空间

3. 若 d 是 E 上距离,证明 ρ ( x, y) = 1+d(x,y)
d(x,y) 也是 E 上的距离函数。(事实上,
这两个距离是等价的)
4. 证明 Levy’s metric 和 Ky Fan metric 确实是距离。
5. 设 ( X , O ) 是一个拓扑空间,如果 X 的任意两个不同的点 x 和 y 都可以用
两个不交的开集 U 和 V 分离,即 x ∈ U, y ∈ V ,且 U ∩ V = ;,则称 X 为
Hausdorff 空间。证明:距离空间是 Hausdorff 空间。Hausdorff 空间中若某
个序列有极限,则极限是唯一的。

2.3 Banach 空间和 Hilbert 空间

2.3.1 Banach 空间
我们熟知的 R 或 Rn 除了是个距离空间,还是个线性空间,即我们在其上定义
了加法与数乘的运算,更一般的:
线性空间定义:L 称为线性 (向量) 空间,如果
(1) 对任意两个元素 x, y ∈ L 可唯一确定第三个元素 z ∈ L,即 z = x + y 且
x + y = y + x,( x + y) + z = x + ( y + z),存在 0 元使 x + 0 = x,存在负元即 ∀ x,有
− x 使 x + (− x) = 0
¡ ¢ ¡ ¢
(2) 对任意实数 α 和 x ∈ L,可确定 α x ∈ L,且 α β x = αβ x,1 · x = x,
¡ ¢
α + β x = α x + β x,α ( x + y) = α x + α y
例如:R n ,C [a, b], l 2
范数的定义:在线性空间中我们可以定义范数 (长度函数)k·k : L → [0, ∞)
满足:
1. 0 ≤ || x|| < ∞ ∀x ∈ L
2. || x|| = 0 ⇔ x = 0
3. ||α x|| = |α| || x||,a ∈ R
4. || x + y|| ≤ || x|| + || y||
线性空间 L 和其上的范数 (L, ||·||) 就构成了一个赋范线性空间.
有了范数,很自然的我们可以定义函数 d ( x, y) = || x − y||,容易验证这是一个
距离函数,所以通常讨论 (L, ||·||) 赋范空间时,也把它视作距离空间.
定义:一个完备的赋范线性空间就叫做 Banach 空间.
(完备性见后文介绍)
2.3. BANACH 空间和 HILBERT 空间 31

注意:赋范空间可由范数诱导出距离,但也可以存在其它的距离,此外,并
非所有的距离都可由范数导出.否则 d (α x, α y) = |α| || x − y|| = |α| d ( x, y)
±
显然 |a − b| (1 + |a − b|) 不满足前述关系.
例子:
1. R 上的绝对值函数就定义了一个范数
µ ¶1
P
n p
2. R 中的 || x|| p =
n
|xi | p
(1 ≤ p < ∞) 定义了一个范数 (证明涉及几个重
i =1
要不等式)
|| x||∞ = max1≤ i≤n | x i | 也定义了一个范数.
³R ´1
3. C [a, b],|| f || p = ab | f ( t)| p dt (1 ≤ p < ∞),|| f ||∞ = maxa≤t≤b | f ( t)|
p

P
4. R ∞ 的一些子集 (即数列全体的子集):l p 代表 ∞ i =1 | x i | < ∞ 的 { x i } 全体.
p

l ∞ 代表所有有界的实数列 l ∞ = {( xn )n ⊆ R : supn | xn | < ∞}.c 0 代表收敛到 0 的数


µ ¶1
P
∞ p
列全体,c 代表收敛的数列全体。在 l p 中可定义 k xk p = |xi | p
,在 l ∞ 中定
i =1
义 k xk∞ = supn | xn |,则 (l p , ||·|| p ),(l ∞ , ||·||∞ ), ( c 0 , ||·||∞ ), ( c, ||·||∞ ) 都是 Banach 空
P

1 | xn − yn |
间。[在 R ∞ 中定义 Frechet 距离 d ( x, y) = 2n 1+| xn − yn | ,则 (R ∞ , d ) 是完备距离
n=1
空间]

练习

1. 证明:l 1 ⊂ l 2 ⊂ c 0 ⊂ c ⊂ l ∞

2.3.2 Hilbert 空间
通常的 n 维欧式空间的距离或者范数是通过内积定义的,将其推广就有了
Hilbert 空间。这是欧式空间最自然的推广。
内积的定义: X 是一线性空间。 X 上的内积是指满足下列条件的函数
〈·, ·〉 : X × X → R :
1. 〈 x, y〉 = 〈 y, x〉
2. ∀a 1 , a 2 ∈ R ,〈a 1 x + a 2 y, z〉 = a 1 〈 x, z〉 + a 2 〈 y, z〉
3. 〈 x, x〉 ≥ 0
4. 〈 x, x〉 = 0 ⇔ x = 0,
( X , 〈·, ·〉) 称为内积空间。
32 CHAPTER 2. 距离空间
p
定义:( X , 〈·, ·〉) 为内积空间,则 k xk = 〈 x, x〉 定义了其上的范数,如果 X 对
于距离 d ( x, y) = || x − y|| 是完备的,则称 X 为 Hilbert 空间。

2.3.3 线性拓扑空间
Banach 空间是在线性空间中利用范数引入拓扑,更一般的,有:
定义:线性空间 X 称为线性拓扑空间(拓扑向量空间)
,如果其被赋予一个
拓扑,且在此拓扑下加法运算和数乘运算是连续的。

2.3.4 几个重要的不等式
为了证明前面举的几个例子中定义的确实为范数.需要有下面几个重要的不等式
(我们只以 l p 为例进行说明)

定理 2.1. 1 < p < ∞,a,b ≥ 0.则 (a + b) p ≤ 2 p (a p + b p ).故 x,y ∈ l p ⇒ x + y ∈ l p

Proof. :(a + b) p ≤ (2 max {a, b}) p = 2 p max {a p , b p } ≤ 2 p (a p + b p )


p q
定理 2.2 (杨格不等式). :1 ≤ p < ∞, 1p + 1q = 1.则 ∀a,b ≥ 0,ab ≤ ap + bq ,等
号仅当 a p−1 = b 时成立.

Proof. 不妨设 a, b > 0.注意 f ( t) = t p−1 的反函数就是 g ( t) = t q−1 ( t ≥ 0)。由图


可知,有

Ra Rb ap q
ab ≤ 0 x p−1 dx + 0 y q−1 d y = p + bq ,
等号成立需 a p−1 = b.

注:等价的表述方式为若 0 < α < 1,a, b ≥ 0,则 aα b1−α ≤ αa + (1 − α)b。即


说明几何平均要小于算术平均
P∞
定理 2.3 (Hölder 不等式). 1 < p < ∞.1p + 1q = 1,x, y ∈ l p .则 i =1 | x i yi | ≤ k xk p k yk q
2.4. 距离空间的拓扑 33

Proof. ¯ ¯ ¯ ¯
X n ¯ ¯ 1X n ¯¯ x ¯¯ p Xn ¯ y ¯q
¯ x y
i i ¯ ¯ i ¯ + 1 ¯ i ¯ 1 1
¯ ¯≤ ¯ ¯ ¯ ¯ ≤ + =1
¯
i =1 k xk p k yk q
¯ p i=1 k xk p q i=1 ¯ k yk q ¯ p q

n → ∞ 即得

定理 2.4 (Minkowski 不等式). 1 < p < ∞.若 x, y ∈ l p ,则 k x + yk p ≤ k xk p + k yk p

1
Proof. 令 q 使 p + 1q = 1.则

X
∞ X
∞ X
∞ X

| x i + yi | p = | x i + yi | | x i + yi | p−1 ≤ | x i | · | x i + yi | p−1 + | yi | · | x i + yi | p−1
i =1 i =1 i =1 i =1

° ° ° °
≤ k xk p · °{| x + y| p−1 }° q + k yk p · °{| x + y| p−1 }° q
µ ¶1 µ ¶1
° ° P
∞ q P
∞ q ¡ ¢ p−1
° p−1 °
注意到 {| x + y| } q = | x i + yi | (p −1)q
= | x i + yi | p
= k x + yk p
i =1 i =1
P

p p−1 p−1
故 | x i + yi | p = k x + yk p ≤ k xk p · k x + yk p + k yk p · k x + yk p
i =1

p−1 ¡ ¢
= k x + yk p k xk p + k yk p ⇒ k x + yk p ≤ k xk p + k yk p

注:上述不等式说明 l p 空间确实是赋范线性空间,要说明其是 Banach 空


间还需验证完备性。

2.4 距离空间的拓扑

2.4.1 极限与收敛
定义:
1. 给定距离空间 (E, d ).x ∈ E ,r > 0.B r ( x) = { y ∈ E : d ( x, y) < r } 称为以 x 为球
心,半径为 r 的开球.{ y ∈ E : d ( x, y) ≤ r } 称为闭球,显然 A ⊂ E 有界,则必
存在 x 和 r ,使 A ⊂ B r ( x)
为方便记,有时候我们说 x 的邻域,就是指任何以 x 为球心的开球,ε-邻域
指半径为 ε 的开球.
34 CHAPTER 2. 距离空间

2. 称点列 { xn } 收敛 (在 E 中) 至 x ∈ E .如果 d ( xn , x) → 0,即 ∀ε > 0,∃ N ,使


得当 n ≥ N 时 d ( xn , x) < ε ⇔ ∀ε > 0,∃ N 使得 { xn : n ≥ N } ⊂ Bε ( x)
注意前面的定义直接与距离相关,后面的定义使用的是拓扑
此外,由三角不等式可知在距离空间中,若点列收敛则其极限必然是唯一的
(唯一性在一般的拓扑空间里未必成立)。
3. { xn } 在 (E, d ) 中是 Cauchy 列,如果 ∀ε > 0,∃ N ≥ 1,使得 m,n ≥ N 时总
有 d ( xm , xn ) < ε ⇔ ∀ε > 0,∃ N ≥ 1,使得 { xn : n ≥ N } 的直径 ≤ ε.
注意:一般来讲,Cauchy 列不一定收敛到 E 中的某一点,有界序列也并不
一定有收敛子序列,但收敛序列一定是柯西列,柯西列一定有界. 例如:
© 1 ª∞
1) E = (0, 1],通常的距离,序列 n n=1 是 Cauchy 列且有界
2) E = R ,离散距离 {n}∞
n=1 是有界序列,但不是 Cauchy 列

定义. 我们称 E 上的两个距离等价,如果它们生成的收敛序列是一样的 (更确切


的,如果诱导的拓扑是一样的),即

d ( x n , x) → 0 ⇔ ρ ( x n , x) → 0
ρ ρ
换言之,∀ x ∈ E, ε > 0,存在正数 r, s 使得 B ds ( x) ⊂ Bε ( x), B r ( x) ⊂ Bεd ( x)。

比如 R n 中,由 k·k1 ,k·k2 ,k·k∞ 引致的距离是等价的.(请证明)


s µ ¶
P
n
逐点收敛与按距离收敛:由于 R 上的通常距离 k x − yk2 = n
| x i − yi | 2

i =1
| x i − yi | ⇒ R n 中的收敛序列 (肯定是 Cauchy 列) 当且仅当它的每一维对应的序
列在 R 中收敛.所以和 R 一样,我们有:R n 中的 Cauchy 列都收敛,R n 中的
有界序列都有收敛子序列 (Bolzano-Weierstrass)
但当 R n 变成 R ∞ ,比如当我们考虑 l p 空间时,逐维的收敛却不能保证整个
向量的收敛,如:e(k) = (0, · · · , 0, 1, 0, · · ·)(第 k 个为 1),每一维都收敛到 0,但 e(k)
¡ ¢
在 l 1 ,l 2 或 l ∞ 中都不收敛到 (0,…,0,…).可以验证 d e(k) , 0 = 1,d ( e(k) , e(m) ) ≥ 1
© ª
对于任意的 k 6= m(这表明 e(k) 不是柯西列)

2.4.2 开集和闭集
定义:(E, d ) 中的 U 称为开集.如果 ∀ x ∈ U ,∃ε > 0 使得 Bε ( x) ⊂ U

例. ; 和 E 是开集.欧式空间 R 中的开区间,R n 中任意开球 Bε ( x) 是开集,赋


予离散距离的距离空间中的单点集
2.4. 距离空间的拓扑 35

注意. 给定拓扑空间 ( X , O ),对于 Y ⊂ X ,总可以定义相对拓扑 {O ∩ Y , O ∈ O }. 需


注意此时 Y 中的开集和 X 中开集的形态的差异。比如若 X = [0, 1],Y = (0.5, 1],
请问 (0.7, 1], (0.5, 0.7] 是开集还是闭集
开集的性质:
1. 对于任意开集族 {Uα }α∈ A ,则 V = ∪α∈ A Uα 为开集 (请补充证明)
2. 有限个开集的交仍为开集 (请补充证明),若无限个开集,则交未必为开集.
¡ ¢
如 ∩∞ 1 1
n=1 − n , n = {0}

3. U 是开集 ⇔ { xn } 收敛至 x ∈ U ,则除去有限个点之外,其余 xn ∈ U .

Proof. U 是开集, xn → x ∈ U 则有 ε > 0 使 n > N 的所有 xn ∈ Bε ( x) ⊂ U 。


U 非开,则 ∀ε > 0,存在 x ∈ U 使 Bε ( x) ∩ U c 6= ;.特别的,∀ n,有 xn ∈
B 1 ( x) ∩ U c ,但 { xn } ⊂ U c ,且 xn → x.
n

注意:这个性质表明在距离空间开集可以通过序列收敛的方法表述。
4. (R 中开集的构造) U 为 R 的开集,则 U 可以唯一的表示成可数个互不相
交的开区间的并.

Proof. 设 x ∈ U .可以通过 a x = inf {a : (a, x] ⊂ U } 和 b x = sup {b : [ x, b) ⊂ U } 得


到包含 x 但包含于 U 的最长的开区间 I x = (a x , b x ).即对于任意开区间 I ,
若 x ∈ I ⊂ U ,则有 I ⊂ I x .
注意到 x, y ∈ U ,则 I x ∩ I y = ; 或者 I x = I y .若 I x ∩ I y 6= ;,则 I x ∪ I y 也为
开区间,由 I x 和 I y 的最大性,必有 I x = I y .这样 U = ∪ x∈U I x 。因为 R 上
互不相交的开区间最多可数,得证.

注:R n 中的开集并没有这样的性质,只能表示为可数个互不相交的半开闭
矩形的并,且表示方法不唯一.
闭集:F 为闭集若 F c = E \F 为开集.
例:;, E 为闭集,欧式空间 R 中的闭区间以及任意有限集合,赋予离散距
离的距离空间中的单点集
注:有非开也非闭的集合,如 Q ,(0, 1]
闭集的性质:
1. 任意闭集族的交为闭集,有限个闭集的并为闭集 (请证明)
36 CHAPTER 2. 距离空间

2. 闭集的等价定义(留作练习):F 闭
⇔ 对所有 ε > 0,有 Bε ( x) ∩ F 6= ;,则 x ∈ F
⇔ 若序列 { xn } ⊂ F 收敛于点 x ∈ E ,则 x ∈ F (F 包含所有极限点)
内部的定义:(E, d ) 中子集 A 的内部 A o 指包含于 A 的最大开集.即
© ª © ª
A o = ∪ U : U 开,U ⊂ A = ∪ Bε ( x) : Bε ( x) ⊂ A, 对于某个 x ∈ A, ε > 0
© ª
= x ∈ A : Bε ( x) ⊂ A 对于某个ε > 0 ,内部的点称为内点.
© ª
A 的闭包 Ā 指包含 A 的最小闭集: Ā = ∩ F : F 闭, A ⊂ F
闭包的性质
1. x ∈ Ā ⇔ 对任一 ε > 0,Bε ( x) ∩ A 6= ;
2. x ∈ Ā ⇔ 存在 A 中序列 { xn }, xn → x.即 Ā 是 A 中所有可能的收敛序列的
极限的全体 (包括常序列)
Fσ 集:可数个闭集的并集
G δ 集:可数个开集的交集
Fσδ 集:可数个 Fσ 集的交集
Borel 集:所有开集生成的 σ 代数称为 Borel σ 代数,其中的元素称为 Borel
集。
例. 在实数空间,有理数集是 Fσ 集,但不是 G δ 集
思考:开集,闭集,Fσ 集,G δ 集,Fσδ 集,G δσ 集等等是否可穷尽 R 中的
Borel 集的形态
极限点 (limit point):x 称为 A 的极限点,如果 ∀ε > 0,(Bε ( x) \ { x}) ∩ A 6= ; ⇔
存在 A 中互异的点列 { xn } → x.(只需 xn 6= x, ∀n) ⇔ x ∈ A \{ x}
凝点 (condensation point):x 称为 A 的凝点,如果 x 的任意邻域与 A 的交
集都是不可数集。
导集 A 0 为 A 的所有极限点的集合. A 0 是闭集且 Ā = A 0 ∪ A
孤立点: x 称为 A 的孤立点,如果存在某个 ε > 0,使 (Bε ( x) \ { x}) ∩ A = ;.
但 x∈ A
边界点: x 称为 A 的边界点,如果 Bε ( x) ∩ A 6= ;,Bε ( x) ∩ A c 6= ;,∀ε > 0

练习

1. A 的边界= A c 的边界, A 的闭包= A 的内部 ∪ A 的边界, 全集 E = A 的内


部 ∪ A 的边界 ∪ A c 的内部
2.4. 距离空间的拓扑 37

2. 下列关系是否正确? X 为距离空间,Y 与 Z 是 X 的子集


X \Y = X \Y , X \Y = X \Y o ,( X \Y ) o = X \Y ,( X \Y ) o = X \Y o
Y ∩ Z = Y ∩ Z ,Y ∪ Z = Y ∪ Z ,(Y ∩ Z ) o = Y o ∩ Z o ,(Y ∪ Z ) o = Y o ∪ Z o
3. 在距离空间 (E, d ) 中,a ∈ E ,r > 0,总有 { x : d ( x, a) < r } ⊂ { x : d ( x, a) ≤ r } [本
题旨在说明在距离空间中,开球的闭包并不一定就是闭球]
4. 请举例说明在距离空间中,开球可以是闭集但不是闭球,闭球可以是开集但
不是开球

2.4.3 稠密和可分
完全集 (Perfect set):A 为完全集若 A = ; 或 A = A 0 (例:R 作为 R 2 子集时,是
完全集)

定理. R 的任意非空完全子集是不可数的 (如 Cantor 集)

Proof. 首先 A 无限(为什么?
).若可数 A = { x1 , x2 , · · ·}.由于 x1 与 x2 皆为极限
点.故可找到包含 x2 的闭区间 [a 1 , b1 ] 满足 (a 1 , b1 ) ∩ A 6= ;,且 x1 ∉ [a 1 , b1 ].而且
[a 1 , b 1 ] 中有无限个 A 中的点.继续下去可找到一系列递减的闭区间套 [a n , b n ],
(a n , b n ) ∩ A 6= ;,且 xn ∉ [a n , b n ],矛盾.

定理 (R n 中闭集的一种构造). 若 A 是 R n 中闭集,则 A 可表示成 A = P ∪ C ,


其中 P 为完全子集,C 为一可数子集。

Proof. 由下面的定理知 R n 有可数基 B 。记 A 的所有凝点的集合为 P ,C = A ∩ P c 。


由于凝点是极限点,所以 P ⊂ A 。因为 C 不是凝点,所以 C 中每一个点 x 都可
S
找到邻域 Vx ∈ B 使得 A ∩ Vx 可数,因为 C ⊂ { A ∩ Vx : x ∈ C },故 C 可数。下
面说明 P 是完全集。为此我们依次证明 P 没有孤立点,P 包含了所有 P 的
极限点。设 x ∈ P ,U 是 x 的一个邻域,则 U ∩ A 不可数,U ∩ C 可数,所以
U ∩ P = (U ∩ A ) ∩ (U ∩ C ) c 不可数,故 x 是 P 的极限点,不是 P 的孤立点。设
x ∈ P c ,要证明 x 不是 P 的极限点,即只需找到一个邻域 V 使 V ∩ P = ;。由于
x 不是 A 的凝点,必有一个邻域 V 使得 V ∩ A 可数。假如有 y ∈ V ∩ P ,则 V 是
y 的邻域,因为 y 是 A 的凝点,所以 V ∩ A 不可数,矛盾,故 V ∩ P = ;,说明
x 不是 P 的极限点。得证。

稠密: A 稠密如果 A = E
可分 (separable):称一个空间可分,如果它含有一个可数的稠密子集
38 CHAPTER 2. 距离空间

注意. 可分距离空间的子集也是可分的。(如何证明?)

定理 2.5. :l p ( p ≥ 1) 可分,l ∞ 不可分.C [0, 1](一致度量) 可分

Proof. 令 D = {( xn ) ∈ l p : xn 为有理数, 只有有限个n使得 xn 6= 0},显然 D 可数。我们


P∞ p P∞ p
证明 D 在 l p 中稠密。设 x = ( xn ) ∈ l p ,ε > 0,则 n=1 | x n | < ∞,lim n= N +1 | x n | =
P∞ N →∞
p
0。可选取 N 使得 n= N +1 | x n | < ε p /2。现在对每个 n,在 D 中选取有理数 y = ( yn ),
使得:
若 1 ≤ n ≤ N ,则 | xn − yn | < (2 N )−1/p ;若 n > N , yn = 0
PN P∞
显然,|| x − y|| pp = n=1 | x n − yn |
p
+ n= N +1 | x n − yn |
p
< N ε p /(2 N ) + ε p /2 = ε p ,说
明 l p 可分。
为证 l ∞ 不可分,假定 D = {( x1n ), ( x2n ), ..., ( x jn ), ...} 为 l ∞ 中的稠密集合,如
下定义 y = ( yn ): (
xnn + 1, 若| xnn | ≤ 1
yn =
0, 否则
不难看出数列 ( yn ) 是有界的即 y ∈ l ∞ ,而且 || y − x||∞ ≥ 1,表明 D 不稠密。
矛盾。

定义 (拓扑空间的基,次基). 若 B 是一开集族,可使得拓扑空间里的任意开集
可由 B 中某些开集并成,则称 B 为该拓扑空间的基。
任意拓扑基 B 应当满足
1. ∀ x ∈ X ,∃B ∈ B ,使得 x ∈ B
2. ∀ x ∈ B1 ∩ B2 , B1 , B2 ∈ B ,∃B ∈ B ,使得 x ∈ B ⊂ B1 ∩ B2
如果集合族 S 中的元素的有限交可构成拓扑空间的基,称 S 为该拓扑空间的
次基。

例. 在全体实数集合 R 上,半开闭区间 [ x, r ) (r 为有理数) 全体作为基,可以生


成一个拓扑。请验证 [ x, r ) 在该拓扑下也是闭集。

定理 2.6. 可分距离空间有可数基。

Proof. B = {Bε ( x) , x 为稠密可数子集 D 中元素,ε 为正有理数 }。


则 B 是由可数个开集组成,且对任意开集 U ,若 z ∈ U ,则存在 ε 使
Bε ( z) ⊂ U ,且总可以找到 x ∈ D ∩ B ε ( z)。
3
取有理数 r ∈ ( 3ε , 23ε ),则 y ∈ B r ( x) ⇒ d ( y, z) ≤ d ( x, y) + d ( x, z) < r + 13 ε < ε
⇒ B r ( x) ⊂ Bε ( z) ⊂ U ,又 z ∈ B r ( x).故 U 为 B 中开集的并.
2.4. 距离空间的拓扑 39

注意. 具有可数基的拓扑空间称为第二可数的 (second-countable)。在这样的拓


扑空间里,任意开覆盖都有可数的子覆盖 [在距离空间中,如果任意开覆盖都有
可数的子覆盖,则空间有可数基],而且这个空间是可分的
¡ ¢o
无处稠密集:(E, d ) 的子集 A 称为无处稠密的,如果 Ā = ;.
¡¡ ¢ o ¢ c ¡ ¢
之所以称为无处稠密,是因为 E = Ā = Ā c ⇒ Ā c 稠密(请证明中间的
等式)
(E, d ) 的 子 集 Y 称 为 第 一 纲 集, 如 果 存 在 可 数 个 无 处 稠 密 集 A n 使 得
S∞
Y = n=1 A n 。不是第一纲集的集合称为第二纲集。
以实数空间为例:称集合 A 在区间 I 中稠密,如果 A 与 I 的任意子区间有
非空交集.称 A 稠密,即指 A 在 R 中稠密,或者 A 的闭包= R。称 A 无处稠
密,如果 A 在任何区间中都不稠密,即任何区间都有一个子区间包含在 A c 之
中.或者说 Ā 无内点,或者 A c 包含一个稠密的开集.
©1 ª
例子:1. R 中,Q 是稠密的,但 Z 和 n , n = 1, 2, · · · 无处稠密.
2. 如果 A 无处稠密,则 A 稠密,但也有可能 A 与 A c 都稠密 (如 Q )
c

直接的性质:
(1) A 在 E 上稠密. A ⊂ B ⊂ E ,则 B 在 E 上稠密
(2) A 稠密 ⇔ E 的任何非空开集与 A 相交
(3) A 无处稠密 ⇔ Ā 无处稠密 ⇔ Ā c 稠密 ⇔ E 的任何非空开集包含与 A 不
相交的非空子开集
(4) A 与 B 在 E 上无处稠密,则 A ∪ B 也无处稠密
(5) A 无处稠密,则 A 的任何子集也无处稠密
比如 (4):AB 无处稠密 ⇒ 对任意区间 I ,存在区间 I 1 ⊂ I − A ,而对于区间
I 1 ,存在区间 I 2 ⊂ I 1 − B ⇒ 对任意区间 I ,存在区间 I 2 ⊂ I − ( A ∪ B) ⇒ A ∪ B 无
处稠密.
我们已经讨论了可数与不可数的定义,也谈到了 Q 的可数性与 R 的不可数
性,在展开下面度量空间的拓扑性质和其它性质的讨论之前,我们从 R 的完备
性和特殊的拓扑性质角度说明 R 的不可数性不是偶然的.更一般的,有一个重
要的定理 Baire 定理,告诉我们,任意的完备度量空间都不能表示为可数个无处
稠密集的并的形式.
定理 2.7 (Baire 定理). R 的任何区间都不是第一纲集.
S

Proof. 不妨假设该区间就为 A = [0, 1].反证法:假设 A = A i ,A i 为无处稠密
i =1
集.由于 A i 无处稠密,存在长度 < 21 的闭区间 I 1 ⊂ A 1c ;因 A 2 在 I 1 中无处稠
40 CHAPTER 2. 距离空间

密,存在长度 < 13µ 的闭区间 ¶


I 2 ⊂ I 1 ∩ A 2c ;继续下去可得一列递降的闭区间,长度
S
n
→ 0,且 I n ⊂ A \ A i .由闭区间套定理,存在 x ∈ I n ,∀ n.显然 ∀ i , x ∉ A i ,
i =1
矛盾.故 A 不可能是第一纲集.

注:单点集也是第一纲集.故 R 不可能只有可数个点.

练习

1. 对于 [0, 1] 上的连续函数全体 C [0, 1],定义两个距离,


Z 1
ρ ( f , g) = ( ( f ( x) − g( x))2 dx)1/2
0
d ( f , g) = max0≤ x≤1 | f ( x) − g( x)|

令 U = { x ∈ C [0, 1] : ρ ( x, 0) ≤ 1}, V = { x ∈ C [0, 1] : d ( x, 0) ≤ 1},证明:V ⊂ U ,V c


在 U 中稠密

2.5 连续性和乘积拓扑
定义. 从拓扑空间 ( M, O M ) 到 ( N, O N ) 的函数是连续的,如果对任意 N 中开集
O , f −1 (O ) 是 M 中开集。

问题:给定非空集合 Ω 和一族函数 f ∈ F ,f : Ω → N f ,( N f , T f ) 为拓扑空间


1. 能否找到 Ω 上拓扑使得 f 都连续——离散拓扑
2. 能否找到最弱的拓扑使得 f 都连续——乘积拓扑
先来看如何利用 R 的拓扑导出 R2 上的拓扑。
µ ¶1
P
2 2
R 上有距离函数 d 1 ,在 R2 定义距离函数 d 2 ( x, y) = ( d 1 ( x i , yi ))2
i =1
R 上的全体开区间(包括空集)构成拓扑基 W = {ω i = (a i , b i )},在 R 2 上定
义拓扑基为 {ω i × ω j : ω i , ω j ∈ W },或者 {ω i × R : ω i ∈ W } ∪ {R × ω i : ω i ∈ W } 构成一
个次基
可以验证,上述两种方法定义的拓扑是一样的
对于任意个拓扑空间的乘积拓扑其定义如下:
Q Q
定义. X α 上的乘积拓扑的拓扑基是形如 Uα 的全体,其中
(1). ∀α ∈ A ,Uα 是 X α 中的开集;
(2). 除了有限个维度外,Uα = X α 。
2.5. 连续性和乘积拓扑 41
Q Q
若定义投影映射 p β : X α → X β : p β ( x) = xβ , X α 上的乘积拓扑的次基也
可以是 { p−1
α (Uα ) : α ∈ A,Uα ∈ Oα },其中 Oα 是 X α 的一个拓扑基。
R
例子: X = R ,R 上实值函数的全体。令 F = { x1 , ..., xn }(R 的有限子集),
对应有 n 个正数 {ε1 , ..., εn },记 ε = min({ε1 , ..., εn }),定义 U ( f , F, ε) = { g ∈ X :
∀ x ∈ F, | g( x) − f ( x)| < ε},那么当 F 取遍 R 的所有有限子集,ε 取遍所有正数时
U ( f , F, ε) 全体就形成了 X 在乘积拓扑下 f 的邻域基,即 f 的任意一个邻域必
包含某个 U ( f , F, ε)。[比较距离空间-概念讲义最后部分]

Q
定理 2.8. 乘积拓扑是 X α 上使得所有投影映射 p β 连续的最弱的拓扑。[称拓
扑空间 X 上的拓扑 τ1 弱于拓扑 τ2 ,如果 τ1 ⊂ τ2 ]

Proof. 如 果 τ 是 使 得 每 个 投 影 都 连 续 的 拓 扑, 那 么 ∀β, 若 Uβ 在 X β 开,
p−
β
1
(Uβ ) ∈ τ,得证。

注意. 根据这个定理,可以知道给定集合 X,一族函数 F = { f α : X → Yα , α ∈ A },


和一族拓扑空间 {(Yα , Tα ), α ∈ A },总可以构造出 X 上最弱的拓扑使得每个 f α 都
连续。具体的做法是,先根据一族拓扑空间生成乘积拓扑,再由一族函数构造高
维函数 f ,即 f 取值的每个维度刚好对应某个 f α ,于是这个乘积拓扑在 f 下的
原像就是那个使得所有函数连续的最弱的拓扑。这个拓扑通常被称为弱拓扑。而
依据距离导出的拓扑通常被称为强拓扑。
R 上的向量空间赋予一定的拓扑后称为拓扑向量空间,如果加法 + 和数乘
· 运算对于 X × X 和 R × X 上的乘积拓扑连续。
Q Q
定理 2.9. X α 上赋予乘积拓扑, f : X → X α 连续 ⇔∀α ∈ A , p α ◦ f 连续。

Proof. ⇒ 显然。
Q
⇐:若 ∀α ∈ A , p α ◦ f 连续,{ p− 1
α (Uα ) : α ∈ A,Uα ∈ O α } 是 X α 上乘积拓扑
Q
的一个次基。注意到 f −1 ( p−α1 (Uα )) = ( p α ◦ f )−1 (Uα ) ,所以 X α 的基本开集在 f
下的原象是 X 中开集。
42 CHAPTER 2. 距离空间

定理实际上表明在乘积拓扑下,逐点收敛与整体收敛是等价的。注意,如果
乘积空间里的拓扑如果不是乘积拓扑,结论就不成立了。

练习
Q
1. 给定一族拓扑空间 {( X α , Tα ), α ∈ A },在 X α 上可定义盒拓扑,盒拓扑的基

Y
∪{ Oα : Oα ∈ Tα }

显然盒拓扑比乘积拓扑更细,其中包含的开集要多很多。试证明:Hilbert
方体 [0, 1] N 若赋予乘积拓扑,则序列 { xn } = { n1 , ..., n1 , ...} 收敛到 0,但在盒拓
Q∞ 1
扑下不收敛。[考虑集合 i =1 [0, i )]

2. B = {[a, b) : a, b ∈ R},T 为 B 生成的拓扑,举例说明存在实值函数使得其


对于 T 是连续的,但是对于 R 上的欧式拓扑不连续

2.6 拓扑空间与距离空间收敛的差异
这里我们举例说明拓扑空间的收敛并不总能表示为距离空间的收敛,考察的对
象是 [a,b] 上的实值函数列 { f n } 的点点收敛

2.6.1 函数列的点点收敛不能通过距离函数描述
记 [a,b] 上的实值函数全体为 F 。假定存在这样的距离函数使得 d ( f n , f ) → 0 等
价于函数列点点收敛,那么对于 x0 ∈ [a, b],令 U ( x0 ) = { f ∈ F : | f ( x0 )| < 1}。
那么 U ( x0 ) 是开集,为此我们说明 (U ( x0 )) c 是闭集。设 { f k } 是 (U ( x0 )) c 中的
一列函数, d ( f k , f ) → 0, f ∈ F 。那么因为点点收敛,| f ( x0 )| ≥ 1。故 f ∈ (U ( x0 )) c ,
即 (U ( x0 )) c 为闭集,U ( x0 ) 是开集。显然 0 函数属于 U ( x0 ),所以存在 n 使得
B 1 (0) ⊂ U ( x0 )。
n
S

令 A n = { x ∈ [a, b] : B 1 (0) ⊂ U ( x)},则 [a, b] = A n 。由于 [a, b] 不可数,应
n n=1
当存在 n 使得 A n 是无穷的。设 X = { x1 , x2 , ...} 为 A n 的可数子集,考虑函数列
{1{ xk } }。不难看出,对任意的 x ∈ [a, b],1{ xk } ( x) → 0 即 {1{ xk } } 点点收敛到 0 函数。
但是 ∀n, 1{xk } ∈ (B 1 (0)) c ,故 ∀n, k, d (1{xk } , 0) ≥ n1 ,说明 {1{xk } } 在距离 d 下不会收
n
敛到 0。矛盾。
2.6. 拓扑空间与距离空间收敛的差异 43

2.6.2 函数列的点点收敛可以看成函数空间赋予一定的拓扑后其中
的收敛
对于 x ∈ [a, b] 及 R 中的非空开集 G ,定义 U ( x,G ) = { f ∈ F : f ( x) ∈ G }。构造 F
上的拓扑 T :记形如 V = U ( x1 ,G 1 ) ∩ ... ∩ U ( xn ,G n ) 的集合的全体为 B ,T 中任
意集合是由 B 中集合的并形成。可以验证 T 确为拓扑(可作为练习)。
如果 U ∈ T , f ∈ U ,那么存在集合 V ∈ B 使得 f ∈ V ⊂ U 。注意 V 的表示形
式,可知 f ∈ V ⇔ ∀ i, f ( x i ) ∈ G i 。所以 F 中的 { f n } 点点收敛到 f ∈ F ⇔ 对包含 f
的所有 B 中集合 V ,存在 N0 使得当 n ≥ N0 时 f n ∈ V 。(可作为练习)

2.6.3 为什么拓扑可以而距离不可以?
距离空间的收敛、开集、闭集是可以通过序列收敛描述的,而一般拓扑空间里的
不是。
如 A = { f ∈ F : 除了一个可数集外 f ≥ 0},若 { f n } 为 A 中序列,且点点收敛
到 f ,则 f ∈ A ,表明 A 在序列点点收敛下是闭的。
但实际上 A 不是拓扑空间里的闭集,因为 A c = { f ∈ F : 在一个不可数集上 f <
0} 不是 T 中的集合。为说明 A c 不在 T 中,令 f ≡ −1,则 f ( ∈ A c 。总可以选出
−1 若 x1 , ..., xn
开集 V ∈ B 使得 f ∈ V = U ( x1 ,G 1 ) ∩ ... ∩ U ( xn ,G n ),定义 g( x) = ,
1 其他
则 g ∈ V ∩ A 。这表明任何包含 f 的开集都与 A 有非空的交集,所以 A c 不是开
集, A 不是闭集。

2.6.4 拓扑空间里的收敛
在拓扑空间中,仅用序列收敛的概念是不够的,通常对应的是网 (Net) 的收敛

定义. 有向集是指非空集合 A 和定义于集合上的具有自反性和传递性的二元关


系 ≺,满足:对于 A 中任意两个元素 a 和 b,存在着 A 中的一个元素 c(可以
为 a 或 b)使得 a ≺ c 和 b ≺ c.
拓扑空间 X 中的网是指从一有向集 A 到 X 的映射 { xα }α∈ A
称拓扑空间上的网 { xα }α∈ A 收敛到点 x,如果对于任意包含 x 的开集 O ,都
存在 α0 ∈ A 使得只要 α0 ≺ a 就有 xa ∈ O 。

例. 1. 非空实数集在通常的大小比较关系下形成有向集
44 CHAPTER 2. 距离空间

2. 集合 S 的有限子集全体 P f (S ) 在包含关系 ⊂ 下是有向集


3. 收敛序列也是收敛的网
4. 函数 f : [a, ∞) → R 可以看成是 R 中的网 { f ( t), t ∈ [a, ∞)}。lim f ( t) = L ⇔
∀ϵ > 0, ∃ M ∈ R 使得当 t ≥ max(a, M ) 时,总有 f ( t) ∈ (L − ϵ, L + ϵ)
P
5. 称无穷和 j∈S x j 收敛到 s 如果网 {s F , F ∈ P f (S )} 收敛到 s,其中
X
sF = xj
j ∈F ∈P f (S)

定理 2.10. f : M → N 连续 ⇔ 若网 { xl }l ∈ I 收敛于 x,则网 { f ( xl )}l ∈ I 收敛于 f ( x)


第 3 章 完备性与紧性

“..three notions which were to play a fundamental part in all later developments
of Functional Analysis: compactness, completeness, and separability”

Dieudonné, J., 1981, History of Functional Analysis

3.1 全有界
在实数空间,我们知道 Cauchy 列是有界的,而且有界序列有收敛的子序列.
但 是 在 一 般 度 量 空 间 中, 有 界 性 与 Cauchy 列 的 关 系 就 没 有 这 么 紧 密。
之 前 我 们 已 举 过 例 子, 有 界 序 列 中 任 何 子 序 列 都 不 是 Cauchy 列 [l 2 空 间,
{(0, ..., 0, 1, 0, ...)}]。在一般度量空间中,要保证序列有子序列为 Cauchy 列,有界
应改为全有界 (totally bounded)
全有界的定义: A ⊂ (E, d ) 为全有界的,如果给定 ε > 0,存在有限个点
S
n
x1 , x2 , ..., xn ∈ E ,使 A ⊂ Bε ( x i ),即 A 被有限个 ε 球覆盖.事实上,可以认为
i =1
这些 ε 球的中心都在 A 中.
引 理: A 全 有 界 ⇔ 给 定 ε > 0, 存 在 有 限 个 集 合 A 1 ,…, A n ⊂ A ,∀ i ,
Sn
diam ( A i ) < ε,且 A ⊂ i =1
Ai
例:1. 由三角不等式,全有界是有界的,但反之不对
2. 有限点集总是全有界的.
下一定理表明了度量空间中 Cauchy 列和全有界的关系.

定理 3.1. { xn } 为 (E, d ) 中的序列, A = { xn : n ≥ 1} 则


(1){ xn } 为 Cauchy 列,则 A 全有界
(2) A 全有界,则 { xn } 有 Cauchy 子序列

45
46 CHAPTER 3. 完备性与紧性

Proof. (1)ε > 0,{ xn } 为 Cauchy 列,存在 N 使 diam { xn : x ≥ N } < ε.这样


A = { x1 } ∪ · · · ∪ { x N −1 } ∪ { xn : n ≥ N } 即为 N 个直径小于 ε 的集合
(2) 不妨设 A 无限,A 可以被有限个直径小于 1 的集合覆盖,其中之一必有
A 中无限个点记为 A 1 .A 1 也全有界,可被有限个直径小于 1/2 的集合覆盖,其
中有某个 A 2 有无限个点.继续下去,可有 A ⊃ A 1 ⊃ A 2 ⊃ · · · , A k 中有无穷点,
± © ª © ª
且直径 < 1 k.于是可取 xn k 使 xn k ∈ A k ,则 xn k 是 Cauchy 列.

定理 3.2. A 全有界 ⇔ A 中每个序列都有 Cauchy 子序列.

Proof. ⇐ 若 A 不全有界,则存在 ε > 0,使 A 不能被有限个 ε− 球覆盖.这样我


们可以找到一个序列 { xn } 使得 d ( xn , xm ) ≥ ε 若 m 6= n,这样 { xn } 就没有 Cauchy
子序列.

注:我们知道,对于一个距离空间 (E, d ),距离 ρ = 1+dd 总是与 d 是等价的,


而在 ρ 下该空间总是有界的,这说明有界与否与该空间的拓扑性质关系不大

3.2 完备性

3.2.1 概念
完备性:距离空间 E 是完备的,如果 E 中任意的 Cauchy 列都收敛到 E 中的某
一点.
例:
1. 欧式空间 R ,R n 都完备.
2. (0, 1) 不完备,但 (0, 1) 与 R 同胚,所以完备性不是拓扑性质
3. l 1 , l 2 ,l ∞ 完备, l ∞ 相当于 B ( X )(有界实值函数全体)[值域是 N],k f k∞ =
sup | f ( x)|
x∈ X

l 2 完备的证明. { f n } 是 l 2 中柯西列.f n = ( f n ( k))∞


k=1 (练习:l 1 、l ∞ 的完备性)
令 f (k) = lim f n (k),因 { f n } 为柯西列,| f n (k) − f m (k)| ≤ k f n − f m k2 → 0 知
n→∞
{ f n ( k)}∞
n=1 为柯西列,故极限存在 ( l 2 收敛 ⇒ 点点收敛)
由于 { f n } 为柯西列,故存在某个 B 使得 k f n k2 ≤ B,
X
n n ¯¯
X
¯2
¯ X
n ¯ ¯
2
| f ( k )| = ¯ lim f ( k ) ¯ = lim ¯ f j ( k)¯2 ≤ B2 ⇒ f ∈ l 2
¯ j→∞ j ¯ j →∞
k=1 k=1 k=1
3.2. 完备性 47

P
N
由 | f n ( k) − f m ( k)|2 ≤ (k f n − f m k2 )2 可知
k=1
P
N P
N
∀N , | f ( k) − f n ( k)|2 = lim | f m ( k) − f n ( k)|2 ≤ ε2 ⇒ k f − f n k2 ≤ ε,当 n
k=1 m→∞ k=1
足够大时.

4. C [a, b] 完备,B [a, b] 完备,([a, b] 上的有界函数)

定理 3.3 (完备性和闭性). 完备度量空间 (E, d ) 的子空间 ( A, d ) 是完备的,当且


仅当 A 在 E 中是闭集.

下面的定理是闭区间套定理和 Bolzano-Weierstrass 定理的推广.

定理 3.4. 对于任何度量空间 (E, d ),下面陈述等价


1. (E, d ) 完备
T∞
2. E ⊃ F1 ⊃ F2 …为递减的闭集列、非空,且 diam (F n ) → 0,则 n=1 F n 6= ;
3. 每一个无穷且全有界的 E 的子集有在 E 中的极限点.

Proof. 1⇒2. 可在 F n 中找 xn .因 F n 递减,{ xk , k ≥ n} ⊂ F n 且 diam { xk : k ≥ n} → 0


表明 { xn } 为 Cauchy 列.由 E 完备性, xn → x ∈ E .由 F n 的闭性, x ∈ F n .
2⇒1. 对于 Cauchy 列 { xn }, 令 F i = { xn }∞
n= i
,则极限 x ∈ ∩∞
n=1 F n ⊂ E
1,2⇒3. 若 A 无穷,全有界,则在 A 中可找到互异点列是 Cauchy 列 { xn }.
令 A n = { xk , k ≥ n},则 A ⊃ A ⊃ A 2 ⊃ · · · . diam ( A n ) → 0.
³ 1´
又 A n ⊃ A n+1 6= ;. diam A n = diam ( A n ) → 0
³ ´
故存在 x ∈ ∩∞
n=1 Ā n 6= ;. d ( x n , x) ≤ diam A n → 0,⇒ x 是极限点.
3⇒1. 设 { xn } 为 E 中 Cauchy 列.则 A = { xn , n ≥ 1} 是全有界的.故存在 E
中的极限点,表明 E 是完备的.

练习

1. 如果 f : R → R 连续,{ xn } 为 Cauchy 列.问 { f ( xn )} 是 Cauchy 列吗?如果


f 满足 Lipschitz 条件呢?
2. 若距离空间上的距离函数还满足 d ( x, z) ≤ max(d ( x, y), d ( y, z)),证明空间中
序列为 Cauchy 列的充要条件是 limn→∞ d ( xn , xn+1 ) = 0
Qn
3. 若 n 个距离空间 ( X i , d i ) 各自都完备,证明乘积距离空间 ( 1 X i , d) 是完备
的,其中 d ({ x i }, { yi }) = max d i ( x i , yi ) 或者 d ({ x i }, { yi }) = Σ i=1 d i ( x i , yi )
n
48 CHAPTER 3. 完备性与紧性

4. 举例或证明:定理中闭集套的直径 → 0 是必需的吗?(完备距离空间中渐缩
的非空闭集列的交是否非空)
5. 举例或证明:定理 3.4 中的全有界能改成有界吗?
提示: X 为自然数集合, d (m, n) = 1 + 1/ (m + n) 若 m 6= n,否则为;令 B n =
© 1
ª
m : d ( n, m) ≤ 1 + 2n = { n, n + 1, . . .}

3.2.2 完备空间的性质(Baire 纲定理)

回忆一些定义:
¡ ¢o
无处稠密集:(E, d ) 的子集 A 称为无处稠密的,如果 Ā = ;.
¡¡ ¢ o ¢ c ¡ ¢
因为 E = Ā = Ā c ⇒ Ā c 稠密
(E, d ) 的 子 集 Y 称 为 第 一 纲 集. 如 果 存 在 可 数 个 无 处 稠 密 集 A n 使 得
S∞
Y = n=1 A n
不是第一纲集的集称为第二纲集.
例子:Cantor 集是 [0,1] 中的第一纲集,作为 R 的子集,Q 是第一纲集,
R\Q 为第二纲集

定理 3.5 (Baire 定理). 1. 完备距离空间是第二纲集。


2. 完备距离空间的稠密开集列的交集是稠密的。
S

Proof. 1. 假定 A ⊂ E 且为第一纲集且 A = A n ,每个 A n 无处稠密,不妨设
n=1
A n 为闭集(为什么?)。设 V 是 E 的任意非空开集,我们证明 V ∩ A c 6= ;。
1
假设 V ∩ A c = ;,因为 A 1c 稠密,总可以取非空开集 U1 ⊂ V ,为半径 < 2
1
的开球,使 U1 ∩ A 1c 为非空开集。由于 U1 ∩ A 1c 开,可取 U2 为半径 < 3 的
开球,使 U2 ⊂ U1 ∩ A 1c .假设 U1 , ...,Un 已选好,∀ j = 1, ..., n − 1,U j+1 非空开,
1
U j+1 ⊂ U j ∩ A cj ,U j+1 的半径 < j +1 。因 Un ∩ A nc 非空开,故存在 Un+1 ,满足
1
Un+1 ⊂ Un ∩ A nc 且半径 < n+1 ,于是得到非空闭球套,直径 → 0.由完备性.知
3.2. 完备性 49
µ ¶
T
∞ ∞ ¡
T ¢ T

c
存在 x ∈ Un+1 ⊂ U n ∩ A n ⊂ U1 ∩ A n ⊂ V ∩ A c 。矛盾。由 V 的任意性,
c
n=1 n=1 n=1
可知 A c 稠密,非空,所以 E 不可能是第一纲集。
2. 注意到证明 1 中 A n 无处稠密时,( A n ) c 是稠密开集。

注意. • Cantor 集作为全空间本身是完备的,这意味着作为其自身的子集,它


是第二纲集。
• 对于开区间 (0, 1)Baire 纲定理也成立,即 (0, 1) 不可以表示为可数个无处稠
密集的并。事实上,只要一个集合与某个完备距离空间同胚,都不可以表示
为无数个可数稠密集的并。

距离空间称为 Baire 空间,如果它的每个非空开集都不是第一纲集.


根据 Baire 纲定理的证明思路,对距离空间 E 而言,下述陈述等价:
1. E 为 Baire 空间
2. 可数个稠密开集的交仍然稠密
S∞ S∞ o
3. 若 E = n=1 F n ,F n 闭.则 n=1 (F n ) 稠密.
Baire 纲定理的应用:
1. Q 不可能表示成 R 中可数个开集的交集 (不是 G δ 集)。不妨设 Q = { q1 , q2 , ...}。
假如 Q = ∩∞ ∞
n=1U n ,U n 开,则 ∩ n=1 (U n \{ q n }) = ;。因为 Q 为 U n 子集,U n
稠密,Un \{ q n } 是稠密开集,根据 Baire 纲定理,可数个稠密开集的交非空,
矛盾。
2. 1Q 不是一列连续函数的简单极限。假如 ∀ x ∈ R, f n ( x) → 1Q ( x),设 Um =
T
∞ S
f −1 ((0.5, ∞)),则因为 f m 连续,∀ n,∪m≥nUm 开。由假设 Q = Um ,
n=1 m≥ n
与 1 矛盾。
Baire 纲定理通常用于存在性的证明。在一个完备空间里,证明满足某个性
质的点的存在,可以讨论该性质不成立的点集的集合,若该集合是第一纲集,即
得证。(有兴趣的可以翻看泛函分析中的开映射定理和闭图像定理的证明,在连
续函数空间一节的练习中我们也会看到该定理的应用)。
利用 Baire 纲定理也可以证明线性空间不可能有可列的 Hamel 基,作为推
论,由单变量或多变量多项式组成的空间不可能装备范数成为 Banach 空间。
注意,一个线性空间 X 的 Hamel 基指这样的向量族 ( e i ) i∈ I , e i ∈ X ,满足:1,
向量族的元是线性无关的,即族内任何有限子集 ( e j ) j∈ J 和任意一组数 (a j ) j∈ J ,
Σ j∈ J a j e j = 0 意味着所有 a j 为 0。2. ( e i ) i∈ I , e i ∈ X 能张成整个空间 X ,即任
50 CHAPTER 3. 完备性与紧性

意 x ∈ X 可以表示成有限个 e i 的线性组合。Hamel 基中元素的个数也就是空间


的维数
同时,我们称 ( xn ) 是 X 的 Schauder 基,如果对任意 x ∈ X ,存在唯一的数
列 (a n ) 使得 x = Σ∞
n=1 a n x n ,其中级数是依范数收敛的
另一方面,第一纲集从拓扑的角度给出了集合较小的描述,无处稠密集直观
上看好像是充满了很多洞的集合。从测度方面讲,一个集合测度为也可以认为集
合很小。注意,这两种方式给出的小集合可能是矛盾的。
T
∞ S

1. 比 如, 令 { q i } 是 全 体 有 理 数 的 一 个 排 列,B = ( q i − 2 i1+ j , q i + 2 i1+ j )。
j =1 i =1
2 S
则对任意 ε >0,可找到 j 使 2j
< ε,这样就有 B ⊂ i(q i − 2 i1+ j , q i + 2 i1+ j ),
P 1 1 2
i |( q i − 2 i+ j , q i + 2 i+ j )| = 2 j < ε. 说明 B 的长度应该为 0。
S

而另一方面 ( q i − 2 i1+ j , q i + 2 i1+ j ) 是 R 中的稠密开集,故其补集是无处稠密
i =1
c
的,也就说 B 是第一纲集。
2. Liouville 数指无理数 z,使得对每个正整数 n,存在整数 p 和 q 使得
p 1
|z − q | > q n , q >1
T∞
若记 E 为 Liouville 数全体,则 E = Q 0 ∩ n=1 G n ,
S∞ S∞ p p
其中 G n = q=2 p=−∞ ( q − q1n , q + q1n )
c
不难看出 E 是第一纲集,即 Liouville 数全体是第二纲集,而同时 E 的测
度为 0。

练习

1. 能否找到 R 上的连续函数序列 ( f n )n∈N 使得,对于给定 x 得到的序列


{ f n ( x), n ∈ N },对于每个无理数 x 有界,而对有理数 x 非有界吗?
2. 设 f 是无穷次可微的连续实函数,且在每个 x ∈ R ,存在一个整数 n( x) ≥ 1
使得 f (n(x)) = 0,证明 f 是多项式

3.2.3 距离空间的完备化
对于任意的距离空间,它并不一定是完备的。就如从有理数到实数一样,任意距
离空间都可以通过往空间里加入一些点而使新的空间是完备的。
定义:距离空间 ( X , d ) 和 (Y , ρ ) 称为等距的,如果存在满射 f : X → Y 使得
∀ x, y ∈ X ,ρ ( f ( x), f ( y)) = d ( x, y),满足前述等式的映射称为等距映射。
3.2. 完备性 51

定义:(Y , ρ ) 称为 ( X , d ) 的完备化,如果存在等距映射 f 使得 f ( X ) 在 Y 中
稠密。

定理 3.6. 完备化在等距的意义上是唯一的,即若 (Y , ρ ) 和 (Yb , ρb) 都是 ( X , d ) 的


完备化,则 (Y , ρ ) 和 (Yb , ρb) 等距。

Proof. 设完备化 (Y , ρ ) 和 (Yb , ρb) 对应的等距映射 y ∈ 分别为 f 和 g,则 g ◦ f −1


是从 f ( X ) 到 g( X ) 的等距映射。因为 f ( X ) 和 g( X ) 分别在 Y 与 Yb 中稠密,可
将 g ◦ f −1 延拓定义到 Y 上:对于 y ∈ Y \ f ( X ),存在序列 { f ( xn )} 收敛到 y,那
么 { g( xn )} = {( g ◦ f −1 )( f ( xn ))} 也是 Yb 中的 Cauchy 列因为映射的等距性。由 Yb 完
备性,序列极限也在 Yb 中。另一方面,对于 by ∈ Yb \ g( X ),总存在序列 { g( xn )} 收
敛到 by,即可以找到 Y 中序列 { f ( xn )} 其极限 y 满足 by = g ◦ f −1 ( y),这就说明了
b 的等距映射。
g ◦ f −1 是从 Y 到 Y

定理 3.7. 任意距离空间都可以完备化。

Proof. 思 路 与 将 有 理 数 完 备 化 为 实 数 的 Cauchy 方 法 一 样。 对 于 距 离 空 间
( X , d ), 令 X seq 代 表 X 中 Cauchy 列 全 体, 建 立 等 价 关 系 { xn } ∼ { yn } 如 果
limn→∞ d ( xn , yn ) = 0。将等价的序列视为同一序列,用 X 代表 X seq 根据上
述等价关系得到的等价类全体。定义 X 中 Cauchy 序列等价类之间的距离:
d ([{ xn }], [{ yn }]) = limn→∞ d ( xn , yn ),可以验证其确为距离函数,与等价类中具体选
哪个 Cauchy 列进行运算无关。
然后需要验证 ( X , d ) 完备(留作练习)。
注意到 f ( x) = [{ x, x, ....}] 是单射并且保持距离不变,所以 f : X → X 是等
距映射
最后, lim d ( f ( xn ), [{ x i }∞
i =1
]) = 0,表明 f ( X ) 在 X 中稠密。
n→∞

3.2.4 完备性的应用:不动点定理,说明方程解的存在性
不动点定理

定义 (压缩映像): f : E → E 称为压缩映像,如果存在常数 α ∈ [0, 1),使得 ∀ x,


y ∈ E , d ( f ( x) , f ( y)) ≤ α · d ( x, y)
注:显然 f 满足 Lipschitz 条件,故 f 连续。此外,定义中要求 α < 1,给
出的不等式与 d ( f ( x) , f ( y)) < d ( x, y) 不等价 [为什么?]。
下面定理说明完备距离空间中 x = f ( x) 在 f 是压缩映像时解总是存在的.
52 CHAPTER 3. 完备性与紧性

定理 3.8 (Banach 不动点定理). (E, d ) 是完备距离空间, f : E → E 为压缩映像,


则 f 有唯一不动点,即满足 f ( x) = x 的点。而且,任给 x0 ∈ E ,序列 { f n ( x0 )} 收

敛于不动点 x。其中, f n (·) 定义为 f n ( x) = f ( f n−1 ( x))。

Proof. 任给 x0 ,考虑 { f n ( x0 )},只需证其为 Cauchy 列,那么 f n ( x0 ) → x.


由 f 连续, f ( x) = limn→∞ f ( f n ( x0 )) = limn→∞ f n+1 ( x0 ) = x ⇒ x 为不动点.
若 x,y 都为不动点,则 d ( x, y) = d ( f ( x) , f ( y)) ≤ αd ( x, y) ⇒ d ( x, y) = 0 ⇒ x = y
¡ ¢ ¡ ¢
注 意 到 d f n+1 ( x0 ) , f n ( x0 ) ≤ αd f n ( x0 ) , f n−1 ( x0 ) ≤ · · · ≤ αn d ( f ( x0 ) , x0 ) =
mα n
³ ´ Xk ³ ´ X
k
d f n+k+1 ( x0 ) , f n ( x0 ) ≤ d f n+ i+1 ( x0 ) , f n+ i ( x0 ) ≤ m αn+ i
i =0 i =0

n → ∞ 时,上式 →⇒ { f n ( x0 )} 是 Cauchy 列.
p
注意. f ( x) = 0.5( x + x2 + 1) 满足 d ( f ( x), f ( y)) < d ( x, y),但不是压缩映像,也没
有不动点
压缩映射的判定:

定理 3.9 (Blackwell 定理). X ⊆ R n ,B( X ) 是赋予一致度量的有界实函数全体,


T : B( X ) → B( X ) 满足:
a.(单调性) f , g ∈ B( X ),∀ x ∈ X , f ( x) ≤ g( x)[记为 f ≤ g],则 ∀ x, (T f )( x) ≤
(T g)( x)
b. ∃β ∈ (0, 1) 使得 (T ( f + a))( x) ≤ (T f )( x) + βa,∀ f ∈ B( X ), a ≥ 0, x ∈ X 。其中
( f + a)( x) = f ( x) + a。
则 T 为压缩映射。

Proof. 注意到 f ≤ g + || f − g||,有 T f ≤ T ( g + || f − g||) ≤ T g + β|| f − g||,以及


T g ≤ T ( f + || f − g||) ≤ T f + β|| f − g|| ⇒ ||T f − T g|| ≤ β|| f − g||。

Banach 不动点定理的应用

例:
p
3
p
3
1. 计算 5 是 F ( x) = x3 − 5 = 0 的解,考虑 F 在 [1, 2] 的情况,因 5 ∈ [1, 2].
让 f ( x) = x − λF ( x), f ( x) 的不动点即 F ( x) = 0 的解,只需适当选择 λ 使
¯ ¯
f : [1, 2] → [1, 2],¯ f 0 ( x)¯ ≤ α < 1。λ = 1
8 满足条件,可通过 xn+1 = f ( xn ) =
x3 −5 p
x − ( n ) 计算 5 的近似值.
3
n 8
3.2. 完备性 53
p
3
n xn 5
1
1 1.5
2 1.703125 1.709976
3 1.710607
4 1.709915

2. 微分方程 y0 = 2 x (1 + y), y (0) = 0.记 y = ϕ ( x),两边积分有


Z x ¡ ¢ ∆ ¡ ¡ ¢¢
ϕ ( x) = 2 t 1 + ϕ ( t) dt = F ϕ ( x)
0
Rx Rx ¡ ¢ x4
ϕ (0) = 0,ϕ1 ( x) = 0 2 t (1 + 0) dt = x2 ,ϕ2 ( x) = 0 2 t 1 + t2 dt = x2 + 2 ,…,
P
∞ 2k
x x2
ϕ ( x) = k! = e − 1
k=1
更 一 般 的, 如 果 g 连 续,|| g( t, w) − g( t, v)|| ≤ γ||w − v||, 对 于 任 意 的 t ∈
[0, T ], w, v ∈ R n 都成立,则初值问题

u0( t) = g( t, u( t)), 0 ≤ t ≤ T ; u(0) = u 0

有唯一解
考虑积分方程 Z t
u ( t) = u 0 + g( s, u( s)) ds
0
以及连续函数空间的范数

||| · ||| : v ∈ C ([0, T ], R n ) → su p t∈[0,T] ( e−rt ||v( t)||)

证明 Z t
F (v)( t) = u 0 + g( s, v( s)) ds
0
定义的映射 F 是压缩映射
3. 最优增长问题解的存在性
最简单的最优经济增长问题可以表示成下面的动态优化问题:
P

max ∞ β tU ( c t )
{(c t ,k t+1 )} t=0 t=0 (A)
s.t. c t + k t+1 ≤ f ( k t ), c t , k t+1 ≥ 0
54 CHAPTER 3. 完备性与紧性

其中 c 代表消费, k 代表资本投入, f 为产出函数 ( f 0 > 0, f 00 < 0),U 为效


用函数 (U 0 > 0,U 00 < 0)。为简单计,这里假定 U 有界。
可以把整个优化过程分解为两步,首先选择 c 0 , k1 ,到下一期再进行决策。
注意到(A)是一个无穷期的优化问题,第二步决策所面对的问题与(A)形
式上完全相同,唯一的区别在于初始的 k 不同。所以如果假定(A)的最优
值为 v(k0 ),优化问题可以表示为
max U ( c 0 ) + βv( k 1 )
{(c 0 ,k 1 )} (B)
s.t. c 0 + k 1 ≤ f ( k 0 ), c 0 , k 1 ≥ 0
原始问题的解就变为求解 v 的问题 [一般而言,问题 A 的解总是 B 的解,
反之则需要一些限制条件]。虽然 v 的具体形式未知,但是根据定义,它应
当满足:
© ª
v( k 0 ) = max U ( f ( k 0 ) − k 1 ) + β v( k 1 )
0≤ k 1 ≤ f (k 0 )

将等式右边看成是对函数 v 的变换 T ,即对于任意函数 w : R+ → R ,可定义


Tw : R + → R
© ª
(Tw)( x) = max U ( f ( x) − y) + βw( y)
0≤ y≤ f (x)

T 是从函数空间到函数空间的映射,v 就是该映射的不动点。
根据 Theorem of the Maximum,T 是从 BC(R )[有界连续函数] 到 BC(R ) 的
映射,再根据 Blackwell 定理,可知 T 是压缩映射,不动点存在且唯一。

练习

1. ( X , d ) 是完备距离空间,映射 F : X → X 满足 ∀ x 6= y, d (F ( x), F ( y)) < d ( x, y)。


构建例子说明这样的映射可能没有不动点,并证明:若存在 x0 ∈ X 使得
{F n ( x0 )} 有收敛子序列,则 F 有唯一的不动点
2. ( X , d ) 是完备距离空间,映射 F : X → X 满足:存在某个自然数 N 使得
F N : X → X 是压缩映射,证明 F 有唯一的不动点
3. ( X , d ) 是完备距离空间,映射 F n : X → X 连续,且有不动点 xn 。假设 F n
点点收敛到 F ,每个 F n 满足 Lipschitz 条件, d (F n ( x), F n ( y)) ≤ L n d ( x, y),
Lipschitz 常数 L n 是使得前述不等式成立的最小的数,∀n, L n ≤ M < ∞。证明,
F 满足 Lipschitz 条件,d (F n ( x), F n ( y)) ≤ Ld ( x, y),且 Lipschitz 常数 L ≤ M 。
若 xn 收敛到 x0 ,则 x0 也是 F 的不动点
3.3. 紧性 55

3.3 紧性

3.3.1 概念
定义 (紧). 集合 K 是紧的,如果 K 的任何一个开覆盖存在有限的子覆盖,即任何
S Sn
满足 K ⊂ Vi 的开集族 {Vi , i ∈ I } 中存在有限个开集 Vi 1 ,…,Vi n 使 K ⊂ j =1
Vi j 。
i∈ I
整个空间是紧的,如果将其视为集合是紧的.

注:紧性从某种意义上是为了使空间呈现某种有限性,最简单的例子是离
散空间是紧的,当且仅当它是有限的,另外,R 中的紧集必须为有界闭区间.
Hermann Weyl (1885-1955):“If a city is compact, it can be guarded by a finite
number of arbitrarily near-sighted policemen”。
紧性在分析中是很重要的性质,许多定理,尤其是有关存在性的定理的条件
都会用到紧性。很多时候这些定理往往在有限的情形下都是显然的,而在无限不
紧的情形下不成立或很难证明。
S∞ ¡ 1 ¢ Sn ³ 1 ´
例. (0, 1) 不紧,首先 (0, 1) = i =1 i , 1 ,但 (0 , 1) =
6 j =1 i j
,1

一些相关的定义:
相对紧:称 A 相对紧,如果 A 的闭包是紧的。
局部紧:称拓扑空间 X 是局部紧的,如果 ∀ x ∈ X ,∃ x 的邻域 U 使得 U 是
紧的。(比如在通常拓扑下,R 是局部紧的,但不是紧的)
注意. 任何一个局部紧的 Hausdorff 空间可以通过加入一个单点,并选择适当的
拓扑变成紧空间。这个紧致化实际上就是把原来的空间 X 嵌入新的紧空间 Y ,
使得 X 在 Y 中稠密。
利用紧性证明的常见思路: X 是一紧空间,P 为 X 的一些开集具有的性质,
而且若开集 U 和 V 都有性质 P,那么 U 和 V 的并集也有性质 P。那么我们可
以知道,只要 X 局部(即每一个点的某个邻域)的具有性质 P,则 X 整体上也
有性质 P。因为这些点的邻域形成 X 的开覆盖,只要从中恰当的选出有限个开
集,即可由归纳法知 X 有性质 P。

例. (1) X 为紧空间, X 上的实函数 f 是局部有界的,则 f 有界。( X 有限时结


论显然)
(2) X 为紧空间,定义于 X 上的实函数列 { f n } 是局部一致收敛的,则 { f n }
在 X 上一致收敛 ( X 有限时结论显然)
56 CHAPTER 3. 完备性与紧性

3.3.2 紧空间的性质
定理 3.10 (紧性的等价定义). K 是紧的 ⇔ K 的任意闭子集族若具有有限交非空
的性质,则全体交非空。⇔ K 的任意基本开覆盖 (由拓扑基形成的开覆盖) 中存
在有限的子覆盖。⇔ K 的任意由拓扑次基形成的开覆盖中存在有限的子覆盖。

Proof. K 紧,设 {Fα } 闭具有有限交非空性质.若 ∩Fα = ;,则 K ⊂ ∪Fαc 为开覆


© ª
盖,可知有 Fαc 1 , · · · , Fαc n 覆盖 K ,故 ∩ni=1 Fαi = ;。矛盾.反之类似.
后两个定义留作练习 [最后一个需要用到 Zorn 引理]。

1. 由该等价定义,可以知道紧距离空间肯定是完备的,因为对于非空闭集
套 F1 ⊃ F2 ⊃ · · · ,任意有限个的交非空,故 ∩∞ F 6= ; 推出闭集套定理成立,等
i =1 i
于说空间是完备的.
2. 由紧的最初定义,可知紧距离空间是全有界的,因为对于任意 ε > 0,
G = {Bε ( x) : x ∈ E } 是 E 的开覆盖.若 E 紧,则可从中选出有限个 Bε ( x i ).
下面的定理说明在距离空间里,紧与序列紧是一致的,而完备性与紧性也关
系密切。

定理 3.11 (距离空间中的紧). 对于距离空间,下面四种陈述是等价的:


(1) ( M , d ) 是紧的 (Heine-Borel 性质)
(2) ( M , d ) 是完备且全有界的
(3) M 的任一无限子集有极限点,(Frechet 紧,Bolzano-Weierstrass 性质)
(4) M 的任一点列有收敛子序列.(列紧)

Proof. (1)⇒(2) 已证.


(2)⇒(3) 由全有界性,不妨设 F n 为 M 的有限子集,使得对每个 x ∈ M ,有
某个 y ∈ F n ,使 d ( x, y) < n1 。设 A 为 M 的任意无限子集,因为存在有限个开球
集 {B1 ( y) : y ∈ F1 } 覆盖 M .则必有某个开球 B1 ( x1 ) 中有 A 中无限个元素,其中
x1 ∈ F 1 . ½ ¾
同 样 方 法, 可 从 F n 中 选 出 xn 使 A ∩ B 1 ( xm ) : m = 1, · · · , n 含有无穷
½ ¾ m

元素。[比如 A ∩ B1 ( x1 ) 无穷, B 1 ( y) , y ∈ F2 有限覆盖 M ,故肯定有 x2 使


2
A ∩ B1 ( x1 ) ∩ B 1 ( x2 ) 无穷]
2
由于 A ∩ B1 ( x1 ) ∩ · · · ∩ B 1 ( xn ) 非空,所以 d ( xm , xn ) < d ( xm , y) + d ( y, xn ) <
n
1
m + n1 → 0 表明 { xn } 是 Cauchy 列,收敛至 x ∈ M .
3.3. 紧性 57
¡ ¢
因为 d ( xn , x) < n2 ⇒ B x, n3 ⊃ B 1 ( xn ), x 的邻域中肯定包含无穷个 { xn } 中的
n
点即 x 的邻域中有 A 中无穷个点 ⇒ x 是 A 的极限点.
(3)⇒(4){ xn } 是无穷点列,不妨设其中互不相同.作为 M 的子集,有极限点
¡ ¢ © ª
x.可以找到 xn1 ,…, xn k ,…使 d xn k , x < 1k .则 xn k 为收敛至 x 的子序列.
(4)⇒(1) 设 U 为 M 的一个开覆盖,对任一 x ∈ M .
令 f ( x) = sup {r : B ( x, r ) ⊂ O ∈ U }。
由于 O 开,故 r 肯定大于 0,所以 f ( x) > 0 且 inf { f ( x) : x ∈ M } > 0.否则若
inf { f ( x)} = 0,即存在 M 中序列 { xn } 使 f ( xn ) < n1 ,设其中有个子列 xn k 收敛到
¡ ¢
x ∈ M .那么存在 O ∈ U ,r > 0 使 B ( x, r ) ⊂ O .当 k 足够大时,d xn k , x < 2r ,就
¡ ¢
有 f xn k > 2r .矛盾.
令 c = min (2, inf { f ( x) , x ∈ M }) > 0. 先 选 x1 , 给 定 x1 , ..., xn , 选 xn+1 使
¡ ¢
d xn+1 , x j > 2c ( j = 1, ..., n).
如果可以一直这样选,则可以得到序列 { xn },d ( xm , xn ) >
c
2 ,但这样的序列不是 Cauchy 列,不存在收敛子列,与假设矛盾,因此肯定只
¡ ¢ ¡ ¢
能选有限个,即意味着 M = ∪ j≤n B c x j .由 f 的定义和 c 的定义,B c x j 在某
¡ ¢ 2¡ ¢ ¡ ¢ 2
个 U j 之中,因为 B c x j ⊂ Binf{ f (x)} x j ⊂ B f (x j ) x j ⊂ U j ∈ U .这样 M ⊂ ∪ j≤nU j .
2
得证.

注:1. 定理表明在距离空间中,紧性比完备性更强一些.
2. 证明中我们也得到如下结论:对于紧距离空间 X 的开覆盖 U ,存在一
个大于的数 δ 使得 X 的任意半径为 δ 的开球包含于 U 的某个元素内部。这
个数 δ 称为 Lebesgue 数。显然若一个数为 Lebesgue 数,则比它小的正数也是
Lebesgue 数。如果空间不紧,如 R 的开覆盖 {(n, n + 1)} ∪ {(n − n1 , n + n1 )},(0, 1) 的
开覆盖 {( n+1 1 , n−1 1 )} 中都找不到 Lebesgue 数
由上一定理我们可立即得到 R n 空间的紧集的形态

定理 3.12 (欧氏空间中的紧集). :欧式空间 R n 的子集 A 是紧的 ⇔ 子集 A 是


有界闭集.

Proof. 紧 ⇒ 有界闭显然.⇐ 可由列紧与紧的等价性得到.

定理 3.13 (Lindelöf 定理). R n 中任何子集的开覆盖有一个可数子覆盖

Proof. 设 {O i } i∈ I 为 A 的开覆盖,∀ x ∈ A ,存在有理点 a x 和有理数 r x > 0,使


© ª
x ∈ B r x (a x ) 且 B r x (a x ) ⊂ O i x ∈ {O i } i∈ I .由 B r x (a x ) , a x 有理数, r x 有理数 可数知只
需可数个 O i 即可覆盖 A .

注:定理的证明表明该结果可以推广至具有可数基的拓扑空间。
58 CHAPTER 3. 完备性与紧性

练习

1. 度量空间 M 紧,证明 M 是可分的,且 M ≤ [0, 1].


S S
提示:对每个 n,可找到有限个 x 使 M = B 1 ( x),则 F n 在 M 中稠密.
x∈ F n n
2
2. 给定函数 f : [a, b] → R ,定义 G : [a, b] → R ,G ( x) = ( x, f ( x)),G 的象称为
f 的图象,证明下述陈述等价:1. f 连续, 2. G 连续, 3. f 的图象是 R 2 中
紧集.
更一般的,有
定理 3.14 (闭图象定理). f : X → Y ,X 为拓扑空间,Y 为紧距离空间。函
数 f 连续当且仅当它的图象 Gr ( f ) = {( x, y) ∈ X × Y : y = f ( x)} 在赋予乘积拓
扑的 X × Y 中是闭的。

3. X 为紧距离空间,K : X → X 满足对于 x 6= y, d (K ( x), K ( y)) < d ( x, y),证明


K 有唯一不动点。

3.3.3 (拓扑)紧空间的性质
现在我们看几个不依赖于度量而只依赖拓扑的与紧相关的性质.
定理 3.15. 若 K 为紧空间,F 是 K 的闭子集,则 F 是紧集.
Proof. F 的任一开覆盖 U ,U ∪ {K \F } 为 K 的开覆盖,其中有有限子覆盖 V ,
V \ {K \F } 为 F 的有限子覆盖且 ⊂ U .

定理 3.16. K 为紧空间, f : K → L 连续满射,则 L 紧.


© ª
Proof. U 为 L 开覆盖,则 f −1 (U ) : U ∈ U 为 K 开覆盖,有有限子覆盖 V =
© ª
f −1 (U ) : U ∈ U ,则 f (V ) 为 L 的有限子覆盖.

推论:K 紧, f : K → R 连续,则 f 有界,且取到最大与最小值.


例:K 不紧,其连续像可能无界,即使有界,也不一定有最大,最小值,如
x → x 在 R 上无界, x → x/ (1 + | x|) 在 R 上有界,但无最大 (小) 值.
x → 1/ x 在 (0, 1] 无界, x → x 在 (0, 1) 有界,无最值.
下面的定理表明,紧空间的乘积空间也是紧的,这样就可以把 R n 中的
紧集和 R 的紧集联系起来.[ X × Y 上的乘积拓扑定义为使得投影 p 1 ( x, y) = x
和 p 2 ( x, y) = y 连续的最弱的拓扑,即 W = { p−1 1 (U ) : U ⊂ X open} ∪ { p−2 1 (U ) : U ⊂
Y open} 是乘积拓扑的一个次基,或者说 W 的有限交全体是一个基。]
3.3. 紧性 59

定理 3.17. 如果 X 和 Y 都是紧的,那么 X × Y 也紧.

Proof. 设 Oα 为 X × Y 的开覆盖,则 ( x, y) ∈ X × Y 在某个 Oα 中的开集,于是存


© ª
在 x、 y 的邻域 Vm 和 Wm 使得 Vm × Wm 在某个 Oα 的开集中 (Oα = ω i m )。记
Um = Vm × Wm .
给定 x0 ∈ X , X × Y 的子集 Y0 = { x0 } × Y 与 Y 同胚,故 Y0 为紧集.
将 Y0 的所有点对应的 {Um } 视作 Y0 的开覆盖,我们可从中选出有限覆
© ª T S
盖 Um j j∈ J .令 Vx0 = Vm j 为 x0 的开邻域,且有 ω i m j ⊃ Vx0 × Y (Vx0 ⊂ Vm j ,
j∈ J j∈ J
ω i m j ⊃ Vm j × Wm j )
© ª
所有的 Vx0 形成 X 的开覆盖,故其中有 Vx1 ,Vx2 ,…,Vxn 覆盖 X .
由于每个 Vxi × Y 都可被有限个 ω i m 覆盖,故 X × Y 也可被有限个 ω i m 覆
盖,得证.

注:Tychonoff 定理表明,不仅有限个紧拓扑空间的积空间是紧的,任意多
的紧拓扑空间的乘积空间仍然是紧的。[关键在于乘积拓扑]
Q
定理 3.18 (Tychonoff 定理). Sα 紧,则在乘积拓扑下 S = α∈ A S α 紧

Proof. 令 F 为 S 中满足有限交非空的闭集族,需证 F 中所有集合的交非空。


利用 Zorn 引理,可将 F 扩张成最大的满足有限交非空的集合族 F0 .
对任一 α,投影 f α (F0 ) 具有如下性质:存在一个点 xα ∈ Sα ,它在所有属于
f α (F0 ) 的集合的闭包之中 [ f α (F0 ) 中集合的闭包也有有限交性质,S α 紧,所以
有全体交非空]
令 x ∈ S 第 α 维度的坐标为 xα ,要证明 x 在 F0 的每个元素的闭包中
设 U 为包含 x 的开集,由乘积拓扑定义,存在有限个 α i 和开子集 Uαi ⊂ Sαi
使得
x ∈ ∩ni=1 f α−i1 (Uα i ) ⊂ U ⇒ ∀ i = 1, 2, · · · , n, xα i ∈ Uα i

表明 Uαi 与每个属于 f αi (F0 ) 的集合都相交,所以 f α−i1 (Uαi ) 与 F0 的每个集合相


交,必然属于 F0 (由 F0 的最大性)⇒ ∩ni=1 f α−i1 (Uαi ) ∈ F0 , 这说明 U 和 F0 的每个
集合都相交,即 x 的任何领域与 F0 的每个集合相交。得证。
Q
作为对照,我们考察可数个距离空间的乘积空间 α∈ N Mα 其中的距离定义

1
d ( x, y) = Σα min[ d α (Projα ( x), Projα ( y))]

60 CHAPTER 3. 完备性与紧性

可以证明 (留作练习),

d ( xn , x) → 0 ⇐⇒ ∀α, Projα ( xn ) → Projα ( x)

而且该距离对应的拓扑与乘积拓扑是一致的。
Q
定理 3.19. ∀α, ( Mα , dα ) 完备,可分,紧 ⇐⇒ ( α∈ N Mα , d ) 完备,可分,紧

Proof. 只证明紧的部分,其余留作练习。
设 xn = ( xα,n ) 为 M 中 点 列, 由 M1 紧 性, 从 n → x1,n 中 找 收 敛 子 列
x1,n11 , x1,n12 , x1,n13 , · · · , n → x2,n 再找收敛子列,一直这样继续。令 n k = n k,k ,
则 xn 的子序列 xnk 收敛
反之,若 M 的某个 α 对应的 Mα 不紧,即 M b 中有序列 xb,n 没有收敛子列,
令 x0 为 M 中一点,定义 xn 使得 Projb ( xn ) = xb,n , Projα ( xn ) = Projα ( x0 ), ∀α 6= b,
则 xn 在 M 中没有收敛子列,矛盾。

定理 3.20. 在距离空间中,任何紧集还是闭集.更一般的,Hausdorff 空间的紧


集是闭的。

Proof. 设 A 为紧集,只需证 A c 为开集.设 z ∈ A c ,对任意 x ∈ A ,可找到互不


相交的开集 U zx 和 Vx .使 z ∈ U zx ,x ∈ Vx .由于 {Vx , x ∈ A } 为 A 的开覆盖且 A 紧,
S
n T
n
x
故存在 { x1 , · · · , xn } 使 A ⊂ Vx i .令 U = U z i 则 U 为包含 z 的开集,且 U 与
i =1 i =1
S
n
c c
Vx i 不相交即 U ⊂ A ⇒ A 为开集, A 为闭集.
i =1

结合前面定理,可以得到
推论:从紧空间到距离空间的任何连续双射 f : E → F 是同胚映射
证明:只需证 f −1 连续,即对于 E 的闭集 X , f ( X ) 在 F 中闭,E 紧 ⇒ E
的闭集 X 紧 ⇒ f ( X ) 紧 ⇒ f ( X ) 闭。#

3.3.4 空间非紧的测量
在前面的 Banach 不动点定理中,定理的条件要求完备性和映射的收缩性,下面
的定理表明在映射只具备连续性的情况下,如果空间是紧的,不动点还是可以存
在的。
Brouwer 不动点定理:
假定 M 是 R n 中非空、凸紧集, f : M → M 是连续的,则 f 有不动点.
3.3. 紧性 61

只证一维情形,即 f : [0, 1] → [0, 1] 连续时 f 有不动点,考虑 f ( x) − x := g ( x),


g (0) ≥ 0 g (1) ≤ 0,故中值定理表明必有某个 x0 使 g ( x0 ) = 0,即 f ( x0 ) = x0 .
高维的情形可以利用 Sperner(德国数学家) 引理给出较为初等的证明。

Sperner 引理:给定一个大三角形 V1 V2 V3 , 并将它三角化(把它画分成有限


多个较细的三角形且每个细三角形的边都是另一个细三角形的边或落在大三角
形的边上)。若将各顶点以下述的规定标记:(1) 顶点 Vi 的标号为 i , i = 1, 2, 3;
(2) 在 Vi V j 边上的顶点只可以用 i 或 j 作为标号;(3) 不在大三角形边上的顶点
可随意以 1, 2 或 3 作为标号。
那么至少存在一个细三角形,其三个顶点的标号分别为 1, 2,3。
引理以及在 2 维情形下对 Brouwer 定理的证明可以参见参考资料中 Aigner
与 Ziegler 的 Proofs From The Book 一书。
为了考察紧的条件可以放松到什么程度,需要引入一些衡量空间紧的程度
的指标。
定义: M 为完备距离空间 (E , d ) 中的有界集合,Kuratowski 非紧性测度定
义为:
[n
χ( M ) = inf{ε > 0 : ∃{ x1 , ....xn }, s.t.M ⊂ i =1 Bε ( x i )}

例:1. 显然,如果 M 是紧集,χ( M ) = 0,这意味着 χ( M ) 越大,M 应该越不紧。


2. l 2 中的单位闭球 M ,χ( M ) = χ( M o ) = 2
Brouwer 定理的一个有趣的用处是证明非负方阵有非负的特征值和特征向
量,即
若 A 为 n 阶非负矩阵,则存在 λ ≥ 0, p ≥ 0 ∈ R n 使得 A p = λ p. 事实上,
Perron Frobenius Theorem 表明如果矩阵所有元素都大于 0,正的 λ 和 p 还是唯
一的(注意很多的转移概率矩阵都是正矩阵,经济学中的投入产出矩阵也是)
62 CHAPTER 3. 完备性与紧性

Proof. 定义集合
X
K = { x = ( x i ) ∈ R n : x i ≥ 0, 1 ≤ i ≤ n, x i = 1}
Pn
不妨设 Ax 6= 0 对所有 x ∈ K ,有 i =1 ( Ax) i > 0 对所有 x ∈ K ,由

1
f ( x) = P n Ax
i =1 ( Ax) i

定义的 f 是紧凸集 K 到自身的连续映射,故存在 p ∈ K 使得 f ( p) = p。得证

练习

1. 证明如下关于完备距离空间中 Kuratowski 非紧性测度的性质:


1. 0 ≤ χ( A ) ≤ diam( A )
2. 0 = χ( A ) ⇔ A 是相对紧的,即 A 紧
3. 对有界集合 A ,B,χ( A ∪ B) = max(χ( A ), χ(B))
4. χ( A ) = χ( A )
2. 证明定理:距离空间 (E, d ) 完备 ⇔ 如果 E ⊃ F1 ⊃ F2 …为递减的非空闭集列,
T
且 χ (F n ) → 0,则 ∞n=1 F n 6= ;
注:这里将完备空间的闭集套定理中的 diam (F n ) → 0 换成了 χ (F n ) → 0。
第 4 章 连续性和连续函数空间

4.1 连续性

4.1.1 基本概念
¡ ¢
定义:( M, d ), N, ρ 是两个距离空间,函数 f : M → N 称 f 在 x ∈ M 处连续,如
果对任意 ε > 0,存在 δ > 0(依赖于 f , x,ε),使得:只要 y ∈ M , d ( x, y) < δ 就
¡ ¢ ρ
有 ρ ( f ( x) , f ( y)) < ε[或写成 f Bδd ( x) ⊂ Bε ( f ( x))];
若 f 在 M 的每点连续,则称 f (在 M 上) 连续。
定理 4.1 (连续的等价定义). f 连续 ⇔ 对任意 x ∈ M ,xn → x 则 f ( xn ) → f ( x)⇔E
是 N 的闭集,则 f −1 (E ) 在 M 中闭 ⇔V 在 N 中开,则 f −1 (V ) 在 M 中开 (留
作练习)
连续函数的例子:f : N → R 任意函数,则 f 连续,因为 {n} 是 N 中的开球.
{ n} = B 1 ( n) ⊂ f −1 (Bε ( f ( n))),∀ε > 0
2
定理 4.2 (Debreu 效用表示定理). X 可分距离空间,≥ 是连续的完全的偏好关系,
则它可以有一个连续函数的效用表示.≥ 连续,即指 { x ∈ X , x ≤ y} 和 { x ∈ X , x ≥ y}
闭,∀ y ∈ X .
Proof. 证明思路:找出 X 的可数稠密子集 D ,可以有效用函数 u(·) 表示,取值
于 [a, b] 的有理数上 (思考如何实现,注意无法按照大小排列)。将 u 的定义域从
D 延拓至 X : u( y) = sup{ u( x), x ∈ D, x ≤ y} = inf{ u( x), x ∈ D, y ≤ x}。最后证明这样
的 u 是连续的。

练习

1. f 连续 ⇔∀B ∈ N , f −1 (B o ) ⊂ [ f −1 (B)] o ⇔∀ A ∈ M , f ( A ) ⊂ f ( A )

63
64 CHAPTER 4. 连续性和连续函数空间
¡ ¢
2. f : ( M, d ) → N, ρ 称 f 为开映射如果 U 在 M 中开,则 f (U ) 在 N 中开, f
为闭映射若 U 在 M 中闭则 f (U ) 在 N 中闭.举例:1. 连续映射,但不是
开映射;2. 开映射但不是连续映射;3. 连续开映射但不是闭映射,可以是
满射;4. 连续闭映射但不是开映射,且是满射。
3. 证明定理

定理. 投影映射 p β 是连续开映射,但不一定是闭映射。

4. 记 B(E,R) 为 E(⊆ R ) 到 R 上的有界函数全体,其上定义了距离函数

d ( f , g) = sup | f ( x) − g( x)|
x∈ E

试证 B(E,R) 到 R 的映射 h(u) = sup u( x) 连续。


x∈ E

下 (上) 半连续的定义: M 上的实值函数 f 称为下 (上) 半连续,如果对任


意实数 α,集合 { x ∈ M : f ( x) ≤ (≥) α} 在 M 中闭;⇔∀α ∈ R ,{ x ∈ M : f ( x) > (<) α}
在 M 中开;⇔ 当 xn → x 时,总有 f ( x) ≤ lim infn→∞ f ( xn )( f ( x) ≥ lim sup f ( xn ))

也可以证明,下半连续性也可以由图像 {( x, y) : f ( x) ≤ y} 的闭性描述。
(注意,
乘积拓扑下,投影映射是开映射)
应用:我们知道 R n 中有界闭集 K 上的连续函数能达到最小值,为此,取 x j
¡ ¢
使得 f x j → λ := inf f ( x)。根据 Weierstrass 定理,存在子列 x j → x ∈ K .由 f 连续
¡ ¢
性,λ := lim f x j = f ( x)。这个结论中的连续性可以减弱为下半连续,即 K 上的下
¡ ¢
半连续函数也能达到最小值.F 下半连续意味着 x j → x 则有 lim inf F x j ≥ F ( x),
¡ ¢
若 有 序 列 x j 使 F x j → inf {F ( x)} = λ, 则 有 子 列 x j , x j → x ∈ K . 于 是
¡ ¢ ¡ ¢
λ ≤ F ( x) ≤ lim inf F x j = lim F x j = λ⇒ F ( x) = λ.
同胚:如果存在 X 到 Y 的双射 f ,使对于 X 的任何开集 A , f ( A ) 为 Y 的
开集,对于 Y 的任意开集 B, f −1 (B) 为 X 的开集,我们就称 X 和 Y 同胚.
性质: X 到 Y 上的双射 f 是同胚,必须且只需 f 和 f −1 都连续.
4.1. 连续性 65

注意:连续双射 f 并不能保证 f −1 连续.比如 X = [0, 1) ∪ {2},Y = [0, 1],


f ( x) = x, x ∈ [0, 1); f (2) = 1.则 f 连续但 f −1 在 1 点不连续。
连续映射能把紧集映成紧集,但完备性却不能保持,即使 f 是一致连续的
同胚也不行.如
f ( x) = 1+|x x| R → (−1, 1) 则 R 完备而 (−1, 1) 不完备

4.1.2 一致连续
一致连续:称 f : M → N 一致连续,如果对任意 ε > 0,存在 δ > 0(不依赖 x),只
要 d ( x, y) < δ 就有 ρ ( f ( x) , f ( y)) < ε,
显然,一致连续 ⇒ 连续如 R 上的 exp (·) 连续但不是一致连续,不过 [a, b]
上的连续函数都是一致连续的。更一般的,下个定理说明定义在紧集上的连续函
数总是一致连续的
¡ ¢
定理 4.3. 度量空间 ( M, d ), N, ρ 。 f 是 M 到 N 的连续映射,若 M 紧,则对
任意 ε > 0,存在 δ > 0.使得当 d ( x, y) < δ 时就有 ρ ( f ( x) , f ( y)) < ε.

Proof. 给 定 ε > 0, 因 f 连 续, 每 个 x 对 应 有 r x > 0 使 当 d ( x, y) < 2r x 时


© ª
ρ ( f ( y) , f ( x)) < ε.于是所有 B r x ( x) 是 M 的开覆盖,故存在有限个 x1 ,…, xn
使 M = ∪1n B r xi ( x i ).
© ª
令 δ = min r x1 , · · · , r xn > 0。若 x, y ∈ M 且 d ( x, y) < δ.则肯定有某个 i 使
d ( x, x i ) < r x i ⇒ ρ ( f ( x) , f ( x i )) < ε.于是 d ( y, x i ) ≤ d ( y, x) + d ( x, x i ) < δ + r x i <
2r xi
⇒ ρ ( f ( x) , f ( y)) ≤ ρ ( f ( x) , f ( x i )) + ρ ( f ( x i ) , f ( y)) < ε + ε = 2ε

紧空间上连续函数一致连续的图示证明:以 f : [a, b] → R 为例.

Proof. 要证 ∀ε > 0,∃δ 使 | x − y| < δ ⇒ | f ( x) − f ( y)| < ε ⇔ 只要 | f ( x) − f ( y)| ≥ ε 则


| x − y| ≥ δ,即 d ( x, y) = | x − y| 在

E = {( x, y) ∈ [a, b] × [a, b] : | f ( x) − f ( y) ≥ ε|}

上的下界 > 0.
注意到 E = g−1 ([ε, ∞)), g ( x, y) = | f ( x) − f ( y)|
66 CHAPTER 4. 连续性和连续函数空间

f 连续 ⇒ E 是 [a, b] × [a, b] 的闭子集 ⇒ E 紧


d ( x, y) = | x − y| 在 E 上连续 ⇒ d 在 E 上取得最小值 δ > 0

Lipschitz 条件: f : R → R ,存在 k < ∞,使 | f ( x) − f ( y)| ≤ k | x − y|,∀ x, y


显然,满足 Lipschitz 条件的函数是一致连续的。应用到距离函数,可知
| d ( x, z) − d ( y, z)| ≤ d ( x, y) ⇒ f ( x) = d ( x, z) 连续 ⇒ d ( x, y) 对于 x 或 y 都是连续的.

4.1.3 连续函数的复合函数
定理 4.4. 1. f : L → M , g : M → N 连续,则 g ◦ f : L → N 连续
2. f , g: M → R 连续.则 h ( x) = ( f ( x) , g ( x)) 作为 M → R 2 的函数连续

[注意与 h : R 2 → R

 x12 x2
x14 + x22
若 x14 + x22 6= 0
h( x1 , x2 ) =
 0 若 x14 + x22 = 0

这个类型函数的区分]
推论: f 、 g: M → R 连续,则 f ± g, f g,max { f , g},min { f , g} 连续
证明:因为 ( x, y) → x + y,max { x, y}, x y 连续
注意:{ f n } : M → R 连续,不能得证 h ( x) = ( f 1 , f 2 , · · ·) 连续
P
故 n f n .sup { f n } 不一定连续 (同学们可试着举反例)

4.1.4 一致收敛,积分号与极限的交换
几个例子:
R1 R1 R1
1. f n ∈ c [0, 1], f n 一致收敛于 f ,则 0 lim f n dx = lim 0 f n dx = 0 f dx
¯Z Z ¯ Z
¯ 1 1 ¯ 1
¯ f dx − f n dx¯¯ ≤ | f − f n | dx ≤ k f − f n k∞ → 0
¯
0 0 0
4.1. 连续性 67

2. f n ∈ c [0, 1], f 是 f n 的简单收敛或点点收敛的极限

函数列定义为:[0, n1 ] 上 f n ( x) = n2 x (1 − nx);[ n1 , 1] 上 f n ( x) = 0
f n 简单收敛于 f = 0,但不是一致收敛:sup | f n ( x) − f ( x)| = n/4 → ∞

Z Z 1 Z
1 n ¡ 2 3 2
¢1 1
f n ( x) dx = n n x − n x dx = > f ( x) dx = 0
0 0 6 0

若 f n 一致收敛于 f ,则 f n 简单收敛于 f ,反之则不对,下面的 Dini 定理


说明了什么时候简单收敛可以推出一致收敛

定理 4.5 (Dini 定理). { f n } 为紧距离空间 X 上的上半连续递减函数列,即


∀ x ∈ X , n ∈ N , f n+1 ( x) ≤ f n ( x),并且 { f n } 在 X 上点点收敛到下半连续函数 f,则
f n 一致收敛于 f 。

Proof. 不妨假定 { f n } 递减点点收敛到 0(相当于考虑 { f n − f })。


S

k
对任意 ε,X ⊆ { x : f n ( x) < ε} ,由紧性,可找到有限个开集 { x : f n i ( x) < ε} i=ε 1
i =1
覆盖 X。因为 { f n } 递减, X = { x : f n kε ( x) < ε}。故 ∀ x ∈ X , n > n kε , f n ( x) < ε。

练习

1. 若 { f n }、{ g n } 为距离空间 X 上的实值函数列.且都一致收敛.证明:f n + g n


一致收敛但 f n g n 并不一定一致收敛.

4.1.5 R 上连续函数列和函数的连续点

连续函数列的一致收敛极限是连续的,而简单极限不一定连续 [ f n ( x) = max(0, 1 −
n| x|)],下面的定理说明了其简单极限的性态 [虽然连续性在简单极限下不保持,
但可测性可保持]

定理 4.6. 若 f 是一列 R 上连续函数的简单极限,则 f 的不连续点是第一纲集.


68 CHAPTER 4. 连续性和连续函数空间

Proof. 只需证 ∀ε > 0,F = { x : ω ( x) ≥ 5ε} 无处稠密.其中


" #
ω ( x) = lim sup f ( t) − inf f ( t)
δ→0 t∈(x−δ,x+δ) t∈(x−δ,x+δ)

代表函数 f 的振幅。令 f = lim f n , f n 连续,定义


© ¯ ¯ ª
E n = ∩ i, j≥n x : ¯ f i ( x) − f j ( x)¯ ≤ ε

则 E n 闭,且 E n ⊂ E n+1 ,∪E n = R .考虑任一闭区间 I .因 I = ∪ (E n ∩ I ).E n ∩ I 不


¯ ¯
所以存在某个 n,E n ∩ I 包含开区间 J ,即 ¯ f i ( x) − f j ( x)¯ ≤ ε,
可能都是无处稠密的.
∀ x ∈ J , i, j ≥ n.让 j = n, i → ∞,则 | f ( x) − f n ( x)| ≤ ε,∀ x ∈ J .对任意 x0 ∈ J ,
存在邻域 O ( x0 ) ⊂ J ,使 | f n ( x) − f n ( x0 )| ≤ ε,∀ x ∈ O ( x0 ),故 | f ( x) − f n ( x0 )| ≤ 2ε,
∀ x ∈ O ( x0 )⇒ ω ( x0 ) ≤ 4ε.即 J 的所有点不属于 F .这样对任一闭区间 I 存在开
区间 J ⊂ I − F ,这表明 F 是无处稠密的.

定理. 若 f 是 R 上实函数,则不连续点集是 Fσ 集,连续点集是 G δ 集

Proof. 显然函数的连续点对应于 ω( x) = 0。注意到 { x : ω( x) < ε} 是开集,而 f 的


S∞ 1
不连续点集可表示为 D = n=1 { x : ω( x) ≥ n },得证。

定理. R 上任意 Fσ 集 E ,存在有界函数 f 使其不连续点恰好是 E

Proof. 令 E = ∪F n ,F n 闭.不妨假定 ∀n ,F n ⊂ F n+1 .A n 为 F n 中的有理内点.


1 P P∞
对任意集合 A ,定义 f n = 1Fn − A n .并令 a n = n! ,则 an > i>n a i , n=1 a n f n ( x)
一致收敛至有界函数 f . f 在所有 f n 都连续的集合上连续.故在 R − E 上肯定
P
连续,而在 F n − F n−1 的每一点上, f 的振幅至少为 a n − i > n a i ,故 f 在E 上
都不连续.

例:1Q 处处不连续,故它不是一列连续函数的简单极限,但是

1Q ( x) = lim lim (cos( m!π x))2n


m→∞ n→∞

4.1.6 点到集合距离的连续性
回忆点 x 到 A 的距离定义为 d ( x, A ) = inf {d ( x, a) : a ∈ A }
显然 d ( x, A ) ∈ [0, ∞),但 d ( x, A ) > 0 即使 x ∉ A ,比如,d ( x,Q ) = 0,∀ x ∈ R .
4.2. 连通性 69

1. 注意到 d ( x, A ) = 0 ⇔ 有一列 A 中的点 {a n } 使 d ( x, a n ) → 0 即 a n → x ⇔ x ∈ Ā


即有 d ( x, A ) = 0 ⇔ x ∈ Ā ,换言之,0 是 {d ( x, a) : a ∈ A } 的极限点当且仅当 x
是 A 的极限点.
2. 进一步的,可以证明映射 x → d ( x, A ) 是连续的,而且还是一致连续的,即
有:| d ( x, A ) − d ( y, A )| ≤ d ( x, y)

Proof. d ( x, a) ≤ d ( x, y)+d ( y, a) ⇒ d ( x, A ) ≤ d ( x, y)+d ( y, a) ⇒ d ( x, A ) ≤ d ( x, y)+


d ( y, A )

注意到若 f 连续,则 E : { x ∈ M : f ( x) = 0} 为闭集,反过来,若 E 闭,E =


{ x ∈ M : d ( x, E ) = 0}
即表明 E 是某个函数 f 的 {0} 的原像.这表明,知道 M 中所有闭集,你可
以知道其上的所有实值连续函数,反过来,知道所有实数连续函数,可以知
道所有闭集.
3. 若 A 为紧集,则存在 y0 ∈ A ,使 d ( x, y0 ) = d ( x, A )

Proof. y → d ( x, y) 是连续函数, d ( x, A ) 相当于该函数在 A 上的下确界。作


为紧集上的连续函数,可以取到最小值,故结论成立。

仅要求 A 闭,结论未必成立。

4.2 连通性
在微积分课程中学过中值定理,现在我们稍稍的推广这个定理.
首先我们来证明

定理. [a, b] 不能分成两个互不相交的开集 (非空).

Proof. 否则 [a, b]= A ∪ B


A 、B 非空开集,且互不相交,由于 A 、B 互为补集,故 A 、B 也是闭集.
假定 b ∈ B,故 (b − ε, b] ⊂ B,对于某个 ε > 0.设 c = sup A ,则 a ≤ c ≤ b.
因 b ∈ B,而 c ∈ A ( A 闭) 故 c < b.又 A 开,故 c > a.由 c 定义,∀ε > 0,
( c, c + ε) ∩ B 6= ;.或者说 ( c, b] ⊂ B ⇒ c ∈ B̄ = B ⇒ c ∈ A ∩ B 矛盾.
70 CHAPTER 4. 连续性和连续函数空间

连通:M 不连通,如果 M 能够表示成两个不平凡的开集的并,即两个不相


交的非空开集的并,否则称 M 连通.换言之, M 连通如果 M 不包含既开又闭
的集合 (除去 ; 和 M )
道 路 连 通: 称 空 间 M 是 道 路 联 通 的, 如 果 ∀ x, y ∈ M , 存 在 连 续 函 数
f : [0, 1] → M 使得 f (0) = x, f (1) = y。这样的 f 称为从 x 到 y 的路径。
例子:
1. M = {( x, 0), x ≤ 0} ∪ {( x, sin( 1x )), x > 0} 连通但不是道路连通的
2. R 中的拓扑基为 {[ x, r )},则在该拓扑下 R 不连通
性质:R 的连通子集都是区间,换言之,R 任何可数子集不是连通的

定理 4.7 (连续与连通). 1. M 不连通 ⇔ 存在连续映射将 M 映满 {0, 1}( M → {0, 1}


的满射)
2. f : ( M, d ) → ( N, p) 连续,E 为 M 的连通子集,则 f (E ) 在 N 中连通.

Proof. 1. f −1 ({0}) 既开又闭

⇒ 由定理第二部分可立即推出中值定理
连续性与连通性之间的联系可帮助我们对同胚概念的理解
1. [a, b],(a, b],(a, b) 两两不可能同胚
以 (a, b] 与 (a, b) 为例,假设同胚,则有连续函数 f ,令 c = f (b),则 a < c < b,
考虑 f 在 (a, b) 上的值域为 (a, c) ∪ ( c, b),不连通,矛盾.
2. [0, 1] 与 [0, 1] × [0, 1] 不同胚,更一般的,R n 与 R m 不同胚,如果 n 6= m。
则必有 f ( x) = 12 .
假如存在同胚映射 f : [0, 1]×[0, 1] → [0, 1]. 但 [0, 1]×[0, 1] \ { x}
©1ª
仍连通,而 [0, 1] \ 2 不连通.矛盾。
注:尽管 [0, 1] 与 [0, 1] × [0, 1] 不同胚,但我们可找到 [0, 1] 与 [0, 1] × [0, 1] 的
一一对应,也能找到连续的满射 f : [0, 1] → [0, 1] × [0, 1]
1. Peano 曲线

P

2a n
2. Lebesgue:Cantor 集 ∆, t ∈ ∆,则 t = 3 ,3 进制小数 0.(2a 1 )(2a 2 )…
n=1
4.3. 连续函数空间 71

定义映射 t → ( x ( t) , y ( t)). x ( t) = 0.a 2 a 4 a 6 …(二进制), y ( t) = 0.a 1 a 3 a 5 …


可以证明 x ( t)、 y ( t) 在 ∆ 上连续,且都映满 [0, 1]
可延拓上 [0, 1] 上,故有 [0, 1]→ [0, 1] × [0, 1] 的连续满射.
同样道理可做出 [0, 1]→ [0, 1]n 的连续满射.

4.3 连续函数空间
一般而言,度量空间 ( M, d ) 上的所有连续函数空间是复杂的,但是紧度量空间
上的连续函数空间,比如 C [a, b] 有比较好的性质,这部分我们讨论一下 C [a, b]
的性质,主要考虑它 (1) 是否完备 (2) 是否可分 (3) 紧子集的性态.

4.3.1 完备性
首先来看完备性,我们在一致度量 d ( f , g) = sup | f − g| 下研究。下图说明了在这
个函数空间里 f 的邻域的形态

(注意,用其它度量该空间可能是不完备的。比如 C [0, 1] 上除了一致距离 d1


R
,也可按照 d2 ( f , g) = 01 | f ( x) − g ( x)| dx 定义距离.
对于如下图定义的函数列 { g n ( x)}. g n 在 d2 下收敛至 1{0} ,并非连续函数。

这个例子也说明了不同距离下的收敛性是不同的。由于 d2 ≤ d1 ,故 d1 下收
敛 ⇒ d2 下收敛.但图中 g n 在 d1 下不是 Cauchy 列,不收敛 (即不是一致收敛
的))
72 CHAPTER 4. 连续性和连续函数空间

定理 4.8. C [0, 1] 在一致度量下是完备的 (即连续函数的一致收敛极限是连续


函数)

Proof. { f n } 为 Cauchy 列. f 为其极限,要证 f 连续.


任给 ε > 0,由 f m 连续性,可知存在 δ 使 d ( x, y) < δ 时 | f m ( x) − f m ( y)| < 3ε 。
由一致收敛性质,当 m 充分大时有 ∀ x,| f ( x) − f m ( x)| < 3ε ,故

| f ( x) − f ( y)| < | f ( x) − f m ( x)| + | f m ( x) − f m ( y)| + | f m ( y) − f ( y)| < ε

⇒ f 连续.
f 的作法:由于 sup | f n ( x) − f m ( x)| < ∞.故给定 x 时 { f n ( x)} 是实 Cauchy
列,将其极限定为 f ( x).即相当于点点极限.此时点点极限也是一致极限.因
为一致极限意味着 sup | f n − f | → 0 当 n → ∞.而 Cauchy 列 ⇒ sup | f n − f m | → 0,
n,m → ∞ 即 ∀ε > 0,∃ n 0 ,当 n,m ≥ n 0 时,sup | f n ( x) − f m ( x)| < ε.让 m → ∞.
x
即有 sup | f n − f | < ε.

练习

1. 考虑赋予一致度量的连续函数空间 C[0,1]。对任意 [0,1] 上的区间 I ,记 A[I ]


为所有在 I 上递增的 C[0,1] 中的连续函数,即 A [ I ] = { f ∈ C [0, 1] : x, y ∈ I, x ≤
y ⇒ f ( x) ≤ f ( y)},B[I ] 为所有在 I 上递减的 C[0,1] 中的连续函数。证明:
(1) 对任意区间 I ⊂ [0, 1],A[I ] 和 B[I ] 是闭集且内部为空
(2) 令 I 1 , I 2 , ... 代表 [0, 21 ],[ 12 , 1],[0, 13 ],[ 13 , 23 ],[ 32 , 1],[0, 14 ],…利用 Baire
S S
纲定理说明 C [0, 1] 6= A[I n] ∪ B[ I n ]
n n
(3) 存在 [0,1] 上的连续函数,它在 [0,1] 的任意子区间上既不递增也不递减。
2. 现在考虑 R 上的连续函数全体 C (R ),定义其子集 C0 (R ),它由满足下面条
件的所有连续函数 f 组成:对任意正数 ε,存在 R 中的紧集 F(与 f ,ε 有
关),使得对于所有的 x ∈ F c ,都有 | f ( x)| < ε。称 C0 (R ) 中的函数在无穷大
处为 0。证明:
(1) C0 (R ) ⊆ C (R ) 但是 C0 (R ) 6= C (R )
(2) 赋予一致度量,C0 (R ) 是 B (R )[有界函数全体] 中的闭集,C0 (R ) 完备。
4.3. 连续函数空间 73

检验函数列是否一致收敛

定理 4.9 (Weierstrass M-test). { f n } 为 M 上的一列实函数,且 ∀n,∃a n ,使对


P P∞
所有 x,| f n ( x)| ≤ a n .如果 ∞n=1 a n 收敛.则 n=1 f n ( x) 是一致收敛的.

P
n P
m
Proof. s n ( x) = f i ( x),则 | s n ( x) − s m ( x)| ≤ ak < ε
i =1 k= n

4.3.2 可分性
现在看 C [0, 1] 是否可分.

定理 4.10 (Weierstrass). 若 f ∈ C [0, 1],ε > 0,则存在多项式 p 使

k f − pk∞ = sup | f ( t) − p ( t)| < ε


t∈[0,1]

即可找到一列多项式 p n 使得 p n 一致收敛至 f .

注:(1) 对于任意实系数多项式 p,总可以找到有理系数多项式 q,使


k p − qk∞ < ε.故定理说明了 C [0, 1] 是可分的.
(2) Weierstrass 定理说明

{1, x, x2 , · · · , x n , · · · }

的线性组合稠密,如果考虑的是

{1, x n1 , x n2 , · · · , x n k , · · · }

的线性组合 ( n1 < n2 < · · · < n k < · · · ),则 Müntz 定理表明上述函数的线性组合稠


密 ⇔ 序列的和 Σ∞ 1
k=1 n k
发散,不要求 n k 为整数

Proof. 我们将明确构造出这个多项式序列.给定连续函数 f .
P
n ¡k¢
定义 Bernstein 多项式 (B n ( f )) ( x) = f n · C nk x k (1 − x)n−k , x ∈ [0, 1]
k=0
易见 B n ( f ) 为最多 n 阶多项式 B n ( f ) (0) = f (0),B n ( f ) (1) = f (1)
可以证明确实有 B n ( f ) 一致收敛于 f .
我们用概率方法来解释这个收敛结果.
将 x ∈ [0, 1] 视为概率,即抛硬币朝上概率.则朝下概率为 1 − x.于是抛 n
次,共 k 次朝上的概率为 C nk x k (1 − x)n−k (二项式分布).事实上, n 次抛平均会
74 CHAPTER 4. 连续性和连续函数空间

有 nx 次朝上 (均值 nx) 对应方差是 nx (1 − x).对应的, n 次中朝上次数的比例


x(1− x)
作为随机变量的均值为 x,而方差是 n ,于是给定 δ > 0,若 F 代表 {0, · · · , n}
中满足 | k/ n − x| ≥ δ 的 k,则
P
n → ∞ 时, C nk x k (1 − x)n−k → 0
k∈F
¯ µ ¶ ¯ ¯ ¯
¯ Xn ¯ ¯X n µ µ ¶¶ ¯
¯ k ¯ ¯ k ¯
¯ f ( x) − C nk f x k (1 − x)n−k ¯ = ¯ f ( x) − f C nk x k (1 − x)n−k ¯
¯ k=0 n ¯ ¯k=0 n ¯
¯ µ ¶¯
X n ¯ ¯
≤ ¯ f ( x) − f k ¯ C k x k (1 − x)n−k
¯ n ¯ n
k=0

取 δ 为满足 | x − y| < δ ⇒ | f ( x) − f ( y)| < 2ε 的数,则 ( M = max | f ( x)|)


ε X X
| f ( x) − B n ( f ) ( x)| ≤ C nk x k (1 − x)n−k + 2 M C nk x k (1 − x)n−k ≤ ε
2 k∉F k∈F

如果 n 足够大。

从证明过程中,大家也可以发现下面的积分算子或核函数逼近连续函数的
方法也是可行的。即如果有一列连续函数 K n ( x, y) 满足:当 n 趋于无穷时,关于
R R R
x 一致的, K n ( x, t) dt → 1, | x− t|≥δ |K n ( x, t)| dt → 0, [ 0, 1]|K n ( x, t)| dt ≤ M 则
Z
f n ( x ) := K n ( x, t) f ( t) dt, n = 1, 2, ...
[0,1]

一致收敛到 f .
Weierstrass 定理可以拓展到紧空间上的连续函数,这里不再讨论.
我们讨论上述定理的一个小应用.即 C [a, b] 上的连续函数 f 是否可由其各
阶矩唯一确定:即
Rb
若已知 µn = a x n f ( x) dx,( n = 0, 1, 2, . . . ) 能否确定 f ?
答案是可以的。这里只证明唯一性,即若 f 、 g 有相同的各阶矩.则 f = g.
等价于
Rb
定理. 若 f ∈ c [a, b], a x n f ( x) dx = 0,∀ n = 0,1,2,…则 f = 0

Proof. 由 Weierstrass 定理,有多项式 p n 一致收敛于 f .故 f P n 一致收敛于 f 2 .


且 Z Z
b b
f 2 ( x) dx = lim f ( x) p n ( x) dx
a n→∞ a
Rb Rb Rb
由 a x n f ( x) dx = 0 ⇒ a f ( x) p n ( x) dx = 0⇒ a f 2 ( x) dx = 0 ⇒ f ( x) ≡ 0
4.3. 连续函数空间 75

注意,对于一般的函数,结论是不对的。从概率的角度看,这就是说分布不
可以由其各阶矩完全确定。

4.3.3 Stone-Weierstrass 定理
记 C ( X ) 为 X 上的实值连续函数全体,C ∗ ( X ) 为有界连续函数全体。我们说一
个线性空间是一个代数,如果在其中可以进行乘法运算,且满足 x( yz) = ( x y) z,
x( y + z) = x y + xz, ( x + y) z = xz + yz,α( x y) = (α x) y = x(α y),其中 α ∈ R 子代数就
是指线性子空间,包括内部元素的乘积

引理. C ∗ ( X ) 在一致度量下的任一闭的子代数都是一个格

Proof. 因 min( f , g) = 0.5( f + g) − 0.5| f − g|,max( f , g) = 0.5( f + g) + 0.5| f − g|,只


要证 f ∈ A ⇒ | f | ∈ A
先设 | f | ≤ 1,由 Weierstrass 定理,可知存在多项式 P e 使得 |P e ( f ) − | f | <
e ⇒ | f | 可由 P e ( f ) 一致逼近,因为 P e ( f ) ∈ A ⇒ | f | ∈ A 。对于其它的 f ,由 f 的
有界性可将其转化成 | f | ≤ 1 的情形,得证。

定义. 生成的子代数:包含某一族函数 D 的最小的子代数 A (D )


可区分点的函数族 A : 对 X 中不同的点 x, y,总存在 f ∈ A ,使得 f ( x) 6= f ( y)

定理 4.11 (Stone-Weierstrass). X 为紧 Hausdorff 空间,D 是 C ∗ ( X ) 中可区分


点,包含常值函数 1 的函数族,则 D 生成的子代数在 C ∗ ( X ) 中稠密,或者说 D
生成的一致闭的子代数为 C ∗ ( X )

Proof. 我们证明对于任意 f ∈ C ∗ ( X ),可被 A (D ) 中函数一致逼近


不妨假设 infx∈ X f ( x) < supx∈ X f ( x),且 infx∈ X f ( x) = −1,supx∈ X f ( x) = 1。定义
−1 1
A 1 = { x ∈ X | f ( x) ≤ }, B1 = { x ∈ X | f ( x) ≥ }
3 3
对于任意的 a ∈ A 1 , b ∈ B1 ,存在函数 h ab 使得 h ab (a) 6= h ab (b)。定义
−4 h ab ( x) − h ab ( b) 2
g ab = +
3 h ab (a) − h ab ( b) 3

则 g ab (a) = −2/3, g ab (b) = 2/3, g ab ∈ A (D )


固定 a ∈ A 1 ,对任意 y ∈ B1 , g a y ( y) = 32 ,所以对 z ∈ y 的某个邻域 O y 都
1
有 g a y ( z) ≥ 3 由紧性,可选出有限个 O y1 , · · · , O yn 覆盖 B1 ,可以定义 g a ( x) =
76 CHAPTER 4. 连续性和连续函数空间

min{ g a y1 ( x), · · · , g a yn ( x)}。g a ( x) ∈ A (D ),g a (a) = −2/3,在 B1 上 g a ≥ 1/3,重复上


述步骤可得到 g( x) ∈ A (D ) 且满足在 A 1 上 g ≤ −1/3,在 B1 上 g ≥ 1/3,于是当
x ∈ A 1 ∪ B1 ,| g( x) − g a ( x)| ≤ 2/3。
定义 h0 ( x) = min{ g( x), 1/3},h1 ( x) = max{h0 ( x), −1/3},则 h1 ∈ A (D )。注意到
如果 x ∈ X \( A 1 ∪ B1 ),| h1 ( x)| ≤ 1/3,| f ( x)| ≤ 1/3,而在 A 1 ∪ B1 上 h1 ( x) = g( x),故
|| f − h 1 || ≤ 2/3
对 f − h1 和 [−2/3, 2/3] 重复上述步骤,可找到 h2 ∈ A (D ) 使得 || f − h1 − h2 || ≤
(2/3)2 。故必有 f ∈ A (D ).

练习

1. X ,Y 为紧距离空间,f ,g 为 X 和 Y 上的连续函数, 定义 ( f ⊗ g)( x, y) = f ( x) g( y)


Pn
证明:乘积空间 X × Y 上的连续函数总可以被形如 i =1 f i ⊗ g i 的函数和一
致逼近,其中 f i 是 X 上的连续函数, g i 是 Y 上的连续函数
p
2. 设函数 f = ( x) ∈ C [0, 1],多项式 p n , n ≥ 0 递推定义为

p 0 ( x) = 0, p n ( x) = p n−1 ( x) + 0.5( x − [ p n−1 ( x)]2 ), n ≥ 1

证明在一致距离下 p n 收敛于 f (提示:Dini 定理)

4.3.4 紧子集
最后讨论连续函数空间里的紧集,这里略去相关的证明,可参考 Kolmogorov 和
Fomin 的书。
定义:设 F = { f i , i ∈ I } 为 M 到 N 的一族连续函数,称 F 是在 x 处等度连
续,如果 ∀ε > 0,∃δ x > 0,使得 d ( x, y) < δ x 时总有 ρ ( f i ( x) , f i ( y)) < ε,∀ y, i .若
每一点都等度连续,则称 F 为等度连续。如果对任一 ε > 0,存在与 x 无关的 δ,
则称为一致等度连续。

例. 给定 m > 0, F 为 (0, 1) → (0, 1) 的函数族,满足 f 0 存在,| f 0 | ≤ m,则 | f ( x) −


f ( y)| ≤ m| x − y|,所以给定任意 ϵ,可取 δ = ϵ/ m,总有 | x − y| < δ → | f ( x) − f ( y)| < ϵ,
表明 F 一致等度连续
¡ ¢
定理 4.12. 从紧距离空间 ( M, d ) 到距离空间 N, ρ 的任何等度连续函数族都是
一致等度连续的.
4.3. 连续函数空间 77

定义. 一族从 M 到 R 的函数称为一致有界的,如果

sup {| f ( x)| : f ∈ F , x ∈ M } < ∞

定理 4.13 (Arzela 阿尔采拉). 紧距离空间上定义的连续函数族 (赋予一致度量


时),是紧的充要条件是该族一致有界且等度连续 (而且是闭的).

Proof. 我们简略说明证明思路。
不妨设 X 为紧距离空间,F 为 C ( X ) 的闭子集。如果 F 是紧集,按照一致
距离定义的集合有界性表明函数族是一致有界的。任意给定 ε,存在有限个函数
族里的函数 f j 使得
F ⊂ ∪nj=1 B( f j , ε/3)

因为只有有限个 f j ,而且都一致连续,所以存在 δ(ε) 使得 d ( x, y) < δ 时总有


| f j ( x) − f j ( y)| < ε/3, ∀ j 。于是对任意 f ∈ F , f ∈ B( f j 0 , ε/3)

f ( x) − f ( y) ≤ | f ( x) − f j 0 ( x)| + | f j 0 ( x) − f j 0 ( y)| + | f j 0 ( y) − f ( y)| < ε

说明函数族等度连续。
反过来,C ( X ) 完备,F 为闭子集,若 F 一致等度连续(紧距离空间上的等
,由下面的定理可知 F 是全有界的,所以它是
度连续函数族也是一致等度连续)
紧的。

作为上述定理的简化形式,我们证明如下定理

定理 4.14. A, B 是全有界距离空间,若 F 是 B A 中一致等度连续的函数族,则


F 在一致度量下是全有界的

Proof. 任意给定 ϵ > 0,选取 δ 使得 ∀ f ∈ F , x1 , x2 ∈ A, d ( x1 , x2 ) < δ ⇒ ρ ( f ( x1 ), f ( x2 )) <


ϵ
4.
令 D 为 A 的有限子集并使得 D 在 A 中 δ− 稠密 ( A ⊂ ∪ x∈D Bδ ( x)),E 是 B
的有限子集且在 B 中 14 ϵ− 稠密。令 G = E D ,则 G 有限,对 G 中任意函数 g 定

1
F g = { f ∈ F : ρ ( f ( x), g( x)) < ϵ, ∀ x ∈ D }
4
我们证明 {F g } 能覆盖 F ,且每个 F g 的直径至多为 ϵ。为此要证每一个 f ∈ F
都存在某个 F g 使得 f ∈ F g 。
78 CHAPTER 4. 连续性和连续函数空间

关键是找到 g。根据已知,D 中的每个点 x,都有 E 中的 y 与 f ( x) 相距小


于 0.25ϵ,如果对每个 x 选择这样的 y,得到的即为 G 中的 g 使得 f ∈ F g
最 后 证 明,∀ f , h ∈ F g , ρ ( f , h) < ϵ。 因 为 ρ (h, g) < 0.25ϵ, ρ ( f , g) < 0.25ϵ ⇒
ρ ( f ( x), h( x)) < 0.5ϵ, ∀ x ∈ D 故对任意 x0 ∈ A ,只需要选择 x ∈ D 使得 d ( x, x0 ) < δ,
于是有

ρ ( f ( x0 ), h( x0 )) ≤ ρ ( f ( x0 ), f ( x)) + ρ ( f ( x), h( x)) + ρ ( h( x), h( x0 )) < ϵ

现在我们利用 Ascoli-Arzela 定理来证明常微分方程初值问题解的存在性,


与应用 Banach 不动点定理时不同,这里不需要假设方程中映射满足 Lipschitz
条件,但是得出的结论也相对更弱一些,实际上只能保证局部的存在性,也不确
保解的唯一性。从证明中我们也得到了 Euler 方法的收敛性。

定理 4.15. 给定 T > 0, r > 0, u0 ∈ R N , g ∈ C ([0, T ] × B(u0 , r ); R N ),则存在 τ ∈ (0, T ]),


使得
u0( t) = g( t, u( t)), 0 ∈ [0, T ], u(0) = u 0

至少有一个解 u ∈ C 1 ([0, τ]; R N )

Proof. 令

M = sup{|| g( t, v)||; ( t, v) ∈ [0, T ] × B( u 0 , r )}, τ = min{ r / M, T }

目标等价于证明下面的积分方程有解
Z t
u ( t) = u 0 + g( s, u( s)) ds, t ∈ [0, τ]
0

将区间 [0, τ] 进行 n 等分,考虑如下的差分方程


u i+1 − u i
= g( t i , u i ), t i = ih = i τ/ n
h

可以验证 u i ∈ B(u0 , r )
然后可以定义分段线性函数
t − ti
û n ( t) = u i + ( u i+1 − u i ), t ∈ [ t i , t i+1 ]
h
4.4. 连续函数的延拓 79

,可知 û n ∈ C 0 [0, τ] 在空间 C 0 [0, τ] 赋予一致距离,可以验证 û n 序列有界而且等


度连续,所以是空间里的紧集合,存在子序列 ûσ(n) 收敛到连续函数 u ∈ C 0 [0, τ]
R ti
注意到此时 u i+1 = u0 + 0 g n ( s) ds,其中 g n ( s) = g( t i , u i ), s ∈ [ t i , t i+1 ], i ∈ [0, n],
有 Z t
û n ( t) = u 0 + g n ( s) ds, t ∈ [0, τ]
0
因为在 [0, τ] 上 g(s, u(s)) 一致连续,当 n 趋于无穷,||| ûσ(n) − u||| → 0,

sup || g σ(n) ( s) − g( s, u( s))|| → 0


0≤ s≤τ

所以 Z Z
t t
sup || g σ(n) ( s) ds − g( s, u( s)) ds|| → 0
0≤ s≤τ 0 0

练习

1. 证明 Rn 中紧集上的任意等度连续函数族是一致等度连续的。
2. X 是紧距离空间,F 是 C ( X ) 的闭子空间,证明 F 是紧集,如果它等度连
续且对任意 x ∈ X ,F x = { f ( x) : f ∈ F } 是 R 中有界集合
3. f (s, t, u) 是定义在 0 ≤ s ≤ 1, 0 ≤ t ≤ 1, −1 ≤ u ≤ 1 上的连续实值函数,对于满
足 || x|| ≤ 1 的 x ∈ C [0, 1],定义
Z 1
F x ( s) = f ( s, t, x( t)) dt
0

显然上述映射是从 C [0, 1] 到 C [0, 1] 的映射,证明该映射是紧映射。[从距离


空间 X 到 Banach 空间 E 的映射是紧的,如果 f ( X ) 在 E 中是紧的]
4. 证明紧距离空间 K 上等度连续而且点点收敛到 f 的连续函数列 f n 也一致
收敛到 f

4.4 连续函数的延拓
一个拓扑空间中的开集可能太少,比如只包括空集和全空间。因此,这一部分我
们引入空间可分离性的概念,以考察空间是否有足够多的开集,从而保证空间上
存在足够多的连续函数。
80 CHAPTER 4. 连续性和连续函数空间

定义. 1. 一个拓扑空间是 T1 空间,如果对于任意两个不同的点,各自有一个


邻域,使得另一个点不在该邻域中。等价于要求每个点都是闭集
2. 一个拓扑空间是 Hausdorff 空间,如果对于任意两个不同的点,各自有一个
邻域使得两个邻域互不相交,也可以说不同点可以由开集分离开
3. 一个拓扑空间 X 是完全正则空间,如果 (它是 T1 空间),且对于点 x ∈ X 和
不包含 x 的闭集 F ,存在连续函数 f : X → [0, 1],使得 f ( x) = 0 且 f (F ) ⊂ 1。
4. 拓扑空间 X 称为正规的,如果 (它是 T1 空间),若对 X 的任意不相交的闭
子集 A 和 B,存在不相交的开集 A 1 和 B1 使 A ⊂ A 1 ,B ⊂ B1

注:图中的完全正则空间和正规空间也假定是 T1 空间

定理 4.16 (Urysohn’s 引理 (乌里松引理)). A 和 B 为度量空间中不相交的闭子


集,则存在整个空间上的连续函数 f : X → [0, 1],使 f | A = 0 , f |B = 1 .

反例:如果去掉闭的要求,比如 A = (0, 1),B = {1} 不能找到这样的连续函数.


d(x,A)
Proof. 令 f ( x) = d(x,A)+ d(x,B) ,则 f 连续,满足要求.

定理 (一般形式的 Urysohn’s 引理). X 是正规空间, A 和 B 为 X 中不相交的闭


子集,则存在整个空间上的连续函数取值于 [a, b],使 f | A = a , f |B = b .

定理 4.17 (Tietze 扩张定理). 若 f : E → R 是 E 上的连续有界函数,E 为


闭 子 集 ⊂ X (度 量 空 间). 则 存 在 连 续 函 数 g : X → R 使 在 E 上 g = f , 且
sup {| g ( x)| , x ∈ X } = sup {| f ( x)| , x ∈ E }
¡1¢
反例:E 闭条件必须,否则比如 X = [0, 1],E = (0, 1), f = sin x
4.4. 连续函数的延拓 81

Proof. 令 M = sup | f ( x)| < ∞.定义


x∈ E
½ ¾ ½ ¾
M M
A 1 = x ∈ E : f ( x) ≤ − , B 1 = x ∈ E : f ( x) ≥
3 3

f 连续 ⇒ A 1 、B1 为 E 中的闭集,也为 X 中闭集.由 Urysohn’s 引理,存


在连续函数 h1 , h1 | A 1 = − 13 M , h1 |B1 = 13 M .
令 f 1 = f − h1 ,则其在 E 上连续.在 A 1 上, f 1 ≤ − 13 M , h1 = − 31 M ,在 B1
上, f 1 ≥ 13 M ,h1 = 13 M .在 A 1C ∩ B1C 上,− 13 M < f < 31 M ,− 13 M ≤ h1 ≤ 13 M ⇒ 在
E 上 | f 1 | ≤ 32 M 。定义
½ µ ¶¾ ½ µ ¶¾
1 2 1 2
A 2 = x ∈ E : f 1 ( x) ≤ − M , B 2 = x ∈ E : f 1 ( x) ≥ M
3 3 3 3
¯ ¡ ¢ ¯ ¡ ¢
可以找到 h2 , h2 ¯ A 2 = − 31 23 M , h2 ¯B2 = 13 23 M
令 f 2 = f 1 − h2 = f − h1 − h2 ,则 f 2 在 E 上连续,且
µ ¶ µ ¶2
2 2 2
| f2| ≤ M = M
3 3 3
¡ 2 ¢n−1
继续下去,可在 M 上得到连续函数 h n 使 | h n | ≤ 13 3 M .且在 E 上
¯ ¯ µ ¶
¯ X n ¯ 2 n
¯ ¯
¯f − h j¯ ≤ M
¯ j =1
¯ 3
P∞
由 Weierstrass M-test, n=1 h n 一致收敛至某个连续函数 g.其中
µ ¶
1 X∞ 2 n−1
| g| ≤ M n=1 =M
3 3

而且在 E 上 f = g.

上述定理可以说明连续函数是足够多的,即任意不同的点 x, y,存在连续
函数,它在 x, y 上的取值不同.

利用 Stone-Weierstrass 定理证明 Tietze 扩张定理. 考虑简单情形,E 是紧集。考


虑这样的连续函数延拓,在 E 上一致,并且在 U c 上取值为 0,其中 U 开,U c
紧而且 E ⊂ U ⊂ X 。易见这样的函数族 C E 限定在 E 上是 C ∗ ( X ) 的子代数。根
据 Stone-Weierstrass 定理,只需证明 C E 可区分点,包含常值函数 1,且在一致
度量下是闭集。
82 CHAPTER 4. 连续性和连续函数空间

对于不同的点 a, b ∈ E ,可找到互不相交的闭集 (为什么?) 分别包含它们,


于是根据 Urysohn 引理可知存在区分这两点的连续函数。类似可知 C E 包含常
值函数。
对于 C E 中函数 f ,令 α = max f ( x), x ∈ E ,β = min f ( x), x ∈ E ,则 || f || =
max{|α|, |β|}。
记 f + 为 f 的延拓,α+ = max{α, 0}, β+ = min{β, 0},
定义 ϕ = min{max{ f + , β+ }, α+ } 可知 || f || = || f + ||,表明 C E 中函数延拓可保持
范数不变,故对于 C E 中一致收敛的函数列 f n ,可找到子列 || f n k − f n k+1 || < 2−k ,
令 g k = f n k − f n k+1 ,可以找到范数不变的延拓 g+k ,不难发现 g+k 的一致收敛极
限限制在 E 上恰好是 f n 的极限,也是 C E 中函数,表明 C E 中函数是闭集。
得证。

练习

1. 在正规空间 X 中, A 和 B 为不相交的非空闭集,问是否存在连续函数
f : X → [0, 1],使得 f −1 (0) = A , f −1 (1) = B

4.5 Banach 空间上的线性泛函


假设 X,Y 是赋范线性空间。X 到 Y 上的线性泛函指映射 l : X → Y ,满足
l (α f + β g) = α l ( f ) + β l ( g),∀α, β ∈ R , f , g ∈ X

定义. 线性泛函 l 是连续的,如果对于任意给定的 ε > 0,存在 δ > 0,使得只要


|| f − g|| X ≤ δ,就有 || l ( f ) − l ( g)||Y ≤ ε。
线性泛函 l 是有界的,如果存在 M > 0,使得 ∀ f ∈ B,|| l ( f ) ||Y ≤ M || f || X

下面的定理说明,对于线性泛函,其连续性和有界性是等价的。

定理 4.18. 赋范空间 X 到 Y 上的线性泛函是连续的,当且仅当它是有界的。

Proof. 由线性泛函的定义, l 是连续的,当且仅当 l 在 0 点是连续的。


假定 l 连续,可知存在 δ > 0,使得只要
³
|| f || X ≤ δ,就有 || l ( f ) ||Y ≤ 1。对于 X
´
中任意元素 h, ||hδ||hX 的范数为 δ,所以 || l δh
|| h|| X
||Y ≤ 1。所以 || l ( h) ||Y ≤ δ1 || h|| X ,
即 l 有界
而若 l 有界,对于给定的 ε > 0,要使得 || l ( f ) ||Y ≤ ε,只需要 M || f ||X ≤ ε,即
ε
|| f || X ≤ M. 得证。
4.5. BANACH 空间上的线性泛函 83

对于 X 到 Y 上的连续线性泛函,一个简单的事实是,这些函数的全体构成
一个线性空间,因为在上面可以进行线性运算和数乘运算。在这个线性空间里,
可以如下定义算子范数:

|| l ( f ) ||Y
|| l || = sup || l ( f ) ||Y = sup || l ( f ) ||Y = sup
|| f || X ≤1 || f || X =1 f 6=0 || f || X

相当于有界性定义中对所有线性泛函 l 都成立的 M 的下确界。


记 L ( X , Y ) 为从赋范线性空间 X 到 Y 上的所有连续线性泛函全体

定理 4.19. 赋予前述的算子范数,L ( X , Y ) 也是一赋范线性空间。

Proof. 需要验证若 l 1 , l 2 ∈ L ( X , Y ),则 l 1 + l 2 , αl 1 ∈ L ( X , Y ),这直接验证定义就


可得到。还需要证明 || l 1 + l 2 || ≤ || l 1 || + || l 2 ||,||αl 1 || = |α| || l 1 ||,|| l 1 || = 0 ⇔ l 1 = 0.
为证此,考虑 ||(l 1 +l 2 ) ( x) || ≤ || l 1 ( x) ||+|| l 2 ( x) || ≤ || l 1 |||| x||+|| l 2 |||| x|| = (|| l 1 || + || l 2 ||) || x||
取 || x|| = 1, 就 有 || l 1 + l 2 || ≤ || l 1 || + || l 2 ||。 而 ||(αl 1 ) ( x) || = |α| || l 1 |||| x||, 故
||α l 1 || = |α| || l 1 ||。

定理 4.20. 若 Y 是 Banach 空间,则 L ( X , Y ) 也是 Banach 空间。

Proof. 设 {l n } 是 L ( X , Y ) 中的 Cauchy 列,则 ∀ x ∈ X ,{l n ( x)} 是 Y 中的 Cauchy


列,因为
|| l n ( x) − l m ( x) || ≤ || l n − l m |||| x|| → 0

所以可定义 l ( x) = liml n ( x),显然 l 是线性的。考虑到

|| l ( x) || = || l ( x) − l m ( x) || + || l m ( x) || ≤ || l ( x) − l m ( x) || + || l m |||| x||

m 充 分 大 时,|| l ( x) − l m ( x) || ≤ ϵ|| x||, 故 || l ( x) || ≤ (ϵ + || l m ||) || x||, 说 明 l 有 界,


l ∈ L ( X , Y ),而且 || l || ≤ ϵ + || l m ||。而这也说明 m 充分大时,|| l − l m || ≤ ϵ,即
l m → l.

如果 Y =R,那么 L ( X , R) 是 Banach 空间,它称为 X 的对偶空间,记为


X ∗。
例:若 1 < p < ∞,则 l p 的对偶空间是 l q ;c 0 的对偶空间是 l 1 ;紧 Hausdorff
空间 K 上的连续实值函数全体赋予一致范数后成为 Banach 空间 (C (K ) , || · ||∞ ),
其对偶空间为 K 上的 Radon 符号测度全体形成的空间
84 CHAPTER 4. 连续性和连续函数空间

w
定义:称 { xn } 弱收敛到 x ∈ X ,记为 xn → x,如果对所有的 ϕ ∈ X ∗ ,ϕ ( xn ) →
© ª w∗
ϕ ( x);称 ϕn 弱 ∗ 收敛到 ϕ ∈ X ∗ ,记为 ϕn → ϕ,如果对所有的 x ∈ X ,ϕn ( x) →
ϕ ( x)[比较点点收敛]。
例:测度的淡收敛,K 为紧 Hausdorff 空间,µ, µ1 , µ2 , · · · 是 K 上的 Radon
R R
测度,如果对于任意的 f ∈ C (K),有 f d µn → f d µ,则称 {µn } 淡收敛到 µ。不
难看出,这本质上就是 C (K )∗ 中元素的弱 ∗
收敛。

弱 收敛的主要用处体现在下面的定理:

定理 (Alaoglu 定理). 若 X 为赋范线性空间,则 X ∗ 中的单位闭球 B = { f ∈ X ∗ : f ≤ 1}


∗ ∗
是弱 紧的,即 B 在弱 拓扑下是紧的,换言之, X ∗ 中的有界序列有弱 ∗

敛的子序列。

定理的证明可参见 Royden 的 Real Analysis 的 P237.


第 5 章 测度

5.1 测度的定义
回忆 σ 代数的定义, X 的子集类 F 称为 X 上的 σ 代数如果

1. X ∈ F
2. A ∈ F ,则 A c ∈ F
3. { A i }∞ ∞
i =1 ∈ F ,则 ∪ i =1 A i ∈ F

定义. 称 X 及其上的 σ 代数 F 为可测空间 ( X , F )。称 F 中的集合为可测的。


如果在可测空间 ( X , F ) 上有一个定义域为 F 的广义实值函数µ µ:F¶ → [0, +∞],
∞ P

满足可数可加性,即对 F 中任意互不相交的集合列 { A i } 有 µ ∪ A i = µ (Ai)
i =1 i =1
,且 µ (;) = 0。称 µ 为测度,( X F ,µ) 称为测度空间.

注意.
µ 1.¶ 由可数可加性容易推出有限可加性,即对有限个互不相交的集合
n P
n
µ ∪ Ai = µ ( A i ).
i =1 i =1
2. 广义实数的运算:若 0 ≤ a ≤ ∞,a + ∞ = ∞ + a = ∞,
(
∞ a>0
a·∞ = ∞·a =
0 a=0
若 a = ∞,a + b = a + cb = c,ab = acb = c
广义实数集合 [−∞, ∞] 是紧的,如果我们定义 −∞ 的邻域需包含某个
[−∞, − n),∞ 的邻域需包含某个 ( n, ∞]

可测空间的例子

1. X 为一集合,则 X 的幂集是 σ 代数,{;, X } 也是 σ 代数

85
86 CHAPTER 5. 测度

2. X 为无限集合,A 为 X 中有限子集的全体,则 A 不是 σ 代数.若 B


是 X 中使 A 或 A c 有限的子集 A 的全体,则 B 是代数而不是 σ 代数.C 是 X
中使 A 或 A c 可数的 A 的全体,则 C 是 σ 代数.
3.σ 代数的交还是 σ 代数.但是 σ 代数的并不是一定是 σ 代数.
4.最重要的 σ 代数之一是 R n 中由全体开集生成的 σ 代数.通常记为
B (R n ).其中的集合称为 Borel 集.[注意:当讨论到 Borel 集时,自然就涉及到
空间的拓扑]

性质. R 中的 Borel σ 代数可由下列集合生成: 1). 全体 R 中闭集,2). 全体形


如 (−∞, b] 的区间,3). 全体形如 (a, b] 的区间.4). 全体形如 (a, b] 的区间,其
中 a, b 为有理数

Proof. 注意 (a, b] = (−∞, b] \ (−∞, a],显然 B (R ) ⊃ B1 ⊃ B2 ⊃ B3 .只需证明


B3 ⊃ B (R ) 即可.这又只需证 B3 中包含所有形如 (a, b) 的开区间.注意 (a, b) =
∞ ¡
S ¤
a, b − n1
n=1

性质. R n 中的 Borel 代数可由下列集合生成: 1) R n 中全体闭集,


2) 形如 {( x1 , · · · , xn ) : x i ≤ b i } 的集合,
3) 形如 {( x1 , · · · , xn ) : a i ≤ x i ≤ b i } 的集合.

对于 B (R n ) 中的其它集合形态的分析:
记 G δ 为可数个开集的交集,Fσ 为可数个闭集的并.则 G δ 、Fσ 都是 Borel
集.

性质. R n 中的闭集都是 G δ .开集都是 Fσ (练习:找一个不是 Fσ 也是 G δ 的


集合)
© ª
Proof. F 为闭集,定义 Un = x ∈ R n , d ( x, F ) < n1 ,则 Un 开,F ⊂ ∩Un ,另一方
面 ∩Un 中的点都满足 d ( x, F ) = 0 ⇒ x ∈ F ⇒ F 是 G δ .
注意 G δ 集的补集是 Fσ ,故开集是 Fσ .

由 G δ 、Fσ 集可以进一步构造 Fσδ 、G δσ 、Fσδσ ,G δσδ ,…集合.可知有下列


包含关系.
© ª
开集 ⊂ {Fσ } ⊂ {G δσ } ⊂ · · ·
© ª
闭集 ⊂ {G δ } ⊂ {Fσδ } ⊂ · · ·
5.1. 测度的定义 87

但这些集合并未穷尽 Borel 集.
注:拓扑空间 X 上另一个重要的 σ 代数是由所有形如 { x ∈ X , f ( x) > 0} 的集
合生成的 σ 代数,其中 f 取遍所有连续函数。称为 Baire σ 代数,记为 B a ( X )。
通常情况下 B a ( X ) ⊂ B ( X )

测度的例子

1.( X , F ).µ ( A ) = A 中元素个数,或 +∞( A 无限个元素),则可验证 µ 为测度


(计数测度)
(
1 x∈ A
2.( X , F ).δ x ( A ) = .则 δ x 是一个测度(Dirac 测度)[这两个例
0 x∉ A
子表明任意可测空间上都存在测度,而且可以找到无穷多的测度]
3.在 (R, B (R )) 上存在一个测度使得 µ ((a, b]) = b − a,这就是 Lebesgue 测
度,但具体如何构造呢,特别是形状比较古怪的 B (R ) 中集合?这是本章的主要
内容.
¡ ¢
由测度的定义,可以很容易的推出下面的一些性质. X , F , µ 为测度空间.
1.若可测集 A ⊂ B,则 µ ( A ) ≤ µ (B)(单调性)
.若 µ ( A ) < ∞.则 µ (B\ A ) =
µ (B ) − µ ( A ) µ ¶
∞ P

2.{ A k } 为任意可测集列,则 µ ∪ A k ≤ µ ( A k ) (次可数可加性)
k=1 k=1
¡ ¢
注意,令 B k = A k − ∪ki=−11 A i ,可将 { A k } 化为一列互不相交的集合
3.{ A k } 为递增可测集列.则 µ (∪k A k ) = limk µ ( A k ).若 { A k } 为递减可测集
列且某个 µ ( A n ) < ∞ 则 µ (∩k A k ) = limk µ ( A k ).[测度的连续性]
µ ¶
P ¡ ¢ P
k ¡ ¢
Proof. 递增时令 B j = A j − A j−1 有 µ ∪ A k = µ B j = lim µ B j = lim µ ( A k )
k j k→∞ j =1
递减时定义 C k = A 1 − A k ,则 {C k } % ∪C k = A 1 − (∩ A k ).µ (∪C k ) = lim µ (C k )
µ ¶
⇒ µ A 1 − ∩ A k = lim µ ( A 1 − A k ) ⇒ µ (∩ A k ) = lim µ ( A k )
k

练习

1. 找一个不是 Fσ 也不是 G δ 的集合


88 CHAPTER 5. 测度

2. ( X , F ) 为可测空间,
(a) 假定 µ 在 F 上非负且可数可加,若存在某个 A 使 µ ( A ) < ∞ 则 µ (;) = 0
(b) 举例说明测度定义中的 µ (;) 不能省略 (即可从定义其它部分推得)
S

3. 称 µ 是 σ 有限的,如果存在集合列 { A n } ⊂ F ,∀n, µ( A n ) < ∞, X = An。
¡ ¢ n=1
证明:如果 σ 有限测度空间 X , F , µ 满足 µ( X ) = ∞,则对任意 m < ∞,存
在可测集 A ∈ F 使得 m < µ( A ) < ∞。
4. 令 X = [0, 1],F 为其中第一纲集的集合。对任意 A ∈ F ,定义 m( A ) = 0,
m( X \ A ) = 1,证明: m 是定义在某个 σ 代数上的测度。

为什么需要可数可加性
[a, b] 上的有界函数 f 的积分应该满足
R R
1. ab f ( x) dx = ab++hh f ( x − h) dx
R R R
2. ab f ( x) dx + bc f ( x) dx + ca f ( x) dx = 0
R R R
3. ab [ f ( x) + g ( x)] dx = ab f ( x) dx + ab g ( x) dx
R
4. ab f ( x) dx ≥ 0,a < b, f ≥ 0
R
5. 01 1dx = 1
R R
6. f n ( x) 单调递增收敛于 f ( x),则 ab f n ( x) dx → ab f ( x) dx
Riemann 积分满足 1~5,但不满足 6
P
n
对于函数 f ,可以用 g = C i 1[yi−1 ,yi ) ,C i ∈ [ yi−1 , yi ) 近似,分划记为 p =
i =1
Rb Pn
{ y0 , · · · , yn },若记 E i = { x : f ( x) ∈ [ yi−1 , yi )}.直观上 a g = C i m (E i )
i =1
Rb Rb
a f 可定义为 a g 的极限 ( p 分划越来越细).其中的关键是测度 m.
测度问题是希望找到满足下面条件的集合函数.
1.m ([0, 1]) = 1(对应 5,单位方体的测度为 1)
2.m (E + h) = m (E ) = m (−E ) E + h = { x + h : x + E }(对应 1,平移旋转翻转测
度不变)
P
3.对于不相交的 {E n }, m (∪n≥1 E n ) = n≥1 m (E n )(对应 6)
其中条件 3,即可数可加性,是 Lebesgue 积分论中与之前的理论差异最大
的地方.
那么为什么要可数可加性呢?若只有有限可加性,则太弱无法保证积分条件
6 成立,同时下面的例子也说明只有有限可加性,与常识多有不符
5.2. R N 空间的 LEBESGUE 测度 89

1. R 中任意区间 I = (a, b].p ( I ) = 0,而 I = (a, ∞),p ( I ) = 1,则 p 是有限可加


的,但是不是可数可加的. p ((a, b]) p ((a, ∞)),当 b → ∞ 时
2. A 和 B 随机挑选整数 1,2,….假定定义在 2 N 上的有限可加函数,即有限
集 A ,m ( A ) = 0,m ( N ) = 1,然后谁挑的数大谁就赢.若先告诉你 A 选取的
数字,请问谁赢的可能性大?
显然在这种情形下,B 赢的可能性为 1.
大家可能会感觉如果选数的概率都一样,也会产生上述结果.但实际上在
N 上不存在可数可加的均匀分布,且要求 2 N 中的每个集合的概率都知道.
只能在部分集合定义均匀分布.即设 P ({ x ∈ N, x ≡ k ( mod n)}) = n1 .
3. 类似的另一个例子是两个信封中各有一些钱,其中一个是另一个的两倍,现
在你随机打开一个,问你是否应当换另一个信封.(Two envelopes problem)
初看上去,若看到 x,另一个为 2 x 或 2x ,均值是 54 x,似乎该换.
但是上述结果实际上假定了 R + 上的均匀分布,但这是不存在的 (如果假定
可数可加性)。
同样类似的,若假设不可数可加性,也会产生意想不到的结果,比如每个点
m { x} = 0 但 m ([0, 1]) 在不可数可加性的假设下也会为.
最后要指出的是 Lebesgue 积分时测度的三个要求在选择公理成立的时候,
不能对所有集合都成立,即不是所有集合都是 Lebesgue 可测的.
事实上,即使只要求条件1 和 3 或者只要求1,2 +有限可加性 也不能保证所
有集合是可测的.(连续统假设) (Banach-Tarski 悖论)

5.2 R n 空间的 Lebesgue 测度

下面我们正式介绍如何构造 R n 空间的 Lebesgue 测度


基本的思路是构造外测度,它对任意集合都有定义,但不满足 (可数) 可加
P
性,只有次可加性 m (∪n≥1 A n ) ≤ n≥1 m ( A n ).然而将外测度限制在某些集合类
上,使之满足可数可加性,这样就得到可测集类和其上的测度.
由于矩体的形态足够简单,而且它的体积是已知的,所以测度的构造就由矩
体开始.
定义:R n 中形如 B = {( x1 , · · · , xn ) ∈ R n : x i ∈ (a i , b i ]} 的集合称为半开闭矩体,
Q
n
它的体积是 |B| = (b i − a i )
i =1
90 CHAPTER 5. 测度

注:若某个 a i = b i ,则 B 为 ;,其体积为 0.另外由前面的条件 1,我们实


际上要求 B 的测度就等于它的体积.
对于半开闭矩体,我们有如下看似简单却不容易证明的性质.
© ª
引理 5.1. 若 B 是 R n 中的半开闭矩体, B j 是一列覆盖 B 的半开闭矩体,则
∞ ¯
P ¯
|B | ≤ ¯B j ¯
j =1

Proof.
n
先证 1 维的情形.若 B = ;,则显然,若
o
B = (a, b],a < b.记 H x = ( x, +∞),
P∞ ¯¯ ¯
¯ (B j ∩ H x 仍然是个半开闭区间)
A = x : a ≤ x ≤ b, b − x ≤ j =1 B j ∩ H x

显然 b ∈ A ,又 A ⊂ [a, b],可令 c = inf A .因为


X¯ ¯ X¯ ¯
b − c = sup ( b − x) ≤ sup ¯B j ∩ H x ¯ ≤ ¯B j ∩ H c ¯ ⇒ c ∈ A
x∈ A x∈ A
¯ ¯ ¯ ¯
假如 c > a,则 c ∈ B k = (a k , b k ].令 x = max (a, a k ) < c,则 ¯B j ∩ H x ¯ ≥ ¯B j ∩ H c ¯
X¯ ¯ X¯ ¯
|B k ∩ H x | = |B k ∩ H c | + c − x ⇒ ¯B j ∩ H x ¯ ≥ ¯B j ∩ H c ¯ + c − x ≥ b − x

P ¯¯ ¯ P¯ ¯
矛盾 ⇒ c = a ⇒ b − a ≤ B j ∩ H a ¯ ≤ ¯B j ¯
现假定 n = r 时结论成立,考虑 n = r + 1 的情形,不妨设 B = (a, b],B j =
¡ ¤ ¡ ¤ Qr ¡ ¢ ¡ ¢
a j , b j ,B 的第 i 维为 α i , β i .记 ς = β i − α i .则 |B| = ς βr+1 − αr+1
© i =1 ª
r +1
任给 ε > 0,ε ∈ R , Hε = x ∈ R , ξr+1 > ξ .记
n ¡ ¢ X∞ ¯ ¯o
A = ξ : αr+1 ≤ ξ ≤ βr+1 , ς βr+1 − ξ ≤ (1 + ε) j=1 ¯B j ∩ Hξ ¯

显然 βr+1 ∈ A ,令 γ = inf A
¡ ¢ ¡ ¢ X¯ ¯ X¯ ¯
ς βr+1 − γ = sup ς βr+1 − ξ ≤ (1 + ε) sup ¯B j ∩ Hξ ¯ ≤ (1 + ε) ¯B j ∩ Hγ ¯
ξ∈ A ξ∈ A

⇒γ∈ A
© ¡ ¢ ª © ¡ ¢ ª
若 γ > βr+1 ,并记 J = x ∈ R γ , x, γ ∈ B , J j = x ∈ R r , x, γ0 ∈ B j
由 B ⊂ ∪B j ⇒ J ⊂ ∪ J j ,而且 J 和 J j 是 R γ 中的半开闭矩体,由假设
P ¯¯ ¯ P ¯¯ ¯¯
ς = |J| ≤ J j ¯ 显然存在某个 m,使 ς ≤ (1 + ε) m
j =1 J j .令

n o
( j)
ξ = max αγ+1 , αγ+1 , j ≤ m, J j 6= Φ < γ
5.2. R N 空间的 LEBESGUE 测度 91
¯ ¯ ¯ ¯ ¯ ¯¡ ¢
则 ¯B j ∩ H ξ ¯ ≥ ¯B j ∩ H γ ¯ + ¯ J j ¯ γ − ξ j ≤ m
¡ ¢ ¡ ¢ ¡ ¢
ς βγ+1 − ξ = ς βγ+1 − γ + ς γ − ξ
X∞ ¯ ¯ ¡ ¢ Xm ¯ ¯
≤ (1 + ε) j=1 ¯B j ∩ Hγ ¯ + γ − ξ (1 + ε) j=1 ¯ J j ¯
X∞ ¯ ¯ Xm ¯ ¯
= (1 + ε) m+1 ¯B j ∩ Hγ ¯ + (1 + ε) j=1 ¯B j ∩ Hξ ¯
X∞ ¯ ¯
≤ (1 + ε) j=1 ¯B j ∩ Hξ ¯
P∞ ¯¯ ¯
¯ P∞ ¯¯ ¯¯
矛盾 ⇒ γ = αr+1 . 即有 |B| ≤ (1 + ε) j =1 ¯ B j ∩ H αγ+1 ¯ ≤ (1 + ε) j =1 B j
P ¯¯ ¯¯
由 ε 的任意性 ⇒ |B| ≤ ∞
j =1 B j 得证

5.2.1 外测度
现在可以定义 Lebesgue
( 外测度 m∗ (R n ) → [0, ∞], )
∞ ¯
P ¯ © ª © ª
m∗ (E ) = inf ¯B j ¯ : B j 为半开闭矩体, E ⊆ ∪∞ B j (这样的 B j 称为 E
j =1
j =1
的 L− 覆盖)
注意. 1. 若只允许用有限个矩体覆盖,则上述定义对应是 Jordan 容度.
2. 由于任意 E 都可以由 {(−n, n]}n∈N ,L− 覆盖,而非空集合的下确界总是
唯一确定的,故外测度对任意子集 E 皆有定义.
∞ ¯
P ¯
3. 如果 E 的所有 L− 覆盖都使 ¯B j ¯ = ∞,则 m∗ (E ) = ∞
j =1

外测度的性质

1. 非负性,m∗ (E ) ≥ 0, m∗ (;) = 0
2. 单调性,若 E 1 ⊂ E 2 ,则 m∗ (E 1 ) ≤ m∗ (E 2 ) (E 2 的 L− 覆盖总是 E 1 的 L−
覆盖)
¡ ¢ P∞
3. 次可加性,m∗ ∪∞ E ≤
k=1 k k=1 m

(E k )
P∞ ∗ P∞ ∗
Proof. 若 k=1 m (E k ) = ∞,结论显然,故设 k=1 m (E k ) < ∞,任意给定
ε > 0.
© ª ∞ ¯
P ¯
对任一 E k ,存在 L− 覆盖 B k, j 使 E k ⊂ ∪∞ B , ¯B k, j ¯ < m∗ (E k ) + 2−k ε
j =1 k, j
j =1
© ª
故 ∪k E k ⊂ ∪k ∪ j B k, µj ,由于
¶ B k, j 可以重新排成一列 {B c }.故其为 ∪k E k 的
P P ¯¯ ¯ P¡ ∗ ¢ P
L− 覆盖.即有 m∗ ∪ E k ≤ B k, j ¯ < m (E k ) + 2−k ε = m∗ (E k ) + ε
k k j k k
92 CHAPTER 5. 测度
¡ ¢ P∞
由 ε 的任意性,有 m∗ ∪∞ E ≤
k=1 k k=1 m

(E k ).(这里的双序列可以看成单
列,是因为这里的序列顺序无关)

4. 矩体的外测度等于其体积即 m∗ (B) = |B|,B 可为开,闭或半开闭.

Proof. 对于 (a, b] 形状的半开闭矩体 B,可取 B1 = B,B j = ;( j > 1),则


© ª
B ⊂ ∪ j B j ⇒ m∗ (B) ≤ |B|.另一方面,前面引理表明
n o
B 的任一 L 覆盖 Bj 满
P∞ ¯¯ ¯¯ P∞ ¯¯ ¯¯ ∗ ∗
足 |B | ≤ j =1 B j 即应有 | B | ≤ inf j =1 B j = m (B) ⇒ m (B) = |B|
对于 (a, b) 开矩体 B,因为其被 (a, b] 覆盖,故 m∗ (B) ≤ |B|.另一方面
(a, b − δ] 包含于 (a, b),可知 |(a, b − δ]| ≤ m∗ (B).令 δ → 0,|(a, b − δ]| → |B|.
故 m∗ (B) = |B|.对于闭矩体 B [a, b].由 (a, b] ⊂ [a, b] ⊂ (a − δ, b] 并令 δ → 0,
可知 m∗ (B) = |B|.

5. 对于任意集合 E , m∗ (E ) = inf m∗ (O ).O 是包含 E 的开集.

Proof. 显然 m∗ (E ) ≤ m∗ (O ).故只要证 inf m∗ (O ) ≥ m∗ (E ),即对任意给定


的 ε > 0,存在某个开集 O 使 m∗ (O ) ≤ m∗ (E ) + ε。对于 E ,可找到 L− 覆
© ª P ¯¯
¯
盖 B j ,使 B j ¯ <¯ m¯∗ (E ) + 2ε ,将半开闭矩体 B j 的边延长一点点即可得
¯ ¯ ¯ ¯
到开矩体 B0j 满足 ¯B0j ¯ ≤ ¯B j ¯ + 2 jε+1 .令 O = ∪∞ B0 开集,且 E ⊂ O .而
j =1 j
¯
P¯ ¯ P¯ ¯ ¯
m∗ (O ) ≤ ¯B0j ¯ ≤ ¯B j ¯ + 2ε < m∗ (E ) + ε,得证.
© ª
注意. 不可能用闭集从内部逼近,即 m∗ (E ) 6= sup m∗ (F ) , F 闭, F ⊂ E

6. 平移不变性:m∗ (E + { x0 }) = m∗ (E )
© ª
Proof. 首先矩体体积是平移不变的,|B| = |B + { x0 }|.由 E 的 L− 覆盖 B j ⇔
© ª
B j + { x0 } 是 E + { x0 } 的 L− 覆盖可知 m∗ (E + { x0 }) = m∗ (E ).

下面的两个性质表明,外测度对于某些特定的集合满足可加性.
7. 若 d (E 1 , E 2 ) > 0,且 E = E 1 ∪ E 2 .则 m∗ (E ) = m∗ (E 1 ) + m∗ (E 2 ).

Proof. 因 为 E 1 的 L 覆 盖 并 上 E 2 的 L 覆 盖, 总 是 E 的 L 覆 盖, 故
m∗ (E ) ≤ m∗ (E 1 ) + m∗ (E 2 ).
下面证明 m∗ (E 1 ) + m∗ (E 2 ) ≤ m∗ (E ).即将 E 的 L− 覆盖分成两部分,一部分
覆盖 E 1 ,另一部分覆盖 E 2 ,为此选取 δ > 0 但 < d (E 1 , E 2 ).取 E 的一个满足
5.2. R N 空间的 LEBESGUE 测度 93
P ¯¯ ¯
B j ¯ < m∗ (E ) + ε 的 L 覆盖 B j ,将每个 B j 分割成直径小于 δ 的矩体,于是
P
得到新的 E 的覆盖 { I k }.I k 的直径 < δ.且 | I k | < m∗ (E ) + ε.显然 I k 最多
只与 E 1 和 E 2 中的一个相交.令 J1 = { I k , I k ∩ E 1 6= ;},J2 = { I k , I k ∩ E 2 6= ;},
则 J1 ∩ J2 = ;.
P
且 E 1 ⊂ ∪ J1 ,E 2 ⊂ ∪ J2 ⇒ m∗ (E 1 ) + m∗ (E 2 ) ≤ | I k | < m∗ (E ) + ε.得证.

8. 若 E 是可数个内部互不相交的半开闭矩体的并,即 E = ∪∞ B .则 m∗ (E ) =
j =1 j
P∞ ¯¯ ¯¯
Bj
j =1

¯ ¯ ¯ ¯
Proof. 作严格包含于 B j 内部的小矩体 B̃ j 使 ¯B j ¯ ≤ ¯B̃ j ¯ + ε2− j ,
¡ ¢
则 ∀ i, j ,d B i , B j > 0,故任给 N ,有
µ ¶ N N ³¯ ´
N X¯ ¯ X ¯
m∗ ∪ B̃ j = ¯B̃ j ¯ ≥ ¯B j ¯ − ε2− j
j =1 j =1 j =1

PN ¡¯ ¯ ¢ ∞ ¯ ¯
N
由 ∪ B̃ j ⊂ E 知 m∗ (E ) ≥¯B̃ j ¯ − ε2− j .令 N → ∞ 有 m∗ (E ) ≥ P ¯B j ¯ − ε.
j =1
P j=1 ¯¯ ¯¯ P∞ ¯¯j=1¯¯
另一方面总有 m∗ (E ) ≤ ∞j =1 B j ,由 ε 的任意性, m ∗
( E ) = j =1 B j

下面是一些直接计算外测度的例子 (在 R 空间)
1. m∗ (R ) ≥ m∗ ((−n, n]) = 2n → m∗ (R ) = ∞
2. m∗ ({ x}) ≤ m∗ (( x − δ, x + δ]) = 2δ → m∗ ({ x}) = 0
P P
3. m∗ (Q ) ≤ q∈Q m

({ q}) = q∈Q 0 = 0
∗ ∗ ∗ ∗
4. m (R ) ≤ m (R \Q ) + m (Q ) ⇒ m (R \Q ) = ∞

m∗ ((R \Q ) ∩ [0, 1]) + m∗ (Q ∩ [0, 1]) ≥ m∗ ([0, 1]) = 1

⇒ m∗ ((R \Q ) ∩ [0, 1]) ≥ 1 又 m∗ ((R \Q ) ∩ [0, 1]) ≤ m∗ ([0, 1]) = 1


故 m∗ ((R \Q ) ∩ [0, 1]) = 1.

练习
( )
N ¯ ¯
P N
1. 定义 R 上集合 E 的 Jordan 外容度 J ∗ (E ) = inf ¯ I j ¯, E ⊂ ∪ I j , I j 为区间
j =1 j =1
³ ´
证明:对任意集合 E , J ∗ E = J ∗ (E )
2. 举例说明存在开集 O ,使其闭包 Ō 的边界的外测度为正.
94 CHAPTER 5. 测度

3. E ⊂ [a, b],m∗ (E ) = 0,证明 E c 在 [a, b] 中稠密


¡ ¢ © ª
4. E ⊂ R , m∗ (E ) = 0,证明 m∗ E 2 = 0,其中 E 2 = x2 : x ∈ E
5. 若 | f ( x) − f ( y)| ≤ k | x − y|.证明 ∀E ⊂ R , m∗ ( f (E )) ≤ km∗ (E )

5.2.2 不可测集
上面有关 Lebesgue 外测度的性质表明其具有平移不变性及正则性 (单位方体测
度为 1),与测度要求只差可数可加性的要求.下面的例子表明,外测度不可能具
有可加性,无论是可数可加还是有限可加的性质.

不具有可数可加性

给定任一实数 x,可以得到集合 I x = { x + q : q ∈ Q },即有理数集 Q 平移 x 就得到


I x 将所有这些集合放在一起,形成集合类 J = { I x : x ∈ R } = { x + Q : x ∈ R }.
显然,当 x1 − x2 为有理数时 I x1 = I x2
应用选择公理,在 J 中的每个集合如 A ,可取出元素 x A ∈ A ∩ [0, 1](因 A
在 R 中稠密),将所有选出的 x A 放在一起,记为集合 E ,E ⊆ [0, 1].
考察集合 X = ∪ ( q + E ),显然 X ⊆ [−1, 2]
q∈Q ∩[−1,1]
1) X 包含区间 [0, 1].为此任取 y ∈ [0, 1],要证 y ∈ X .首先由 J 定义, y 必
定属于某个 I x ,也就是 I y .这个集合中取出构成 E 的元素 x0 自然也在 I y 中,故
x0 − y 是有理数.因为 y,x0 ∈ [0, 1].x0 − y ∈ [−1, 1].这意味着存在 q ∈ Q ∩ [−1, 1]
使 y = q + x0 ,注意到 x0 ∈ E , y ∈ q + E 即 y ∈ X .
2) 如果 q1 6= q2 ,那么 ( q1 + E ) ∩ ( q2 + E ) = ;,否则 E 中有两个元素, x1 和
x2 满足 q 1 + x1 = q 2 + x2 ,但此时 x1 与 x2 相差为有理数,属于同一 I x .不可能
同时出现于 E ,矛盾.
P
3) 由外测度的平移不变性. m∗ ( q + E ) = m∗ (E ).故 m∗ ( q + E ) =
q∈Q ∩[−1,1]
P
m∗ (E ),又 m∗ ( X ) ∈ [+1, +3],因为 [0, 1] ⊆ X ⊆ [−1, 2]
q∈Q ∩[−1,1]
P P
而 m∗ ( q + E ) 只能为或 ∞.故 m∗ ( X ) 6= m∗ ( q + E )(不可数
q∈Q ∩[−1,1] q∈Q ∩[−1,1]
可加)

不具有限可加性
P
由前可知 1 ≤ m∗ ( X ) ≤ m∗ ( E ) ⇒ m∗ ( E ) > 0
q∈Q ∩[−1,1]
5.2. R N 空间的 LEBESGUE 测度 95

存在正整数 n > 0 使 m∗ (E ) > n1 .设 I 为 Q ∩ [−1, 1] 的一个有限子集,元素


共 3n 个. µ ¶
∗ ∗ P P
若 m 有限可加,则 m ∪ (q + E) = m∗ ( q + E ) = m∗ (E ) > 3 n · n1 = 3,
q∈ I q∈ I q∈ I
µ ¶
但 m∗ ∪ ( q + E ) ≤ m∗ ( X ) ≤ 3,矛盾. m∗ 不是有限可加的.
q∈ I

5.2.3 Lebesgue 可测集


既然外测度不满足可加性,为使其满足可数可加性,必须对集合加以限制.下面
可测性条件实际上是 Caratheodory 提出来的.

定义. E ⊂ R n ,若对 R n 的任意子集 A 都成立 m∗ ( A ) = m∗ ( A ∩ E ) + m∗ ( A ∩ E c ),


则称 E 是 Lebesgue 可测的,在不引起歧义时我们简称可测。如果 E 可测,其
Lebesgue 测度就是 Lebesgue 外测度 m (E ) = m∗ (E ).

[原始定义:E ⊂ [a, b] 可测若 b − a = m∗ (E ) + m∗ ([a, b] \E )]

注意. 1. A = ( A ∩ E ) ∪ ( A ∩ E c ).( A ∩ E ) ∩ ( A ∩ E c ) = ;,即 A 被 E 分成两部分之


后,m∗ 就呈现出可加性.
2. m∗ 有次可加性,故要验证 E 的可测性,只需有 m∗ ( A ) ≥ m∗ ( A ∩ E ) +
m∗ ( A ∩ E c )
3. 前述定义其实并不直观,也不是 Lebesgue 最初的定义方式,一个更为直
观的定义是 E 可测如果 ∀ε > 0,存在开集 O ,使 E ⊂ O 且 m∗ (O \E ) ≤ ε
4. 通常验证某个集合是否可测直接用定义是困难的,事实上我们下面要先
说明可测集合类形成一个 σ 代数.
5. 既然称 m 为测度,就要验证 m∗ 在可测集类上是具有可数可加性的.

定理 5.1. 设 m∗ 为 R n 上的外测度 (事实上我们只用到 m∗ (;) = 0,单调和次


可数可加性条件).令 F = {E ⊂ R n : ∀ A ⊂ R n , m∗ ( A ) = m∗ ( A ∩ E ) + m∗ ( A ∩ E c )} ←
可测集全体,则 F 为 σ 代数,m∗ 在 F 上是测度,具有可数可加性.

Proof. 为证 F 为 σ 代数,先证 ;,R n ∈ F ,再证 F 为对求余集,并运算封闭,


最后证其对单调上升集合列求极限封闭 (相当于代数+单调类)
1. ;,R n ∈ F 是显然的
¡ ¢
2. 若 E ∈ F ,由 m∗ ( A ) = m∗ ( A ∩ (E c )) + m∗ A ∩ (E c ) c 知 E c ∈ F .
96 CHAPTER 5. 测度

3. 若 E 1 ∈ F ,E 2 ∈ F ,即 ∀ A ,
¡ ¢
m∗ ( A ) = m∗ ( A ∩ E 1 ) + m∗ A ∩ E 1c
¡ ¢
= m∗ ( A ∩ E 2 ) + m∗ A ∩ E 2c
¡ ¢
m∗ ( A ∩ (E 1 ∪ E 2 )) + m∗ A ∩ (E 1 ∪ E 2 ) c
¡ ¢ ¡ ¢
= m∗ ( A ∩ (E 1 ∪ E 2 ) ∩ E 1 ) + m∗ A ∩ (E 1 ∪ E 2 ) ∩ E 1c + m∗ A ∩ E 1c ∩ E 2c
¡ ¢ ¡ ¢
= m∗ ( A ∩ E 1 ) + m∗ A ∩ E 2 ∩ E 1c + m∗ A ∩ E 1c ∩ E 2c
¡ ¢
= m∗ ( A ∩ E 1 ) + m∗ A ∩ E 1c
= m∗ ( A ) ⇒ E 1 ∪ E 2 ∈ F

4. 若 {E n } ∈ F ,且 E n ∩ E m = ;, n 6= m,要证 ∪E n ∈ F .
¡ ¢
m∗ ( A ) = m∗ ( A ∩ E 1 ) + m∗ A ∩ E 1c
¡ ¢ ¡ ¢
= m∗ ( A ∩ E 1 ) + m∗ A ∩ E 1c ∩ E 2 + m∗ A ∩ E 1c ∩ E 2c
¡ ¢
= m∗ ( A ∩ E 1 ) + m∗ ( A ∩ E 2 ) + m∗ A ∩ E 1c ∩ E 2c ∩ E 3
¡ ¢
+ m∗ A ∩ E 1c ∩ E 2c ∩ E 3c = · · ·

µ µ ¶c ¶

X
n
∗ ∗ n
m ( A) = m (A ∩ Ek) + m A ∩ ∪ Ek
k=1 k=1
µ µ ¶c ¶
Xn
∗ ∗ ∞
≥ m (A ∩ Ek) + m A ∩ ∪ Ek
k=1 k=1

令 n → ∞,由 m∗ 的次可加性.
µ µ ¶c ¶

X

∗ ∗ ∞
m ( A) ≥ m (A ∩ Ek) + m A ∩ ∪ Ek
k=1 k=1
µ ¶ µ µ ¶c ¶
∗ ∞ ∗ ∞
≥m ∪ (A ∩ Ek) + m A ∩ ∪ Ek
k=1 k=1

⇒ ∪ Ek ∈ F
k=1


5. 令 A = ∪ E k .则由 4 得到
k=1
µ ¶ µ ¶
∗ ∞ X

∗ ∗ ∞ X

m ∪ Ek ≥ m (E k ) ⇒ m ∪ Ek = m∗ ( E k )
k=1 k=1 k=1 k=1

这表明 m∗ 在 F 上具有可数可加性.
5.2. R N 空间的 LEBESGUE 测度 97

注:1) 定理证明中并未用到 R n 空间特殊性质,故该证明事实上可适用于抽


象测度相关的性质 µ µ ¶c ¶
∗ P

∗ ∗ n
2) 第 4 步可直接应用 m ( A ) = lim m ( A ∩ E k ) + lim m A ∩ ∪ Ek ,
n→∞ k=1 n→∞ 1
因为后面两个极限都存在.下面定理说明了可测集类中包含了哪些具体的集合.

定理 5.2. 半开闭矩体 (a 1 , b1 ] × · · · × (a n , b n ] 是 Lebesgue 可测的

Proof. 记 B = (a 1 , b1 ] × · · · × (a n , b n ].要证 ∀ A ,m∗ ( A ) ≥ m∗ ( A ∩ B) + m∗ ( A ∩ B c ).


若 m∗ ( A ) = ∞.显然结论成立.若 m∗ ( A ) < ∞,任给 ε > 0,要证
¡ ¢
m∗ ( A ∩ B ) + m∗ A ∩ B c ≤ m∗ ( A ) + ε
© ª∞ S
∞ ∞ ¯
P ¯
由外测度定义,存在 B j 使 A⊂ Bj 且 ¯B j ¯ ≤ m∗ ( A ) + ε
j =1
j =1 j =1
S
∞ ¡ ¢ P ¡ ¢
因 A∩B ⊂ B j ∩ B ⇒ m∗ ( A ∩ B ) ≤ m∗ B j ∩ B
j =1
[¡ ¢ ¡ ¢ X ¡ ¢
A ∩ Bc ⊂ B j ∩ B c ⇒ m∗ A ∩ B c ≤ m∗ B j ∩ B c

注意到 B j ∩ B 是半开闭矩体,B j ∩ B c 则可由有限个不相交的半开闭矩体并


¡ ¢ ¯ ¯ ¯ ¯ Sn
成 (半开闭矩体全体是半环),可知 m∗ B j ∩ B = ¯B j ∩ B¯,¯B j ∩ B c ¯ = 1 Di ⇒

¡ ¢ P
n
m Bj ∩B c
= |D i |
1
又因 B j 是由不相交的半开闭矩体 B j ∩ B 和 {D i }ni=1 并成.故
¯ ¯ ¯ ¯ n
¯B j ¯ = ¯B j ∩ B ¯ + P | D i |
1
¡ ¢ X¡ ∗ ¡ ¢ ¡ ¢¢
m∗ ( A ∩ B ) + m∗ A ∩ B c ≤ m B j ∩ B + m∗ B j ∩ B c
X¯ ¯
= ¯B j ¯ ≤ m∗ ( A ) + ε

注意. 由于半开闭矩体 B = (a, b] 所生成的 σ 代数即 R n 中的 Borel 集.故前两


个定理表明 R n 中的 Borel 集都是 Lebesgue 可测的.自然,所有开集,闭集也都
是可测的.但 Lebesgue 可测集并不局限于 Borel 集.这个将在后续学习中提到.

练习

1. 证明 R 的子集 B 是 Lebesgue 可测的,当且仅当对于 R 中任意开区间 I ,


m∗ ( I ) = m∗ (B ∩ I ) + m∗ (B c ∩ I )
98 CHAPTER 5. 测度

5.2.4 单调类定理和 Carathedory 扩张定理


回忆单调类定理:C 为一集合类,若 C 为 π 类,则 C 生成的最小 λ 类= C 生
成的最小 σ 代数

定理 5.3. (Ω, F ) 为可测空间,C 为 Ω 上的 π 类,F = σ (C ).若 µ 和 v 是 C


上的有限测度,且 ∀C ∈ C ,µ (C ) = v (C ),µ (Ω) = v (Ω),则在 F 上,µ = v.
© ª
Proof. 令 G = A ∈ F : µ ( A ) = v ( A ) ,则 C ⊂ G ,我们证明 G 是 λ 类.
显然 Ω ∈ G ,若 A ,B ∈ G , A ⊂ B.
则 µ (B − A ) = µ (B ) − µ ( A ) = v (B ) − v ( A ) = v (B − A ) ⇒ B − A ∈ G
若 A n ∈ G , A n ↑ A 则 µ ( A ) = lim µ ( A n ) = lim v ( A n ) = v ( A ) ⇒ A ∈ G
由单调类定理,F = σ (C ) ⊂ G ⇒ F = G ,得证.

定理 5.4 (Carathedory 测度扩张定理). 设 C 为 Ω 上的一个半环,µ 为 C 上的


可数可加非负集函数,µ (;) = 0,则 µ 可以扩张成 σ (C ) 上的测度.若 µ 在 C
上 σ 有限,且 Ω 可表示为 C 中可数个集合的并,则扩张是唯一的.且扩张所
得的测度在 σ (C ) 上也是 σ 有限的.

Proof. 类似 Lebesgue
½ 测度构造,对任意 A ⊂ Ω
¾ 构造外测度
P
∞ S

µ∗ ( A ) = inf µ ( An) : An ∈ C , A ⊂ An ,
n=1 n=1
称 A 是 µ∗ 可测,即 ∀D ⊂ Ω,µ∗ (D ) = µ∗ ( A ∩ D ) + µ∗ ( A c ∩ D ),可证 µ∗ 在
µ∗ 可测集类上为测度,µ∗ 可测集类包括 σ (C ):C 中集合 µ∗ 可测且 µ∗ 可测集
形成 σ 代数.
(唯一性证明) 设 µ 在 C 上 σ 有限,Ω 可表示为 C 中可数个集合的并.由
P
C 为半环,知存在 Ω 的可数划分 { A n } 使 Ω = A n , A n ∈ C ,µ( A n ) < ∞,若 µ1
T T
与 µ2 是在 σ (C ) 上的两个测度扩张,则因为 A n C 是 π 类,且 A n C ⊂ C .由
P T
上一定理,可知 µ1 与 µ2 在 σ ( A n ∩ C ) 上一致.∀B ∈ σ (C ) 有 B = (B A n ),故
P T P T P T
µ1 (B) = µ1 ( (B A n )) = µ1 (B A n ) = µ2 (B A n ) = µ2 (B),得证.

注意. 1. 定理表明 Lebesgue 测度是 Borel 集上唯一使矩体测度等于体积的测度


2. 定理中的 σ 有限性不能去掉,比如 R 上半开闭区间形成的半环上,定义
( (
0 A=; 0 A=;
m0 ( A ) = ,则 m 1 ( A ) = 仍为 B (R ) 上的测度
∞ A 6= ; ∞ A 6= ;
但 m 2 ( A ) = A 中元素个数也是 m 0 在 B (R ) 上的测度扩张.
5.2. R N 空间的 LEBESGUE 测度 99

3. C 是半环的条件不能少的.比如 X = {1, 2, 3, 4},B = 2 X ,C = {{1, 2} , {1, 3}},


则 B = σ (C ).设 µ { j } = 1/4( j = 1,2,3,4),α {1} = α {4} = 1/6,α {2} = α {3} = 1/3.
则在 C 上,α {1, 2} = µ {1, 2} = 1/2,α {1, 3} = µ {1, 3} = 1/2.

5.2.5 Lebesgue 可测集与 Borel 集


定理 5.5. 若集合 E 的 Lebesgue 外测度 m∗ (E ) = 0,则 E 及 E 的任意子集都是
可测的.

Proof. 只需证 E 可测.因为 E 的子集外测度也为,∀ A ,m∗ ( A ∩ E ) ≤ m∗ (E ) = 0


又 m∗ ( A ) ≥ m∗ ( A ∩ E c ),故 m∗ ( A ) ≥ m∗ ( A ∩ E ) + m∗ ( A ∩ E c ),得证.

注意. 由于 Cantor 集外测度为,故其可测,而且其势为 c.意味着可测集的基数


至少为 2 c .而 R 的全体子集的基数也为 2 c .表明可测集的基数就是 2 c ,于是无
法从基数角度说明是否存在不可测集.
现在我们基本上知道了 Lebesgue 可测集的构成:即包含了所有 Borel 集,
也包括所有测度为的 Borel 集的子集。我们进一步考察 Lebesgue 可测集与 Borel
集之间的关系.
1.存在非 Borel 集的 Lebesgue 可测集
需要用到一个定理,即

N
定理. 若 C 为 X 的子集族且 ; ∈ C ,C ≥ 2,F = σ (C ).则 F ≤ C

N
于是因 R 上的 Borel 集可由有理端点的区间生成.故其基数 ≤ N = R ,而
Lebesgue 可测集的基数是 2R ,故必有非 Borel 集的 Lebesgue 可测集.
2.若 E ⊂ R n 是 Lebesgue 可测的,则
© ª © ª
m (E ) = inf m (U ) : U 开, E ⊂ U = sup m (K ) : K 紧, K ⊂ E
© ª
注意. 1. 虽然紧集改为闭集结论也对,但所表示的意思不同,即 K : K 紧, K ⊂ E
© ª
与 K : K 闭, K ⊂ E 不等价,比如 E = Q ;2. 前面类似的定理针对的是外测度和
任意集合;3. 有界集合的 Lebesgue 测度有限,但若集合测度有限未必是有界的

Proof. 前一部分业已证明,只需证后半部分。先设 E 有界,且可以找到有界闭集


C,使 E ⊂ C ,对于 C −E 找开集 U 使 C −E ⊂ U ,m (U ) < m (C − E )+ε,令 K = C ∩U c ,
100 CHAPTER 5. 测度

K 闭且有界,故 K 紧且 K ⊂ E . m (C ) ≤ m (K ) + m (U ) < m (K ) + m (C − E ) + ε ⇒
m (E ) − ε ≤ m (K ) 由 ε 的任意性,知 E 有界时结论成立.
¡ ¢
再设 E 无界时,定义 B j = [− j, j ]n ,E j = E ∩ B j ,则 m (E ) = lim j m E j ,且
E j 有界 [将无界集合化为有界集合的常用手段].
¡ ¢
若 m (E ) < ∞,∀ε > 0,可找到 j 使 m E j ≥ m (E ) − 2ε ,及紧集 K ⊂ E j 使
¡ ¢
m (K ) ≥ m E j − 2ε ,即有 m (K ) ≥ m (E ) − ε
¡ ¢
若 m (E ) = ∞,∀b > 0,有某个 j 使 m E j > b 及紧集 K ⊂ E j ,使 m (K ) > b.
得证.

[若只限于 Borel 集合,可以利用单调类定理进行证明,只需说明开矩体具


有性质,然后具有性质的全体是 λ 类]
注意. 1. 上述定理说明了 Lebesgue 测度是正则的.

定义 (正则的定义). A 为 R n 中包含 Borel 集的 σ 代数,其上的测度称为正


则测度.如果
(a) R n 的紧子集 K 的测度 µ (K ) < ∞
© ª
(b) A 中任意集合 A 满足 µ ( A ) = inf µ (U ) : A ⊂ U,U 开
© ª
(c) 任意开集 U 满足 µ (U ) = sup µ (K ) : K ⊂ U, K 紧

定理. 在完备可分的距离空间里,任意有限的 Borel 测度都是正则的。

2. 由定理不难得到下面的推论,即

推论. 对任意可测集 E ,任给 ε > 0.存在包含 E 的开集 U ,使 m (U \E ) < ε;


存在包含于 E 的闭集 K ,使 m (E \K ) < ε。

因为取 E = R n ,K 为紧集时,总有 m (E \K ) =
其中后者的闭集不能改成紧集.

Proof. 前者在 m (E ) < ∞ 时,可直接找 m (U ) < m (E ) + ε,而在 m (E ) = ∞


S

时,可将 E 表示成 D i ,其中 m (D i ) < ∞(因 R n 上 Lebesgue 测度是 σ 有
i =1
限的).
S∞
找 U i 满足 D i ⊂ U i . m (U i ) < m (D i ) + 2− i ε,则 i =1
U i 即为所求.

3. 若假定可测集 E 的测度有限,可进一步得到下面的推论.
5.2. R N 空间的 LEBESGUE 测度 101

推论. m (E ) < ∞,则存在紧集 K ,K ⊂ E 且 m (E \K ) ≤ ε.也存在有限个闭


Sn
矩体 B j ( j = 1,…, n) 使得 m (E ∆F ) ≤ ε,其中 F = j =1
B j.
© ª∞
Proof. 第一条性质是显然的,第二条性质因为总有 E 的闭矩体覆盖 B j j =1
S ∞ ¯
P ¯ ∞ ¯
P ¯
满足 E ⊂ Bj ¯B j ¯ ≤ m (E ) + ε .由数列的收敛性知存在 n 使 ¯B j ¯ < ε .
2 2
j =1 j = n+1
Sn
令 F= j =1
Bj 有
à ! à !
[
∞ [

m (E ∆F ) = m (E \F ) + m (F \E ) ≤ m Bj +m Bj −E
j = n+1 j =1
X
∞ ¯ ¯ X∞¯ ¯
≤ ¯B j ¯ + ¯B j ¯ − m ( E ) ≤ ε
n+1 1

这表明任意的可测集合基本上是一些区间 (矩体) 的并.


4. 若不仅限于开集和闭集的逼近方式,可以证明 Lebesgue 可测集实际上只与
Borel 集差一个零测集.更确切的说,

定理. 对于任意集 E ,E 是可测的当且仅当:


(a)E 可以表示成 E = H \ Z1 ,H 是 G δ 集,m ( Z1 ) = 0
(b)E 可表示成 E = K ∪ Z2 ,K 是 Fσ 集,m ( Z2 ) = 0

Proof. 若 (a)(b) 成立,E 的可测性显然,反之,由 E 可测,∀k,可作包含 E


± ±
的开集,G k 及包含于 E 的闭集 F k ,满足 m (G k \E ) < 1 k, m (E \F k ) < 1 k.
T
∞ S

取 H= G k ,K = F k .即为所求.
k=1 k=1

由此结论结合紧集的连续象还是紧集的论断容易推得:若 T : R n → R l 为连
续映射,E ⊂ R n 是 Lebesgue 可测集,则 T (E ) 也是 Lebesgue 可测的。因为
T 将零测集映为零测集,将 Fσ 集映为 Fσ 集。注意,若 E 是 Borel 集,则
T (E ) 未必为 Borel 集。
5. 这里提到的诸多可测集的性质也可以作为 Lebesgue 可测集的定义,即这些
性质的逆命题也成立.以下是部分常见的逆命题,大家可尝试证明等价性。
设 E 为 R n 的子集,任给 ε > 0
102 CHAPTER 5. 测度

(a) 若存在开集 O ,使 E ⊂ O , m∗ (O \E ) < ε.则 E 可测


© ª
(b) m∗ (E ) < ∞ 且 m∗ (E ) = sup m (F ) , F ⊂ E, F 是紧集 ,则 E 可测
(c) 存在闭集 F 和开集 G 使 F ⊂ E ⊂ G ,且 m∗ (G \F ) < ε,则 E 可测
(d) 存在闭集 F ,使 F ⊂ E , m∗ (E \F ) < ε,则 E 可测
[思路:可证明满足上述条件的集合也组成 σ 代数,再说明 Borel 可测]

练习

在前面我们考察了 R 上的 Lebesgue 测度, m (R ) = ∞,但 R 上也可能有其它的


测度,这个练习旨在帮助大家回顾测度的构造过程.
1. µ 是可测空间 (R, B (R )) 上的测度,定义 Fµ : R → R ,Fµ ( x) = µ ((−∞, x]),
µ ( R ) < ∞.
证明:Fµ 有界,单调增,右连续, lim Fµ ( x) = 0
x→−∞
2. 若 F : R → R 是有界,单调增,右连续且 lim F ( x) = 0 的函数,证明存在测
x→−∞
度 µ,定义在 (R, B (R )) 之上,满足½F ( x) = µ ((−∞, x]),∀ x ∈ R ,且 µ (R¾
) < ∞.
P
∞ S

提示:定义如下列测度 µ∗ ( A ) = inf (F ( b n ) − F (a n )), A ⊂ (a n , b n ]
n=1 n=1

5.2.6 测度空间的完备化
前面说明了任一 Lebesgue 可测集只与 Borel 集只差一个零测集.这实际上说明
了 Lebesgue 集是 Borel 可测度空间完备化后得到的,为此引入如下定义.
© ª
定义. :测度空间 Ω, F , µ 是完备的,如果任意 A ∈ F ,µ ( A ) = 0( A 称为零测
度),则 A 的所有子集都属于 F .

注:显然零测集的任意子集的测度也为 0.
¡ ¢ ³ ´
定理 5.6. 任意测度空间 Ω, F , µ 存在包含它的最小的完备测度空间 Ω, F , µ̄ ,
满足 F ⊆ F ,µ̄ |F = µ.
© ¯ ª
Proof. 令 C = A ¯ A ∈ F , µ ( A ) = 0 ,C = {B |B ⊆ A, A ∈ C },
© ¯ ª
F̄ = Ā ¯ Ā = ( A \ N1 ) ∪ N2 , A ∈ F , N1 ∈ C¯ , N2 ∈ C¯ ,µ̄ (( A \ N1 ) ∪ N2 ) = µ ( A )
1. 在 F 定义中取 A = ;, N1 = ; 可知 C ⊂ F 即 F 含有所有 F 中零测
集的子集.若另有 B ∈ F ,µ̄ (B) = 0 且 B ∉ F .则 B = ( A \ N1 ) ∪ N2 ,µ ( A ) = 0.
S
B 的子集 B0 也可表示成 B1 ∪ B2 的形式.其中 B1 = ( A \ ( N1 N2 )) ∩ B0 ,B2 =
5.2. R N 空间的 LEBESGUE 测度 103

T
N2 B0 ⇒ B2 ∈ C¯ ,因 N1 ∪ N2 也是某个零测集的子集即 N1 ∪ N2 ∈ C¯ ,改写 B1 ,
有 B1 = A \ ( N1 ∪ N2 ∪ ( A \B0 )).
因 N1 ∪ N2 ∪ ( A \B0 ) ∈ C¯ ,故 B0 = A \ ( N1 ∪ N2 ∪ ( A \B0 )) ∪ B2 ∈ F ⇒ F 包含
所有 µ̄ -零测集的所有子集.
S
2. F 为 σ 代数,;、Ω ∈ F 显然.若 B ∈ F ,B = ( A \ N1 ) N2 ,则
¡ ¢[
B c = A c \ N2 ( N1 \ N2 ) ∈ F

S
若 B1 ∈ F ,B2 ∈ F ⇒ B1 ∪ Bµµ2 = (( A 1¶∪ A 1∪N
µ 2 ) \ ( M¶¶ ¶ 2 ∪ N2 )
µ 1 )) ( M
S
∞ S
∞ S
∞ S

若 {B n } ∈ F ⇒ Bn = An \ Nn ∪ Mn
¡n=1 ¢ n=1 n=1 n=1
3. 注意包含 Ω, F , µ 的完备测度空间必然包含 F ,以及 C¯ ,显然 F ⊂
¡ ¢ ¡ ¢
σ F ∪ C¯ 故只能有 F = σ F ∪ C¯

推论. (R n , L (R )n , m) 是 (R n , B (R )n , m) 的最小完备化测度空间.

Proof. 因为任意 Lebesgue 可测集 A ,存在 Borel 集 B∗ ,B∗ ,使 B∗ ⊂ A ⊂ B∗ .


m (B ∗ \B ∗ ) = 0
于是 A = (B∗ ∪ N1 ) \ N2 ,其中 N1 和 N2 是 B∗ \B∗ 的子集.

练习

1. 考虑测度空间 ([0, 1] , L [0, 1] , m),定义 d ( A, B) = m ( A ∆B), A 、B ∈ L [0, 1].


如果将 m ( A ∆B) = 0 的两个集
³
A 和 B 都视为同一个集.试证 ´ (L [0, 1] , d ) 是
R
一个完备的距离空间. m ( A ∆B) = [0,1] |1 A − 1B | dm
[提示:对于 Cauchy 列 { A n },考虑 lim sup A n ]
n→∞
2. 举例:R 中紧集,测度为正但是没有内点。
3. 假定 E 1 和 E 2 是 R n 中的紧集,E 1 ⊂ E 2 ,证明对任意 a,若满足 m(E 1 ) <
a < m(E 2 ),则存在紧集 E ,满足 E 1 ⊂ E ⊂ E 2 , m(E ) = a。
4. 假定 A ⊂ R 2 是连续函数 y = f ( x) 的图像,证明 m( A ) = 0。

5.2.7 可测集与矩体
人们可能会想知道正测度集 A ( A c 也是正测度集) 是否可以很均匀的分布于整个
±
空间,即对某个 r ,0 < r < 1. 满足 m ( A ∩ J ) m ( J ) = r .
对任意区间 J ,0 < m ( J ) < ∞.
104 CHAPTER 5. 测度

下面定理说明这个是不可能的,即肯定有某个区间中 A 比较多,有些区间 A 的
点比较少.

定理 5.7. 设 E 是 R 中 Lebesgue 可测集, m (E ) > 0,那么 ∀ε > 0,存在某个有


界区间 J ,使得 m (E ∩ J ) > (1 − ε) m ( J )
P
Proof. 设 m (E ) < ∞,ε < 1.取 Jn = (a n , b n ],使 E ⊂ ∪∞
n=1 Jn ,且 m ( Jn ) ≤ m(E)
1−ε
P P P n≥1
故 m (E ) ≤ m (E ∩ Jn ) ⇒ (1 − ε) m ( Jn ) ≤ m ( E ∩ Jn )
n≥1 n≥1 n≥1
⇒ 至少有某个 n 使得 (1 − ε) m ( Jn ) ≤ m ( Jn ∩ E )

我们知道在 R 上,无理数的测度为正,但它不包含任意区间 [当然包含区间


的集合测度应当为正],但是任意正测度集的差必然包含区间.

定理 5.8 (Steinhaus). 对于任意 R 中可测集 E , m (E ) > 0,存在 ε > 0,使

E − E := { x − y : x, y ∈ E } ⊃ [−ε, ε]

Proof. 由前述定理,取区间 J 使 m (E ∩ J ) > 3m ( J )/4.令 ε = m ( J )/2.那么若


| x| ≤ ε,
(E ∩ J ) ∪ ((E ∩ J ) + x) ⊂ J ∪ ( J + x) 及 m ( J ∪ ( J + x)) ≤ 3 m ( J )/2
由于 m ((E ∩ J ) + x) = m ((E ∩ J )) ⇒ ((E ∩ J ) + x) ∩ (E ∩ J ) 6= ;,否则
3 3
2 m( J ) ≥ m (((E ∩ J ) + x) ∪ (E ∩ J )) = 2 m(E ∩ J ) > 2 m( J ),矛盾
⇒ x ∈ (E ∩ J ) − (E ∩ J ) ⊂ E − E .

下面例子表明,零测集的差集也可能包含区间.即对于 [0, 1] 上的 Cantor 集


∆.有 ∆ - ∆ = [−1, 1]

Proof. (Steinhaus):只需证明 y − x = b 对任意 b ∈ [−1, 1] 有 ∆ 中的解,即 y = x + b


与 ∆ × ∆ 相交.
右图给出了如何得出 ∆ × ∆ 的过程 ∆ × ∆ = lim A n

由图中可以看出 y = x + b 总与 A n 相交,两者交集为闭集.注意到 A n ⊃ A n+1 .


由闭集套定理可知 ∆ × ∆ 与 y = x + b 有交点,得证.
5.3. 可测集的 LIPSCHITZ 变换 105

注意. 这一结果表明即使 R n 中的两个集合 A 和 B 都是零测集,他们的和


(Minkowski 和) A + B = { x + y : x ∈ A, y ∈ B} 的测度 (假定 A + B 可测) 可以任意
大。事实上我们有这样的不等式 (Brunn Minkowski 不等式):对于 R n 中的任
1
意非空 Borel 集合 A 和 B,(可以证明此时 A + B 可测),都有:(m( A + B)) n ≥
1 1
( m( A )) n + ( m(B)) n

练习

1. 证明:在 R n 中,若 A 或 B 开,则 A + B 开;若 A 与 B 都闭,则 A + B 为


Fσ 集。
2. 在 R 中,若 Lebesgue 可测集 E 满足:对某个处处稠密集中的任意点 x,总
有 m(E ∆(E + x)) = 0,则 m(E ) = 0 或者 m(E c ) = 0
3. µ 是完备可分距离空间 X 中 Borel 集合的测度,且 µ( X ) = 1,则存在子集 E
可以表示为可数紧集的并,且 µ(E )=1

5.3 可测集的 Lipschitz 变换


我们已经说明在欧氏空间中平移变化并不影响集合的可测性,现在进一步考虑
其他的变换。
Cantor 函数可以帮助我们构建一个可测集的连续像不可测的例子。
但是如果变换是 Lipschitz 的,则可测性能够保持。

定理 5.9. 如果 y = T ( x) 是 R n 到自身的 Lipschitz 变换,则 T 将可测集映为


可测集

Proof. 注意到 R n 中任意闭集可表示为可数紧集的并的形式,任意闭集在 T 下


的象是 Fσ 集。故 Fσ 集在 T 下的象也是 Fσ 集。
而由 |T x − T y| ≤ c| x − y|,容易证明 0 测集在 T 下的象也是零测集。
由前面结果,任意可测集都是 Fσ 集合并上一个零测集,得证。

作为特例,如果 T 是线性变换,即 T 可表示为 n × n 的矩阵时,变换后的


集合的测度也能很快得出。

定理 5.10. 如果 y = T ( x) 是 R n 到自身的线性变换,E 是可测集,则 m(T (E )) =


| det T | m(E )
106 CHAPTER 5. 测度

Proof. 首先注意任意矩体 B 满足 |T (B)| = | det T ||B|,这可以从线性变换总是以


下三种基础变换的复合得到。
1. ( x1 , x2 , · · · ) 7→ (δ x1 , x2 , · · · )
2. ( x1 , x2 , · · · ) 7→ ( x1 + xd , x2 , · · · )
3. 任两个坐标互换
P
这样,考察 E 的 L-覆盖 {B j }, |B j | ≤ m(E ) + ϵ,同时有 T (E ) ⊂ ∪T (B j ),
P P
m(T (E )) ≤ m(T (B j )) = | det T | |B j | ≤ | det T | m(E ) + | det T |ϵ
由 ϵ 的任意性结论得证。

5.4 测度与集合列的极限
前面已经证明单调集列的测度与极限运算可以交换,即有测度的单调收敛定理,

定理 5.11. 对可测集列 {E n }
若 E n ↑ E .则 limn→∞ m (E k ) = m (limn→∞ E n )
若 E n ↓ E . m (E 1 ) < ∞,则 limn→∞ m (E k ) = m (limn→∞ E n )
而对于一般的可测集合列.只能有
µ E n ) ≤ ¶lim infn→∞ m ³(E k ) (Fatou
m (lim infn→∞ 引理集合形式)
S
∞ ´
如果还有 m E n < ∞,则 m lim E n ≥ lim m (E n )
n=1 n→∞ n→∞
µ ¶ µ ¶
S
∞ T
∞ T

Proof. 只证 Fatou 引理.m E j = lim inf m E j ≤ lim inf m (E n )
n=1 j = n n→∞ j=n n→∞
R
注意. 1. 对于集合 E ,总对应示性函数 1E ,容易想象应该有 1E dm = m(E ),
于是集合的收敛可以对应示性函数的点点收敛,测度的收敛就对应积分的
收敛,由可测集合列的性质可以大致知道可积函数列的一些性质
2. 对于概率测度 P ,因为总有限,故有:
³ ´
P (limn→∞ E n ) ≤ limn→∞ P (E n ),P limn→∞ E n ≥ limn→∞ P (E n )

P
∞ ³ ´
引理 (Borel-Cantelli lemma). 若有可测集列 {E n }. m (E k ) < ∞.
则 m limn→∞ E n =
n=1
0.
³ ´ µ ¶
S
∞ P

Proof. m limn→∞ E n = limk→∞ m E i ≤ lim m (E i ) = 0
i=k k→∞ i = k

可应用于周民强《实变函数论》习题 2 第一组第 9 题。
5.5. 补充 107

3. 在概率空间中 P (E ) = 1,说明 E 几乎一定发生.习题 2 第一组第 8 题表明,


¡ ¢
如果 ∀n ∈ N ,P (F n ) = 1,那么 P ∩∞
n=1 F n = 1,即若 F n 几乎一定发生,那
么所有 F n 都发生的概率也为 1.但这只对可数个事件成立,如果是不可数
个事件同时发生,就不一定成立.
© ª
考虑掷硬币无穷次形成的空间 Ω = (ω1 , ω2 , · · ·) , ω i = 正或者反
¡ © ª¢
F = σ {ω ∈ Ω, ωn = W } : n ∈ N, ω ∈ 正, 反 [为什么这样定义呢?]
n o
# k≤ n:ωk =正}
F = ω: { n → 1 1
2 朝上比例逼近 2 的事件.F ∈ F [为什么是成立的?
]
令 A 代表映射 α: N → N ,α (1) < α (2) < · · · 的集合
n o
# k≤ n:ωα(k) =正}
任给 α ∈ A ,定义 Fα = ω : { n → 12
T
注意 P (Fα ) = 1,但 Fα = ;
α∈ A
要说明:1. A 中元素不可数 (对角线方法)
T
2. 若有 ω ∈ Fα ,则 ω 中朝上比例不可能逼近 12 .事实上,由 ω ∈ Fα1 知
α∈ A
ω 中约有一半朝上 [取 α1 ( n) = n],考虑朝下的那部分指标,肯定也有对应的
α2 (1) < α2 (2) < · · · ,其中也应有一半朝上,矛盾.
或者,取 ω 中朝上的序号作为 α,则对应的事件中向上的比例恒为 1,不可
能趋向 1/2。

5.5 补充

5.5.1 测度的扩张
在前面已经讨论了如何从半环上的可数可加集函数开始构造半环生成的 σ 代数
上的测度,按照类似方法不难从代数开始构造测度。基本步骤有三:
1. 定义外测度
2. 利用 Caratheodory 条件界定可测集,说明可测集合形成 σ 代数,且外测度
在可测集上可数可加
3. 说明代数里的集合也是可测集
不难发现,扩张中的前提是集函数在半环或代数上就是可数可加的,但这一
条件并不容易验证,往往有限可加是容易成立的,下面给出两个定理,说明何时
有限可加可以加强为可数可加。
108 CHAPTER 5. 测度

定理 5.12. R 为半环,µ 在 R 上非负有限可加,若存在集合类 M ⊂ R 使得:


M n ∈ M 非空递减,∩∞ M 非空。且 ∀R ∈ R , µ(R ) = sup{µ( M ) : M ∈ M , M ⊂ R },
i =1 i
则 µ 可数可加

定义. X 的子集类 K 称为紧类,若对任意交集为空的集合序列 {K n }∞


1 ⊂ K ,总
存在 N ,使得 ∩1N K n = ;

定理 5.13. A 为代数,µ 在 A 上非负有限可加,若存在紧类 K , 对任意 A ∈ A ,


ε > 0,存在 K ε ∈ K , A ε ∈ A ,使得 A ε ⊂ K ε ⊂ A, µ( A \ A ε ) < ε,则 µ 可数可加。
特别的,如果 K ⊂ A ,且 ∀ A ∈ A ,µ( A ) = supK ⊂ A,K ∈K µ(K ),则 µ 可数可加。

5.5.2 Bernoulli 测度
这里补充说明如何严格定义可数次抛硬币形成的空间里的概率测度。
令 X = {0, 1}N ,可以看成是可数次抛硬币形成的空间。给定非空集合 S ∈ N,
可知对应那些次抛硬币形成了空间 X S = {0, 1}S 。显然,AS = { A ∈ X : ΠS ( A ) ⊆ X S }
()ΠS 为投影映射)
,代表已知 S 对应那些次抛硬币结果的集合,是一个 σ 代数。
假如每次抛硬币独立,向上概率为 p,很容易对有限的 S 定义 AS 中集合的概
率,而且 A = ∪{AS : S < ∞} 是一个代数,但并非 σ 代数。利用前述定理证明在
A 上有可数可加的概率,再将其扩张成为 σ(A ) 上的测度。
第 6 章 可测函数

回忆 Lebesgue 积分需要计算集合 f −1 (( yn , yn+1 ]) 的测度,但是前面的学习表


明,并非所有集合都是 Lebesgue 可测的,这意味着并不是所有函数都可以求
f −1 (( yn , yn+1 ]) 的测度,可以求其测度的函数即可测函数。虽然并非所有函数都
是 Lebesgue 可测的,它已经包含了一大类函数,足够应付平常的应用.
一个需要可测性的例子,在统计中,我们往往需要通过观测到的样本数据
{ X i (θ )} 估计参数 θ ,通常假定 θ 为未知固定参数(Bayesian 方法假定其随机),
而统计量表示为样本数据的函数即 T ( X 1 , X 2 , . . . , X n ),为分析统计量的性质,需
要有 T 的可测性。若 T 的形式可以解析表达,可测性通常较容易验证,否则可
能是困难的。如极大似然估计或更一般的 M 估计量,

T = argmax f (θ ; X 1 , X 2 , . . . , X n )

相当于给定参数的最优化,需要有最优值关于参数的可测性(有相应的连续函
数的 Maximum Theorem)

6.1 映射的可测性
¡ ¢
定义. (可测映射) 令 Ω, Φ, µ 为测度空间(只需要可测空间 (Ω, Φ),无需引入
测度)
1. (E, E ) 为另一可测空间,映射 f : Ω → E 称为 (Φ, E ) -可测的 (简称为可测
的),如果对一切 A ∈ E , f −1 ( A ) ∈ Φ
³ ´
2. R = R ∪ {−∞, ∞},B (R ) 和 B R 表示 R 和 R 上的 Borel σ -代数
³ ³ ´´
若 f : Ω → R 是 (Ω, Φ) 到 R, B R 的可测映射,称 f 为 Φ 可测函数
若 f : Ω → R 是 (Ω, Φ) 到 (R, B (R )) 的可测映射,称 f 为 Φ 实值可测函数

109
110 CHAPTER 6. 可测函数
³ ³ ´´
3. 若 f : R n → R 是 (R n , B (R n )) 到 R, B R 的可测映射,称 f 为 Borel 可
测函数
³ ´´ ³
n n n
若 f : R → R 是 (R , L (R )) 到 R, B R 的可测映射,称 f 为 Lebesgue
可测函数.

[如何考虑 f : Ω → R m ?]

性质 (如何判断映射可测). :如果 E = σ (C ),则 f : Ω → E 是 (F , E ) -可测的 ⇔


对所有 A ∈ C , f −1 ( A ) ∈ F .
© ª
Proof. : A ⊂ E : f −1 ( A ) ∈ F 是含 C 的 σ 代数,故也包含 E .

推论. 如果 f : R n → R 是连续函数,那么 f 是 Borel 可测的,也是 Lebesgue


可测的

Proof. U 在 R 中开,则 f −1 (U ) 开.

下列表述等价, f : Ω → R 是 Φ 可测的 ⇔∀α ∈ R , f −1 ((a, ∞]) ∈ Φ


⇔ ∀a ∈ R , f −1 ([a, ∞]) ∈ F ⇔ ∀a ∈ R, f −1 ([−∞, a)) ∈ F
⇔ ∀a ∈ R , f −1 ([−∞, a]) ∈ Φ

定义. (Ω, Φ) 可测空间,f 是 Ω 上的函数,E ∈ Φ,称 f 在 E 上可测,如果对任意


Borel 集 A ,f −1 ( A )∩ E ∈ Φ,或等价于说 f |E 是 nΦE 可测,其中 Φ³ E ´o
= { A ∩ E : A ∈ Φ}
³ ³ ´´
若只给定空间 Ω,f : Ω → R ,通过定义 F = f −1 ( A ) : A ∈ B R = f −1 B R ,
可知 Φ 为 σ 代数,且 f 是 Φ -可测的,换言之,Φ 也是使 f 可测的 σ 代数,
称 Φ 为 f 生成的 σ 代数.

定义. (与概率的关系):设 (Ω, Φ, P ) 为概率空间,Ω 上的 Φ 实值可测函数 X 称


¡ ¢
为 (实值) 随机变量. X : (Ω, F ) → (R, B ),∀ A ∈ B ,定义 P X ( A ) = P X −1 ( A ) =
P ({ω : X (ω) ∈ A }),称 P X 为 X 的概率分布,F ( x) = P X ((−∞, x]) 称为 X 的分布
函数.

6.1.1 复合映射的可测性
定理 6.1 (复合映射的可测性). 设 f 是 (Ω, F ) 到 (E, E ) 的可测映射,g 是 (E, E )
到 (G, Γ) 的可测映射,则 g ◦ f 是从 (Ω, F ) 到 (G, Γ) 的可测映射.
¡ ¢
Proof. ∀ A ∈ Γ, g−1 ( A ) ∈ E , f −1 g−1 ( A ) = ( g ◦ f )−1 ( A ) ∈ F
6.1. 映射的可测性 111

推论. 若 X = ( X 1 , · · · , X n ) 是 (Ω, Φ) 到 (R n , B (R n )) 上的可测映射,h 是 R n 到 R


的实值 Borel 可测函数,那么 f = h ◦ X = h ( X 1 , X 2 , · · · , X n ) 是 (Ω, Φ) 到 (R, B ) 的
可测函数.

±
推论. 若 f 1 ,f 2 是 (Ω, Φ) 上的实值可测函数,则 c 1 f 1 + c 2 f 2 ,f 1 · f 2 ,f 1 f 2 ( f 2 处
处不为 0),max ( f 1 , f 2 ),min ( f 1 , f 2 ),| f | = max ( f , 0) + max (− f , 0) 都可测.

Proof. h ( f 1 , f 2 ) = c 1 f 1 + c 2 f 2 ,h : R 2 → R 是连续函数,其它类似.

注意. 若 f ,g 都是 Lebesgue 可测的实值函数,复合函数 f ◦ g 并不一定 Lebesgue


可测,即使 g 连续,但若 f Borel 可测,g 为 Lebesgue 可测,则 f ◦ g 是 Lebesgue
可测的.

6.1.2 向量函数的可测性与各个维度函数的可测性
定理 6.2. X = ( X 1 , · · · , X n ) 是 (Ω, F ) 到 (R n , B (R n )) 上的可测映射 ⇔ 每个 X i
都是 (Ω, F ) 到 (R, B (R )) 上的可测映射。

Qn Q
Proof. ⇒:对任意 i =1 (a i , b i ),X
−1
( ni=1 (a i , b i )) ∈ F 即知 X i −1 ((a i , b i )) ∈ F(取
其它的 (a i , b i ) 为 (−∞, ∞))
⇐ :考虑如下形式的矩体 J = R ×· · ·× (a i , b i ) ×· · ·× R ,X −1 ( J ) = X i −1 ((a i , b i )) ∈
F ,因为全体形如 J 的矩体生成的最小 σ 代数为 B (R n ),结论得证。

思考:在无限维成立吗?

6.1.3 处处收敛和几乎处处收敛
定理 6.3 (可测函数的点态极限). (Ω, Φ) 上的可测函数列 { f n }:Ω → R ,则
sup f n ,inf f n ,lim f n ,lim f n 及 lim f (若每点都存在) 都是可测函数.

T

Proof. 证明:∀a, { x : sup f n ( x) ≤ a} = { x : f n ( x) ≤ a}
n=1
inf { f n } = − sup {− f n } ,lim f n = inf sup f n ,lim f n = sup inf f n .
112 CHAPTER 6. 可测函数

练习

1. 举例说明:存在 f : R → R 不是 Lebesgue 可测的,但 | f | Lebesgue 可测,存


在不可数多个的 Borel 可测函数的上确界不是 Borel 可测的.
2. 设 f 、 g 都是 Lebesgue 可测的,则 { x : f ( x) > g ( x)} 是 Lebesgue 可测集.
3. 证明 R n → R 的 Lebesgue 可测函数与某个 Borel 可测函数几乎处处相等.
³ ³ ´´
4. f : (Ω, F ) → R, B R 可测的充要条件是 f −1 (−∞) 和 f −1 (+∞) ∈ Φ,以及对
任意 B ∈ B (R ), f −1 (B) ∈ Φ.
5. R n 上的一切广义实值上 (下) 半连续函数都是 Borel 可测的
6. 存在 Lebesgue 可测集 A ,以及 Lebesgue 可测函数 f ,使 f −1 ( A ) 不 Lebesgue
可测.[提示:[0,1] 上不可测集合 B 在 Cantor 函数 f 下的象 f (B) 是 Cantor
集的子集,这也说明了存在不是 Borel 集合的 Lebesgue 可测集 f (B)。]
7. 举例说明存在单调函数,使得其在任何非空开区间上不可能和某个连续函
数处处相等
8. f : R → R 称为可加的如果 ∀ x, y ∈ R, f ( x + y) = f ( x) + f ( y),证明 Lebesgue 可
测的可加函数必为线性函数,即 f ( x) = f (1) x.
(a) 证明对于有理数 q, f ( qx) = q f ( x),故连续可加函数是线性的
(b) 若 f 在非空开集上有界, f 线性
(c) 若 f 为 L-可测,可加,考虑 Γ = { x ∈ R : | f ( x)| ≤ R },若其测度为正,证
明存在开区间 (−δ, δ) 使得 | f ( x)| ≤ 2R

前面的定理表明点点收敛的极限仍是可测的,但如果不是点点收敛而是几
乎处处收敛呢?
¡ ¢
定义 (几乎处处). 给定测度空间 Ω, Φ, µ ,称性质 P ( x) 几乎处处为真,即存在
零测集 (测度为 0 的集合或测度为 0 的集合的子集) N ,使得 ∀ x ∈ Ω\ N ,P ( x) 成
立,记为 P ( x)µ − a, e.
¡ ¢
定理 6.4. 若 Ω, Φ, µ 为完备测度空间,其上可测函数列 f n ( x) → f ( x),µ − a, e.
则 f 可测.

Proof. 存在 A, µ ( A ) = 0,在 Ω\ A 上,f n ( x) → f ( x),于是 f 在 Ω\ A 上可测,而在


零测集 A 上,由于测度空间的完备性,f 必可测,故 f 在整个空间是可测的.
6.2. 可测函数的性质 113
¡ ¢
定理 6.5. 完备测度空间 Ω, Φ, µ 的可测函数 f ,改变零测集上的取值得到的函
数 g 仍可测,即 g ( x) = f ( x)µ − a, e.

Proof. { x : g ( x) < a} = { x : f ( x) = g ( x) < a} ∪ { x : f ( x) 6= g ( x) < a}


¡ \ ¢ ¡ \ ¢
= { x : f ( x) = g ( x)} { x : f ( x) < a} ∪ { x : f ( x) 6= g ( x)} { x : g ( x) < a}

注意 { x : f ( x) 6= g ( x)} 是零测集.

例 (为什么要完备测度空间):R 上的 Cantor 集 ∆,若 A ⊂ ∆ 但 A 不是


Borel 集,令 f = 1 A .f n = 0 ,显然 f n 是 Borel 可测的,但 f 不是 Borel 可测的.
但去掉 A 之后, f n → f ,即 f n → f ,µ − a, e.

6.2 可测函数的性质

6.2.1 可测函数与简单函数
下面要说明可测函数和简单函数之间的关系.

定义. :(Ω, Φ) 为可测空间,简单函数指仅取有限个值的可测函数,即存在有限个


S
互不相交的可测集合 A 1 ,A 2 ,…,A n ∈ Φ,及互不相同的数 a 1 ,…,a n ,Ω = Ai,
Pn
f ( x) = i =1 a i 1 A i ( x).

定理 6.6. :每个可测函数一定是简单函数逐点收敛的极限.
每个有界可测函数一定是简单函数的一致收敛的极限
每个正值可测函数一定是简单函数递增逐点收敛的极限.

Proof. 任意可测函数 f .定义

X
n ·2 n
−1 i n
f n ( x) = n
1 x: i < f ≤ i+1 o + n1{ f >n} − n1{ f ≤−n}
(x) 2n
i =− n·2n 2 2n

f n 是简单函数,若 f ≥ 0 则 f n % f .否则也有 ∀ x, f n ( x) → f ( x).


因为给定 x,n 足够大时,| f n ( x) − f ( x)| ≤ 2−n → 0
若 f 有界,则存在 β 使 | f ( x)| < β,对于一切 n > β,都有 sup | f ( x) − f n ( x)| ≤
x∈Ω
1
2n → 0,这表明 f n ( x) 一致收敛至 f ( x).
114 CHAPTER 6. 可测函数

注:在简单函数中,若可测集合 A i 都是矩体,则称 f 为阶梯函数 (真简单


函数),可以使每个可测函数成为一列阶梯函数的几乎处处收敛的极限.
前面的定理说可测函数可以看成简单函数的逐点极限,但简单函数是不连
续的.

6.2.2 可测函数与连续函数—Lusin 定理
下面的卢津 (Lusin) 定理阐述了可测函数与连续函数的关系,即可测函数可由连
续函数近似
定理 6.7 (Lusin 定理). f : R n → R 是 Lebesgue 实值可测函数.E ⊂ R n ,且
m (E ) < ∞,则对任意 ε > 0,存在紧集 K ⊂ E ,使得 (1) m (E \K ) < ε (2) f |K ( f 限
于 K 上) 是连续的.
Proof. 证明 1:不妨设 E 紧,且 m(E ) = 1。映射 g : x 7→ ( x, f ( x)) 从 E 到 E × R 是
可测的。所以存在紧集 S ⊂ E × R 使得 m( g−1 (S )) > 1 − ε。令 K 为 S 在 X 上的
投影,显然 K 是紧集且 m(K ) = m( g−1 (S )) > 1 − ε。假设 { xn } ⊂ K , xn → x0 ∈ K ,
则 S 中序列 {( xn , f ( xn ))} 有子序列收敛到 S 中某一点,只能是 ( x0 , f ( x0 )),于是
f ( xn ) → f ( x0 ),表明 f 在 K 上连续。
© ª∞
Proof. 证明 2:对任意正整数 i ,令 B i j j=1 ⊂ R 为不相交的 Borel 集合列.使
S
R= ∞ B .
j =1 i j
¡ ¢ T ¡ ¢ S
diam B i j < 1/i .定义 A i j ∆E f −1 B i j .则 A i j 可测且 E = ∞ A .
j =1 i j
n T
定义测度 v 在 R 的子集上,即 v (B) = m (B E ),v 相当于限制在 E 上的
Lebesgue 测度.
由 Lebesgue 测 度 的 正 则 性, 可 知 对 可 测 集 A i j , 存 在 紧 集 K i j ⊂ A i j ,
¡ ¢ ±
m A i j \K iµj < ε 2 i+ j ¶ µ ¶
S
∞ S∞ S
∞ ³S ´
则 m E \ K i j = m j=1 A i j \ K i j ≤ m ∞ j =1
A i j \ K i j < 2εi
j =1 j =1
³ S ´ ³ S∞ ´
由于 lim m E \ N K
j =1 i j
= m E \ K
j =1 i j
可知存在 N ( i ) 使得
µ N →∞ ¶
S
N(i) S
m E\ K i j < 2εi .记 D i = N(i) j =1
K i j ,则 D i 紧.
j =1
对每个 i 和 j ,选定 b i j ∈ B i j ,并定义函数 g i : D i → R : g i ( x) = b i j .若
x ∈ K i j ( j ≤ N ( i ))
由于 K i j 为互不相交的紧集,之间的距离为正.不难知道 g i 是连续的,而
T∞
且 ∀ x ∈ D i ,| f ( x) − g i ( x)| < 1/i ,令 k = i =1
D i ,则 K 仍为紧集,而且
6.2. 可测函数的性质 115

P

m (E \K ) ≤ m (E \D i ) < ε ,因为 ∀ x ∈ D i ,| f ( x) − g i ( x)| < 1/i ,∀ x ∈ K 亦成
i =1
立.
即 g i 是一致收敛于 f 的,由 g i 在 K 上连续性,知 f 为 K 上的连续
函数.

[证明思路:先考虑取值可数的可测函数,在对应的原象可测集合中找充分
逼近的紧集,其中的有限个紧集的并即为所求;再考虑一般的可测函数,可以表
示成取值可数的可测函数列的一致极限,对每一个可选取充分接近的紧集,其上
函数连续,故极限也连续]
注意. 1. f 限于 K 上是连续的,意味着 { x i } ∈ K ,且 x i → x∈ K .则 f ( x i ) → f ( x0 ),
这与 f 作为整个空间的函数在 K 上每个点处连续不同,即指 { x i } 不一定属
于 K , x i → x∈ K ,而有 f ( x i ) → f ( x0 )(参见习题)
2. 可以放宽假设 m (E ) < ∞,即允许 m (E ) = ∞,同时不要求 K 为紧集,只需
为闭集.
证明:令 E ik = { x ∈ R n , i − 1 ≤ | x| < i },则 m (E i ) < ∞.可在其中找到闭集 F i ,
± S
m (E i \F i ) < ε 2 i ,令 K = i F i .我们证明 K 为闭集,且 m (E \K ) < ε, f 限
于 K 上是连续的.
* 注意 K 中任何收敛序列有界,故必可以由 N 个 E i 覆盖,也就是说在有
限个 F i 内,而有限个 F i 的并仍是闭集,表明收敛序列的极限仍属于 K ,故
K 为闭集.
S S
或者, A m = n≤ m F n 闭,B m = n> m F n , H m = { x : | x| ≥ m},B m ⊂ H m

F = A m ∪ Bm = A m ∪ Bm ⊂ A m ∪ Hm ⊂ F ∪ Hm
\
⇒F⊂ m ( F ∪ H m ) = F ∪ (∩ m H m ) = F ⇒F=F
© ª
* m (E \K ) < ε,显然, f |K 的连续性则由于 K 中收敛于 x0 的序列 x j 有
© ª
界,且 F i 互不相交可知 x j 中除有限项外必属某个 F m ,由 f 在 F m 上限
制连续,知 f |K 连续
3. ε 不 能 为 0, 即 不 可 以 只 除 去 一 个 零 测 集, 比 如 E = [0, 1],E 1 ⊂ [0, 1],
且 m (E 1 ) = 0.5,类 Cantor 集 f ( x) = 1E 1 .对任意零测集 E 0 ⊂ E ,E − E 0
稠密,而 E 1 \E 0 非空,取 x0 ∈ E 1 \E 0 ⊂ E \E 0 , f ( x0 ) = 1,∀δ > 0,∃ x1 ∈
(E \E 0 ) ∩ ( x0 − δ, x0 + δ),而 E 1 为类 Cantor 集,即第一纲集 (或者说就是无
处调密集).(E \E 0 ) \E 1 是稠密的,故 ∃ x1 ∈ (E \E 1 ) ∩ ( x0 − δ, x0 + δ) ∩ (E \E 0 ),
116 CHAPTER 6. 可测函数
© ª S T³ ´
f ( x1 ) = 0.若 F = E \E 0 ,则可取 x j 满足 x j ∈ (E \ (E 0 E 1 )) x0 − 1j , x0 + 1j ,
¡ ¢
f x j = 0, x j → x0 而 f ( x0 ) = 1,说明在 F 上, f 不连续.当然,有时候可
以只除去零测集.
4. 我们知道,连续函数可由多项式逼近,但 Lusin 定理中的连续函数却不能改
为多项式.比如 E = [0, 1], f ( x) = e x .不可能找到多项式 P n 使其在某个很
小的正测度集外 f = P n .因为在 [0, 1] 上 f = P n ( x) 的解最多只有 n 个,所
以 { x : f ( x) = P n ( x)} 是零测集,矛盾.事实上连续函数的多项式逼近并不要
求它们在大部分点上相等,这与 Lusin 定理要求不同.
5. Lusin 定理中的 f 在闭集 F 上的限制是连续的,则根据连续函数延拓定理,
可以把 f 延拓成 R n 上的连续函数 g,从而使得 ∀δ > 0,存在连续函数 g ( x),
满足
m ({ x ∈ E : f ( x) 6= g ( x)}) < δ
进一步的,可以知道对任意的实值可测函数 f ,存在连续函数列 { g k ( x)} 几
乎处处收敛于 f ,即 lim g k ( x) = f ( x),a, e(当然也有连续函数列 { g k },依测
k→∞
度收敛于 f 表明连续函数全体在可测函数距离空间中稠密)
6. Lusin 定理的逆命题也是对的,即若在 E 上可找到闭集 F ,m (E \F ) < ε,f |F
连续,则 f 可测.
± ¯
证明:对任一 k = 1,2,…有闭集 F k ⊂ E 使 m (E \F k ) < 1 k,使 f ¯Fk 连续.
S
令 H= ∞ F 则 H ⊂ E . m (E \ H ) = 0.令 Z = E \ H
k=1 k
{ x ∈ E : f ( x) > a} = { x ∈ H : f ( x) > a} ∪ { x ∈ Z : f ( x) > a} ( Z 零测集,故其可测)
[∞
= k=1 { x ∈ F k : f ( x) > a} ∪ { x ∈ Z : f ( x) > a}
¯ ¯
由 f 在 F k 的可测性可知 f 可测,( f ¯F 连续,故 f ¯F 可测)#k k

7. 可测集与可测函数逼近的比较.
如果允许忽略小的正测集,可测集可由开集或闭集逼近;若要只差零测集,
则由能用 G δ 或 Fσ 集逼近.
在可测函数逼近时,若允许相差小的正测集,可由连续函数逼近。这时候 g
© ª © ª
连续,对于任意的 α, β, x : α < g ( x) < β 和 x : α ≤ g ( x) ≤ β ,恰好为开和
闭集.
若只要求差零测集的对可测函数的逼近,这个近似可测函数的函数 g(恰好
是一列连续函数的点点收敛极限,Baire-1 函数)必须满足对于任意的 α, β,
© ª
x : α < g ( x) < β 是 Fσ 集.
6.2. 可测函数的性质 117

8. 在纯粹的测度空间里,由于没有开集的概念,自然也无法讨论连续性,自然
无法应用 Lusin 定理。
9. 在 Lusin 定理中,函数的取值有限性(几乎处处是有限的)是重要的
10. Blumberg 定理:对于任意 R 上的实值函数 f ,存在 R 的可数稠密子集 D
使得 f |D 连续。这个稠密子集的势不可以是阿列夫 1.

6.2.3 近一致收敛—Egoroff 定理
我们在讨论连续函数空间部分讨论了逐点收敛和一致收敛,也在可测函数部分
讨论了几乎处处收敛,显然呢,一致收敛是最强的,而且可以看成是一致度量下
的收敛.而逐点收敛或几乎处处收敛则不可能是与度量空间的某种收敛相联系,
这一点留作习题.下面将讨论几乎处处收敛在施加何种条件后能得到更强的一致
收敛,叶戈罗夫定理说明,除去一个任意小的正测度集 (非 0 测集) 之外,几乎
处处收敛就是一致收敛.

定理 6.8 (Egoroff). { f k } 是一列实值可测函数,定义域可测集 E 的测度 m (E ) < ∞,


且 lim f k ( x) = f ( x),a, e(f 几乎处处有限)
.则任意给定 ε > 0,存在可测集 E ε ⊂ E ,
k→∞
m (E \E ε ) < ε,且在 E ε 上 f k 一致收敛于 f .(E ε 甚至可为闭集)

Proof. 分两步,首先证明对任意正数 δ,ε,存在 E 中可测集 F 和正整数 n0 ,使


得 m (E \F ) < δ,且当 n ≥ n0 时,∀ x ∈ F ,| f ( x) − f n ( x)| < ε.
为此,对每个 m,令
\
E m = { x ∈ E : | f ( x) − f n ( x)| < ε, n ≥ m} = {| f − f n | < ε}
n≥ m

E m 可测且 E m ⊂ E m+1 .因 f k → f a · e.E m % E \ Z , m ( Z ) = 0.


由于 m (E ) < ∞. 有 m (E m ) → m (E \ Z ) = m (E ),即 m (E \E m ) → 0.于是
¡ ¢ ¡ ¢
可取正整数 n0 使 m E \E n0 < δ2 ,又可取 E n0 中闭集使 m E n0 \F < δ2 ,故
m (E \F ) < δ,而 ∀ x ∈ F ,| f ( x) − f n ( x)| < ε, n ≥ n 0 .
第二步,在给定 ε > 0 之后,选取 E 中闭集 F m 和整数 K m ,使 m (E \F m ) <
±
ε m ,∀ x ∈ F ,k > K ,| f ( x) − f ( x)| < 1/ .令 F = T∞ F ,则 F 为闭集,且
2 m m k m m=1 m
T∞ S P
E \F = E \ m=1 F m = (E \F m ) ⇒ m (E \F ) ≤ m m (E \F m ) < ε
m
1
对任意 m,当 k > K m 时,sup | f ( x) − f k ( x)| ≤ sup | f ( x) − f k ( x)| < m
x∈ F x∈ F m
这表明 f k 在 F 上一致收敛于 f .
118 CHAPTER 6. 可测函数

[证明思路:关键是给出处处收敛的点的集合的表示方法。将极限表示成上
下极限,有
{ x : lim f j ( x) = 0} = { x : lim f j ( x) ≤ 0} ∩ { x : lim f j ( x) ≥ 0},故不收敛的点为 { x :
lim f j ( x) > 0} ∪ { x : lim f j ( x) < 0} 。
根据习题可知

1
{ x : lim f j ( x) > 0} = ∪ ∩ ∪ {− f j ( x) < − }
n k j≥k n
1
{ x : lim f j ( x) < 0} = ∪ ∩ ∪ { f j ( x) < − }
n k=1 j ≥ k n
所以有
1 1
m(∪ ∩ ∪ {| f j ( x)| > }) = 0 ⇒ ∀ n, m(∩ ∪ {| f j ( x)| > }) = 0
n k j≥k n k j≥k n
由于 m (E ) < ∞ 有 ∀n, ∃ k n 使得 m( ∪ {| f j ( x)| > n1 }) < 2nε+1
j≥k n
1
记 F n = ∪ {| f j ( x)| > n },则在 E \ ∪ F n 上 ∀ n,∃ k n 与 x 无关,只要 j > k n ,
j≥k n
∀ x ∈ E \ ∪ F n , | f j ( x)| ≤ n1 ,故 f j 一致收敛于 f ]
注意. 1. 定理中的 m (E ) < ∞ 条件不能去除,比如 R 上, f n = 1[n,n+1] , f = 0,
∀ x ∈ R , f n ( x) → f ( x),设 A 为可测集,且 m ( A c ) < 1,则对任意 n,存在
T
xn ∈ A [ n, n + 1],从而总有 | f ( xn ) − f n ( xn )| = 1,表明 f n 在 A 上并不一致
收敛于 f .
¡ ¢n
2. 定理中 E ε 与 E 不能改为只差零测集.比如, f n ( x) = 1 − x2 ,E = [−1, 1].
f n → f = 1{0} ,逐点收敛而非一致收敛,否则 f 也连续。但对任意小的 r > 0,
在 [−1, 1] \ (−r, r ) 上 f n 一致收敛于 0。要使 f n 在 [−1, 1] \B 上一致收敛于
0.B 必须包含 0 的一个邻域,否则在 [−1, 1] \B 上,可以找到任意接近 0
的数.
换言之,∀δ < 0,给定 n ∈ N ,总可以找到 x ∈ [−1, 1] \{0},使得 | f n ( x)| > δ,
说明 f n 不一致收敛于 0.
3. Egoroff 定理的假设和结论中,∀ε < 0,存在集合 E 使得 m (E ) < ε,而在 E c
上 f n 一致收敛于 f ,称这样的 f n 是近乎一致收敛于 f .记作 f n → f a · u.
下面我们证明,若 f n → f a · u.则 f n → f a · e.
证明:对于每个正整数 m,可以找到 E m , m (E m ) < 1/m,在 E m
c
上 fn 一
S∞ c
致收敛于 f ,也逐点收敛,于是
µ ¶
fn 在 m=1 E m 上也逐点收敛至 f ,而
¡¡S∞ c
¢c ¢ T

m m=1 E m =m E m = 0.#
m=1
6.3. 可测函数的收敛 119

这表明在一般情况下,一致收敛 → 近一致收敛 → 几乎处处收敛,而在定义


域测度有限时,还有近一致收敛 ⇔ 几乎处处收敛
4. Egoroff 与 Lusin 定理是 Little wood’s 三原则中的另两个:
(1) 任一可测集几乎是有限个矩体的并.
(2) 任一可测函数几乎都是连续的.
(3) 任一逐点收敛的可测函数列几乎是一致收敛的.
这里的几乎都指除去任意小的正测集.

练习

1. 令 f 为 [0, 1] 上无理数集的示性函数.
证明:1) f 处处不连续;2) 任给 ε > 0,找出一个集合 B,m (B) < ε,B ⊂ [0, 1],
使得 K = [0, 1] \B 紧, f |K 连续.(旨在说明限制于 K 上连续,并不意味着
作为全空间函数在 K 上每点连续)
2. { f n } 在可测集 D 上几乎处处收敛至 f , f n 可测实值正数,D ⊂ R n .证明
存在可测集
µµ ¶ ¶E 1 ⊂ E 2 ⊂ · · · ⊂ D .使得在每个 E k 上 f n 一致收敛至 f .并且
c
S

m Ek = 0.
k=1

6.3 可测函数的收敛

6.3.1 依测度收敛
先看一个习题:
¡ ¢
练习 A: Ω, F , µ 为测度空间,其上的 F 可测函数 f : Ω → [−∞, +∞],定义
© ª
k f kµ = inf r : µ ({ x : | f ( x)| > r }) ≤ r
³n o´
1. 证明:µ x : | f ( x)| > k f kµ ≤ k f kµ
2. 证明:k f + gkµ ≤ k f kµ + k gkµ
µ
→ f (依测度 µ 收敛)⇔ k f n − f kµ → 0
3. 证明: f n −
© ª
4. f = 1 A ,A ∈ F ,证明对任意 0 ≤ c < ∞k c f kµ = inf c, µ ( A ) ,进而说明一般
情况下 k c f kµ 6= ck f kµ .
120 CHAPTER 6. 可测函数

注意. 上述习题表明所有可测函数形成的空间中若定义距离 d ( f , g) = k f − gkµ ,


并视满足 k f − gkµ = 0 的 f 与 g 为同一函数,则可测函数空间成为一距离空间
(记为 L0 ),也就是说依测度收敛可以用某种度量的收敛来表示,类似于一致收
敛可以用一致度量下的收敛来表示,故以下的依测度收敛的性质也可从度量空
间角度予以理解.
除了上述定义的距离(在概率论中它称为 Ky Fan 距离),如果 µ(Ω) < ∞,
R | f − g|
按照 d ( f , g) = 1+| f − g| d µ 定义的距离对应的收敛也与依测度收敛等价。
¡ ¢
定义. Ω, F , µ 为测度空间,E ∈ F , f 和 { f k } 是定义在 E 上的几乎处处有限的
可测函数,称 { f k } 在 E 上依测度收敛于 f
如果 ∀ε > 0, lim µ ({ x ∈ E : | f ( x) − f k ( x)| > ε ) = 0.
k→∞

注:定义中要求几乎处处有限,如非几乎处处有限,依测度收敛定义需进行
相应调整。为什么?
由于

µ({ x ∈ E : | f ( x) − g( x)| > ε})


≤ µ( x ∈ E : | f ( x) − f n ( x)| > 2ε ) + µ( x ∈ E : | g( x) − f n ( x)| > 2ε ) → 0

即 ∀ε > 0,µ ({ x : | f ( x) − g ( x)| > ε}) = 0 ⇒ µ ({ x : | f − g| > 0}) = 0 ⇒ f = g, µ − a.e.,表


明依测度收敛的极限在几乎处处的意义上是唯一的.

6.3.2 依测度收敛与几乎处处收敛
现在来比较一下依测度收敛和几乎处处收敛,从定义上看,几乎处处收敛指除去
某个确定的点集之后,剩下的每个点上形成的数列都收敛,而依测度收敛只要求
函数越来越接近,不接近的点集测度越来越小,并不要求不接近的点集固定在某
个范围,所以可能导致点点都不收敛。具体见下面的定理与例子.
µ
定理 6.9. 若可测函数列 f n → f , µ − a.e..定义域 E 的 µ (E ) < ∞,则 f n −
→ f.

Proof. 根据 Egoroff 定理,可知 ∀δ, ε > 0,存在 E 中集合 F 和正整数 n0 ,使


µ ( E \ F ) < δ.
且当 n ≥ n0 时,∀ x ∈ F ,| f ( x) − f n ( x)| ≤ ε.
µ
由 µ ({ x ∈ E : | f ( x) − f n ( x) > ε|}) ≤ µ (E \F ) < δ 可知 f n −
→ f.
6.3. 可测函数的收敛 121

注意:在 µ (E ) = ∞ 时,几乎处处收敛不能推出依测度收敛,比如在 R 上,
¡© ¯ ¯ ª¢
f n ( x) = nx .则 f n → 0, a.e.,但 m x : ¯ nx ¯ > ε ↛ 0, n → ∞.即 f n 0.
在概率空间中,几乎处处收敛 ⇒ 依概率收敛,几乎处处收敛有时也称为以
概率 1 收敛。不过无论 µ (E ) < ∞ 还是 = ∞,依测度收敛都不能推出几乎处处
£ ¤ £ ¤
收敛,比如,在 [0, 1] 上,给定 k > 0,将 [0, 1] 等分为 k 个区间, 0, 1k , 1k , 2k ,
£ ¤ £ ¤ £ ¤ £ ¤ £ ¤ £ ¤
…, k−k 1 , 1 .再将所有这些区间排起来得到 0, 12 , 21 , 1 , 0, 13 , 31 , 23 , 32 , 1 ,
£ 1¤ £1 1¤ £1 3¤ £3 ¤ £ 1¤
0, 4 , 4 , 2 , 2 , 4 , 4 , 1 , 0, 5 ,…分别记为 A 1 ,A 2 ,A 3 ,…,A n ,…,对应
定义 f n = 1 A n .不妨令 A n 对应的是 k n 等分的区间,则 n → ∞, k n → ∞,故
有 m ({ x ∈ [0, 1] : | f n − 0| > ε}) ≤ k1n → 0
m
表明 f n −→ 0,但是 ∀ x ∈ [0, 1],总是属于无穷多个 { A n } 中的集合,于是 f n
在 E 上无一点收敛于 0.虽然依测度收敛不能保证序列的几乎处处收敛,但可
以保证其中有几乎处处收敛的子序列(Riesz 定理).
练习 B:找出上述例子里 { f n } 中几乎处处收敛到 0 的一个子序列 { f n k }
几个常见的可用作反例的函数列: f n = n−1 1(0,n) , f n = 1(n,n+1) , f n = n1[0,1/n]
这几个函数列分别一致,点点和几乎处处收敛到 0,但是积分都是 1
µ
定理 6.10 (Riesz 定理). 如果在 E 上 f n −
→ f ,则存在子序列 f n j → f , µ − a.e.
µ
³n o´
Proof. 由 f n −
→ f 可知,给定 j = 1,2,…存在 n j 使得 n ≥ n j 时 µ x : | f − f n | > 1j
<
¯ ¯ 1o n
1
2j
而且在选取 n j 过程中可要求 n1 < n2 < · · · .记 E j = x : ¯ f − f n j ¯ > j , H m =
S∞
E .
j=m j
¡ ¢ ¡ ¢P∞ − m+1 T

则 µ E j < 21j ,µ (H m ) ≤ j=m µ E j = 2 .再令 H = H m ,则 µ ( H ) = 0.
¯ ¯ 1 m=1 ¯ ¯
由于在 E \E j 上,¯ f − f n j ¯ ≤ j ,故当 j ≥ m 时,在 H m c
上总有 ¯ f − f n j ¯ ≤ 1j ,那
S
么在 ∞ c c
m=1 H m = H 上就有 f n j → f .即在 E 上 f n j → f , µ − a.e..

6.3.3 依测度收敛与近一致收敛
下面的定理说明了近一致收敛要强于依测度收敛.

定理 6.11. f n → f , a.u. 即 ∀ε > 0,∃ 集合 E ε 使 µ (E \E ε ) < ε,而在 E ε 上 f n 一


µ
致收敛于 f ,则 f n −
→ f.

Proof. 如果 f n → f a.u.,∀ε, δ > 0,由 f n 在 E ε 上一致收敛于 f .


¡© ª¢
µ x ∈ E : | f n ( x) − f ( x)| ≤ δ, n足够大时 ≥ µ (E ε ) ⇒
122 CHAPTER 6. 可测函数

µ ({ x ∈ E : | f n ( x) − f ( x)| > δ}) ≤ µ (E \E ε ) < ε, n足够大时 ⇒

lim µ ({ x : | f n ( x) − f ( x)| > δ}) = 0


n→∞

关于各种收敛的关系,目前有

我们已经知道在某种对应依测度收敛的距离下,可测函数空间是一个距离
空间,所以其中有对应的 Cauchy 列.

定义. (依测度 Cauchy 列):{ f k ( x)} 是 E 上几乎处处有限的可测函数列,若 ∀ε > 0


¡© ¯ ¯ ª¢
lim µ x ∈ E : ¯ f k ( x) − f j ( x)¯ > ε = 0 .则称 { f k } 为 E 上的依测度 Cauchy 列.
k→∞
j →∞

下面的定理表明可测函数在依测度收敛对应的距离下,是一个完备度量空
间.

定理 6.12. { f k ( x)} 依测度收敛到某个几乎处处有限的可测函数 ⇔{ f k } 为依测度


Cauchy 列.

Proof. “⇒” | f n ( x) − f m ( x)| ≤ | f n ( x) − f ( x)| + | f m ( x) − f ( x)|a.e..


¡© ª¢ ¡© ª¢
µ ({| f n − f m | > ε}) ≤ µ | f n − f | > ε/2 + µ x : | f m − f | > ε/2

令 n, m → ∞,则后两项 → 0 故 lim µ ({ x : | f n − f m | > ε}) = 0


n,m→∞
“⇐”.若 { f k } 为依测度 Cauchy 列,则仿照 Riesz 定理,可从中选出子序列
© ª ¡© ª¢
f n j 使得 µ x : | f n ( x) − f k ( x)| ≥ 2− j < 2− j ,若 n, k ≥ n j , n 1 < n 2 < · · ·
© ¯ ¯ ª S∞ T

记 E j = x : ¯ f n j+1 ( x) − f n j ( x)¯ ≥ 2− j ,H m = E .H =
H m ,则 µ ( H ) = 0
j=m j
ª© m=1
当 x ∈ E \ H 时, f n j ( x) 是 Cauchy 数列,事实上,x ∉ 某个 ¯ H m ,即当¯ j ≥ m
¯ ¯ ¯ ¯
时,x ∉ E j 等价于 ¯ f n ( x) − f n ( x)¯ < 2− j .故只要 i, j > m,¯ f n ( x) − f n ( x)¯ < 2−m
j +1 j i j

由实数空间的完备性,定义 f ( x) = lim f n j ( x),∀ x ∈ E \ H ; f ( x) = 0,∀ x ∈ H .


j →∞
c µ
而在任意 Hm 上, f n j 一致收敛于 f ,即 f n j → f , a.u. ⇒ f n j −
→f
6.3. 可测函数的收敛 123

µ © ¯ ¯ ª © ¯ ¯ ª
→ f ,注意到 { x : | f n − f | > ε} ⊂ x : ¯ f n − f n j ¯ > ε/2 ∪ x : ¯ f n j − f ¯ > ε/2 ,
为证 f n −
当 n 足够大时,由 { f n } 为 Cauchy 列知只要 n j 足够大右边第一项的测度可足够
© ª µ
小,同时 f n j 的收敛性亦保证右边第二项测度足够小,故 f n −
→ f 成立.

注:定理证明中用到一个技巧,即为证某个序列的收敛性,不直接从整个序
列入手,而是寻找其中的子序列,或者由空间的紧性找到收敛子序列,或者如定
理中那样找一个收敛较快的子序列.
下面定理说明了如何利用子序列判断序列是否依测度收敛.
¡ ¢
定理 6.13. 测度空间 Ω, F , µ ,µ (Ω) < ∞,则 f n 依测度收敛至 f ⇔ { f n } 的任一子
© ª © ª
序列 f n k 有子序列 f n k ( j) 几乎处处收敛于 f .
© ª
Proof. “⇒”{ f n } 的任一子序列 f n k 亦依测度收敛于 f ,故由 Riesz 定理结论
成立.
f ,则存在 ε,δ > 0.使 µ ({ x : | f n − f | > ε}) ≥ δ 对无穷个 n 都
“⇐” 若 f n
© ª © ª © ª
成立,不妨记为 n j ,则子序列 f n j 并不依测度收敛于 f , f n j 的任意子序列
也不依测度收敛,也不可能几乎处处收敛到 f .

依测度收敛的子序列准则的一个用处是证明依测度收敛序列的连续映像仍
旧依测度收敛.
µ µ
定 理 6.14. µ (Ω) < ∞, f n −
→ f ,gn −
→ g,φ ( x1 , x2 ) 是 R 2 上 的 连 续 函 数, 则
µ µ µ
→ φ ( f , g).特别的, f n g n −
φ ( f n, gn) − → f g,α f n + β g n −
→ αf + βg
© ª © ª n o © ª
Proof. 对任意子序列 f n j , g n i ,可找到子子序列 f n j (k) , g n i (l) 几乎处处
© ª n o n o n o
收敛于 f 和 g.(先在 f n j 找子序列 f n j (k) ,再在 g n j (k) 中找 g n j (k)l ,最
n o n o n o © ª ³ ´
后以 f n j (k)l 、 g n j (k)l 为 f n j (k) 、 g n i (l) )。由 φ 的连续性,φ f n j (k) , g n i (l) →
µ
φ ( f , g)a.e..因子序列的任意性,可知 φ ( f n , g n ) −
→ φ ( f , g).

注:定理结论适用于概率空间,但请思考结论是否成立如果 µ(Ω) = ∞。事实


上,几乎处处收敛,依概率收敛以及依分布收敛序列的连续映像都还是保持原来
的收敛性质的。
我们知道,大数定律的表述分强弱两种形式,弱型即依概率收敛。以后我们
会说明,依概率收敛可以推出依分布收敛,而依分布收敛是中心极限定理所使用
的,在统计中用的最多 (事实上是因为它差不多是最弱的收敛)。
124 CHAPTER 6. 可测函数

最后我们简要说明为什么几乎处处收敛不可以用某种距离下的收敛来表示
(在前面的章节我们已说明处处收敛是不可以用距离对应的收敛来表示的),这
是因为在度量空间中的任意 (收敛) 序列满足以下性质:序列 {a n } 的任一子序列
都有进一步的子序列收敛到 a,则 a n → a.(反证,{a n } 不收敛,则不是 Cauchy
© ª ¡ ¢ © ª
列,即可找到 a n j 使得 ∃ε > 0, d a n j , a > ε.即 a n j 中不可能有收敛子序列
可以收敛至 a,矛盾)
于是,假如几乎处处收敛可以用距离表示,那么该序列的任一子序列都有几
乎处处收敛的子序列,这表明在测度有限的时候该序列也是依测度收敛的。但我
们知道存在 { f n } 依测度收敛于 f 而不几乎处处收敛于 f ,于是出现矛盾.
要使几乎处处收敛能用距离描述,则必须在它和依测度收敛是等价的情形
下才成立。

练习

1. (Ω, P (Ω), P ) 为概率空间,Ω 可数。证明在 Ω 上,几乎处处收敛与依概率收


敛等价。
2. 问下面的函数列在给定区间 ∆ 上是否依测度收敛或几乎处处收敛:
nx nx − nx2
f n ( x) = 2 , ∆ = (0, 1) ;f n ( x) = 2 , ∆ = (1, ∞);f n ( x) = nxe , ∆ = (0, 1);
1+ n 2 x 1+ n 2 x
¯ ¯ − 1
f n ( x) = nx
, ∆ = (0, 1); f n ( x) = ¯ x − n1 ¯ 2 , ∆ = (0, 1)。
(1+n2 x4 )ln(n+1)
3. 函数列 f n 在 (0, 1) 上都单调,且依测度收敛到 f 。证明存在与 f 几乎处处
相等的某个单调函数 f 0 ,使得函数列 f n 在 f 0 的连续点上点点收敛。

6.3.4 函数形式的单调类定理
定理 6.15. 设 (Ω, F ) 为一可测空间,C 为生成 F 的一个代数。令 H 为 Ω 上
的一族非负实值函数,如果它满足下列条件:
(1) f , g ∈ H ,α, β ≥ 0 ⇒ α f + β g ∈ H
(2) f n ∈ H , n ≥ 1, f n ↑ f 且 f 有限或者 f n ↓ f ⇒ f ∈ H
(3) ∀ A ∈ C , I A ∈ H
则 H 包含 Ω 上的所有非负实值 F 可测函数

Proof. 令 T = { A ∈ F : I A ∈ H },则 (3) 表明 T ⊃ C ,(2) 说明 T 为单调类,故


单调类定理说明 T = F 。由于任意非负可测实值函数可以表示为一列递增非负
可测实值函数的点点极限,结论成立。
6.4. 应用:极大似然估计 125
© ª
定义. 设 (E, E ) 为一可测空间,H 为 Ω 到 E 中的一族映射,令 F = σ ∪ f ∈H f −1 (E ) ,
则 F 是使得 H 中所有元素为可测的最小 σ 代数。称 F 为函数族 H 在 Ω 上
¡ ¡ ¢¢
生成的 σ 代数。若 (E, E ) = R̄, B R̄ ,我们用 σ ( f , f ∈ H ) 表示 F 。

下一定理刻画了 σ ( f ) 可测函数的形态

定理 6.16. 设 f 为 Ω 到可测空间 (E, E ) 中的映射,σ ( f ) 为 f 在 Ω 上生成的 σ


代数 f −1 (E ),则为要 Ω 上的数值函数 ϕ 为 σ ( f ) 可测,当且仅当存在 E 上的 E
可测函数 h,使得 ϕ = h ◦ f 。如果 ϕ 为实值可测函数,则 h 可取为实值函数。

Proof. 只证必要性。设 A ∈ σ ( f ),则存在 B ∈ E ,使得 A = f −1 (B),即 I A = I B ◦ f 。


于是对任意 σ ( f ) 可测简单函数 ϕ,存在 E 上一 E 可测函数 h 使得 ϕ = h ◦ f 。设
ϕσ ( f ) 可测函数,则存在一列 σ ( f ) 可测简单函数 ϕn 收敛到 ϕ,根据已得到的结
果,存在一列 E 上的 E 可测实值函数 h n ,使得 ϕn = h n ◦ f 。令 h = limsupn→∞ h n ,
则 ϕ = h ◦ f 。若 ϕ 为实值函数,可令 h0 = hI |h|<∞ ,则 ϕ = h0 ◦ f 。

定理的结论在经济学金融学研究中有诸多用途,一个典型的应用就是不对
称信息模型。注意,直观的看,随机变量的可测性实际上也代表了该变量实现值
的可观测性。
考虑一个简单的模型,股票未来的回报为 u + v,股票的供给为随机变量 s,
u, v, s 独立。市场上有两类人,一类是知情交易者,知道 u,另一类为不知情交
易者,只能观测到股票价格。市场均衡要求股价是关于 σ (u, s) 可测的,即价格
是 u, s 的函数。

6.4 应用:极大似然估计
定理 6.17. 假定参数空间 Θ 为 R 中的开区间,对于任意 θ ∈ Θ,对数似然函数
∂ log f (X ,θ ) 2
log f ( x, θ ) 对 θ 的一二三阶偏导数都存在且局部可积,E θ [( ∂θ
) ] < ∞。考虑
来自 Fθ 的独立同分布观测,存在一列似然方程的根 θ̂n 几乎处处收敛到 θ ,而
且是渐近正态和有效的。

Proof. 将对数似然函数在 θ 处 Taylor 展开,


∂ log f ( x, λ) ∂ log f ( x, θ ) ∂2 log f ( x, θ ) 1
− = (λ − θ ) + ξ (λ − θ )2 H ( x )
∂λ ∂θ ∂2 θ 2
P ∂ log f (X i ,θ) P ∂2 log f (X i ,θ) P
记 A n = n1 ∂θ
,B n = n1 ∂θ 2
,C n = n1 H(X i)
126 CHAPTER 6. 可测函数

1 ∂ log L(λ)
则 θ̂n 是 n ∂λ
= A n + B n (λ − θ ) + 12 ξC n (λ − θ )2 = 0 的根
可以证明 A n , B n , C n 都是一列方差有限的独立随机变量的均值,由强大数定
律,
A n → 0, B n → −vθ , C n → E θ [ H ( X )] 。取充分小的正数 ε < vθ /E θ [ H ( X )],则当
n 充分大时,| n1 ∂ log∂λL(λ) |θ−ε − vθ ε| ≤ | A n | + ε|B n + vθ | + 12 ε2 |C n | ≤ 34 vθ ε,

1 ∂ log L(λ) 1 3
| |θ+ε + vθ ε| ≤ | A n | + ε|B n + vθ | + ε2 |C n | ≤ vθ ε
n ∂λ 2 4
有此可知 [θ − ε, θ + ε] 中含有似然方程的解,故下面的极限也是方程的解
∂ log L(λ)
θ̂nε = inf{λ : θ − ε ≤ λ ≤ θ + ε, = 0}
∂λ

首先要证明 θ̂nε 可测。只需证明 ∀ t ≥ θ − ε,


∂ log L(λ) ∂ log L(λ)
{θ̂nε > t} = {infθ−ε≤λ≤ t ∂λ
> 0} ∪ {supθ−ε≤λ≤ t ∂λ
< 0} 可测
∂ log L(λ)
注意到 ∂λ
连续,infθ−ε≤λ≤t ∂ log∂λL(λ) 中可假定 λ 为有理数,于是可测性
得证。
接下来选取一列¯θ̂n 。由前面的分析,对于数 kε ,存在充分大的数 Nk ,使得
¯
∂ log L(λ) ¯
当 n > Nk 时 ¯λ=θ̂ ε = 0, a.s.,记对应几乎处处成立的状态集合为 Ωk ,令
∂λ ¯ n,
k
Ω = ∩k Ωk ,则 P (Ω) = 1。不妨假设 Nk 递增,当 Nk ≤ n < Nk+1 时,定义 θ̂n = θ̂n, ε ,
k
则在 Ω 上 θ̂n 处处收敛到 θ ,即说明极大似然估计是强相合的。
第 7 章 积分与期望

¡ ¢ R
本章将在测度空间 Ω, F , µ 上定义可测函数 f 的积分, Ω f d µ.为方便计,同
时也为了表明积分可以看成是一种算子或泛函,有时候也写成 µ ( f ).

7.1 积分的定义
积分的定义从简单函数开始.
¡ ¢ Pn R P
n
定义. 1. Ω, F , µ 上的非负简单函数 f = k=1
a k 1E k ,则 Ω f dµ = a k µ (E k ),
k=1
S
n
若有 a k = 0,µ (E k ) = ∞,定义 a k µ (E k ) = 0,此外 Ω = Ek
k=1
¡ ¢
2. Ω, F , µ 上的非负简单函数全体记为 Φs .若 f 为非负可测函数,则
Z ½Z ¾
f d µ = sup ϕ d µ : 0 ⩽ ϕ ⩽ f , ϕ ∈ Φs
Ω Ω

这里的 ⩽ 意味着在每个点上都小于等于
思考:我们在定义测度的时候用的是外测度,即覆盖集合的区间的长度总和
的下确界。这里是上确界,在定义中若取 f 为每个可测集的示性函数,则测
度也是包含于集合的一些可测集测度之和的上确界,为什么?另外,这里的
积分定义相当于一种下积分,能用上积分定义吗?测度采用下确界与积分使
用上确界反映出两者之间的对偶性质。
R R
对于 E ∈ F ,定义 Ω f 1E d µ E f dµ =
¡ ¢ R R R
3. Ω, F , µ 上的实值可测函数 f ,定义 E f d µ = E f + d µ − E f − d µ.
其中 E ∈ F , f + = max ( f , 0), f − = max (− f , 0).
R R R
若 E f + dµ 与 E f − d µ 都为 ∞,则 E f d µ 无定义,否则称积分存在。如果
R +
R −
E f d µ < ∞, E f d µ < ∞,称 f 是可积的 (绝对可积).

127
128 CHAPTER 7. 积分与期望

在 E 上所有可积函数的全体记为 L1 (E ).以后我们会看到它是一个 Banach


空间,在赋予一定的范数,并视几乎处处相等的函数等价之后.
注意. 1. 在定义中,为保证定义的无歧义,需要说明若简单函数 f 既可写成
Pn Pm P P
k=1
a k 1E k ,又可表示成 k=1
b k 1 A k ,则 a k µ (E k ) = b k µ ( A k ).这是因为
a i = b j 若 E i ∩ A j 6= ;.

Xn X
n X
m ¡ \ ¢ X n X
m ¡ \ ¢ X m ¡ ¢
a µ E =
k=1 k ( k )
aiµ E i A j = b jµ E i A j = b jµ A j
i =1 j =1 i =1 j =1 j =1

为避免歧义,亦可使简单函数的表现形式唯一,即要求每个 a k 各不相同。
R R R
2. 由 定 义 可 直 接 推 出 若 f ⩽ g 且 f dµ 和 gd µ 都 有 定 义, 则 f dµ ⩽
R
gd µ(单调性)
R
3. 非负简单函数亦满足非负可测函数积分的定义.(单调性保证 Ω ϕd µ ⩽
R R R
Ω f d µ,取 ϕ = f ,有 Ω ϕd µ ⩾ Ω f d µ)
4. 对于非负简单函数 f 和 g 以及实数 α > 0,则 µ (α f ) = αµ ( f ),µ ( f + g) =
µ ( f ) + µ ( g)(线性性)

7.1.1 积分的性质
在非负可测函数积分的定义中,积分是一族简单函数积分的上确界,回忆上确界
相当于最大的极限点,是否可以将积分视为一列简单函数积分的极限呢?下面的
定理表明非负可测函数积分确实是一列递增简单函数列的积分的极限.

定理 7.1. 对非负可测函数 f ,若有非负简单函数列 { f n },f n ⩽ f n+1 且 lim f n = f ,



R R
lim f n d µ = f d µ(显然,这样的简单函数列是存在的)
R R R
Proof. 由 f d µ 的定义,要证 lim f n d µ = f d µ,只需对任意满足 0 ⩽ g ⩽ f 的
R R R ©R ª R
简单函数 g,lim f n d µ ⩾ gd µ.这样 lim f n d µ ⩾ sup gd µ = f d µ。
R R R R
又 ∀n, f n d µ ⩽ f d µ,故 lim f n d µ = f d µ.
P P n P T P
不妨设 g = m n
i =1 a i 1 A i , f n = j c j 1B j = i, j c j 1 A i B j = i f n 1 A i
R P R P ¡ T ¢ R P
f n d µ = i A i f n d µ = i, j c nj µ A i B j , gd µ = i a i µ ( A i )
R P ¡ T ¢
我们证对所有的 i 都有 lim A i f n d µ = lim j c nj µ A i B j ⩾ a i µ ( A i )
n→∞ n→∞
若 a i = 0 结论自然成立,故考虑 a i > 0:
对于任意给定 ε > 0,令 F n = { x ∈ A i : f n ( x) ⩾ (1 − ε) a i }
7.1. 积分的定义 129

则由 f n 的递增性,不难知道 F1 ⊂ F2 ⊂ · · · ,由于 lim f n = f ⩾ g


[∞
A i = { x ∈ A i : lim f n ( x) ⩾ (1 − ε) a i } = n=1 F n ⇒ µ ( A i ) = lim µ (F n )
n→∞
R R
因为 A i f n d µ ⩾ F k f n d µ ⩾ (1 − ε) a i µ (F n ),两边取极限,有
R R
lim f n d µ ⩾ (1 − ε) a i µ ( A i ),由 ε 的任意性有 lim f n dµ ⩾ a i µ ( A i )
n→∞ A i n→∞ A i
Z Z X X Z
f n dµ = f n1 A i dµ = i
f n1 A i dµ
R P R
⇒ lim f n dµ ⩾ i aiµ (Ai) = gd µ.
n→∞
R
推论. 若有非负可积简单函数列 { f n }, f n ⩾ f n+1 且 lim f n = 0,则 lim f n d µ = 0

Proof. 可以直接用前面定理的结论。这里给出另一个证明,其实就是将前述证明
进行改写。
设 f 1 的支集的测度为 M (必然有限),最大值为 C ,那么所有其它函
数 的 支 集 都 在 f 1 的 支 集 当 中, 最 大 值 也 小 于 等 于 C 。 所 以 对 任 意 δ > 0,
R
f n ⩽ M δ + C µ({ f n > δ}),集合列 { f n > δ} 是嵌套递减的,并且集合列极限为
R
零测度集,由测度的连续性,µ({ f n > δ}) → 0,所以 f n → M δ。由 δ 的任意性,
得证。(这个证明更清楚地表明了简单函数的单调收敛性质来源于测度的连续性,
本质上也就是测度的可数可加性。)

注意. (由推论反推定理,其中 f 是简单函数)


:先考虑 f 是可积的,那么对 f − f n
R
应用推论结果即可。若 f 的积分值为 ∞,那么在某个正测度集 A 上有 f 1A = ∞
而且其上 f 为常数 a,即或者 a = ∞ 或者 µ( A ) = ∞, a > 0,或者皆有可能。a = ∞
R
时,{ x : f n ( x) > N } ↑ A ,表明 f n 1 A > N µ({ x : f n ( x) > N }) → N µ( A ),a 有限时,
R
{ x : f n ( x) > a(1 − ε)} ↑ A ,表明 f n 1 A > a(1 − ε)µ({ x : f n ( x) > a(1 − ε)}) → a(1 − ε)µ( A ),
R
都说明 f n 1 A → ∞
有了上述定理,我们可以证明对一般的可积函数,积分也具有线性性.
R¡ ¢ R R
定理 7.2. 若 f 、 g 非负可测,α、β ⩾ 0,则 α f + gβ d µ = α f d µ + β gd µ
R¡ ¢ R R
若 f 、g 可积,α、β ∈ R ,则 α f +β g 可积,且 α f + β g d µ = α f d µ+β gd µ.

Proof. 选简单函数列 0 ⩽ f n ↑ f ,0 ⩽ g n ↑ g,则 0 ⩽ α f n + β g n ↑ α f + β g


R¡ ¢ R R
α f n + β g n d µ = α f n d µ + β g n d µ,两边求极限即可
R + R R R
f d µ < ∞, f − d µ < ∞ ⇒ α f + d µ < ∞, α f − d µ < ∞ ⇒ α f 可积
130 CHAPTER 7. 积分与期望
R R
g+ d µ < ∞, g− d µ < ∞,又有 ( f + g)+ ⩽ f + + g+ ,( f + g)− ⩽ f − + g−
R R R R
⇒ ( f + g)+ d µ ⩽ f + d µ + g+ d µ < ∞, ( f + g)− d µ < ∞ ⇒ f + g 可积.
同样由 (α f ) = α f + + α f − (α > 0) 及 f + g = f + + g+ − ( f − + g− ) 可知积分对于
可积函数的线性性.

注意到积分定义中若定义域测度为时,积分值也为,故改变测集上的函数
值,函数的积分不变,所以积分的许多性质对于几乎处处相等的函数是不变的.

定理 7.3. 设 f 、 g 都是非负可测函数,则
R
1. f = 0, µ − a.e.,⇔ f d µ = 0
R
2. µ ({ x : f ( x) > 0}) > 0 ⇔ f d µ > 0
R
3. f d µ < ∞ ⇒ f < ∞, µ − a.e.
R R
4. f ⩽ g, µ − a.e..⇒ f d µ ⩽ gd µ
R R
5. f = g, µ − a.e..⇒ f d µ = gd µ

定理 7.4. 对于可积函数 f 、 g,有


R
1. | f | d µ = 0 ⇔ f = 0, µ − a.e.
R R
2. f ⩾ g, µ − a.e..⇒ f d µ ⩾ gd µ
¯R ¯ R
3. ¯ f d µ¯ ⩽ | f | d µ

留作练习,可应用 Chebychev 不等式即 f ⩾ 0 可积 α > 0,E α = { x : f ( x) > α},


R R ¡ ¢ R
则 µ (E α ) ⩽ α1 f d µ. 如:2. f d µ = 0 ⇒ µ ( f > 0) = 0,因 µ f ⩾ n1 ⩽ n f d µ = 0,
µ ¶
S© ª R R
µ ( f > 0) = µ f⩾n ,
1
因 µ ( f ⩾ n) ⩽ n1 f d µ,
3. f d µ < ∞ ⇒ f < ∞, µ−a.e..
n
Z
1
µ ( f = ∞) ⩽ µ ( f ⩾ n ) ⩽ f dµ → 0
n

练习

1. 证明在 R n 中,如下定义的 Lebesgue 可测函数的 Lebesgue 积分与前面给出


的定义一致:
对于有界非负可测函数,定义:

Z ½Z ¾
f dm = sup ϕ dm : 0 ⩽ ϕ ⩽ f , ϕ为简单函数且在某个测度有限的集合之外取值为0
Ω Ω
7.2. 积分的几个收敛定理 131

对于一般非负可测函数,定义:
Z ½Z ¾
f dm = sup ϕ dm : 0 ⩽ ϕ ⩽ f , ϕ ∈ Φ f
Ω Ω

其中 Φ f 是满足 ϕ ⩽ f a.e. 且在某个测度有限的集合之外取值为 0 的有界非


负可测函数全体。
2. 设 A 是 R n 中使 A 或 A c 可数的子集 A 的全体,测度 µ( A ) = 0 若 A 可数,
µ( A ) = ∞ 若 A c 可数,说明在测度空间 (R n , A, µ) 上练习 1 给出的积分定义
与讲义前面给出的定义不一致。

7.2 积分的几个收敛定理

7.2.1 单调收敛定理
R
定理 7.5 (单调收敛定理). 设 f n 单调增,且 f n → f , µ − a.e.. f 1 d µ > −∞,则 f
R R
积分存在且 f n dµ ↑ f d µ.

Proof. 先 假 设 f 1 ⩾ 0, 对 每 个 n 可 找 到 非 负 简 单 函 数 f nm ↑ f n . 令 g n =
max ( f 1n , · · · , f nn ),则 g n 是简单函数 (需要证明,留作思考题),而且 0 ⩽ g n ↑ f .
(留作思考题)
R R R R
故 g n dµ ↑ f d µ.注意到 g n ⩽ f n ↑ f ,有 f n dµ ↑ f d µ.
如果只用简单函数单调收敛的结果,可以考虑任意满足 0 ⩽ h ⩽ f 的简单函
数 h,有
Z Z Z Z Z Z
h = lim min( h, g n ) ⩽ lim g n ⩽ lim fn ⇒ f ⩽ lim fn

R R
若 f ⩽ 0,令 g n = − f n ↓ − f = g.有 0 ⩽ gd µ ⩽ g n d µ < ∞
R R
因为 0 ⩽ g 1 − g n ↑ g 1 − g.故 ( g 1 − g n ) d µ ↑ ( g 1 − g) d µ < ∞
R R R R R
由于 g 1 d µ 为有限值,两边相减有 g n d µ ↓ gd µ ⇒ f n d µ ↑ f d µ
R R
对于一般的 f n ,有 f n+ ↑ f + , f n− ↓ f − , f − d µ < +∞,由前可知 f n+ d µ ↑
R + R R R R
f d µ + ∞ > f n− d µ ↓ f − d µ ⩾ 0 ⇒ f n d µ ↑ f d µ
R R
注:令 f n = −1[n,∞) ,则 f n ↑ 0,但 f n d µ = −∞ ↛ 0.说明 f 1 d µ > −∞ 是
必需条件.
132 CHAPTER 7. 积分与期望
µ ¶
R P
∞ ∞ R
P
推论. 若 { f n } 是非负可测函数,则 f n dµ = f n dµ
n=1 n=1
PN P∞
Proof. 取 g N = n=1 f n ,则 gN ↑ n=1 f n .由单调收敛定理即得证.
R
推论. 若 f 是非负可测函数,则映射 E → E f dµ 是 (Ω, F ) 上的测度,特别的,
R ∞ R
P
若 {E n } 是互不相交的可测集合列,就有 S∞ f dµ = f d µ.
n=1 E n n=1
En

注:推论 2 表明 (Ω, F ) 上只要有一个测度,就可以有无穷多个测度.


反过来,若 v,µ 都是 (Ω, F ) 上的测度,是否可能存在非负可测函数 f 使
R
得 v (E ) = E f d µ,∀E ∈ F 呢?这是 Radon-Nikodym 定理所讨论的内容.

7.2.2 Fatou 引理
单调收敛定理说明了极限和积分号在一定情形下可以交换,但通常情形两者无
法交换,我们有:

定理 7.6 (Fatou 引理). 设 { f n } 为一列积分存在的可测函数,若存在某积分存在


R
的函数 g, gd µ > −∞ 且 ∀n ⩾ 1, f n ⩾ ga.e..则 lim inf f n 积分存在,
n→∞
Z Z
lim inf f n d µ ⩽ lim inf f n dµ
n→∞ n→∞

Proof. 令 g n ( x) = infk≥n f k ( x),则 g n ↑ lim inf f n .又 g 1 ≥ g, a.e..


R R
有 g 1 d µ ⩾ gd µ > −∞, 由 单 调 收 敛 定 理,lim inf f n 的 积 分 存 在, 且
R R R R R
lim inf f n d µ = lim g n d µ = lim inf f k d µ ⩽ lim inf f k d µ = lim inf f n d µ.
n→∞ n→∞ k⩾ n n→∞ k⩾ n n→∞

R R R
推论. 若 f n 非负可测,且 f n → f a.e.,则 f d µ ⩽ lim inf f n d µ ⩽ lim sup f n d µ
n→∞ n→∞
R
注意. µ 为 Lebesgue 测度. f n = 1[n,n+1] ,则 f n → f = 0,但 f n d µ = 1,∀ n,而
R
f d µ = 0,表明 Fatou 引理中的不等号是可以严格成立的,同时这个例子也可帮
助记忆不等号的方向.

练习

1. 在 Fatou 引理中,设 f n ≤ 0,将 Liminf 改为 Limsup,结论中的 ≤ 改为 ≥,


重新证明,并举例说明,若只改三个中的一个或两个条件,结论是不对的.
7.2. 积分的几个收敛定理 133

7.2.3 控制收敛定理
现在我们讨论可积函数的收敛问题,不过首先回顾一下可积的一些等价定义:
R R R¡ ¢
f 可积 ⇔ f + 和 f − 都可积 ⇔ f + < ∞, f − < ∞ ⇔ f + + f − d µ < ∞
R
⇔ | f | d µ < ∞ ⇔ | f | 可积
¯R ¯ R
此外,容易观察到的是 f 可积,则 ¯ f d µ¯ ⩽ | f | d µ
¡ ¢ ¡ ¢
为方便记,记 Ω, F, µ 上的可积函数全体为 L1 Ω, F , µ .
¡ ¢
定理 7.7 (控制收敛定理). 设 f n ,g ∈ L1 Ω, F , µ ,| f n ( x)| ⩽ g ( x),f n ( x) → f ( x) , a.e..
R R R
则 f ∈ L1 ,且 f n dµ → f d µ, lim | f n − f | dµ = 0
n→∞
R R
Proof. 因 | f | ≤ | g| a.e..故 | f | d µ ≤ | g | d µ < ∞, f ∈ L 1
对非负函数 g − f n 及 g + f n 应用 Fatou 引理,得到
Z Z Z Z Z
gd µ − f d µ ⩽ lim inf ( g − f n) dµ = gd µ − lim sup f n dµ
Z Z Z Z
f d µ ⩽ lim inf f n dµ ⇒
f n dµ f d µ = lim
R
由于 | f n − f | ⩽ 2 g, a.e..而 | f n − f | → 0, a.e..故 lim | f n − f | d µ = 0
n→∞
µ R R
推论. 若定理中 f n → f , a.e..改为 f n −
→ f (依测度收敛), f n d µ → f d µ 仍然
成立.
© ª © ª
Proof. 对 { f n } 的任一子列 f n k 存在子列 f nk j 使 f nk j → f , a.e..
R R R R
故 f nk j d µ → f d µ,由子列选取的任意性,可知 f n d µ → f d µ.
R R R
注意. 1. lim | f n − f | d µ = 0,这比 lim f n dµ = f d µ 更强.
n→∞ n→∞
1
我们称这样的 f n 是 L − 收敛到 f ,或者说平均收敛到 f ,同样的,要推出
平均收敛,在有控制情形下,几乎处处收敛也可改为依测度收敛.
R
2. 1[n,2n] → 0 但 1[n,2n] d µ = n ↛ 0,(µ 为 Lebesgue 测度),说明控制函数 g ∈ L1
是不可以省略的.
3. 与单调收敛定理类似,控制收敛定理对于级数求和也有类似结论,事实上,
取 µ 为计数测度积分就和求和一样了.
4. 定理的条件可以改为 f n 被两个可积函数的函数夹住,那么结论仍然成立。
5. 测度有限时,控制收敛定理的如下推广给出了点点收敛的函数极限与积分
可交换的充分必要条件。
134 CHAPTER 7. 积分与期望
¡ ¢
定义. 函数族 F ⊂ L1 µ 称为一致可积的,如果
Z
lim sup | f |d µ = 0
C →∞ f ∈F {| f |>C }

显然,有限测度空间中有限个可积函数或是被可积函数控制的函数族都是
一致可积的
¡ ¢
定理 7.8. Lebesgue-Vitali 定理:{ f n } ⊂ L1 µ ,µ (Ω) < ∞,f n → f , a.e.,则 f 可积
L1 L1
且 f n → f 的充分必要条件是 { f n } 一致可积。若 f n 非负,则 f n → f 的充分必要
R R
条件是 lim f n d µ = f d µ < ∞
n

下面给出了一致可积但不存在可积控制函数的一族函数:
£ ¤
令 A n = 2−n−1 , 2−n ,将其划分为 n 个等测度的可测集合 A 1n . . . A nn ,定义
f ni ( x) = 1x 1 A ni ( x),考虑函数列 f 11 , f 21 , f 22 , f 31 , . . .,不难验证它是一致可积的,同时也
处处 L1 收敛到 0。但控制该函数列的函数必然控制 x−1 ,所以不可能是可积的。
作为控制收敛定理的一个简单应用,我们证明 Lebesgue 积分的微积分基
本定理

定理 7.9 (微积分基本定理). 若 f : [a, b] → R 连续,F : [a, b] → R 可求导且导数


R
为 f ,则 [a,b] f dm = F ( b) − F (a)

Proof. 对自然数 n,将区间 n 等分,记对应的分割点为 c n,i = a + i (b − a)/ n,令 f n


R
在 [ c n,i , c n,i+1 ) 上取值 n(F ( c n,i+1 ) − F (C N,I )/(b − a)),容易计算得知 [a,b) f n dm =
F ( b)− F (a)。另一方面,导数性质表明存在 | t− u n ( t)| ≤ ( b − a)/ n 使得 f n ( t) = f ( u n ( t)),
所以有 | f n | ≤ max| f |,注意到 f 连续, f n f ,由控制收敛定理,极限与积分可以
R
交换,有 [a,b) f dm = F ( b) − F (a),因为 b 点测度为 0,结论成立

练习

1. g ( x) = 1
x ln x , x > 1. f n = c n 1 A n , c n ⩾ 0, A n 可测且 ⊂ [2, ∞)
R∞
证明或者说明下面结论不正确:f n ( x) → 0 且 ∀ x,| f n ( x)| ⩽ g ( x);则 2 f n ( x) dx →
0, n → ∞
2. 证明 Lebesgue-Vitali 定理。
¡ ¢
3. 证明有限测度空间中,函数族 F ⊂ L1 µ 一致可积的充要条件是 sup f ∈F ∫ | f | d µ <
∞(一致有界),且对任意 ε > 0,存在 δ > 0,使得任何满足 µ( A ) ⩽ δ 的 A
上的函数 f ∈ F 的积分小于 ε,即 sup f ∈F ∫ A | f | d µ ⩽ ε(一致绝对连续)。
7.3. 可积函数空间的性质 135

7.3 可积函数空间的性质
继续看 L1 空间,容易看出在可积函数全体中可定义如下的距离函数.
R
d ( f , g) = | f − g| d µ(视 f 1 = f 2 , µ − a.e. 的 f 1 和 f 2 为同一函数)
¡ ¢
所以,在以上距离下,L1 Ω, F , µ 就成了一个度量空间,L1 收敛即对应距
离下的收敛.
下面的定理说明了 L1 空间是完备的.

定理 7.10 (Riesz-Fischer 定理). L1 空间是完备的度量空间.

Proof. 类似依测度收敛的证明,我们要在 L1 空间的 Cauchy 列中找几乎处处收


敛的子序列.
R
设 { f n } 是 Cauchy 列,即 | f n − f m | d µ → 0, n, m → ∞
© ª∞ R¯ ¯
选择子序列 f n k k=1 使得 ¯ f n k+1 − f n k ¯ d µ ⩽ 2−k ,∀k ⩾ 1
∞ ¡
P ¢ ¯ ¯ P∞ ¯ ¯
定义 f ( x) = f n1 ( x)+ f n k+1 ( x) − f n k ( x) ,g ( x) = ¯ f n1 ( x)¯+ ¯ f n ( x) − f n ( x)¯
k+1 k
k=1 k=1
Z Z X Z Z X
¯ ¯ ∞ ¯ ¯ ¯ ¯ ∞
g ( x) d µ = ¯ f n ¯d µ + ¯ fn − f nk ¯d µ ⩽ ¯ f n ¯d µ + 2− k < ∞
1 k+1 1
k=1 k=1

而 | f | ⩽ g ⇒ | f | ∈ L1 ,同时还有 f n k ( x) → f ( x) a.e..
R¯ ¯
由控制收敛定理,也有 f n k 是 L1 收敛到 f 的,即 ¯ f n − f ¯d µ → 0
k

由 { f n } 为 Cauchy 列, 对 任 意 ε, 存 在 足 够 大 的 N , 当 n, m > N 时,
R R¯ ¯ R R
| f n − f m | d µ < 2ε ,若 n k > N ,并使得 ¯ f n k − f ¯ < 2ε ,有 | f n − f |d u ⩽ | f n − f nk | d µ+
R
| f nk − f | d µ < ε,只要 n > N ,得证.
L1
推论. 若 f n −→ f ,则存在子序列 f n k → f , µ − a.e..

推论. 空间 L1 是 Banach 空间

注意:由积分的线性性,不难知道 L1 也是线性空间,而且可以定义范数
R
k f k = | f | d µ(k f + gk ⩽ k f k + k gk,k c · f k = | c| k f k,k f k = 0 ⇔ f = 0)
,这样 L1 就
变成一个完备的赋范线性空间,即 Banach 空间.
我们现在从泛函的角度重新看一下控制收敛定理和其它几个收敛定理.
将积分写成 µ ( f ) 的形式.
1. 单调收敛定理 0 ⩽ f n ↑ f ⇒ µ (lim f n ) = lim µ ( f n )
2. Fatou 引理 f n ⩾ 0,则 µ (lim inf f n ) ⩽ lim inf µ ( f n )
136 CHAPTER 7. 积分与期望

3. 控制收敛定理 | f n | ⩽ g, f n → f ,则 µ (lim f n ) = lim µ ( f n )


这表明,泛函关于单调序列的逐点收敛,或者被控制的可积函数列的逐点收
敛表现出泛函的连续性,而在一般情况下,泛函关于非负可测函数列是逐点下半
连续的.然而 µ ( f ) 也是线性泛函,所以这表明线性泛函并不一定是连续的.
¯ ¯ ¯R ¯ R ¯ ¯
但是 µ ( f ) 是 L1 空间的线性泛函,¯µ ( f )¯ = ¯ f d µ¯,k f k = | f | d µ ⇒ ¯µ ( f )¯ ⩽
k f k.
故 µ ( f ) 作为 L1 空间的线性泛函,既是连续的,也是有界的.
R 1
思 考:1. 如 何 将 上 述 证 明 推 广 到 L p 空 间 ( | f | p d µ) p < ∞, d ( f , g) =
R 1
( | f − g | p d µ) p ), p > 1?
[本质上分两部分:一是说明在赋范线性空间中完备性等价于绝对收敛序列
收敛。二是利用单调收敛定理说明绝对收敛序列几乎处处收敛,并利用控制收敛
定理说明收敛。]
2. 与依测度收敛那个证明不同的地方在哪里?
注意. 1. 处处收敛而不是平均收敛的例子我们已经见过,下面构造一个处处
不收敛,但是平均收敛的例子。在 [0, 1] 上.给定 k > 0,将 [0, 1] 等分为 k
£ ¤ £ ¤ £ ¤ £ ¤
个区间, 0, 1k , 1k , 2k ,…, k−k 1 , 1 .再将所有这些区间排起来得到 0, 12 ,
£1 ¤ £ 1¤ £1 2¤ £2 ¤ £ 1¤ £1 1¤ £1 3¤ £3 ¤ £ 1¤
2 , 1 , 0, 3 , 3 , 3 , 3 , 1 , 0, 4 , 4 , 2 , 2 , 4 , 4 , 1 , 0, 5 ,…分别记
为 A 1 , A 2 , A 3 ,…, A n ,…,对应定义 f n = 1 A n .不妨令 A n 对应的是 k n
L1
等分的区间,则 n → ∞,k n → ∞, f n −→ 0,但是 ∀ x ∈ [0, 1],总是属于无穷
多个 { A n } 中的集合,于是 f n 在 E 上无一点收敛于 0 .
2. 现在来看一下 L1 收敛,L p 收敛以及依测度收敛的关系
假定全空间测度有限,
L p 收敛, p > 1 ⇒ L1 收敛 ⇒ 依测度收敛;只要 p > q > 1,就有 L p 收敛
⇒ L q 收敛
这里只证明 L1 收敛 ⇒ 依测度收敛,大家可以尝试用 Holder 不等式证明其
他部分(Holder 不等式也表明 ∞ > p > q ⩾ 1 时,L p ⊂ L q 。这个关系恰与
l p ⊃ l q 相反,不过 l p 看成 L p 的特殊情形时,对应的测度不是有限的。若无
测度有限条件,通常 L p ≮ L q ,例如 (0, ∞)[Lebesgue 测度] 上的 f t ( x) = x−t ,
则 f t 1(0,1) ∈ L p ⇔ p < t−1 , f t 1(1,∞) ∈ L p ⇔ p > t−1 ,说明一个函数不在 L p 中
的两种可能:一是在靠近某一点时 | f | p 增长太快,二是在无穷远处趋于过
慢。第一种情况下 p 越大函数越不可能在 L p 中,而后一种是 p 越小函数
越不可能在 L p 中,所以通常情况下 L p 、L q 间的包含关系无法确定)
7.3. 可积函数空间的性质 137
R R
令 E n,ε = { x : | f n − f | ⩾ ε},则 | f n − f |d µ ⩾ E n,ε | f n − f | d µ ⩾ εµ(E n,ε ),故
Z
−1
µ(E n,ε ) ⩽ ε | fn − f | → 0

利用这个结果,可以证明 L p 空间完备性。
(首先说明 L p 收敛 Cauchy 列是
依测度收敛 Cauchy 列,其中有几乎处处收敛子序列,然后证明其极限就是
L p 收敛极限)

再来看可积空间 L1 的一些稠密子集.[结论也适用于 L p 空间吗?]

定理 7.11. 下面几类函数是 L1 (R n ) 中的稠密子集,即 ∀ f ∈ L1 ,ε > 0 可找到下


R¯ ¯
面函数类 Φ 中的函数 ϕ,使得 ¯ f − ϕ¯ d λ < ε.1) 简单函数, 2) 阶梯函数,3) 具
有紧支撑集的连续函数,支撑集指 supp ( f ) = { x : f ( x) 6= 0}.[可以进一步要求函数
的光滑性]

Proof. 因对一般函数可分 f + 和 f − 分别考虑,故不妨设 f ⩾ 0


R
同时,由控制收敛定理总有 lim | f 1[−n,n] − f | = 0,即我们可假定 f 有
n→∞
紧支集

1. 简单函数,∀ f ⩾ 0 且 f ∈ L1 ,存在上升的简单函数列 ϕk ↑ f ,故控制收敛定


R¯ ¯
理表明 ¯ f − ϕk ¯ d λ → 0, k → ∞.这说明简单函数族在 L1 中稠密.由于 f
有紧支集,这里的简单函数也有紧支集
2. 只需证明对任意有紧支集的简单函数可以用阶梯函数逼近,由于简单函数只
是有限个特征函数的线性和.所以只需用阶梯函数逼近特征函数,因为对任
³ ´ SN
意有界可测集 E ,存在有限个互不相交的矩体 B j ,使 m E ∆ j =1
B j < 2ε,
R¯ P ¯
表明 ¯1E − j 1B j ¯ dm < 2ε.
3. 注意阶梯函数对应矩体都是有界的,可做比它略大一点点的开矩体,由连续
函数延拓性质,可做一连续函数,在里面的闭矩体上取值 1,大的开矩体外
取,中间部分取到 1 之间,这是一个有紧支撑的函数,而且与闭矩体上的特
征函数可以充分接近,故具紧支撑的连续函数也是稠密的.

推论. C [a, b] 在 L1 ([a, b]) 中稠密,而且给定 f ∈ L1 [a, b],ε > 0,存在有理系数


Rb ¡ ¢
的多项式 p 使 a | f − p| dm < ε.故 L1 [a, b] 是可分空间.(实际上 L1 R 1 也是
可分的)
138 CHAPTER 7. 积分与期望
R
推论. 如果对于任意具有紧支撑的函数 g,可积函数 f 满足 g f = 0,则 f = 0

Proof.

注意. 1. 上述定理实际上给了我们另外一种构造 Lebesgue 积分的方法,或者


说理解角度.
记 S ( J ) 代表所有阶梯函数的全体,并视只有有限个点不同的阶梯函数
R
为同一函数,那么不难证明 S ( J ) 是一个赋范线性空间 k f k = | f ( t)| dt =
Pm
k=1
| c k | | Jk |
L ( J ) 就代表 Lebesgue 可积函数的全体,是 S ( J ) 在范数 k·k 下的完备化.
1

L1 ( J ) 中的每个元素或者说可积函数对应于 S ( J ) 中函数形成的 Cauchy 列


{S n }.
R ¯ ¯
¯S j − S k ¯ dt → 0, j , k → ∞
J
R R
这个元素的积分就定义为 J f dm = lim J S n ( t) dt
R R n→∞
范数 k f k = J | f | dm = lim J |S n ( t)| dt
n→∞
2. 可积函数可由其它较为简单的函数如简单函数或连续函数逼近的事实使得
我们在证明中可以将可积函数分解为两部分进行考虑,参见课本上定理 4.12
,196 页的例 4(Riemann-Lebesgue 引理的推广)等等。
(积分的绝对连续性)
¡ ¢
如积分的绝对连续性:若 f ∈ L1 µ ,则 ∀ε > 0,∃δ > 0 使得:只要 µ (D ) < δ,
就有 ∫ | f | d µ < ε[将函数分解为简单函数 f 1 和残余部分 f 2 ,对于 f 1 结论显
D
然 (有限个示性函数的和),而 f 2 的绝对值的积分可以任意小]
3. 我们简单说明一下 2 中提到的 Riemann-Lebesgue 引理在 Fourier 分析中的
应用:
R
定义. Fourier 变换: f ∈ L1 (R ), fˆ( y) = (2π1)1/2 R e− i yx f ( x) dx
R
逆变换: f ∈ L1 (R ), fˇ( x) = (2π1)1/2 R e i yx f ( y) d y

在适当条件下,总有 f ( x) = (2π1)1/2 ∫ fˆ( y) e ix y d y


不难得到,如果 f ∈ L1 (R ),控制收敛定理表明 fˆ( y) 是连续的,而 Riemann-
Lebesgue 引理说明 lim fˆ( y) = 0,相当于 fˆ ∈ C0 (R )
| y|→∞
R
Fourier 级数: f ∈ L1 (R ), fˆ(n) = R e− inx f ( x)dx 称为 Fourier 系数
P

f 的 Fourier 级数是 fˆ( n) e inx ,我们关心该级数什么时候以何种方式收
n=−∞
敛于 f ( x)
7.4. 对偶空间 139

[1922 年,年仅 19 岁的 Kolmogrov 给出了一个 Fourier 级数几乎处处发散


的例子,1966 年,Carleson 证明了 Luzin 于 1915 在其博士论文中提出的猜
想 f ∈ L2 (R ) 的 Fourier 级数几乎处处收敛]
下面练习中的定理给出了点点收敛的一个必要条件:

练习
P

1. Cantor-Lebesgue 定理:假定在具有正测度的集合 E ⊂ [a, b] 上 a n cos( nx + θn )
n=1
处处收敛,证明 lim a n = 0(反证法,说明有子序列 cos(nx + θn ) → 0,利用
n→∞
控制收敛定理和 Riemann-Lebesgue 引理)
2. T 为距离空间,f : X × T → R,而且对任意的 t ∈ T ,f t : x 7→ f ( x, t) 是可积的。
对任意的 x ∈ X ,映射 t 7→ f ( x, t) 连续。且存在可积函数 g( x) 使得对于所有
R
x,| f ( x, t)| ≤ | g( x)|,证明 Φ( t) = X f ( x, t) d µ( x) 连续。
3. 判定函数可积的一些方法:
P

若假定 µ 有限,可测函数 f 可积等价于级数 µ ( x : | f ( x)| ⩾ n) 或者级数
n=1
P

nµ( x : n + 1 > | f ( x)| ⩾ n) 收敛。
n=1
对于一般的测度, f 可积等价于 t 7→ µ( x : | f ( x)| > t) 在 (0, ∞) 上 Lebesgue 可
R R
积,且 | f | d µ = 0∞ µ( x : | f ( x)| > t)dt.
[最后的等式表明 Lebesgue 积分可以通过 Riemann 积分定义,因为 t 7→ µ( x :
| f ( x)| > t) 是单调函数,是 Riemann 可积的]

7.4 对偶空间
设 V 为一赋范线性空间。V 上的连续线性泛函的全体可以看成是 V 上的所有线
性泛函的子空间,称为 V 的对偶空间 (dual space),记为 V ∗ 。
对于 V 上的连续线性泛函 F ,定义:
|F ( x)|
kF k = inf{K : ∀ x ∈ V , |F ( x)| ⩽ K k xk} = sup{ : x 6= 0}
k xk

则其为 F 在 V ∗ 中的范数。即 V ∗ 也是一赋范线性空间。


¡ ¢
定 理 7.12. (1) 若 1 < p < ∞, 1p + 1q = 1, 则 对 任 意 F ∈ (L p Ω, F , µ )∗ , 存 在
¡ ¢ ¡ ¢ R
g ∈ L q Ω, F , µ 使得对任意 f ∈ L p Ω, F , µ ,F ( f ) = f gdµ。
140 CHAPTER 7. 积分与期望
¡ ¢
(2) 对于 L1 Ω, F , µ 上的任一连续线性泛函 F ,如果 Ω 是 σ 有限的,我们
¡ ¢ R
可以将该泛函表示成积分的形式,即存在 y ∈ L∞ Ω, F , µ 使 F ( x) = x yd µ.
© ª
注意. L∞ = f 可测, 且存在正的有限常数K, 使 k f ( x)k ⩽ K, a.e.
k f k∞ = inf {K : | f ( x)| ⩽ K, a.e.} 称为 | f | 的本性上确界.
¡ ¢
空间 L∞ Ω, F , µ 与有界可测函数空间(赋予极大值范数)类似但不同
:F 是 C[0,1](Banach 空间)上的连
一个类似的结论(Riesz-Markov 定理)
R
续线性泛函,当且仅当在 [0,1] 上存在符号 Borel 测度 µ,使得 F ( f ) = [0,1] f dµ

1. 现在我们利用上述对偶定理的结论证明 Radon-Nikodym 定理

定理 7.13 (Radon-Nikodym 定理). (Ω, F ) 是可测空间,v,µ 是 F 上的两


个概率测度,v 对 µ 绝对连续,即若 µ ( A ) = 0 则有 v ( A ) = 0.那么,存在
R
非负可积函数 g 使得 v (E ) = E gd µ。( g 在 µ − a, e 相等的意义上唯一,即
dv
Radon-Nikodym 导数 dµ )

¡ ¢ R ¡ ¢
Proof. 考虑 Hilbert 空间 L2 v + µ ,则 l ( x) = xd µ 是 L2 v + µ 上的连续
R R¡ ¢ ¡ ¢
线性泛函,由前述表示定理 xd µ = x yd µ + v , y ∈ L2 µ + v ,∀ x.
R R ¡ ¢
⇒ x (1 − y) d µ = x ydv,∀ x ∈ L2 µ + v ⇒ 0 ⩽ y ⩽ 1,µ, a.e
在 µ− 零 测 集 上 修 改 y 使 之 满 足 y > 0, 因 v 对 µ 绝 对 连 续, 仍 成 立
R R
x (1 − y) d µ = x ydv.
1− y R R ¡ ¢
令 u = x y, g = y ,则有 udv = u gd µ, g ∈ L1 µ ,取 u = 1E ,即有
R
v (E ) = E gd µ.

注意. 1. 证明中得到的 g 也是概率测度 v 关于 µ 的概率密度函数。


2. 与条件期望的关系:

定义. 条件期望的定义:(Ω, F , P ) 为概率空间,H 为 F 的子 σ 代数, X 可


积,则给定 H 的 X 的条件期望为满足下列条件的随机变量(在几乎处处相
等的意义上是唯一的):
R R
(1)E [ X | H ] 为 H 可测且可积 (2)∀ A ∈ H , A X dP = A E [ X |H ] dP

条件期望与 Radon-Nikodym 导数:对于可积的 X ,定义不定积分 v ( A ) =


R
A X dP , A ∈ F ,则 v 关于 P 绝对连续.若仅限于子 σ 代数 H 上,v 亦关
于 P 绝对连续,则 E [ X | H ] 即为 v 关于 P 在 (Ω, H ) 上的 Radon-Nikodym
导数
7.5. RIEMANN 积分与 LEBESGUE 积分 141

2. 对偶空间与弱收敛
X 是赋范线性空间,{ xn } 为其中的序列,我们知道 || xn − x|| → 0 意味着 xn
依范数收敛到 x。
w
定义. 称 { xn } 弱收敛到 x ∈ X 如果对任意 φ ∈ X ∗ ,φ( xn ) → φ( x)。记为 xn −
→ x。

7.5 Riemann 积分与 Lebesgue 积分


我们将讨论限制在区间 [a, b] 上,但实际上很多结论可以拓展到高维的有界集
合上.
Rb Rb
我们用 a f d λ 代表 Lebesgue 积分, a f dx 代表 Riemann 积分
首先用上、下积分的方式重新给出 Lebesgue 积分的定义.

定理 7.14. 假设 nf 是 [a, b] 上有界的可测函数,定义 o R− nR o


R Rb b
f d λ = sup a Ld λ : L ⩽ f , L简单函数 , f d λ = inf a U d λ : f ⩽ U,U 简单函数 ,

R R− R
则 f dλ = f dλ = f dλ

Proof. 令 M n= sup | f | < ∞,选定 n ∈oN ,∀k ∈ [−n, n],


S
令 E k = x : nk M ⩾ f ( x) > (k−n 1) M ,则 E k 互不相交,可测且 [a, b] = −n n E k ,
M P
n
M Pn
定义 Un = n k 1E k , L n = n − n ( k − 1) 1E k
−n
R− Rb P
n
则简单函数 Un ,L n 满足 L n ⩽ f ⩽ Un , f d λ ⩽ a Un d λ = M
n kλ (E k )
−n

Z Z b MX n
f dλ ⩾ L n dλ = ( k − 1) λ (E k )
a n −n

Z− Z
MX n M
⇒0⩽ f dλ − f dλ ⩽ λ (E k ) = ( b − a ) → 0, n → ∞
n −n n

注意. 之前 Lebesgue 积分的定义实际上是下积分,定理表明在测度有限时,有


界函数的积分也可通过上、下积分相等而给出.这里的测度有限不可以改成测
度无限,也就是说,全空间测度无限时,比如 R ,不能用上、下积分方式定义
Lebesgue 积分,因为上积分 (对于正函数 f ),总是 +∞.
142 CHAPTER 7. 积分与期望

P
n S
n
回忆阶梯函数 f = c k 1 I k ,[a, b] = I k ,I k 互不相交,I k 为区间,是特殊
k=1 1
形式的简单函数.
下面我们通过阶梯函数的形式给出 Riemann 可积的定义

定义. 设 f 为 [a, b]n 上的函数,令 o


R Rb
f ( t) dt = sup a R ( t) dt : R ⩽ f , R 阶梯函数 ,

R− nR o
b
f ( t) dt = inf a s ( t) dt : f ⩽ s, s阶梯函数 ,
R R− Rb
如果 f ( t) dt = f ( t) dt,则 f 是 Riemann 可积的,记为 a f ( t) dt.

比较 Riemann 积分与 Lebesgue 积分,差别在于阶梯函数与简单函数的区


别,于是由简单函数包括阶梯函数,可得如下不等式.
Z Z Z− Z−
f ( t) dt ⩽ f dλ ⩽ f dλ ⩽ f ( t) dt
− −

由此可以知道,Riemann 可积则 Lebesgue 可积,且两者积分值相等


回忆对任意可测函数的逼近,可以有一列简单函数逐点收敛,如果可测函
数有界,则成为一致收敛,而对于阶梯函数列,只能有几乎处处收敛,比如若
f = 1Q C T [a,b] ,可以由 1[a,b] 从上面逼近,注意简单函数列到可测函数的收敛中
可以要求函数列单调,而对于阶梯函数列,若还要求单调,则几乎处处收敛都
不一定能满足,比如同样 f = 1Q C T [a,b] ,对于小于等于 f 的阶梯函数,最接近的
只能是 R ≡ 0(在 [a, b] 上)。不难想象,如果 f 也能够由阶梯函数列较好的逼近,
Riemann 积分和 Lebesgue 积分就一样了.下面的目标就是说明这样的 f 应当是
几乎处处连续的.
设 f 在 [a, b] 上有界, x0 ∈ [a, b],δ > 0,记

m δ ( x0 ) = inf { f ( x) : x ∈ ( x0 − δ, x0 + δ) ∩ [a, b]}

Mδ ( x0 ) = sup { f ( x) : x ∈ ( x0 − δ, x0 + δ) ∩ [a, b]}

定义 m ( x0 ) = limδ→0 m δ ( x0 ),M ( x0 ) = limδ→0 Mδ ( x0 ),ω ( x0 ) = M ( x0 ) − m ( x0 ),分


别称为 f 的下边界,上边界和振幅.注意 m,M ,ω 与 lim inf f ( x)、lim sup f ( x)、
x→ x0 x→ x0
lim sup f ( x) − lim inf f ( x) 不同,差别是后面的式子没有考虑 x0 点的函数取值.可
x→ x0 x → x0
以看出 f 在 x0 连续 ⇔ ω ( x0 ) = 0.
先证明对任意函数 f ,阶梯函数最多只能逼近到它的下、上边界.
7.5. RIEMANN 积分与 LEBESGUE 积分 143

引理. f 在 [a, b] 上有界,m 为 f 的下边界,则 m 是 Lebesgue 可测的,且若阶


R R
梯函数 R ⩽ f ,则在 R 的任意连续点 x 上有 R ( x) ⩽ m ( x), f ( t) dt = ab md λ.

Proof. m 可测,若 m ( x0 ) > α,则 ∃β > α 使 f 在 x0 的某个开邻域内 f > β,故


其上 m > α ⇒ { x : m ( x) > α} 开集.若 x0 为 R 的连续点,则应当 ∃ 某个 δ 使
( x0 − δ, x0 + δ) 上 R 取常数,而 R ⩽ f ⇒ m δ ( x0 ) ⩾ R ( x0 ),故有 R ( x0 ) ⩽ m ( x0 ).于
R R R
是 f ( t) dt ⩽ md λ = ab md λ,只需证反向不等式成立.给定 n ∈ N .将 [a, b] 2n
− −
等分为 { I k },只第一个为闭,其它为半开闭区间.定义 R n 在 [a, b] 上,在 I k 上
S

取值为 inf { f ( x) : x ∈ I k },R n 为阶梯函数且 R n ⩽ f ,D n 为分割点,则 D = Dn
n=1
是可数的.
设 x0 ∈ D C ,α < m ( x0 ),可选 δ > 0 使 m δ ( x0 ) > α,记 I n ( x0 ) 为第 n 次分
划时包含 x0 的区间,则 n 足够大时,I n ( x0 ) ⊂ ( x0 − δ, x0 + δ) ⇒ m ( x0 ) ⩾ R n ( x0 ) ⩾
m δ ( x0 ) > α
注意到对任意 δ → 0,都有 n 满足上式,可知 lim R n ( x0 ) = m ( x0 ),∀ x0 ∈ D C
n→∞
Rb
故 R n → ma.e..注意 m 有界, m 是可积的,控制收敛定理 ⇒ a R n dλ →
Rb Rb
md λ.而对于阶梯函数.Riemann 积分与 Lebesgue 积分一样.故 R n ( t) dt →
Rab R Rb Rb a

a md λ,又 f ⩾ R n 有 f ( t) dt ⩾ lim a R n d λ = a md λ,得证.


− n→∞

注意上述引理对于 f 的上边界 M 有类似结论.

定理 7.15. f 为 [a, b] 上的有界函数,则 f 是 Riemann 可积的,当且仅当 f 几


R R
乎处处连续,而且此时 ab f ( t)dt = ab f d λ.
R Rb Rb Rb R−
Proof. 由引理, f ( t) dt = a md λ ⩽ a f dλ ⩽ a Md λ = f ( t) dt(*)

Rb
f 是 Riemann 可积 ⇔ a ( M − m) d λ = 0 ⇔ M = ma.e. ⇔ f 几乎处处连续.
而 f Riemann 可积时,(*) 中不等号都为等号.故结论成立.

练习

1. 记 [a, b] 上的所有 Riemann 可积函数全体为 R [a, b],直接证明.R [a, b] 是


Rb
R 上的向量空间, f 7→ a f ( t) dt 是 R [a, b] 上的线性泛函.
2. 举例说明 Riemann 可积函数不一定是 Borel 可测的
3. 举例说明存在一个 Lebesgue 可积函数,任意与它几乎处处相等的函数都不
是 Riemann 可积的
144 CHAPTER 7. 积分与期望
R R
4. 若 f ∈ L1 (R ),证明 f dλ = lim [a,b] f d λ(说明了 Lebesgue 积分应
a→−∞,b→+∞
该怎么求,是 Riemnann 积分的极限)
5. f 在 [0,1] 上 Lebesgue 可积,{Πn = { Jn,k , k ⩽ Nn }} 是 [0,1] 上不断加细的划
Pn
N
分序列,λ( Jn,k ) ⩽ δn → 0,证明存在点 ξn,k ∈ Jn,k ,使得 | f (ξn,k )λ( Jn,k ) −
R k=1
[0,1] f d λ| → 0 [找一列连续函数 L1 逼近 f ,并且不等于 f 的点的测度趋于,
再利用 [0,1] 上连续函数的一致连续性选取点]
值得指出的是,前面定理表明 [a, b] 上的 Riemann 积分是 Lebesgue 积分的
特殊情形.但对于无穷区间上的反常积分,广义 Riemann 积分与 Lebesgue 积分
就不一样了,因为 Lebesgue 可积是绝对可积的,而 Riemann 积分并不然(但如
果函数绝对值是广义 Riemann 可积的,那么广义 Riemann 积分与 Lebesgue 积
分一样,大家可将其作为练习).
例如计算 Z ∞ sin x π
dx =
0 x 2
注意,
Z ¯ ¯
∞ ¯ sin x ¯ Z kπ Z
¯ ¯ = +∞因 |sin x| 1 kπ 2
¯ x ¯ dx ⩾ |sin x| dx =
0 (k−1)π x kπ (k−1)π kπ


Z Z
n
− tx sin x
n
− tx e−nt ( t sin n + cos n) − 1
f n ( t) = e dx, f n1 ( t) = − e sin xd x = ,
0 x 0 1 + t2
1 ¯¯ 1 ¯¯ 1 1 + t −t
lim f n1 ( t) = − , f n ( t) ⩽ + e
n→∞ 1+ t 2 1+ t 2 1 + t2
故 f n1 可积,令 g n = f n1 1[0,n] ,
Z Z ∞ Z ∞
n 1 π
lim f n1 ( t) dt = lim g n ( t) dt = − dt = −
0 0 0 1+ t 2 2
Rn Rn
因 0 f n1 ( t) dt = f n ( n) − f n (0),| f n ( n)| ⩽ 0 e− xn dx ⩽ 1
n ⇒ f n ( n) → 0, f n (0) → π2
Z n+1 Z n+1 Z n+1
sin x sin x cos n cos ( n + 1) cos x
dx = f n (0) + dx = f n (0) + − − dx
0 x n x n n+1 n x2
R∞
n → ∞ 可知 0 sinx x dx = π2 .
其中用到定理:
7.6. RIESZ-MARKOV 表示定理 145

定理 7.16.¯ f ( x, y¯) 定义在 E × (a, b) 上,f (·, y) 在 E 上可积,f ( x, ·) 可微,若有可积


¯ ¯ R R
函数 g 使 ¯ ∂ f ∂(x,y)
y ¯ ⩽ g ( x),∀ ( x, y) ∈ E × (a, b).则
d
dy E f ( x, y) dx = ∂
E ∂y f ( x, y) dx.
∞ R
P P∞
类似的一个定理是若 | f k ( x)| dx < ∞,则 1 f k ( x) 几乎处处收敛于函数
n=1
∞ R
P R
f ∈ L1 且 f k ( x) dx = f ( x) dx.
k=1

我们知道 Lebesgue 积分与 Lebesgue 测度关系密切.下面的定理证明 Rie-


mann 积分与 Jordan 可测集之间有类似 Lebesgue 积分与 Lebesgue 可测集的
关系.

定理 7.17. 有界集 E 是 Jordan 可测的 ⇔ 如果 1E 是 Riemann 可积的.

Proof. (只证明一维情形) 不妨设 E ⊂ [a, b].


“⇒”Jordan 可测是指 Jordan
½ 外测度与内测度相等的集合. ¾
∗ P
n S
n
Jordan 外测度 v ( A ) = inf |B i | : A ⊂
B i , {B i } 是有限个互不相交 (半开闭) 矩体
½ i =1 i =1 ¾
Pn Sn
内测度 v∗ ( A ) = sup |B i | : A ⊃ B i , {B i } 是有限个互不相交 (半开闭) 矩体
i =1 ½ i =1 ¾ ½ ¾
n R
P S n R
P S
E 为 Jordan 可测表明 sup 1B i ( t) dt, E ⊃ B i = inf 1B i ( t) dt, E ⊂ B i
i =1 i =1
n R
P R P
n P
n S
注意 1B i ( t) dt = 1B i ( t) dt, 1B i ( t) 是阶梯函数,E ⊃ B i ⇒ 1E ⩾
i =1 i =1 i =1
P
n
1B i 不难看出,1E 的 Riemann 积分恰好处于上面等式两端的数之间,故 1E
i =1
是 Riemann 可积
“⇐” sup{∫ab R ( t)dt : R ⩽ 1E , R 阶梯函数} = inf{∫ab s( t)dt : 1E ⩽ s, s阶梯函数},
因所有 ⩽ 1E 的阶梯函数在 E 上取值都 ⩽ 1,在 E c 上取值都 ⩽ 0.
R P ¯R ¯
R n dt → sup {},则存在 R n = a i 1B i ¯ R n dt − sup {}¯ < ε,显然
若有 R n 使
0 P R 0
R n = 1B i 也满足 sup {} −n R n dt < ε, o
R
这表明 v∗ (E ) = sup ab R ( t) dt : 阶梯函数R ⩽ 1E ,类似方法可证 v∗ (E ) =
inf {}

7.6 Riesz-Markov 表示定理


定理 7.18. 设 I 为 R n 空间上有紧支集的连续函数全体 C C (R n ) 上的正线性泛
函,则在 R n 上存在唯一的,正则的 Borel 测度 µ,使 µ (K ) < ∞ 对所有紧集
R
K ⊂ R n 成立,则对所有的 f ∈ C C (R n ), I ( f ) = f d µ.
146 CHAPTER 7. 积分与期望
© ª
注意. 正则即指对任意开集 E 有 µ (E ) = sup µ (K ) : K ⊂ E, K (内正则)
© ª
对任意 Borel 集 E 有 µ (E ) = inf µ (U ) : U ⊃ E,U (外正则)
正泛函的意思是只要 f ⩾ 0 处处大于,则 I ( f ) ⩾ 0.
C C (R n ) 空间上的范数定义为 k f k = sup { f ( x) : x ∈ R n }

Proof. 1. 首先证明唯一性,假定 µ 和 v 都是 R n 上的正则 Borel 测度,且


R R
f dµ =
f dv = I ( f ) 我们要证明 µ 和 v 和开集上都是一致的. 事实上若 U 为开集,
©R ª
只要 f ⩽ 1U ,就有 I ( f ) ⩽ µ (U ),所以 µ (U ) ⩾ sup f d µ : f ∈ C C (R n ) , 0 ⩽ f ⩽ 1U
另一方面若 K 紧且 K ⊂ U ,那么由 Urysohn 引理,可知存在 f ∈ C C (R n )
R
在 K 上取值为 1,而且 f < 1U ,即有 µ (K ) ⩽ f d µ = I ( f ),由于 µ 是内正则的,
µ (K ) → µ (U ).故
½Z ¾
¡ ¢
µ (U ) ⩽ sup f dµ : f ∈ CC R n
, f < 1U

这就说明在开集 U 上,µ (U ) = v (U ) = sup { I ( f ) : f ∈ C C (R n ) , f ⩽ 1U }


又根据 µ 的外正则性,所有 Borel 集的测度由开集决定,故在 Borel 集上 µ,
v 也一致.
2. 唯一性的证明提供了构造的思路,即先通过 I 构造测度 µ 再将 I 表示为积
分形式,更具体的,先在开集 U 上定义 µ (U ) = sup { I ( f ) : f ∈ C C (R n ) , f ⩽ 1U },然
© ª
后在任意集合 E 上定义 µ∗ (E ) = inf µ (U ) : U ⊃ E,U ,显然开集 U ⊂ V ⇒ µ (U ) ⩽
µ (V ),故 U 开时 µ ∗ (U ) = µ (U ),然后逐次证明 (1).µ 在开集上是外测度,
(2).µ∗ 为外测度,
(3).开集是 µ∗ − 可测的,从而 µ∗ 成为 Borel 测度,对于可测集 µ∗ 可写为
µ
(4).µ 是内正则的,且 µ (K ) = inf { I ( f ) : f ∈ C C (R n ) , f ⩾ 1K },K 紧.
R
(5). I ( f ) = f d µ,若 f ∈ C C (R n ).
(1).对于 µ,µ (;) = 0,及 U1 ⊂ U2 ⇒ µ (U1 ) ⩽ µ (U2 ) 是显然的.
S∞ P∞ ¡ ¢
只要证若 U = 1 U j ,则 µ (U ) ⩽ 1 µ U j 。若 f ⩽ 1U ,K = supp ( f ) 为 f 的
支撑集.
S∞ S∞
则K⊂ 1 U j ,由于 K 是紧集,K ⊂ U j .可以找到 g 1 ,…,g n ∈ C C (R n ),
1
P P
g j ⩽ 1U j , 1n g j = 1(在 K 上),那么这样就有 f = 1n f g j , f g j ⩽ 1U j
P ¡ ¢ P ¡ ¢ P ¡ ¢ P∞ ¡ ¢
故 I ( f ) = 1n I f g j ⩽ 1n µ U j ⩽ ∞
1 µ U j ⇒ µ (U ) ⩽ 1 µ U j ( g 1 ,g 2 的找
S
法,找紧集 K 1 ⊂ U1 ,K 2 ⊂ U2 ,K = K 1 K 2 .由 Urysohn 引理找 1K i ⩽ h i ⩽ 1Ui ,
g 1 = h 1 , g 2 = h 2 − min ( h 1 , h 2 ), h i ∈ C C (R n ),归纳即得)
7.6. RIESZ-MARKOV 表示定理 147
¡S∞ ¢
(2).µ∗ (;) = 0, A ⊂ B ⇒ µ∗ ( A ) ⩽ µ∗ (B) 也是显然的,要证 µ∗ 1 Ai ⩽
P∞
1 µ∗ ( A i ) 为此,选取开集 U i ⊃ A i ,µ (U i ) ⩽ µ∗ ( A i ) + 2− i ε.则
¡[∞ ¢ ¡[∞ ¢ X∞ X∞ ∗ X∞ − i X∞ ∗
µ∗ 1 Ai ⩽ µ 1 U i ⩽ 1
µ (U i ) ⩽ 1
µ ( A i ) + 1
2 ε = 1
µ (Ai) + ε

由 ε 的任意性,得证.
(3).由于 µ∗ 为外测度,对任意集合 E 都有 µ∗ (E ) = µ∗ (E ∩ A ) + µ∗ (E ∩ A c )
的集合 A 称为 µ∗ 可测集,其全体为 σ 代数,µ∗ 在其上成为测度即可数可加的.
为证开集为 µ∗ − 可测.
只需对任意集合 E 和开集 U ,µ∗ (E ) ⩾ µ∗ (E ∩ U ) + µ∗ (E ∩ U c ).µ∗ (E ) < ∞,先
设 E 也是开的,那么 E ∩ U 是开集,给定 ε > 0,可找到 f ∈ C C (R n ) 使 f ⩽ 1E T U ,
I ( f ) ⩾ µ (E ∩ U ) − ε,K = supp ( f ) 是紧集.K c 是开集且 K c ⊃ U c
因 E ∩ K c 开集,找到 g ∈ C C (R n ), g ⩽ 1E T K c , I ( g) ⩾ µ (E ∩ K c ) − ε,
此时 f + g ⩽ 1E ⇒ µ (E ) ⩾ I ( f + g) = I ( f ) + I ( g) ⩾ µ (E ∩ U ) + µ (E ∩ K c ) − 2ε
¡ ¢
⩾ µ ∗ ( E ∩ U ) + µ ∗ E ∩ U c − 2ε

ε 是任意的,故 E 是开时,µ∗ (E ) ⩾ µ∗ (E ∩ U ) + µ∗ (E ∩ U c )
若 E 是任意集合,找开集 V ⊃ E ,并且 µ (V ) < µ∗ (E ) + ε,这样就有
¡ ¢ ¡ ¢
µ∗ (E ) + ε > µ (V ) ⩾ µ∗ (V ∩ U ) + µ∗ V ∩ U c ⩾ µ∗ (E ∩ U ) + µ∗ E ∩ U c

再次由 ε 的任意性得知 µ∗ (E ) ⩾ µ∗ (E ∩ U ) + µ∗ (E ∩ U c ).
(4).设 K 为紧集, f ∈ C C (R n ), f ⩾ 1K .令 Uε = { x : f ( x) > 1 − ε},则 Uε 为
开集,若 g ⩽ 1Uε
± ±
有 f (1 − ε) − g ⩾ 0 ⇒ I ( g) ⩽ I ( f ) (1 − ε),因为 K ⊂ Uε ,故

µ (K ) ⩽ µ (Uε ) ⩽ (1 − ε)−1 I ( f ) ⇒ µ (K ) ⩽ I ( f )

反之,对于任意包含 K 的开集 U ,可找到 f ∈ C C (R n ),使得 1K ⩽ f ⩽ 1U ⇒


I ( f ) ⩽ µ (U )
由 U 的任意性而且 µ 的外正则性,有 ε + µ (K ) ⩾ I ( f )
这表明对任意紧集 K ,µ (K ) = inf { I ( f ) : f ∈ C C (R n ) , f ⩾ 1K }
故 K 为紧集时 µ (K ) < ∞。在开集 U 上,任取 α < µ (U ),可选 f ∈ C C (R n )
使 f ⩽ 1U , I ( f ) > α,记 K = supp ( f ).若 g ∈ C C (R n ),且 g ⩾ 1K .则 g − f ⩾ 0,
⇒ I ( g) ⩾ I ( f ) > α ⇒ µ (K ) > α,故 µ (K ) 可任意逼近 µ (U ) 表明 µ 的内正则性.
148 CHAPTER 7. 积分与期望
R
(5).下面证表示定理,只要证 f ∈ C C (R n , [0, 1]) 时 I ( f ) = f d µ 成立就可以,
其它情形是这些函数的线性组合. 

 f ( x) ⩽ ( n − 1) ε
 0
设 ε 为任意小的某个正数,定义 f n ( X ) = f ( x) − ( n − 1) ε ( n − 1) ε < f ( x) ⩽ nε


 ε nε < f ( x)
P n
那么 f = n f n ,而且 f n ∈ C C (R ),存在 N ,使 n > N 时 f n = 0.
记 K 0 = supp ( f ),K n = { x : f ( x) ⩾ nε} 也为紧集,则

ε1K n ⩽ f n ⩽ ε1K n−1


R R R R
显然 ε1K n d µ ⩽ f n d µ ⩽ ε1K n−1 即 εµ (K n ) ⩽ f n d µ ⩽ εµ (K n−1 ),
而根据 (4) 的结论 εµ (K n ) ⩽ I ( f n ),另外可找到开集 U ⊃ K n−1 ,使 µ (U ) <
µ (K n−1 ) + δ,f n ⩽ ε1K n−1 ⩽ ε1U ⇒ µ (U ) ⩾ 1ε I ( f n ) ⇒ εµ (K n−1 ) ⩾ I ( f n ) 由 δ 的任意性
P P P
由 f = n f n 可知,ε 1N µ (K n ) ⩽ I ( f ) ⩽ 0N −1 εµ (K n )
XN Z X N −1
ε 1 µ (K n ) ⩽ f d µ ⩽ 0 εµ (K n )
¯ R ¯ ¡ ¢
所以 ¯ I ( f ) − f d µ¯ ⩽ ε µ (K 0 ) − µ (K N ) ⩽ εµ (K 0 ) → 0
R
故 I ( f ) = f d µ.

注意. 1.在证明中测度的构造不能直接用 µ (E ) = I (1E ),因为 1E 不是连续函数.


2.要直接用 1E 定义 µ (E ),可以先将 C C (R n ) 中定义 L1 范数 k f kL0 = I (| f |),
再将空间完备化得到 L1 空间,再定义 µ (E ) = k1E kL1 .
3.定理提供了另外一种构造 Lebesgue 测度的方法,R 上的 Riemann 积分
就是 C C (R ) 上的正线性泛函,容易验证对应这个泛函导出的测度在 [a, b] 上值
为 (b − a).
(参见 Royden Real Analysis 第 16 章 The Daniel Integral)

定义. 设 Ω 为一集合,H 为 Ω 上的一族实值函数组成的线性空间,且满足


f , g ∈ H ⇒ f ∧ g ∈ H , ? f ∨ g ∈ H , f ∈ H ⇒ f ∧ 1 ∈ H ,则称 H 为一向量格

定义. H 为 Ω 上的向量格,I 是 H 上的正线性泛函:


¡ ¢
f , g ∈ H , α, β ∈ R ⇒ I α f + β g = α I ( f ) + β I ( g),f ∈ H , f ⩾ 0 ⇒ I ( f ) ⩾ 0,若 I
使得 f n ∈ H , f n ↓ 0 ⇒ I ( f n ) → 0,或者 f n ∈ H , f n ↑ f ∈ H ⇒ I ( f ) → lim I ( f n ),称
n→∞
I 为 H 上的 Daniel 积分。
7.6. RIESZ-MARKOV 表示定理 149
¡ ¢ ¡ ¢
Daniel 积分的例子:1)测度空间 Ω, F , µ ,H = L1 Ω, F , µ , I ( f ) = ∫ f d µ
2)A 为 Ω 上的一个代数,H 是 A 可测的简单实值函数,µ 是 A 上
的有限可加测度且对单调递减到空集的集合列是连续的,那么按照 I ( f ) =
P
aµ ({ f = a}) 定义的 I 是 Daniel 积分
a∈Range( f )
3)Ω 是 R n ,H = C c (Ω), I 为 Riemann 积分

练习

1. 证明完备可分距离空间中的 Borel 测度 µ 都是 Radon 的,即对每个 Borel


集合 B 和任意 ε > 0,存在紧集 K ⊂ B 使得 µ(B − K ) < ε
2. 证明任何 Radon 测度1 是 τ- 可加的,即对于任意递增的开集网 (U i ) i∈ I ,都

[
µ( i∈ I U i ) = lim µ(U i )
i

1 文献中对 Radon 测度的定义并不完全一致,这里主要强调了内正则性。通常定义会要求测


度是局部有限的。Bourbaki 直接把 Radon 测度描述为连续函数空间上的线性泛函。
第 8 章 乘积测度

学习乘积测度有几个目的,一个是说明高维积分与低维积分之间的关系,另一个
是与随机变量的独立性相关.当然,乘积概率空间与随机过程之间也有密切的联
系,本节会简要进行说明。主要考察两个(有限个)测度空间的乘积空间,也会
介绍无穷个测度空间乘积的特殊性。

8.1 乘积测度的定义
¡ ¢ ¡ ¢
定义. 给定两个测度空间 Ω1 , F1 , µ1 和 Ω2 , F2 , µ2 ,
乘积空间为 Ω = Ω1 × Ω2 = {( x1 , x2 ) , x1 ∈ Ω1 , x2 ∈ Ω2 },
乘积 σ 代数指 F1 ⊗F2 = σ ({ A 1 × A 2 : A 1 ∈ F1 , A 2 ∈ F2 }),称 A 1 × A 2 ,A 1 ∈ F1 ,
A 2 ∈ F2 ,这样的集合称为可测矩形。
对于任意 E ⊂ Ω1 × Ω2 ,定义 E 在 x ∈ Ω1 ,及 y ∈ Ω2 处的截口为 E x =
{ y ∈ Ω2 , ( x, y) ∈ E },E y = { x ∈ Ω1 , ( x, y) ∈ E }.
类似的,若 f ( x, y) 为 Ω 上的函数,称 f 的截口为 f x ( y) = f ( x, y), f y ( x) =
f ( x, y)

下面引理表明高维空间可测集合的截口在各自的空间里可测,可测的函数
也有类似性质.

引理 8.1. (Ω1 , F1 ) 和 (Ω2 , F2 ) 为可测空间,则 (Ω, F ) = (Ω1 × Ω2 , F1 ⊗ F2 ) 也是


可测空间
(a) 若 E ∈ F ,则 E x ∈ F2 ,∀ x;E y ∈ F1 ,∀ y ∈ Ω2 .
(b) 若 f 为 F 可测,则 f x 是 F2 可测的, f y 是 F1 可测的.

Proof. (a) 令 A = { A 1 × A 2 : A 1 ∈ F1 , A 2 ∈ F2 },则对于 A 中集合,引理成立,令


E = {E ∈ F : ∀ x ∈ Ω1 , y ∈ Ω2 , E x ∈ F2 , E y ∈ F1 },不难验证 E 是 λ 类

150
8.1. 乘积测度的定义 151

而 A 是 π 类,由单调类定理,E = σ (A ) = F ,得证.
¡ ¢ ¡ ¢y
(b) 注意到 ( f x )−1 (D ) = f −1 (D ) x ,( f y )−1 (D ) = f −1 (D ) 或者集合 { f ( x, y) > a} ∈
F,
{ y : f x ( y) > a} = { y ∈ Ω2 : ( x, y) ∈ { f ( x, y) > a}}

接下去,我们要证明在上存在唯一的使可测矩形满足 µ ( A 1 × A 2 ) = µ1 ( A 1 ) ×
µ2 ( A 2 ) 的测度 µ.相当于若是概率空间,说明存在相互独立的随机变量.
¡ ¢ ¡ ¢
定理 8.1. Ω1 , F1 , µ1 和 Ω2 , F2 , µ2 为两个 σ 有限的测度空间,在 F1 ⊗ F2
上存在唯一的乘积测度 µ = µ1 × µ2 ,使 µ ( A 1 × A 2 ) = µ1 ( A 1 ) µ2 ( A 2 ).∀ A 1 ∈ F1 ,
R R
A 2 ∈ F2 ,且 ∀E ∈ F1 ⊗ F2 ,µ (E ) = Ω1 µ 2 ( E x ) d µ 1 = Ω 2 µ 1 ( E
y
) d µ2 .

Proof. 首先要证 x → µ2 (E x ) 及 y → µ1 (E y ) 是 F1 与 F2 可测的,然后说明定义


的 µ 确实为 F1 ⊗ F2 上测度,即验证 (在矩形集合上的) 可数可加性.最后说明
唯一性,这主要是由于 C = { A 1 × A 2 , A 1 ∈ F1 , A 2 ∈ F2 } 是半环 [只要 F1 与 F2 是
半环,C 就是半环],由测度扩张定理可知测度扩张的唯一性.
S∞
证 x → µ2 (E x ) 为 F1 可 测。 设 Ω2 = 1 D n ,µ 2 ( D n ) < ∞ , 令 v n ( B ) =
µ2 (B ∩ D n ),∀B ∈ F2 。这样 x → µ2 (E x ) 的可测性等价于 x → vn (E x ) 的可测
性,实际上变为考虑有限测度,应用单调类定理,可证
© ª
A = E ∈ F1 ⊗ F2 : x → vn (E x ) 是F1 可测 = F1 ⊗ F2

因为 π 类 C ⊂ A ,同时 A 是 λ− 类
[(vn ( A 1 × A 2 )x ) = vn ( A 2 ) 1 A 1 ( x);
vn ((F − E ) x ) = vn (F x − E x ) = vn (F x ) − vn (E x ),
¡ ¢ ¡ ¢ ¡ ¢
vn (∪E i ) x = vn ∪(E i ) x = lim vn E i x ]
R R
µ 在 F 上为测度,设 λ1 = Ω1 µ2 (E x ) d µ1 ,λ2 = Ω2 µ1 (E y ) d µ2 。则 λ1 (;) =
λ2 (;) = 0.
S∞ y
若 {E n } 为 F1 ⊗F2 中互不相交的集合,E = E n 。则 ∀ y ∈ Ω2 ,E n 互不相交,
1
S R P ¡
R y¢ PR ¡ y¢
且 Ey = ∞ 1 E
y
n ,λ2 ( E ) = Ω2 µ 1 ( E y
) d µ 2 = Ω2 µ 1 E n d µ 2 = Ω2 µ 1 E n d µ2 =
P
λ2 (E n ),说明 λ2 也是测度。同样,λ1 也是测度.
同样应用单调类定理,可证

B = {E ∈ F1 ⊗ F2 : λ1 (E ) = λ2 (E )} = F1 ⊗ F2

在 C 上,容易看出 λ1 ( A 1 × A 2 ) = µ2 ( A 2 ) µ1 ( A 1 ) = λ2 ( A 1 × A 2 ),即 λ1 与 λ2 在 C
上一致,C ⊂ B ,又 B 是 λ 类,故 B = F1 ⊗ F2
152 CHAPTER 8. 乘积测度

S
∞ S ¡ ¢ S
此外,Ω1 = D i ,µ1 (D i ) < ∞,Ω2 = G j ,µ2 G j < ∞,Ω1 × Ω2 = Di × G j,
1 i, j
¡ ¢ ¡ ¢
µ Di × G j = µ1 (D i ) µ2 G j < ∞ ⇒ µ 为 σ 有限.

大家可能注意到之前的讨论并没有涉及测度空间是否完备,下面的定理指
出了完备测度空间的结论只是将处处改为几乎处处.
¡ ¢ ¡ ¢
定理 8.2. Ω1 , F1 , µ1 , Ω2 , F2 , µ2 完备 σ 有限,F 为 F1 ⊗ F2 的完备化,µ
¡ ¢
为 µ1 × µ2 的完备化,则对于 Ω1 × Ω2 , F , µ 中集合 E ,除了 Ω1 上的某个
µ1 零测集外,对于 Ω1 上的任意 x,函数 x 7→ µ2 (E x ) 都是 F1 可测的,且
R
µ ( E ) = Ω1 µ 2 ( E x ) d µ 1

S
Proof. 不妨设 E ∉ F1 ⊗ F2 ,否则结论由前面定理必然成立.E = E 0 E 1 ,E 0 ∈
T
F1 ⊗ F2 ,E 0 E 1 = ;,µ (E 1 ) = 0,存在 E 2 ⊃ E 1 ,µ (E 2 ) = 0,E 2 ∈ F1 ⊗ F2 .
R
此时 0 = µ (E 2 ) = Ω1 µ2 (E 2x )d µ1 ⇒ µ2 (E 2x ) = 0,µ1 − a.e.
⇒ µ2 (E 1x ) = 0(因 E 1x ⊂ E 2x )µ1 − a.e..E 1x 几乎对所有 x 都是 µ2 可测的.
⇒ x 7→ µ2 (E 1x ) 除去某个零测集恒为 0,故是 F1 可测函数 (在那个零测集上,
µ2 (E 1x ) 可能无法定义,即 E 1x 不可测)
又 x → µ2 (E 0x ) 是 F1 可测的,故 x → µ2 (E x ) = µ2 (E 0x ) + µ2 (E 1x ) 除去某个
R R
零测集外是 F1 可测的.µ (E ) = µ (E 0 ) = Ω1 µ2 (E 0x ) d µ1 = Ω1 µ2 (E ) d µ1

练习
¡ ¢ ¡ ¢
1. Ω1 , F1 , µ1 及 Ω2 , F2 , µ2 为 σ 有限测度空间,E ∈ F1 ⊗ F2 ,则下列条件
等价:
¡ ¢
(1) µ1 × µ2 (E ) = 0
(2)µ1 (E y ) = 0,µ2 − a.e.y
(3)µ2 (E x ) = 0,µ1 − a.e.x
2. T = {( x, y) ∈ [0, 1]2 : x − y ∈ Q }。证明 T 的测度为 0,但与任意形如 A × B 的
集合又有交集,其中 A 与 B 是 [0,1] 中的具有正测度的集合。[考察 A-B 的
性质]
3. 定义于 [0, 1]2 上的 Lebesgue 可测函数 f 满足,对于几乎处处的 x 和几乎处
处的 y,函数 z 7→ f ( x, z) 以及 z 7→ f ( z, y) 为常数函数,证明 f 几乎处处等于
某个常数。
8.1. 乘积测度的定义 153

8.1.1 R n 中的乘积测度
在考察积分之前,我们将前面的几个性质应用到 R n 空间去,假定 Ω1 ,Ω2 都
为 R ,那么 Ω1 × Ω2 为 R 2 ,先看 (R, B (R ) , λ) × (R, B (R ) , λ).由于对于最平凡的
矩形,R 2 空间的 λ2 ( A × B) = λ ( A ) · λ (B),故在乘积测度空间 R 2 上的测度应该
还是和普通的 Lebesgue 测度一样,关键在于乘积 σ 代数 B (R ) ⊗ B (R ) 到底与
¡ ¢
B R 2 有何关系.
¡ ¢
注意到普通的矩形生成的 σ 代数是 B R 2 ,而可测边矩形包含普通矩形,
¡ ¢
生成的 σ 代数为 B (R ) ⊗ B (R ),故 B R 2 ⊂ B (R ) ⊗ B (R ).而另一方面,任意的
可测边矩形 A × B 可以表示为 ( A × R ) ∩ (R × B),而由投影函数 π ( x, y) = x 的连续
性可知 π 是 Borel 可测的.
¡ ¢
A × R = π−1 ( A ) ⇒ 在 A 是 Borel 集时,A × R 也是 Borel 集 ⇒ A × B ∈ B R 2
¡ ¢ ¡ ¢ ¡ ¢
因 B R 2 是 σ 代数 ⇒ B (R ) ⊗ B (R ) ⊂ B R 2 ⇒ B R 2 = B (R ) ⊗ B (R )
¡ ¢ ¡ ¢
但 是 对 于 Lebesgue 可 测 集 我 们 有 B R 2 ⊂ L (R ) ⊗ L (R ) ⊂6= L R ∈ =
¡ ¢
L (R ) ⊗L (R ) 的完备化,B R 2 ⊂6= (R ) ⊗ (R ) B (R ) ⊂6= (R ),选择 A ∈ L (R ) \B (R ),
¡ ¢
则 A × A ∈ L (R ) ⊗ L (R ),但 A × A ∉ B R 2
¡ ¢
L (R ) ⊗ L (R ) ⊂6= L R 2 是由于任意 Lebesgue 可测边矩形 A × B = ( A × R ) ∩
(R × B)
S
A = A0 A 1 , A 0 为 Borel 集,λ ( A 1 ) = 0 ⇒ A × R = ( A 0 × R ) ∪ ( A 1 × R ),
而 λ ( A 1 × R ) = λ ( A 1 ) · ∞ = 0, A 0 × R 为 Borel 集,故 A × R 是 Lebesgue 可
¡ ¢
测的 ⇒ A × B 是 R 2 中 Lebesgue 可测的 ⇒ L (R ) ⊗ L (R ) ⊂ L R 2 ,
选取 R 中不可测集 M ,单点集 { x},则 { x} × M 测度为 0 是 R 2 中 Lebesgue
可测的,但在 x 上截口不可测故 ∉ L (R ) ⊗ L (R ).
¡ ¢ ¡ ¢
由于 L R 2 也是 B R 2 的完备化,故也必然是 L (R ) ⊗ L (R ) 的最小完
备化.
注意,对一般的拓扑空间 X ,通常只有 B ( X ) ⊗ B ( X ) ⊂ B ( X × X )
对比拓扑空间的乘积的概念
X 、Y 为拓扑空间, X × Y 的拓扑基为 {U × V ,U ⊂ X , V ⊂ Y },称为积拓扑。
在积拓扑下,投影映射 π1 ( x, y) = x 和 π2 ( x, y) = y 都是连续的,
函数 f : Z → X × Y 即 f ( z) = ( f 1 ( z) , f 2 ( z)) 连续 ⇔ f 1 , f 2 都连续.
这样 B ( X ) ⊗ B (Y ) 中可测矩形 A × B = ( A × Y ) ∩ ( X × B),由投影函数连续性
即可知 A × B ∈ B ( X × Y )
154 CHAPTER 8. 乘积测度

8.1.2 截口可测与整体可测
F1 ⊗ F2 可测集的截口都可测,但截口都可测不能保证集合是 F1 ⊗ F2 可测的。
例如,设 Ω 是一不可数集合,F 是由 Ω 的所有单点集生成的最小 σ 代数,则
对角线集合 D = {( x, x) : x ∈ E ⊂ Ω} ⊂ Ω × Ω 的截口可测,但 D ∉ F ⊗ F 如果 E ∉ F
(为什么?
)。[另一个类似的结论,Nedoma’s Pathology:X 为可测空间且 X > 2ℵ0 ,
则 X 2 的乘积 σ 代数不包括对角线集合 D = {( x, x) : x ∈ X }]
对于函数 f ( x, y) 也一样,即若 f x 、f y 都可测不能保证 f 可测.大家可对比 f
在每个变量连续不能保证 f 连续。[作为比较,f ( x) = ( f 1 ( x) , f 2 ( x)):Ω0 → Ω1 × Ω2 ,
可测当且仅当 f 1 和 f 2 是可测的.(关键 f −1 ( A × B) = f 1−1 ( A ) ∩ f 2−1 (B))]
什么时候能得证 f : Ω1 × Ω2 → Ω0 可测,在只给定各变量的一些性质条件下?
下面的定理表明一个变量连续,另一变量可测时可以保证 f 可测.
P
定理 8.3. (S, ) 可测空间, X 和 Y 为距离空间。 f : S × X → Y 满足:∀ x ∈ X ,
P
f x = f (·, x) 是 ( , B (Y )) 可测的;∀ s ∈ S , f s = f ( s, ·) 是连续的。若 X 可分,则
P
f 是 ⊗B ( X ) 可测.

Proof. 设 { x1 , x2 , · · ·} 为 X 的稠密子集。
由 f s 的连续性,f (s, x) ∈ 闭集 F ⇔ ∀ n,∃ xm 使得 d X ( x, xm ) < n1 ,dY ( f (s, xm ) , F ) <
1
n ½ ¾
−1 T
∞ S

⇒ f (F ) = s ∈ S : f ( s, xm ) ∈ B 1 (F ) × B 1 ( xm ),
n=1 m©=1 ªn n© ª
其中 B 1 ( xm ) = x ∈ X , d X ( x, xm ) < n1 ,B 1 (F ) = y ∈ Y , dY ( y, F ) < n1
n ½ n ¾
x
注意到 B 1 (F ) 为开集,因 f 可测, s ∈ S : f (s, xm ) ∈ B 1 (F ) 可测
n P n
⇒ f −1 (F ) ∈ ⊗B ( X )

8.1.3 无穷个测度的乘积
目前我们讨论的是两个空间的乘积,由数学归纳法呢,我们可以将构造方法推广
至任意有限个空间的乘积,不过对于无穷个空间的乘积,乘积 σ 代数及测度的
©Qn Q ª
构造不同,是从投影函数角度出发,或者说是 σ i =1 Ai × I \{1,··· ,n} Ωα (n 取遍
所有可能的有限数),可以证明无穷的乘积概率空间存在,而对于 σ 有限测度空
间,无限个空间的乘积测度未必是 σ 有限的.
例子:考虑两点的测度空间 {0, 1},每个点的测度为 1。不难看出乘积空间
{0, 1}∞ 上的乘积测度实际上是一个计数测度,不是 σ 有限的。
8.2. FUBINI- TONELLI 定理 155

定理 8.4. 在无穷维 Hilbert 空间不存在正测度满足:平移旋转测度不变,任意点


都有邻域测度有限。

Proof. 只需证明对于满足平移旋转测度不变的测度,任意非空球的测度都是无
穷。不妨设球 E 中心是 0 点,球半径 r。 e i = {0, ..., 0, 1, 0, ...} 表示第 i 维取值为
1,其余维度取值为 0。考虑中心为 2r e i ,半径为 r
4 的开球 E i ,则 E i 中任意点
与 0 点的距离小于 r,且不同的 E i 互不相交。注意到 E 中有无穷个 E i ,E 的
测度只能无穷

8.2 Fubini- Tonelli 定理


R
由于 x → µ2 (E x ) 相当于 x → Ω2 f x d µ 2 在 f = 1E 时的取值,前面定理推广到函数
情形就得到了重积分与累次积分关系的 Fubini 定理,先不考虑空间是否完备。
¡ ¢ ¡ ¢
定理 8.5 (Tonelli 定理,针对非负可测函数). Ω1 , F1 , µ1 和 Ω2 , F2 , µ2 为 σ 有
¡ ¢
限测度空间, Ω1 × Ω2 , F1 ⊗ F2 , µ1 × µ2 为乘积测度空间, f 是 Ω1 × Ω2 上非负
广义实值 F1 ⊗ F2 可测函数,则 ∀ x ∈ Ω1 , f x 为 F2 可测.
R R
(a) 函数 y → f y d µ1 , x → Ω2 f x d µ2 分别为 F2 ,F1 可测
³ ´ R ³R ´
Ω1
R ¡ ¢ R R
(b) Ω1 ×Ω2 f d µ 1 × µ 2 = Ω1 Ω2 f x d µ 2 d µ 1 = Ω 2 Ω1 f y d µ 1 d µ 2

Proof. 先考虑示性函数 f = 1E ,E ∈ F1 × F2 ,由前面定理结论成立.


由积分线性性质, f 为非负简单函数时,(a)、(b) 也成立.
当 f 为一般的非负可测函数时,可找一列递增简单函数列 σn ( x, y) → f ( x, y),
R R y R y y
∀ y ∈ Ω2 , Ω1 f y d µ1 = Ω1 lim σn d µ1 = lim Ω1 σn d µ1 (单调收敛定理,σn ↑ f y ),由
R y R y
Ω1 σ n d µ1 的 F2 可测性知极限 Ω1 f d µ1 可测,(a) 成立.
同样用单调收敛定理,可证 (b) 也成立.

定 理 8.6 (Fubini 定 理). 在 Toneili 定 理 条 件 下, 若 f 是 可 积 的, 即 f ∈


¡ ¢
L1 Ω1 × Ω2 , F1 ⊗ F2 , µ1 × µ2 ,(a)(b) 也 成 立. 其 中 (a) 可 进 一 步 表 述 为 对 于
¡ ¢ ¡ ¢
几乎处处的 x ∈ Ω1 , f x ∈ L1 µ2 ;对于几乎处处的 y ∈ Ω2 , f y ∈ L1 µ1 。

Proof. 考虑 f + 及 f − ,由于积分值都有限,故结论成立.几乎处处的出现是由
于可积函数只需要几乎处处有定义,或者是几乎处处有限。

对于完备化测度空间,定理的表述略有不同.
156 CHAPTER 8. 乘积测度
¡ ¢ ¡ ¢
定理 8.7 (Fubini-Tonelli 定理). Ω1 , F1 , µ1 , Ω2 , F2 , µ2 完备且 σ 有限,f 非负,
¡ ¢
F1 × F2 可测或 ∈ L1 Ω, F , µ 其中 Ω = Ω1 × Ω2 ,F = F1 ⊗ F2 ,µ = µ1 × µ2 ,则
(a) 对于几乎所有 y ∈ Ω2 , x 7→ f ( x, y) 是 F1 可测或可积,
µ1 − a.e. x ∈ Ω1 , y 7→ f x ( y) 为 F2 可测或可积
R
(b) x 7→ Ω2 f x ( y)³d µ2 是 F´1 可测或可积
R R R R ³R ´
(c) Ω f d µ = Ω1 Ω2 f d µ2 d µ1 = Ω2 Ω1 f d µ1 d µ2 .
Proof. 设 H ∈ F1 ⊗ F2 ,则 H = G ∪ F ,G ∈ F1 ⊗ F2 ,µ (F ) = 0,G ∩ F = ;,存在
E ∈ F1 ⊗ F2 ,F ⊂ E ,µ (E ) = 0。∀ x ∈ Ω1 , H x = G x ∪ F x ,前面有定理证明,对几
R R
乎所有的 x ∈ Ω1 , H x ∈ F2 ,µ2 (H x ) = µ2 (G x ) = Ω2 1G d µ2 = Ω2 1H d µ2
这说明 f = 1H 时 (a) 成立,{1H ( x, y) > a, x} = H x 或 ; 或 Ω,视 f 为 f x ( y),
1H 相当于 1H x ( y),由非负简单函数,再一般可测函数不难验证 (a) 成立.其余
部分与不完备情形类似。略.

注意. 测度 σ 有限是必要的。比如 Ω1 = (0, 1),F1 = (Ω1 ),µ1 = λ;Ω2 = (0, 1),


F2 = (Ω2 )[幂集],µ2 为计数测度。令 f ( x, y) = 1{ x= y} 。则
R R R
∀ x, Ω2 f ( x, y)µ2 ( d y) = 1,故 Ω1 Ω2 f ( x, y)µ2 ( d y)µ1 ( dx) = 1
R R R
∀ y, Ω1 f ( x, y)µ1 ( dx) = 0,故 Ω2 Ω1 f ( x, y)µ1 ( dx)µ2 ( d y) = 0
可积性要求的必要性见下面的例子。

练习(重积分与累次积分不等的例子及其它)
xy
1. 在 [−1, 1]2 上, x2 + y2 > 0 时 f ( x, y) = , f (0, 0) = 0,
( x2 + y2 )2
R R
对所有 y, −11 f ( x, y) dx = 0,对 ∀ x, −11 f ( x, y) d y = 0,因此累次积分= 0,
R R R R R
但二重积分 −11 −11 | f ( x, y)| dxd y ≥ 01 dr 02π | r | d φ = 2 01 dr
sin φ cos φ
r =∞

 2n 1 1 1 1

 2 2n ≤ x ≤ 2n−1 , 2n ≤ y ≤ 2n−1
2. [0, 1]2 , f ( x, y) = −22n+1 2n1+1 ≤ x < 21n , 21n ≤ y < 2n1−1


 0
8.3. 高维 RIEMANN 积分 157
R 1 ³R 1 ´ R1R1 R 1 ³R 1 ´
0 0 f ( x, y) dx d y = 0 , 0 0 | f ( x, y) | d ydx = ∞ , 0 0 f ( x, y) d y dx = 1
¡ .
¢ ¡ 2 ¢2 RR RR
3. f ( x, y) = x2 − y2 x + y2 ,f (0, 0) = 0, [0,1]2 f dxd y = − π4 , [0,1]2 f d ydx =
π
4.
³ 2´
1 − x2 t R∞ ∂f ∂2 f ∂f
4. f ( x, t) = p e , t > 0. −∞ f ( x, t) = 1,2 ∂ t = ∂ x2 ,令 g ( x, t) = ∂t
2Rπ t R R∞R∞
∞ ∞
问是否有 −∞ s g ( x, t) dtdx = s −∞ g ( x, t) dxdt
R R∞
5. f 在 R d 上可积,α > 0,E α = { x : | f ( x)| > α},证明 Rd | f ( x)| dx = 0 m (E α ) d α
6. F 是 R 中闭集且 F c 有界, d ( x, F ) 为 x 到 F 的距离, t > 0,证明

Z
( d ( y, F )) t
g ( x) = 1+ t
dy
R | x − y|

在 F 上可积,因此在 F 上也几乎处处有限
R R ∞ ¡R r ¢ R∞ x π
7. 应用: εr sinx x dx = 0 ε e− x y sin xdx d y,令 ε → 0+ ,r → ∞,说明 0 sin
dx = 2

8.3 高维 Riemann 积分

定理 8.8. f 为 R p+ q 上有紧支撑的 Riemann 可积函数,则对于 y ∈ R p , y →


R R
Rq f ( y, z) dz 与 y → R q f ( y, z) dz 是 Riemann 可积的,且
R R R
R p+ q f ( x) dx = R p R q f ( y, z) dzd y

Z Z
= f ( y, z) dzd y
Rp Rq

注:若 f 连续,则定理中的上下积分可加强为 Riemann 积分


举例:f ( x, y) = 1q ,若 x = qp ,p、q ∈ N 且互质,p ≤ q,y ∈ Q ∩ [0, 1],其他时
候 f = 0.那么 f 是 R 2 上 Riemann 可积的(为什么?验证无理点连续)
,但对每
R R
个 x = qp , y → f ( x, y) 不是 Riemann 可积的, R f ( x, y) d y = 0, R f ( x, y) d y = 1q .
158 CHAPTER 8. 乘积测度

8.4 概率与测度

分析术语 概率术语
测度空间 概率空间
σ 代数 σ域
可测集 事件
可测函数 f 随机变量 X
f 的积分 X 的期望
Lp p 阶矩有限
依测度收敛 依概率收敛
R 上的 Borel 概率测度 分布
测度的 Fourier 变换 分布的特征函数

8.4.1 独立性
(Ω, F , P ) 为一概率空间,若不做特殊声明,以下提到的 σ 代数都是 F 的子 σ 代

定义:若 A 、B 两事件 ( A ∈ F ,B ∈ F ) 满足 P ( A ∩ B) = P ( A ) P (B) 则称 A
与 B 独立.
µ 对于 ¶A 1 ,…, A , n 个事件,若对任意 m ≤ n.1 ≤ k 1 < k 2 < · · · < k m ≤ n,
T
m Q
m ³ n´
P Ak j = P A k j ,称 ( A 1 ,…, A n ) 相互独立.
j =1 j =1
µ ¶
T
对一族事件 = { A t , t ∈ T },若对 T 的任何非空有限子集 s,有 P As =
Q s∈ S
P ( A s ),称中事件相互独立.
s∈ S
对一族事件类 {F , t ∈ T },如果从每个事件类 Ft 中任取一事件 A t ,{ A t , t ∈ T }
中事件相互独立,则称事件类 {Ft , t ∈ T } 独立.
© ª
对一族随机变量,若由随机变量 { X t , t ∈ T } 生成的 σ 域 σ ( X t ) = X t−1 (B (R )) , t ∈ T
为独立事件,称 { X t , t ∈ T } 相互独立.
注意:Ft ⊂ F ,σ ( X t ) ⊂ F ,上述对事件类独立的定义等于说任取任何有限
个事件类都独立 ⇔ 一族事件类独立.

定理 8.9. 设 {Ft , t ∈ T } 为独立事件类,若每个 Ft 为 π 类,则 {σ (Ft ) , t ∈ T } 为


独立事件类.
8.4. 概率与测度 159

推论. 设 ( X t , t ∈ T ) 为一族随机变量,则它们相互独立的必要条件是对 T 的任一


¡ ¢ Qn ¡ ¢
有限子集 {s 1 , · · · , s n },及 x1 ,…,xn ∈ R .
有 P X s1 ≤ x1 , · · · X s n ≤ xn = P Xsj ≤ xj
j =1

随机向量是 Ω → R k 的可测映射 ⇔ 每个分量都是随机变量.


¡ ¢
象随机变量一样,随机向量 X = ( x1 , x2 , · · · , xk ) 的分布定义为 µ ( A ) = P X −1 ( A ) ,
¡ ¢
A ∈ B R k ,联合分布函数为 F ( x1 , · · · , xk ) = P ( X 1 ≤ x1 , · · · , X k ≤ xk )
这样,一族随机变量 { X i } 独立也等价于任意有限个 { i 1 , · · · , i n },
¡ ¢ ¡ ¢
F (xi1 , · · · , xi n ) = Fi1 xi1 · · · Fi n xi n

练习

1. 如果 ( X − Y ) 与 X 独立,( X − Y ) 与 Y 独立,则 X − Y 几乎处处为常数.假


设 X,Y 方差存在。
2. X 、Y 独立且都服从分布 P ( X = 1) = P ( X = −1) = 12 ,则 X 和 X Ȳ 独立.(其
实 Y 为对称分布即可)
下面举两个独立性的概念在数论中的应用
1. Euler 函数 φ (n) 定义为 1 至 n 的整数中与 n 互质的整数个数.
那么因为 n 有唯一的因子分解为 n = q1 e 1 … q k e k ,与 n 互质即与 q1 ,…, q k
互质.
±
在 1 至 n 中任取一整数 a,这与 q i 互质的概率是 1 − 1 q i ,而对于不同的
质数,a 与 q i 互质以及 a 与 q j 互质这两个事件是独立的,故任取 a 与 n 互质
k ³
Q ± ´ ± k ³
Q ´
的概率是 1 − 1 q i ,而这个应该等于 ϕ ( n) n ⇒ ϕ ( n) = n 1 − q1
i
i =1
P
∞ Q ³i=1 ´
1 1 1
Riemann Zeta function ς (s) = n s = 1 + 2s + · · · = 1− q−s (Euler 给出 后
n=1 q
一步)
P
∞ Q
( 1−1q−s = q− es , 1−1q−s 会给出 q 1 − e 1 s … q k − e k s = 1
n s ,n 与质数因子分解是
e=0
一一对应的)
Riemann Hypothesis 关于 ς (s) 复值零点的假说,与质数的分布情况相关
现在我们来求任意取两个正整数,它们互质的概率有多少?
首先说明如何给可列多个正整数赋予概率测度,显然不能给每一个正整数
P

单独赋予概率,否则 P n = 1 就无法满足,所以实际上是给某些整数集合赋予
n=1
概率,即对任意正整意 c 和 n.集合 { x ≡ c mod (n)} 的概率为 n1 .
160 CHAPTER 8. 乘积测度

这样,若 m, n 互素,
1
P ( x ≡ b mod ( m) , x ≡ c mod ( n)) = = P ( x ≡ b mod ( m)) P ( x ≡ c mod ( n))
mn
任取 x、 y,记其互质的概率为 p.同时有,
(A) x、 y 最大公约数为 n ⇔ x ≡ 0 mod (n), y ≡ 0 mod (n), nx , ny 的最大公约
数为 1.(这三个条件是独立的)
概率 () = n1 · n1 · p
P
∞ P

p 1
1= P ( x、 y 最大公约数为 n)= n2
= p · ς (2) ⇒ p = ς(2) = π62
n=1 1
(B) x、y 互质 ⇔ x 和 y 不能同时有素数 q 为因子,即 ∀ 素数 q,x 6≡ 0 mod ( q)
或 y 6≡ 0 mod ( q),对每个 q,
Y¡ ¢
P { x 6≡ 0 mod ( q) y 6≡ 0 mod ( q)} = 1 − p−2 ⇒ P ( x, y) = 1 − p−2
q

8.4.2 独立随机变量的 Borel-Cantelli 引理


P ¡ ¢
设 { A n , n ≥ 1} 为一列事件,若
P ( A n ) < ∞,则 P { A n } 中有无穷多个事件发生 = 0
T
∞ S

{ A n } 有无穷多个事件发生即 lim A n = An
k=1 n= k

引理 8.2 (Borel-Cantelli 引理). ³设 { A n ,´n ≥ 1} 为一列事件 (可测集),则


P∞
1. 若 P ( A n ) < ∞,则 P lim A n = 0
n=1
P

2. 若 { A n , n ≥ 1} 相互独立,则 P ( A n ) = ∞ ⇒..,也就是说,在 { A n } 相互
³ ´ n=1
独立时,P lim A n ∈ {0, 1}(Borel 0-1 律)
n→∞
³ ´ µ ¶
S
∞ P

Proof. 1. P lim A n ≤ P An ≤ P ( A n ) → 0, k → ∞
µ ¶ µn=k ¶ n= k ½ ¾
S
m T
m
c Qm P
m
2. 1−P An = P A n = (1 − P ( A n )) ≤ exp − P ( A n ) → exp {−∞} =
n= k n=µk k ¶ µ ¶ n= k
³ ´ T
∞ S ∞ S

0, m → ∞。P lim A n = P A n = lim P A n = 1.
k=1 n= k n→∞ n= k

定理 8.10 (Kolmogrov 0-1 律). 设 { X n , n ≥ 1} 为独立随机变量,其尾 σ 代数


T
∞ © ª
D= σ X j , j > n 中元素 (尾事件) 的概率为 0 或 1.
n=1
¡ ¢ ¡ ¢ ¡ ¢
Proof. σ X j , 1 ≤ j ≤ n 与 σ X j , j > n 独立 ⇒ σ X j , 1 ≤ j ≤ n 与 D 独立
¡ S
∞ ¢
⇒A = σ X j , 1 ≤ j ≤ n 与 D 独立 ⇒ σ (A ) 与 D 独立 ⇒ D ⊂ σ (A ) 与 D
n=1
T
独立 ⇒ ∀D ∈ D ,P (D ) = P (D D ) = P 2 (D ) ⇒ P (D ) = 0 或 1.
8.4. 概率与测度 161
n o © ½ ¾
ª X 1 +...+ X k
引理. 注意 lim X n = c lim X n = c 以及 lim k 都是尾事件,因此若
k→∞ P
Xi
{ X n } 独立,可知 lim X n ,lim X n 几乎一定是常数,而 lim k 则或者几乎处处
k→∞
收敛,或者几乎处处不收敛,如果收敛,则几乎处处为常数.(强大数律)

应用:
P
1. 扔硬币,朝上概率 p > 0, A n = {n},那么 P ( A n ) = p,所以 P ( A n ) = ∞,
¡ ¢
P 有无穷次朝上 = 1
2. 猴子随机打字,不停的打,打出莎士比亚全部作品的概率= 1.原因在于
任一文章都是字母的有限序列 ³n o´
3. 如果 X 1 ,…,X n …独立随机变量,则 P ω : lim X n (ω) = 0 = 1 ⇔ X n → 0,
n→∞
a, s.
¡ ¢ ¡ ¢
⇔ 对任意 ε > 0,P | X n | > ε只对有限个不成立 = P | X n | ≤ ε对充分大的 n = 1
¡ ¢
记 A n = {ω : | X n (ω)| > ε} ↑ P lim inf A nc
¡ ¢ P

⇔ P (lim sup A n ) = 1 − P lim inf A nc = 0⇔ P ( A n ) < ∞,∀ε > 0
n=1
(用另一种方法说明了是否收敛)

8.4.3 独立随机变量与乘积测度空间
X X −1
回忆概率分布与随机变量是对应起来的 Ω −→ R ,σ ( X ) ←−−− B (R )
¡ ¢
给定 (Ω, F , P ) 上随机变量 X ,其概率分布 P X ( A ) = P X −1 ( A ) 是 (R, B ) 的
一个概率测度,这样若有两个随机变量 X 1 , X 2 ,就有概率分布 (测度)P1 ,P2 .
显然我们可以在 (R, B ) 上构造另外的随机变量,使之分布为 P1 和 P2 .实际
上,在可测空间 (R, B ) 定义测度为 P1 ,则测度空间 (R, B , P1 ) 中的随机变量,
Y ( x) = x 的分布就为 P1 .于是定义乘积测度空间 (R × R, B ⊗ B , P1 × P2 ),在其定
义 Y1 ( x, y) = x,Y2 ( x, y) = y,则 Y1 ,Y2 连续当然也是 Borel 可测函数.
而且 P (Y1 ≤ a, Y2 ≤ b) = P1 ( x ≤ a) P2 ( y ≤ b) = P1 (Y1 ≤ a) P2 (Y2 ≤ b)(P = P1 × P2 )
即说明 Y1 和 Y2 独立且各自的分布分别为 P1 和 P2 ,而且 Y1 ,Y2 是定义
¡ ¡ ¢¢
在同一可测空间 R 2 , B R 2 上的.

8.4.4 转移概率
定义. 给定可测空间 (Ω1 , F1 ),(Ω2 , F2 ),转移概率 P21 是从 Ω1 × F2 到 [0,1] 的
映射使得:(a) 对任意 x ∈ Ω1 ,P21 ( x, ·) 是 (Ω2 , F2 ) 上的概率;(b) 对任意 A ∈ F2 ,
162 CHAPTER 8. 乘积测度

P21 (·, A ) 是 (Ω1 , F1 ) 可测的。

则上述乘积测度的相关分析可应用于转移概率情形:

定理 8.11. 可测空间 (Ω1 , F1 ),(Ω2 , F2 ),P21 是 Ω1 × F2 上的转移概率,P1 是


(Ω1 , F1 ) 上的概率,则在 (Ω1 × Ω2 , F1 ⊗ F2 ) 存在唯一的概率 P 使得 P ( A 1 × A 2 ) =
∫ P21 ( x1 , A 2 ) P1 ( dx1 )。
A1
对于 (Ω1 × Ω2 , F1 ⊗ F2 ) 上的正随机变量 X,Y ( x1 ) = ∫ X x1 ( x2 ) P21 ( x1 , dx2 ) 几
Ω2
乎处处有定义且 F1 可测, ∫ X dP = ∫ ∫ X x1 ( x2 ) P21 ( x1 , dx2 ) P1 ( dx1 )
Ω1 ×Ω2 Ω_1 Ω2

8.4.5 无穷个概率空间的乘积
定理 8.12. 给定 (Ωn , Fn , P n )∞
n=1 ,令
Q Q
Ω= 1≤ n<∞ Ω n ,R
= { n A n , A n ∈ Fn ; 除了有限个 n, A n =Ωn },F = σ(R )
Q Q
对于任意 A = n A n ∈ R ,定义 P ( A ) = n P n ( A n ),则 R 上的 P 可以延拓
成定义在 F 上的概率测度。R 中集合称为可测柱集。
Q
Proof. 定义 Z n = σ({ i A i , A i ∈ F i , 除了前 n 个 A i , A i = Ω i }),Z n ↑ Z ,则 Z 是代
数。[Z = ∪n Z n 不是 σ 域,F = σ(Z )]。根据有限个概率空间乘积的性质不难证
明,P 在 Z 上是有限可加的。关键是证明 P 在 Z 上也是可数可加的。只需证
明对 Z 中递减集合列 H i ↓ ;,满足 P (H i ) = 0,即若 Z 中递减集合列 H i 满足
∀ i, P ( H i ) ≥ ε > 0,则 ∩ i H i 6= ;。
对于给定的 H i ,肯定有 H i ∈ Z n(i) ,为简单计,不妨设 H i ∈ Z i 。注意总有
Q
∞ Q
i
Hi = Bi × Ω j , B i ∈ σ( F j )。当 i > 1 时,由 Fubini 定理,
j = i +1 j =1
R R ∆R
P (B i ) = Ω1 (... Ω i 1B i ( x1 , ..., x i ) dP i ...) dP1 = Ω1 h i,1 ( x1 ) dP1 。
H i+1 ⊂ H i ⇒ B i+1 ⊂ B i × Ω i+1 ⇒ 给定 x1 , h i,1 ( x1 ) 关于 i 递减,设极限为
R
h 1 ( x1 )。则由控制收敛定理 Ω1 h 1 ( x1 ) dP1 = lim P (B i ) ≥ ε,故存在 x1∗ ∈ Ω1 使
h 1 ( x1∗ ) ≥ ε。而且,x1∗ ∈ B1 ,否则 1B i ( x1∗ , ..., x i ) = 0⇒ h i,1 ( x1 ) = 0⇒ h 1 ( x1∗ ) = 0,矛盾。
R
当 i > 2 时,由 Fubini 定理, h i,1 ( x1∗ ) = Ω1 h i,2 ( x2 )dP2 ,类似推理可知存在
x2∗ ∈ Ω2 使 h i,2 ( x2∗ ) ≥ h 2 ( x2∗ ) ≥ ε,( x1∗ , x2∗ ) ∈ B2 。一直继续下去,最终会找到 x∗j ∈ Ω j ,
( x1∗ , ..., x∗j ) ∈ B j ,说明 ( x1∗ , ..., x∗j , ...) ∈ ∩∞ 1 H i ,定理得证。

注:定理结论对于任意多(不限于可数)概率空间的乘积也成立。
8.4. 概率与测度 163

8.4.6 随机过程
定义. 随机过程:定义为 (Ω, F , P ) 上的一族随机变量 { X t , t ∈ T }.
空间 Ω 称为样本空间, X t 称为过程在 t 时刻的状态。对于固定的 ω ∈ Ω,
t 7→ X t (ω) 称为 ω 的轨道。通常 T 取整数集合 (离散随机过程) 或者实数集合 (连
续随机过程)。
随机过程可以看成是定义于 T × Ω 上的函数
给定 T 上的 σ 代数 T ,如果 T × Ω 上的映射 ( t, ω) 7→ X t (ω) 关于乘积 σ 代
数 T ⊗ F 可测,称随机过程 { X t , t ∈ T } 可测。

在随机过程可测的情况下,我们可以通过描述任意多个时点上发生各种事
件的概率的方式确定这个随机过程,相当于概率论里用分布来刻画随机变量。
Kolmogorov 相容性定理表明,在一定条件下,我们只需要知道随机过程的
所有有限维分布的情形就可以描述整个过程的分布。

定理 8.13. Kolmogorov 相容性定理:若对所有正整数 n,概率测度族 (R n , B (R n ), P n )


相容,即

P n+1 ((a 1 , b 1 ] × ... × (a n , b n ] × R ) = P n ((a 1 , b 1 ] × ... × (a n , b n ])

则在 (R N , B (R N )) 上存在唯一的概率测度 P 满足
P (ω = (ω1 , ...) : ω i ∈ (a i , b i ], 1 ≤ i ≤ n) = P n ((a 1 , b 1 ] × ... × (a n , b n ])。

证明:略。
注:定理对于不可数多的情形也成立。[不过值得指出的是,对于不可数个
Q Q
的乘积空间, B (R ) ⊂ B (R T ),定理只保证 B (R ) 上概率存在,这使得定理结
论难以直接用于讨论,比如 Brownian Motion 的存在性问题。因为布朗运动要
Q
求轨道是连续的,但是连续函数全体 C 却并非是乘积 σ 代数 B (R ) 中的集合。
Q Q S
为说明这一点,首先注意到 i∈T B (R i ) 实际上等于 {n1 ,n2 ,...}⊂T B (R i ),
i ∈{ n 1 ,n 2 ,...}
Q Q
故若集合 C 是 i∈T B (R i ) 可测,则 C 对于某个乘积 σ 代数 B (R i ) 也
i ∈{ n 1 ,n 2 ,...}
可测,这意味着连续函数可以由可数个点的取值所决定,显然是不可能的。]
与前一定理的区别在于前面的定理给定的是乘积测度,换言之各随机变量
是独立的,而在 Kolmogorov 定理里是可以不独立的。所以在后者定理的推广中
会要求概率测度是紧的,类似前面 Borel 测度的内正则性。
164 CHAPTER 8. 乘积测度

例. 先证明存在 X n ⊂ [0, 1],X n+1 ⊂ X n ,∩∞


n=1 X n = ;,每个 X n 的 Lebesgue 外测
度为 1。记 Bn 为 X n 中的 Borel σ 代数(Bn = { X n ∩ B, B ∈ B ([0, 1])})
,定义 Bn
上的集函数
µn ( X n ∩ B) = λ( X n ∩ B),其中 X n ⊂ X n ,λ( X n ) = λ∗ ( X n )。可证明 µn 是概率
测度。
Qn Qn Q
对任意 n,定义 πn : X n → i =1 X i ,π n ( x) = ( x, ..., x)。在 i =1 B i = B ( ni=1 X i )
上定义测度 µn = µn ◦ π−n 1 。则可验证概率测度 {µ } 是相容的。注意到集合 n
Qn
∆n = { x = ( x1 , ..., xn ) : x1 = ... = xn } ∈
∀ n, µn (∆n ) = 1。所以如果
i =1 B i ,而且
Q∞ Q

i =1 B i 中存在概率测度 µ 与 {µ } 相容,则必然有 µ(Ω n ) = 1,Ω n = ∆ n × X k。
n
k= n+1
T
∞ T

但因为 X n = ;, Ωn = ;,不可能存在相容的 µ。
n=1 n=1

8.4.7 Brownian Motion

定义. 布朗运动或维纳过程是概率空间 (Ω, F , P ) 上随机过程 {Wt , t ≥ 0},满足:

1. P (W0 = 0) = 1
Q
2. 对于 0 ≤ t0 < t1 < ... < t k ,P (Wt i − Wt i−1 ∈ H i , i ≤ k) = P (Wt i − Wt i−1 ∈ H i )
i≤k
3. 若 0 ≤ s < t,Wt − Ws ∼ N (0, t − s)
4. 样本轨道连续。

根据 Kolmogorov 定理,我们容易知道存在满足条件 (1)(2)(3) 的随机过程,


但是并不能保证该过程轨道连续,而且事实上所有连续的轨道函数集合还不是
可测的。为了证明布朗运动的存在性,我们要说明已找到的满足 (1)(2)(3) 的随
机过程 { X t } 中绝大部分是连续的。为此,令 Q 2 = {m2−n : m, n ≥ 0} 为二进制有理
数集合,Ω q = {ω : Q 2 → R },F q 为对应的由有限维集合生成的 σ 代数,由相容性
定理可在 (Ω q , F q ) 构造出概率 P q 使得有限维分布与布朗运动一致。可以证明,
对于 T < ∞,在 Q 2 ∩ [0, T ] 上样本轨道 ω : Q 2 → R 一致连续的 ω 的集合 W 的概
率 P q (W ) = 1。由 Q 2 的稠密性和轨道的一致连续性,只在 Q 2 上定义的过程可延
拓为 [0, ∞) 上的随机过程 {Wt },且轨道连续。
(具体过程大家可参阅如 Brownian
Motion and Stochastic Calculus)
8.4. 概率与测度 165

练习

1. 概率空间 (Ω, F , P ),X 与 Y 是相互独立的随机变量。若 Borel 可测函数


F : R 2 → R 是有下界的,利用 Fubini 定理证明 x2 7→ f ( x2 ) = E [F ( X 1 , x2 )] 是
Borel 可测的,而且有:
E [F ( X 1 , X 2 )] = E [ f ( X 2 )]。
2. 概 率 空 间 (Ω, F , P ), 随 机 变 量 X 1 , X 2 , ..., X n 相 互 独 立 且 P 可 积, 证 明
X 1 X 2 · · · X n 可积且有 E [ X 1 X 2 · · · X n ] = E [ X 1 ]E [ X 2 ] · · · E [ X n ]。

8.4.8 随机积分
定义 (鞅). 设 (Ω, F , P ) 为一概率空间,{Ft , t ∈ R+ } 是一族单调增加的 F 的子 σ
代数,即 Ft ⊂ Fs ,若 t < s。随机过程 X = { X t } 称为鞅,如果:
1. X t 可积,2. X t 是 Ft 可测的,3. X s = E [ X t |Fs ],∀s < t

布朗运动就是最常见的连续时间鞅
随机积分的定义方法类似 Lebesgue 抽象积分,先界定 σ 代数与测度,再有
对应的可测函数,然后再定义积分。
如前所述,对于随机过程 { X t },可视其为 R+ × Ω 上的函数,
考察由所有形如 {0} × F0 ,(s, t] × F (F0 ∈ F0 , F ∈ Fs , s < t) 的集合(称之为可
预测矩形)生成的 σ 代数 G 。(比较乘积测度的定义)
对于适应过程 { Z t }[即 Z t 是 Ft 可测的], Z t ∈ L1 , ∀ t,定义集函数
λ Z (( s, t] × F ) = E (1F ( Z t − Z s )),λ Z ({0} × F0 ) = 0
注意若 { Z t } 是鞅,则 λZ ≡ 0 而 λ(Z)2 ⩾ 0。于是给定连续的 L2 鞅 M t ,在
G 上可定义测度(通过测度扩张定理得到):µ M (( s, t] × F ) = λ(M)2 (( s, t] × F ) =
£ ¤
E 1F ( M t − M s )2 。
¡ ¢ R
现在可以在 L2 R+ × Ω, G , µ M 上定义随机积分 X dM .
R R
显然 X 为可预测矩形的示性函数时, X dM = 1(s,t]×F dM = 1F ( M t − M s ),
F ∈ Fs ;
R R
X dM = 1{0}×F0 dM = 0,F0 ∈ F0 ;
P
n
1. X 是简单过程,即 X = c j 1(s j ,t j ]×F j + c 0 1{0}×F0 ,( s j , t j ] × F j 互不相交.
j =1
R P
n ¡ ¢ n¡R ¢2 o R
则 X dM ≡ c j 1F j M t j − M s j ,且有 E X dM = R+ ×Ω X 2 d µ M ,
j =1
R R
说明 X dM ∈ L2 (Ω)。并有 1[0,t] X dM = 1[0,t] (s) X (s, ω) 是鞅
166 CHAPTER 8. 乘积测度
¡ ¢
2. 可以证明简单过程全体在 L2 R+ × Ω, G , µ M 中是稠密的,
¡ ¢
于 是 对 于 任 意 Y ∈ L 2 R + × Ω, G , µ M , 可 找 到 简 单 过 程 序 列 X n , 使 得
R
| X n − Y |2 d µ M → 0
R R ¡R R ¢2
Y dM 就定义为 X n dM 在 L2 (Ω) 中的极限即 E [ X n dM − Y dM ] → 0
R R
作为 1[0,t] X n dM 的 L2 极限, 1[0,t] Y dM 也是鞅.
注:G 可测的随机过程即连续、Ft 可测过程
第 9 章 单调函数与可微性

9.1 Borel-Kolmogorov 悖论
假设我们知道 ( x, y) 是二元正态分布,相关系数是 ρ :
p
1 − ρ2 1
p(d xd y) = exp{− ( x2 + y2 − 2ρ x y)}d xd y
2π 2
R p
1−ρ 2
则边际分布为 p(d x) = p(dxd y)d y = p exp{− 12 (1 − ρ 2 ) x2 }d x

给定 y = y0 时的条件密度为:
p( dxd y) 1 1
p(d x| y = y0 ) = = p exp{− ( x − ρ y0 )2 }d x
p( y0 ) 2π 2

另一方面,在密度中直接令 y = y0 ,并标准化后条件密度应该是
1
p(d x| y = y0 ) = A exp{− ( x2 + y02 − 2ρ x y0 )}d x
2
与前述结果相同。
现在假设 u = 1+yx2 ,那么 ( x, u) 对应的密度函数是
p
1 − ρ2 1
p(d xd u) = exp{− ( x2 + u2 (1 + x2 )2 − 2ρ xu(1 + x2 ))}(1 + x2 )d xd u
2π 2
p
1−ρ 2
边际分布 p(d x) = p exp{− 12 (1 − ρ 2 ) x2 }d x 与前面相同

问给定 u = 0 时的条件密度是多少?
思路 1:在二元密度函数中令 u=0,并标准化:p(d x| u = 0) = A exp{− 12 ( x2 )}(1 +
x2 )d x
思路 2: u=0 就是 y=0, p(dx| u = 0) = p(d x| y = 0) = A exp{− 21 ( x2 )}d x
哪个是正确的?

167
168 CHAPTER 9. 单调函数与可微性

条件密度应当看成条件概率的极限:
P ( x ∈ dx, | u| < ε)
p( dx| u = 0) = lim
ε→0 P (| u| < ε)
ε
P ( x ∈ dx, | y| < ε) P ( x ∈ dx, | u| < 1+ x2 )
p( dx| y = 0) = lim = lim
ε→0 P (| y| < ε) ε→0 P (| u| < 1+εx2 )
注意:分母趋于 0 的方式影响极限的结果。
上述问题与 Borel 提出的如下问题有关:
一个随机变量在单位球面上是均匀分布的,请问它在球面上最大圆环上的
条件分布是什么?
直观的看,由对称性,条件分布应当仍然是均匀分布。但是如果注意到球面
上的均匀分布相当于经度 θ 在 [-π,π] 上均匀分布,纬度 ϕ 在 [-π/2,π/2] 上分布
密度为 cos(ϕ)/2,则:
情况 1:最大圆是赤道,ϕ=0,则 p(θ |ϕ = 0) = 21π ,均匀分布
情况 2:最大圆是经线,θ =0,则 p(ϕ|θ = 0) = 12 cos(ϕ),非均匀分布
P(θ ∈E,ϕ∈[a,b])
条件概率应该理解为 P (θ ∈ E |ϕ = 0) = lim ,
a→0,b→0 P(ϕ∈[a,b])
P(θ ∈E,ϕ=0)
而不是 P (θ ∈ E |ϕ = 0) = P(ϕ=0)

9.1.1 测度的分解和 Radon–Nikodym 导数


定理 9.1 (测度的 Jordan 分解). ( X , F , µ) 为符号测度空间,则存在两个测度
µ+ ,µ− ,其中之一有限,且 µ = µ+ − µ−

定理 9.2 (Hahn 分解). ( X , F , µ) 为符号测度空间,|µ| < ∞,则 X 可分解为正集


X + 和负集 X − 。称 A ∈ F 为正集,如果任意 A 的可测子集的符号测度为正。称
A ∈ F 为负集,如果任意 A 的可测子集的符号测度为负。

Proof. 令 M = sup{µ( A ) : A 为正集},则有一列正集 A n ,µ( A n ) → M 。令 A = ∪ A n ,


则 A 为正集,且 µ( A ) ≤ M ,而另一方面 µ( A ) ≥ µ( A n ) ⇒ µ( A ) = M < ∞
我们证明 X \ A 是负集,否则 X \ A 中应该有子集 B1 测度为正。若此集合
为正集,则 A 并上此集合仍为正集,矛盾。否则 B1 中有 C1 测度严格为负,而
B1 \C 1 测度仍为正
所以如果 X \ A 中没有正集,则一定能够找到一列不相交的 C i ,测度都为
负,而且

µ( X \ A ) = µ(( X \ A )\(∪C i ) + µ(∪C i ) ⇒ |µ(∪C i )| < ∞ ⇒ µ(C i ) → 0


9.2. 单调函数 169

注意在寻找 C i 的过程中总可以使得 |µ(C i )| ≥ 12 sup{|µ(C )| : µ(C ) < 0, ∀ i, C ∩ C i = ;}


于是若 ( X \ A )\∪1∞ C i 仍不为正集,则存在 D ⊂ ( X \ A )\∪∞
1 C i ,使得 µ(D ) < 0,这
与寻找出来的 C i 性质矛盾,如果是正集,也矛盾。这就表明 X \ A 是负集。
于是可定义 µ+ ( A ) = µ( A ∩ X + ),µ− ( A ) = µ( A ∩ X − ).

定义. µ,ν 为符号测度,如果 |µ|( A ) = 0 ⇒ |ν( A )| = 0,称 ν 关于 µ 绝对连续,记


为 ν¿µ

定理 9.3 (Radon–Nikodym). 有限测度 ν 关于有限测度 µ 绝对连续,当且仅当,


R
存在可积函数 f 使得 ν( A ) = A f d µ,∀ A ∈ A

Proof. 不妨设 µ,ν 为非负测度。令


Z
F = { f ∈ L (µ) : f ≥ 0, f d µ ≤ ν ( A ), ∀ A ∈ A }
A
Z
M = sup{ f d µ : f ∈ F }
X

显然可找到 f n ∈ F 且积分收敛至 M ,令 g n = max( f 1 , · · · , f n ),则 g n ∈ F ,由单调


R
收敛定理, f = lim g n ∈ F , X f dµ = M
R
下面证明 ν( A ) = f dµ
RA
令 η( A ) = ν( A ) − A f d µ,则 η( A ) ≥ 0,η ¿ µ,需要证明 η ≡ 0
反证,若非如此,考察符号测度 η − µ/ n,由 Hahn 分解,对每一个 n 都存
在 X n+ ,则 f˜n = f + 1 X n+ / n 满足,∀ A ∈ A ,
Z Z Z Z
f˜n = f+ d µ/ n ≤ f + η( A ∩ X n+ ) ≤ ν( A ∩ X n− ) + ν( A ∩ X n+ ) = ν( A )
A A A ∩ X n+ A ∩ X n−

所以 f˜n ∈ F
注意到 η( X n− ) − µ( X n− ) ≤ 0 ⇒ η( X n− ) → 0。定义 X 0− = ∩∞ − −
1 X n 则 η( X 0 ) = 0,表
明至少有一个 X n+ 使得 η( X n+ ) > 0,否则 η( X ) = η( X n− ) ⇒ η( X ) = η( X 0− ) = 0 表明
R R
η = 0,所以 µ( X n+ ) > 0,对应的,有 X f˜n d µ > X f d µ。矛盾

9.2 单调函数
定义. E ⊂ R ,函数 u : E → R 称为单调的,如果 (1)(2)(3)(4) 中任意一个满足.
(1) 递增,若 u ( x) ≤ u ( y),对所有 x, y ∈ E , x < y
170 CHAPTER 9. 单调函数与可微性

(2) 严格增,若 u ( x) < u ( y)


(3) 递减,若 u ( x) ≥ u ( y)
(4) 严格减,若 u ( x) > u ( y)

单调函数通常并不一定连续,所以问题在于到底有多不连续.

定理 9.4. I ⊂ R 为一区间,u : I → R 为单调函数,则 u 最多只有可数个不连续


的点,反之,给定 R 中可数点集 E ,存在单调函数 u : R → R 使其不连续点恰
为 E.

Proof. 第一部分显然,第二部分 E = { xn }n∈N ,则令 u n ( x) = − n12 若 x < xn ,u n ( x) =


1 ∆P

n2
若 x ≥ x n , u ( x) = u n ( x),则 u ( x) 在 R \E 连续。
1

定 义. 给 定 区 间 I ⊂ R , 递 增 函 数 u : I → R 称 为 阶 跃 函 数, 若 它 能 表 示 成
P
u ( x) = u n ( x)
n∈ E 

 x < an
 0
E ⊂ N ,u n ( x) = sn x = a n ,{a n }n∈E ⊂ I ,{ s n }n∈E ⊂ [0, ∞),{ t n } ⊂ [0, ∞),


 s +t x > an
n n
s n + t n > 0,∀ n ∈ E .

定理 9.5 (递增函数的反函数). I ⊂ R 为有下界的区间,u : I → R 为递增函数,J


为包含 u ( I ) 的最小区间,v : J → R 定义为 v ( y) = inf { z ∈ I : u ( z) ≥ y}, y ∈ J
则:(1) v 递增且左连续,若 u 严格增,则 v 连续
(2) v (u ( x)) ≤ x,∀ x ∈ I ,当且仅当 u 在某个区间 [ z, x] ⊂ I , z < x 上取常数
时不等号成立.

一个应用:随机变量 X 的分布为 F ,U ∼ unif (0, 1),则通过 F − (U ) 可模拟


X 的取值。F − 为 F 的反函数。

9.3 单调函数的可微性
定义 (可微性,导数). E ⊂ R ,x0 是 E 的一个极限点,给定函数 u : E → R .若极限
u(x)− u(x0 )
lim x− x0 存在(且有限),则极限称为 u 在 x0 的导数,称 u 在 x0
x∈E,x→ x0
点可微。
导数不一定存在,但下面的 Dini 导数,通常都存在:
9.3. 单调函数的可微性 171

f (x0 + h)− f (x0 )


右上导数 D + f ( x0 ) = lim+ h ,
h→0
f (x0 + h)− f (x0 )
右下导数 D + f ( x0 ) = lim h ,
h→0+
− f (x0 + h)− f (x0 )
左上导数 D f ( x0 ) = lim− h ,
h→0
f (x0 + h)− f (x0 )
左下导数 D − f ( x0 ) = lim h
h→0−
对于单调函数的可微性,最重要的定理是 Lebesgue 定理,即单调函数几乎
处处可微。这一结果严重依赖于单调性,因为事实上大多数连续函数是无处可
微的.Banach(1931) 证明了在 C [0, 1] 赋予一致度量的连续函数空间中,D + f 和
D + f 在某点都有限的函数在 C [0, 1] 中是第一纲集(利用 Baire 纲定理,参见连
续函数部分练习),最早的无处可微的连续函数是 Weierstrass 给出的.
P

定理 9.6. 0 < a < 1, b ∈ N 为奇数,ab > 1 + 32 π,则函数 u ( x) = a n cos ( b n π x)
n=0
连续但无处可微。
另一个直观的例子是:
(
x 0 ≤ x ≤ 12
1. 定义 φ ( x) = 1
,通过 φ ( x + k) = φ ( x) 延拓到整个 R 上,k ∈ Z
1− x 2 ≤x≤1

±
2. 再定义 u n , n = 0,1,2,…, u n ( x) = ϕ (4n x) 4n

P

3. 最后定义 f ( x) = u n ( x),则 f ( x) 连续但处处不可微
n=0
P
连续性是由于 f n ( x) = 0n u i ( x) 连续,且 f n ( x) 一致收敛到
f ( x)
± ±
不可微性可以分别考察,x 具有 m 4k ,m ∈ Z ,k ∈ N 形式和不具有 m 4k 形
式时的情况
172 CHAPTER 9. 单调函数与可微性

练习

1. F 在 [a, b] 上连续,证明 D + (F ) ( x) 是可测函数


2. 凸函数的一些性质。设 I 是 R 内的一个开区间,f 是 I 上的一个(实值)凸函
[称 f 为凸函数,如果 ∀ x, y ∈ I, λ ∈ (0, 1),f (λ x+(1−λ) y) ≤ λ f ( x)+(1−λ) f ( y)。
数.
可以证明 f 在开区间 I 上必然连续。注意若只要求 f ( x+2 y ) ≤ f (x)+ f (y)
2 ,不足
以保证 f 为凸函数,Sierpinsk 定理表明,只需要附加 f 为 Lebesgue 可测的
条件, f 即为凸函数(事实上 Lebesgue 可测 +Mid-convex 确保了函数的连
续性)。]
(a) 证明 ∀ x ∈ I , f +0 ( x) 和 f −0 ( x) 都存在且有限,此外,在 I 上 f +0 和 f −0 还
是递增函数,且 f −0 ≤ f +0 ,于是在 I 上 f 0 几乎处处存在且有限.
(b) 证明对于 I 的任意有界闭子区间 [a, b],存在常数 M ≥ 0,使对于一切 x,
y ∈ [a, b],有 | f ( x) − f ( y)| ≤ M | x − y|
(c) 开区间 I 上的实值函数 f 是凸函数的充要条件是 f 在 I 上连续且
D 2 f ≥ 0.
其中 D 2 f ( x) = lim sup f (x+h)+ f (x
h2
− h)−2 f (x)
h↓0

定理 9.7 (Lebesgue). 若 f : I → R 是区间 I 上的单调函数,则 f 几乎处处可微.

f (x0 + h n )− f (x0 )
证明思路:定义导出数为 λ = lim hn 对某个序列 h n → 0,记作
n→∞
D f ( x0 )
(在给定点 x 可能存在很多不同值的导出数)
(1) 若 f : [a, b] → R 严格增,E ⊂ [a, b],0 ≤ p < ∞,若 E 中每个 x,存在某
个导出数使 D f ( x) ≤ p,则 m∗ ( f (E )) ≤ pm∗ (E )
(2) f : [a, b] → R 严格增,E ⊂ [a, b],0 < q < ∞,对每个 x ∈ E ,存在 f 的某
个导出数使 D f ( x) ≥ q,则 m∗ ( f (E )) ≥ qm∗ (E )
⇒ 递增函数 f 具有无穷值导出数的点集测度为 0.

g(x+ h)− g(x) f (x+ h)− f (x)


Proof. g ( x) = f ( x) + x,则 g 严格增,而且 h = h +1
故 { x : D f ( x) = ∞} = { x : D g ( x) = ∞},后者在 g 下的像集 ⊂ [ g (a) , g (b)],由
∀ q > 0, g( b) − g(a) ≥ qm∗ ({ x : D g ( x) = ∞}) ⇒ m∗ ({ x : D g ( x) = ∞}) = 0.

⇒ 递增函数 f ,0 ≤ p ≤ q < ∞.若 E 中每个 x,存在 f 的两个导出数使


D 1 f ( x) ≤ p < q ≤ D 2 f ( x),则 m (E ) = 0
9.3. 单调函数的可微性 173

Proof. 不妨设 f 严格增,否则考虑 f + x,则 qm∗ (E ) ≤ m∗ ( f (E )) ≤ pm∗ (E )。

(3) f 在 [a, b] 上递增,则 f 几乎处处有有限导数.

Proof. 若 E 为 f 的不可微点,则 E 上每个点至少有两个不同的导出数即


[
E = { x : D 1 f ( x) < D 2 f ( x)} = { x : D 1 f ( x) ≤ p < q ≤ D 2 f ( x)} ⇒ m (E ) = 0
p<q
p, q ∈ Q

另外 f 0 = ∞ 的点的测度也为 0,即得证.

下面证明 (1) 和 (2)。


大致而言,由于每个点有 D f ( x) ≤ p,可以由极限性质知在 x 的小邻域 I ( x)
内,
P
| f ( I ( x))| ≤ p | I ( x)|,于是所有的 { f ( I ( x))} 可以覆盖 f (E ),形式上有 x∈ E | f ( I ( x))| ≤
P
p x∈E | I ( x)|,但这里的求和项可能是不可数的,但我们可以将其变成可数的并
∗ ∗
使其和接近 m ( f (E )) 与 m (E ).这正是 Vitali 覆盖定理的主要结论.

定义 (Vitali 覆盖). I 为 R 中长度 > 0 的闭区间族,E ⊂ R ,V ⊂ I .若对任意一


个 x ∈ E ,ε > 0,存在 V ∈ V ,使 x ∈ V , m (V ) < ε,则称 V 为 E 的 Vitali 覆盖

定理 (Vitali 覆盖定理).
µ V 为 ¶E 的 Vitali 覆盖,则在 V 中存在互不相交的可数
S

闭区间列 {Vk } 使 m E \ Vk = 0.若给定 ε > 0,则存在有限个互不相交的 {Vk }
µ ¶ k=1
S
n
使 m∗ E \ Vk < ε。
k=1

(1) 的证明. 给定 ε > 0,设 G 为包含 E 的有界开集,使 m (G ) < m∗ (E ) + ε,对


f (x0 + h n )− f (x0 )
于 x0 ∈ E ,存在 {h k } → 0 使 lim hn = D f ( x0 ) ≤ p
n→∞
考虑区间 d n ( x0 ) = [ x0 , x0 + h n ] 或 [ x0 + h n , x0 ] 及 ∆n ( x0 ) = [ f ( x0 ) , f ( x0 + h n )]
或 [ f ( x0 + h n ) , f ( x0 )]
{∆n ( x0 ) , x0 ∈ E, n ≥ 1} 覆盖 f (E ),
显然 {d n ( x0 ) , x0 ∈ E, n ≥ 1} 覆盖 E , m (∆n ( x0 )) >
f (x0 + h n )− f (x0 )
0( f 严格增),因为 h n → 0,不妨假定 d n ( x0 ) ⊂ G , hn < p+ε ∀n
m ( d n ( x0 )) = | h n |,m (∆n ( x0 )) = | f ( x0 + h n ) − f ( x0 )| ⇒ m (∆n ( x0 )) < ( p + ε) m ( d n ( x0 ))
这意味着 h n → 0 时 m (∆n ( x0 )) → 0,所以 {∆n ( x0 ) : x0 ∈ E, n ≥ 1} 是 f (E ) 的
Vitali 覆盖.
174 CHAPTER 9. 单调函数与可微性
µ ¶
© ª ∗ S

由覆盖定理,可找到互不相交的区间 ∆n i ( x i ) 使 m f (E ) \ ∆n i ( x i ) =
µi=1 ¶
∗ P
∞ ¡ ¢ P
∞ ¡ ¢ S

0,故 m ( f (E )) ≤ m ∆n i ( x i ) < ( p + ε) m d n i ( x i ) = ( p + ε) m d n i (xi ) ≤
i =1 i =1 i =1
( p + ε) m (G ) ⇒ m∗ ( f (E )) ≤ ( p + ε) ( m∗ (E ) + ε),ε → 0 ⇒ m∗ ( f (E )) ≤ pm∗ (E ).

(2) 的证明. 给定 ε > 0,设 G 为包含 f (E ) 的有界开集,使 m (G ) < m∗ ( f (E )) + ε,


f (x0 + h n )− f (x0 )
对于 x0 ∈ E ,存在 {h k } → 0 使 lim hn = D f ( x0 ) ≥ q
n→∞
考虑区间 d n ( x0 ) = [ x0 , x0 + h n ] 或 [ x0 + h n , x0 ] 及 ∆n ( x0 ) = [ f ( x0 ) , f ( x0 + h n )]
或 [ f ( x0 + h n ) , f ( x0 )]
{∆n ( x0 ) , x0 ∈ E, n ≥ 1} 覆盖 f (E ),
显然 {d n ( x0 ) , x0 ∈ E, n ≥ 1} 覆盖 E , m (∆n ( x0 )) >
f (x0 + h n )− f (x0 )
0( f 严格增),因为 h n → 0,不妨假定 d n ( x0 ) ⊂ G , hn > q−ε ∀n
m ( d n ( x0 )) = | h n |,m (∆n ( x0 )) = | f ( x0 + h n ) − f ( x0 )| ⇒ m (∆n ( x0 )) > ( q − ε) m ( d n ( x0 ))
如果 x0 ∈ E 是连续点,那么 n 充分大(n( x0 ))时 ∆n ( x0 ) ⊂ G ,令 S 为 f 的连续
点集合,则 E \S 至多可数,注意到 {d n ( x0 ) : x0 ∈ S, n ≥ n( x0 )} 是
µ S 的 Vitali 覆盖,

© ª S

由覆盖定理,可找到可数个互不相交的区间 d n i ( x i ) 使 m∗ S \ d n i ( x i ) = 0,
i =1
所以: µ ¶
∗ P
∞ ¡ ¢ 1 P
∞ ¡ ¢ 1 S

1
m (S ) ≤ m d n i (xi ) < (q−ε)
m ∆n i ( x i ) = (q−ε) m ∆n i ( x i ) ≤ (q− ε)
m (G ) ⇒
i =1 i =1 i =1
∗ ∗
m (E ) = m (S ) ≤ 1
(q−ε) (

m ( f (E )) + ε),ε → 0 ⇒ m∗ ( f (E )) ≥ qm∗ (E ).

注意. 1. Lebesgue 定理中几乎处处条件不可以改进,即若 m (E ) = 0,则可找到


连续递增函数 f ,在每个点 x ∈ E 上, f 0 ( x) = +∞(书上例 4,P245).但是
反过来,给定零测集 E ,要构造出单调函数使得 E c 上导数处处存在,目前
却还不知道如何构造 (Hewitt & Stromberg)
2. Denjoy-Young-Saks 定 理: 对 于 [a,b] 上 的 任 意 函 数 f, 对 于 几 乎 所 有 的
x ∈ [a, b],下列情形之一总会发生:
(a) f 0 ( x) 存在且有限
(b) −∞ < D + f ( x) = D − f ( x) < +∞,D − f ( x) = +∞,D + f ( x) < −∞
(c) −∞ < D − f ( x) = D + f ( x) < +∞,D + f ( x) = +∞,D − f ( x) < −∞
(d) D + f ( x) = D − f ( x) = +∞,D + f ( x) = D − f ( x) = −∞
而且,D̄ = lim suph→0,h6=0 ( f ( x + h) − f ( x))/h 和 D = lim infh→0,h6=0 ( f ( x + h) −
f ( x))/ h 是广义实值可测函数,f 具有有限导数的点的集合(记为 D )Lebesgue
可测。在 D 上 f 0 可测。事实上,D 是 Borel 集, f 0 |D 是 Borel 可测的。
9.3. 单调函数的可微性 175

定理 9.8 (Vitali 覆盖定理). 假设 E 为 R n 中的任意集合。∀ x ∈ E ,存在一列中心为


x 的闭方体 {Vk ( x)}∞
k=1
收缩到 x。则从所有这些方体 V = {Vk ( x) : x ∈ E, k = 1, 2, ...}
中可选出互不相交的方体序列 {S k } 使:m∗ (E − ∪S k ) = 0。

Proof. 首先假定 E 处于顶点在原点的单位立方体 Q 内部。不妨设 V 中方体也


在 Q 内部。

选择 S1 ∈ V 使 |S1 | = m(S1 ) ≥ 0.5 sup{|V | : V ∈ V },
再选 S2 ∈ V 使 |S2 | ≥ 0.5 sup{|V | : V ∈ V , V ∩ S1 = ;},S2 ∩ S1 = ;,
再选 S3 ∈ V 使 |S3 | ≥ 0.5 sup{|V | : V ∈ V , V ∩ (S1 ∪ S2 ) = ;},S3 ∩ (S1 ∪ S2 ) = ;,
一直继续,这个过程或者在进行到有限步后停止,或者可无限继续下去。若
只可进行有限次,则 E ⊂ ∪ i S i 。所以设可无限继续,得到 {S k }。
我们证明,对任意 V ∈ V ,V ∩(∪∞ S ) 6= ;。
k=1 k
否则,存在 S ∈ V ,S ∩(∪∞ S ) = ;,
k=1 k
且 |S | ≥ 0.5 sup{|V | : V ∈ V , V ∩ (∪∞ S ) = ;}。因为 S k 都不相交且在 Q 内部,必
k=1 k
然有 k 趋于无穷时 |S k | → 0,故存在 j 0 使 |S j0 | < 0.5|S |,这意味着:
j −1
0.5|S | > |S j 0 | ≥ 0.5 sup{|V | : V ∈ V , V ∩ (∪k0=1 S k ) = ;} ≥ 0.5|S |,矛盾。
下面证明 m∗ (E −∪S k ) = 0,只需要证 ∀ε > 0,存在 h 使得 m∗ (E −∪hk=1 S k ) ≤ ε。
任给 η > 0,选 h 使得 ∪∞ |S | ≤ η,我们有
k= h+1 k

E − ∪hk=1 S k ⊂ ∪{S : S ∈ V , S ∩ (∪hk=1 S k ) = ;}


= ∪{S : S ∈ V , S ∩ (∪hk=1 S k ) = ;, S ∩ (∪∞ S ) 6= ;}
k= h+1 k
= ∪∞
j
j=h
(∪{S : S ∈ V , S ∩ (∪k=1 S k ) = ;, S ∩ S j+1 6= ;})
⊂ ∪∞
j=h
(∪{S : S ∈ V , |S | ≤ 2|S j+1 |, S ∩ S j+1 6= ;})

[第一个包含是由于 ∪hk=1 S k 是紧集,对每个 x ∈ E − ∪hk=1 S k ,必存在 V j ( x) ∈ V 使


得 V j ( x) ∩ (∪hk=1 S k ) = ;]
对于 {S : S ∈ V , |S | ≤ 2|S j+1 |, S ∩ S j+1 6= ;} 中的 S,只需保持 S j+1 中心不
变,将整个方体扩大固定的某个倍数 ω 后必然包含 S。所以, m∗ (E − ∪hk=1 S k ) ≤
P

ωn |S j | ≤ ωn η,只需要使 ωn η ≤ ε,定理结论成立。
j = h+1
若 E 不在 Q 内部,我们可将前面结果应用于整个空间上各单位方体内部与
E 的交集即可。

注意. R n 中 Vitali 覆盖定理的闭方体也可改为闭球。定理揭示了 Lebesgue 测度


与 R n 空间拓扑的一种关系,对于 R n 上的其它测度结论就不一定成立.Vitali
定理有一个有趣的推论.
176 CHAPTER 9. 单调函数与可微性

推论. R 上任意多(不局限于可数)长度 > 0 的区间的并是 Lebesgue 可测集,即


S
若 ( I α )α∈ A 是 R 上区间族,则 E = α∈ A I α 是可测的.
© ª
Proof. C = 闭区间 J ⊂ I α , 某个α ∈ A ,则 C 为 E 的 Vitali 覆盖.

练习

1. f : (a, b) → R 可微,证明 f 0 是 Borel 可测的,若 f 是几乎处处可微,证明


f 0 是 Lebesgue 可测的.
2. 证明 f 递增 ⇔ f 在每点的所有导出数都是非负的.
3. f 是 R 上 的 实 值 函 数,E 是 可 测 集 且 f 在 E 上 的 每 一 点 都 可 微, 证
明 m ( f (E )) ≤ ∫ | f 0 ( x)| dx. 而且对任意 E 上的零测集 O , m ( f (O )) = 0. 若
E
∀ x ∈ E, | f 0 ( x)| ≤ L,则 m ( f (E )) ≤ L × m (E )

现在来看 Lebesgue 定理的推论。

定理 9.9. 若 f ( x) 是 [a, b] 上的单调函数,则对任意 [ c, d ] ⊂ [a, b],


Rd¯ 0 ¯
¯ ¯ 0
c f ( x) dx ≤ | f ( d ) − f ( c)|.即单调函数 f 的导函数 f 总是局部 Lebesgue
可积的.

0
Rb
Proof. 只需考虑 f 单调增, a f ( x) dx ≤ f ( b) − f (a)
¡ ¡ ¢ ¢±¡ ¢
令 f n ( x) = f x + n1 − f ( x) n1 ,则 f 0 ( x) = lim f n ( x),a, e. x ∈ [a, b]
n→∞ hR i
Rb 0 Rb ¡ ¢ R
由 Fatou 引理 a f ( x) dx ≤ lim a f n ( x) dx = lim n ab f x + n1 dx − ab f ( x) dx
n→∞ n→∞
"Z Z # "Z Z #
b+ n1 b b+ n1 a+ n1
= lim n f ( x) dx − f ( x) dx = lim n f ( x) dx − f ( x) dx
n→∞ a+ n1 a n→∞ b a

½ ¾
1
R a+ n1
≤ lim n nf ( b) − a f (a) dx = f ( b) − f (a)
n→∞

注意. 若等号成立,则 f 必连续,但反过来 f 连续,等号却未必成立.比如在


[−1, 1] 上特殊函数 1[0,1] ,它的导数除了 0 点之外都为 0,故导函数积分为 0,不
等号严格成立,事实上存在 [0, 1] 上严格单调增加的连续函数 f ,但 f 0 几乎处处
为 0.
例:严格增、连续、且几乎处处导数为 0 的函数:
¡ ¢ ¡ k k+1
¢
取 λ ∈ (0, 1),令 f 0 = x,记区间 α, β = 2n , 2n , k = 0,1,2,…,2n − 1
9.3. 单调函数的可微性 177

¡ ¢ ¡ ¢ ³ ´ ¡ ¢
α+β 1−λ 1+λ
定 义 f n+1 (α) = f n (α), f n+1 β = f n β , f n+1 2 = 2 f n (α) + 2 f n β,
f ( x) = lim f n ( x).
n→∞
如果只要求 f 连续单调,而不是严格单调,则 Cantor 函数就满足要求.
±
X∞ c
n X∞ c
n 2
∆→x= n
→ f ( x) = n
⇒ f ( x) = sup { f ( y) : y ∈ ∆, y ≤ x}
n=1 3 n=1 2

定义. f 是 [a, b] 上的实值函数,若 f 0 几乎处处为零而 f 在 [a, b] 上不恒为常数,


则 f 为 [a, b] 上奇异函数.(显然,阶跃函数是奇异函数)

由单调函数的 Lebesgue 定理,可以得到很有用的逐项求导定理.


P

定理 9.10 (Fubini 逐项微分). { f n } 为 [a, b] 上的单调增函数列, f n ( x) 处处收
n=1
P
∞ P

敛于 f ,则 f 0 = dx
d
f n ( x) = f n0 ,a, e.x ∈ [a, b].
n=1 n=1

P
n P
n
Proof. 作部分和 s n = f i ,余项为 R n = f i ,则 s n ,R n 都是单调增,几乎
i =1 i = n+1
P
n
处处可微,由求导运算的线性性质 f 0 = s0n + R 0n = f i0 + R 0n ,a, e.
i =1
P
n P
n
由 R 0n ≥ 0,可知 f i0 ≤ f 0 < ∞, lim f0 几乎处处存在且有限因为是单调
i =1 n→∞ i =1 i
有界的,而 R 0n 作为 n 的函数,是单调减的序列,故也是有极限的.
R
令 R = lim R 0n ,要证 R = 0,a, e.⇔ Rdx = 0,故
R R R
Rdx = lim R 0n dx ≤ lim R 0n dx ≤ lim [R n ( b) − R n (a)] = 0,因为 R n ( x) → 0.

¡m¢
回忆:在无理点连续,有理点不连续的函数存在,( f n = n1 ,无理点为 0)
在有理点连续,无理点不连续的函数不存在.
注意:存在处处可微,而且无处单调的函数,但仍可表示成两个单调函数的
差,这也表明单调函数全体并不是线性空间。
178 CHAPTER 9. 单调函数与可微性

本章主要讨论了单调函数几乎处处可微的性质,其目的是为了学习求导运算
与积分运算之间的联系.我们知道,在 Riemann 积分框架下,若 f 是 Riemann
Rx
可积的,且在 x0 处连续,则 F ( x) = f ( t) dt 在 x0 处可微,F 0 ( x0 ) = f ( x0 ).反
a
R
过来,若 F ( x) 可微,F 0 ( x) 是 Riemann 可积的,则 ax F 0 ( x) dx = F ( x) − F (a)。
[Newton-Leibniz 公式,Riemann 积分框架下,只讨论连续可微的函数即导函数
连续的]
我们需要知道在 Lebesgue 积分中,上述微积分基本定理的推广是什么样
子的。
值得指出的是,在我们这里讨论的微分都还是大家已经遇到过的微分概念,
并不像积分那样,有 Riemann 积分到 Lebesgue 积分的推广。我们也会看到可微
函数与可积函数虽然也都可以形成线性空间,但也有很多区别。
下面的例子表明,处处可微的函数的微分可以是 Lebesgue 不可积的
(
x2 sin x12 x 6= 0
f ( x) = 在 [0,1] 上处处可微,但 f 0 = 2 x sin x12 − 2 1x cos x12 不
0 x=0
可积。
一个有趣的结论是处处可微的函数的导函数可以是几乎处处不连续的,但
不可能处处不连续。

9.4 覆盖定理和极大函数
1.适用于一般测度的覆盖定理

定理 9.11 (Besicovitch 覆盖定理). 设 F 是 R n 中的一族闭球,满足 sup{diam(B), B ∈


F } < ∞,若 A 是 F 中球心的集合,则存在常数 Nn ,及集族 F1 , ..., F Nn ⊂ F 使
得每个 F i ( i = 1, ..., Nn ) 是 F 中可数个不相交的球的集合,且 A ⊂ ∪ N n

i =1 B∈F i
B。

推论. µ 为 R n 上的 Borel 测度,F 是 R n 中的一族闭球,A 是 F 中球心的集合。


µ( A ) < ∞。对任意 A 中的点 a,inf{ r |B r (a) ∈ F } = 0。那么对任意开集 U ⊂ R n ,存
在 F 中可数个不相交的球的集合 G ,使得 ∪B∈G B ⊂ U ,µ(( A ∩ U ) − ∪B∈G B) = 0。

2.Maximal function 极大函数


对于 R n 上的函数 g( x),λ(α) = m{ x : | g( x)| > α} 称为 | g| 的分布函数。λ(α)
R∞ R
是递降的函数。我们业已证明,对于 g ∈ L p , 0 α p d λ(α) = R n | g( x)|
p
dx
9.4. 覆盖定理和极大函数 179

如果考虑函数 g( x) 局部的大小情况,定义极大函数:
Z
1
M ( g)( x) = sup | g( y)| d y
r >0 m(B r ( x)) B r (x)

下面的定理描述了函数 M ( g) 的性质:

定理 9.12. (a) g ∈ L p (R n ), 1 ≤ p ≤ ∞,则 M ( g) 几乎处处有限


R
(b) g ∈ L1 (R n ),则对任意 α > 0,m({ x : M ( g)( x) > α}) ≤ Kα R n | g( y)| d y,其中
K 为常数且只和 n 有关,比如 K = 5n 。
(c) g ∈ L p (R n ), 1 < p ≤ ∞,则 M ( g) ∈ L p (R n ),|| M ( g)|| p ≤ K p || g|| p ,其中 K p
为常数且只和 p 和 n 有关。

推论. 若 g ∈ L p (R n ), 1 ≤ p ≤ ∞(只需局部可积),则对几乎处处的 x,
Z
1
lim g( y) d y = g( x)
r →0 m(B r ( x)) B r (x)

练习

作为练习,我们可以证明 g ∈ L1 (R n ) 时定理的结论。显然只需证明 (b)。


1. 证明 M ( g) 可测
2. 给出 { x : M ( g)( x) > α} 的一个 Vitali 覆盖,应用 Vitali 覆盖定理证明 (b) 成

第 10 章 有界变差函数

10.1 有界变差函数
在前面,我们讨论了单调函数,但单调函数并不构成一个线性空间,我们这里讨
论包含所有 [a, b] 上单调函数的最小向量空间,即有界变差函数:

定义. f ( x) 是 [a, b] 上的实值函数,任取分划 a = x0 < x1 < · · · < xn = b 作和式


X
n
Vf ( x0 , · · · , xn ) = | f ( x i ) − f ( x i−1 )|
i =1

对一切可能的分划,和式 Vf 的上确界称为 f 在 [a, b] 上的全变差,记为


© ª
Vab f = sup Vf ( x0 , · · · , xn ) : { x0 , · · · , xn } ⊂ [a, b]

若 Vab f < ∞,称 f 为有界变差函数,[a, b] 上有界变差函数全体记为 BV [a, b].

注意. 上述定义中的闭区间可以是其他类型的区间。若 Ω 为 R 上的开集,则


可 表 示 为 可 数 个 互 不 相 交 的 开 区 间 的 并, 于 是 可 定 义 f 在 Ω 上 的 全 变 差
P
VΩ ( f ) = VI n ( f )。注意如果讨论开区间上的变差时,下面针对闭区间上的变差的
讨论未必仍成立。
单调函数的导数在某种意义上可以理解为测度,而有界变差函数的导数是
符号测度
¾ ½
¯ ¯
¯ ¯
开集 U 上的有界变差函数也是满足 sup ∫ f φ dx, φ ≤ 1, φ ∈ C C (U ) < ∞ 的
1 0
U
函数 f ∈ L1 (U )
几何意义
如图所示,[a, b] 上函数 f 对应的曲线长度可以近似为:

180
10.1. 有界变差函数 181

n q
X ³ ´
lim ( x i − x i−1 )2 + ( f ( x i ) − f ( x i−1 ))2 ∈ Vab f , b − a + Vab f
n
i

表明曲线求长和变差有界是等价的。Vab f 相当于函数在曲线上移动时在 Y
轴的投影所走过的距离。

例. 1. [a, b] 上任意单调增函数 f 的任一分划对应的


Xn Xn
Vf ( x0 , · · · , xn ) = 1
| f ( x i ) − f ( x i−1 )| = 1
f ( x i ) − f ( x i−1 ) = f ( b) − f (a) < ∞

故 f 是有界变差的,且 Vab f = f (b) − f (a).


2. 若 f 在 [a, b] 上满足 Lipschitz 条件,即 | f ( x) − f ( y)| ≤ M | x − y|,则
P P
Vf ( x0 , · · · , xn ) = 1n | f ( x i ) − f ( x i−1 )| ≤ M 1n ( x i − x i−1 ) = M ( b − a) < ∞,说明 f
是有界变差的
3. 若 f 是在 [a, b] 上处处可微且导数有界或者导函数 Riemann 可积,则函
数是有界变差的。
¡π¢
4. 连续函数未必是有界变差的,比如 f ( x) = x n sin x (0 < x ≤ 1),0( x = 0)
则 n ≥ 2 时 f ∈ BV [0, 1],但 n = 1 时, f ∉ BV [0, 1]

注意到我们给了一个连续有界但不是有界变差的例子,这表明 B[a,b] 比
BV[a,b] 大,同时在一致度量下 BV[a,b] 包含 C[a,b] 的稠密子集如全体有理系数
多项式,所以 C[a,b] 包含于 BV[a,b] 的闭包,这也表明 BV[a,b] 在一致度量下并
非闭集,不是完备度量空间,不过换成其它度量之后,BV[a,b] 是完备的

练习

1. 设 u、v ∈ BV [a, b].证明: u ± v ∈ BV [a, b] uv ∈ BV [a, b]


u
若 v ( x) ≥ c > 0∀ x ∈ [a, b],则 v ∈ BV [a, b]
2. f ∈ BV [a, b],则 f 是有界函数
¡ ¢ ¯ ¯
3. f , g ∈ BV [a, b],α, β ∈ R ,则 Vab α f + β g ≤ |α| Vab f + ¯β¯ Vab g
4. f ∈ BV [a, b],Vab f = 0,则 f 为常数
5. f ∈ BV [a, b],[ c, d ] ⊂ [a, b],则 f ∈ BV [ c, d ]
182 CHAPTER 10. 有界变差函数

6. f ∈ BV [a, b], a < c < b,则 Vab f = Vac f + Vcb f


© ª
7. f n ∈ BV [a, b], Vab f n 有界, f n 处处收敛于 f ,则 f ∈ BV [a, b] 且 Vab f ≤
sup Vab f n .
n
(
x sin πx , 1
n ≤x≤1
注意: f n = 1 1
n sin nπ, 0≤x< n
可以验证 f n 是有界变差的,但 Vab f n → ∞,lim f n 也不是有界变差的
下面的 Jordan 分解定理说明了有界变差函数与单调函数之间的关系.
定理 10.1. f 是 [a, b] 上的有界变差函数,则 f 可表示为两个单调递增函数差的
形式.
Proof. 令 g ( x) = 12 Vax f + 21 f ,h ( x) = 12 Vax f − 12 f
¯ ¡ ¢¯0 0 ¡ ¢
当 x0 > x 时,¯ f ( x) − f x0 ¯ = Vf x, x0 ≤ Vxx f = Vax f − Vax f
0 ¡ ¢ 0 ¡ ¢
故 Vax f + f ( x) ≤ Vax f + f x0 ,Vax f − f ( x) ≤ Vax f − f x0 ,即 g, h 递增.

注意:Jordan 分解并不唯一,因为 g, h 加上常数后,仍然成立.


推论. f ∈ BV [a, b],则 f 几乎处处可微.(因为单调函数几乎处处可微)
y
注意到对于 y > x, | f (y)y−−xf (x)| ≤
Vx ( f )
y− x ,取极限即可得到
R 0
R d
推论. f ∈ BV [a, b],则 [a,b] | f ( x)|d x ≤ x
[a,b] dx (Va f )d x

进一步的,前述推论中的不等号可进一步改为等号
在 Jordan 分解定理证明中我们看到如果 g ( x) = Vax f 在 x0 处连续则 f 在 x0
处连续,下面的定理表明,若 f 连续,则 g 也连续.
定理 10.2. f 与 Vax f 有相同的 (左) 右方连续的点.
Proof. f 在 x0 右连续,a ≤ x0 < b,∀ε > 0,∃δ ∈ (0, b − x0 ) 使当 x ∈ ( x0 , x0 + δ)
Pn
时 | f ( x) − f ( x0 )| < 2ε ,在 [ x0 , x0 + δ] 取分划 { x0 , · · · , xn } 使得 1 |f ( x i ) − f ( x i−1 )| >
x +δ ε
Vx00 f − 2
Pn x +δ x +δ x +δ
由于 | f ( x i ) − f ( x i−1 )| ≤ Vx10 f ,Vx00 f = Vxx01 f + Vx10 f
i =2
x +δ x +δ
Xn ε Xn
Vxx01 f = Vx00 f − Vx10 f < 1 | f ( x i ) − f ( x i−1 )| + − 2 | f ( x i ) − f ( x i−1 )| < ε
2
¯ x0 ¯
¯ x ¯ x x1
⇒ 当 x ∈ ( x0 , x1 ) 时, Va f − Va f = Vx0 f < Vx0 f < ε,这说明 x0 是 Vax f 的右连
续点.
推论. f 与 Vax f 有相同的连续点,连续的有界变差函数也是两个连续的单调增
函数的差.
10.2. 有界变差函数空间 183

练习
(
p x2 sin2 1x 0<x≤1
1. f : [0, 1] → R , u : [0, 1] → [0, 1], f ( z) = z, u ( x) =
0 x=0
证明: f 、 u 是有界变差的,但 f ◦ u 不是 (说明有界变差函数的复合函数不
一定是有界变差的.)(若 u 是有界变差的,f ◦ u 有界变差的充分条件之一是
f 是 Lipschitz 的,另一个可以考虑的是 u ◦ f 是有界变差的充要条件)
2. 设 F 为 [0, 1] 到 [0, 1] 上的函数,且对所有定义在 [0, 1] 上的有界变差函数 f ,
复合函数 f ◦ F = f (F ( x)) 是有界变差函数,则对于任意 [a, b] ⊂ R ,F −1 ([a, b])
可以表示为有限个区间(包括单点集)的并集。
3. 设 T 是半开闭区间全体,
½ C ⊂ T ,定义 ¾
P
n
V ( f , C ) = sup | f ( b i ) − f (a i )| , {(a i , b i ]} 是C 中互不相交的区间列 。
i =1
C 称为 E ⊂ R 的全覆盖,若 ∀ x ∈ E, ∃δ > 0 使 y − x ∈ (0, δ),( x, y] ∈ C ;
称为细致覆盖,若 ∀ x ∈ E, ∀ε > 0,存在 ( x, y] ∈ C 使 y − x < ε。
于是 ∀E ⊂ R,可定义
© ª © ª
Vf (E ) = inf V ( f , C ) : C 是E 的全覆盖 ,v f (E ) = inf V ( f , C ) : C 是E 的细致覆盖
证明:对任意连续函数 f ,Vf 和 v f 都是外测度,且 v f ≤ Vf ;对任意连
续的 f ,Vf ((a, b]) = v f ((a, b]) = Vab f ;C 是 ( c, d ] 的细致覆盖,则对连续 f ,
| f ( d ) − f ( c)| ≤ V ( f , C )
P
∞ P

4. 若对任意自然数 n,f n : [0, 1] → {0, 31n },定义 f = f n ,证明 V01 f ≥ 16 V01 f n
n=1 n=1

10.2 有界变差函数空间
考察空间 BV [a, b],定义 f → k f k = | f (a)|+ Vab f ,可以验证 k·k 是空间的范数.故
自然的问题是 BV [a, b] 完备吗?可分吗?结论是完备但不可分.
° °
° °
不可分例子:BV [0, 1],°1[α,1) − 1[β,1) ° = 2 若 α 6= β,而所有 1[α,1) 的函数中
间不可能找到可数稠密子集 (可分距离空间 X 的任意子集 A 仍然是可分的:设
¡ ¢
{ x i } 为稠密子集,取 yi,n ∈ A ∩ B x i , n1 即可)
现在证明 BV [a, b] 是完备的,并介绍 Helly 定理,它与分布的收敛或者说
弱收敛概念密切相关,在概率统计中有着广泛的应用,首先介绍几个引理,第一
个的证明用到了对角化的技术
184 CHAPTER 10. 有界变差函数

引理 10.1. Helly 选择原理:{ f n } 为一族定义在 X 上的实值函数,若存在常数


K ,对所有 n 和 x 都有 | f n ( x)| ≤ K ,D 为 X 的可数子集,则存在 { f n } 的子序列
© ª
f n i 在 D 上点点收敛

Proof. 记 D = { xk , k ≥ 1},先看集合 { f n ( x1 )},这是有界集,由 Bolzano-Weierstrass


© ª
定理,存在收敛子序列 f n11 ( x1 ), f n12 ( x1 ),….注意此时 f n1i ( x2 ) 也是有界集,可
从中再找到收敛子序列 f n21 ( x2 ), f n22 ( x2 ),…,继续这个过程可得可数个子序列
f n11 ( x1 ) , f n12 ( x1 ),… → A 1
f n12 ( x2 ) , f n22 ( x2 ),… → A 2
……
f n1k ( xk ) , f n2k ( xk ),… → A k
© ª
其中,每个子序列都是从上一个序列中找出来的,则 f n11 , f n22 , · · · , f nkk , · · · 就
© ª © ª
是我们要的子序列,∀k, f nkk ( xk ) , f nkk++11 ( xk ) , · · · 是 f n1k ( xk ) , f n2k ( xk ) , · · · 的子序
列,故收敛于 A k .

注意到 [a, b] 中的有理数可数且稠密,若在可数集 D 上 f 递增,[a, b] = D̄ ,


则通过定义 g ( x) = sup { f ( t) : a ≤ t ≤ x, t ∈ D },可将 f 延拓到 [a, b] 上,且 g |D = f .
用这种方法,上述定理对于 [a, b] 上的增函数可做到点点收敛.

引理 10.2. { f n } 是一族 [a, b] 上的增函数,且 | f n ( x)| ≤ K ,∀n, x,则存在子序


© ª
列 f n i 处处收敛到 [a, b] 上的增函数 f ,且 | f ( x)| ≤ K
© ª
Proof. 由 Helly 选择原理,可找到 f n i 在 [a, b] 中的有理数集上点点收敛至函
数 φ,而且 φ 是增函数,故可延拓至 [a, b] 上的增函数 f .由于 f 为增函数,它
的不连续点最多可数,先来看 f 的连续点上是否有 f ( x) = lim f n i ( x)
对于连续点 x,∀ε > 0,可找到有理数 p、q,使 p < x <
) q,f ( q) − f ( p) < ε/2,
f n i ( x) ≤ f n i ( q) ≤ f ( q) + ε/2 ≤ f ( x) + ε
当 i 足够大时, ⇒ f ( x) = lim f n i ( x)
f n i ( x) ≥ f n i ( p) ≥ f ( p) − ε/2 ≥ f ( x) − ε
© ª © ª
对于可数的不连续点,对 f n i 再度应用 Helly 选择原理,使其子序列 f n k
© ª © ª
在不连续点上处处收敛,显然 f n k 作为 f n i 的子序列,在连续点上也是收
敛的.

现在注意到 BV [a, b] 总可表示成两个增函数的差,我们不难得到 Helly 定理.

定理 10.3 (Helly 定理). 若 { f n } 为一族实值函数 ∈ BV [a, b],且 ∀n,k f n k ≤ K ,


© ª
则存在子序列 f n i 点点收敛到函数 f ,且 f ∈ BV [a, b],k f k ≤ K .(类似某种紧
性,有界集合中有收敛子列,Alaoglu 定理)
10.2. 有界变差函数空间 185

Proof. k f n k = | f n (a)| + Vab f n ≤ K ⇒ | f n ( x)| ≤ | f n (a)| + | f n ( x) − f n (a)| ≤ K


令 Vn = Vax f n ,则 Vn 递增而且 |Vn ( x)| ≤ Vab f n ≤ K ,另外 (Vn − f n ) 也是增
函数.
|Vn − f n | ≤ |Vn | + | f n | ≤ 2K ,而 f n = Vn − (Vn − f n )
© ª © ª
先对 Vn 用前述引理,可找到 Vn i → v1 ,再在 Vn i − f n i 中应用引理找
© ª
到子序列 Vn k − f n k → v2 ,则在子序列 {n k } 上.Vn k → v1 ,Vn k − f n k → v2 ,故
f n k → v1 − v2 。
由 v1 和 v2 为增函数可知 f = v1 − v2 为 BV [a, b] 中函数。
由有界变差函数的性质,可知 k f k ≤ K .

推论. BV [a, b] 在 k f k = | f (a)| + Vab f 之下是完备的

Proof. 设 { f n } 为 Cauchy 列,则 { f n } 有界,即存在 K ,使 k f n k ≤ K ,∀n.故可找到


© ª
子序列 f n i 处处收敛至 f ∈ BV [a, b].∀ε > 0,∃ N ,使 n,m ≥ N 时.k f n − f m k ≤ ε,
° ° ¯ ¯ ¡ ¢
则对所有的 n, n i ≥ N 也有 ° f n − f n i ° ≤ ε 即 ¯ f n (a) − f n i (a)¯ + Vab f n − f n i ≤ ε
¡ ¢ ¡ ¢
当 f n i 处处收敛于 f 时, f n i (a) → f (a),Vab f − f n j ≤ lim Vab f n i − f n j ≤ ε
n i →∞
若 n i , n j ≥ N ⇒ | f n (a) − f (a)| + Vab ( f n − f ) ≤ 2ε 若 n 充分大,即 k f n − f k ≤ 2ε.

注:证明中用到性质,即若 [a, b] 上的函数 f n 逐点收敛至 f ,则 Vab f ≤


P
lim Vab f n ,事实上,∀ t < Vab f ,可找到分割使 t < 1m | f ( x i ) − f ( x i−1 )|,找足够大的
n→∞ Pm Pm
ε
n 使 | f n ( x i ) − f ( x i )| ≤ 2m ,则 t< 1 | f ( x i ) − f ( x i−1 )| ≤ 1 | f n ( x i ) − f n ( x i−1 )| + ε ≤
Vab f n + ε
刚才讨论的增函数有界时存在子序列收敛的性质,在概率论中用处广泛,事
实上我们在练习中讲过概率测度是与右连续的增函数是对应起来的,所以我们
讲的增函数的收敛实际上与概率测度的收敛关系密切(现在来看一下有界变差
函数和测度之间的关系,事实上我们的一个习题已经说过,R 上的有限 Borel 测
度和其上的右连续增函数是对应的。虽然任意增函数 F 并不一定右连续,但我
们总可以通过令 G(x)=F(x+) 得到右连续的增函数 G。同样的情形适用于有界
变差函数和符号测度。这也就是为什么我们对有界变差函数收敛的讨论可以和
测度的收敛联系起来。
).更确切的,在概率论中经常遇到的是依分布收敛和概率
(分布函数) 测度的弱收敛的概念.

定义. 概率分布函数 F n 弱收敛至概率分布 F ,如果在 F 的连续点上,都有


F n ( y) → F ( y)
186 CHAPTER 10. 有界变差函数

注意如果是一般的符号测度对应的分布函数,弱收敛要求分布函数列的每
个子序列包含子子序列使得除了至多可数点之外,逐点收敛。而对于原来的序
列,可能是几乎所有点都不收敛。例如取课上讲的依测度收敛而处处不收敛的例
子中区间端点构造函数 1[x,1] − 1[y,1] ,可知这个函数弱收敛于 0,但除 0 和 1 之
外点点不收敛。

定理 10.4. 随机变量 X n ,依分布收敛于 X ∞ ,如果分布函数 F n ( x) = P ( X n ≤ x)


弱收敛:
R
X n ⇒ X ∞ ⇔ ∀ 有界连续函数 g,E g ( X n ) → E g ( X ∞ )[E g ( X n ) = g( x)dF n ( x)]
⇔ ∀ 开集 G .lim infn→∞ P ( X n ∈ G ) ≥ P ( X ∞ ∈ G )
⇔ ∀ 闭集 K .lim supn→∞ P ( X n ∈ K ) ≤ P ( X ∞ ∈ K )
⇔ 所有满足 P ( X ∞ ∈ ∂ A )=0 的集合 A ,limn→∞ P ( X n ∈ A ) = P ( X ∞ ∈ A ).

定理 10.5. 分布函数的 Helly 定理:任意概率分布函数序列 F n 中存在子序列


F n(k) ,可找到右连续的增函数 F ,在 F 的任意连续点 y 上,F n(k) ( y) → F ( y).

但 F 并不一定是概率分布函数,比如若 a + b + c = 1.F n ( x) = a1{x≥n} +


b1{ x≥−n} + cG ( x)
G 分布函数,则 F n ( x) → F ( x) = b + cG ( x),lim x↓−∞ F ( x) = b,lim x↑∞ F ( x) =
b+ c = 1−a
注意:
1. 分布函数的弱收敛也可从下面的 Levy 距离中推出
ρ (F,G ) = inf {ε : F ( x − ε) − ε ≤ G ( x) ≤ F ( x + ε) + ε, x}
则 ρ (F n , F ) → 0 ⇔ F n ⇒ F .
2. 依照前述的 BV 空间 k·k 定义的收敛比弱收敛更强.(依全变差收敛)
F n = 1³ 1
´c → F = 1(−∞,0] c ,kF n − F k = 2(这里的 F n 与 F 所对应的概率
−∞,
n
只在单点上有正概率)

10.2.1 有界变差函数的导数
Rb
回忆对于单调增函数有 f 0 dx ≤ f ( b) − f (a),那么对于 f ∈ BV [a, b],v = Vax f ,
a
¡ ¢
f = v − (v − f ),v 和 v − f 都是增函数 ⇒ f 0 = v0 − v0 − f 0 , a.e.
¯ ¯
| f ( y) − f ( x)| ≤ Vx f = v ( y) − v ( x) ⇒ ¯ f 0 ¯ ≤ v0 , a.e.
y
R b¯ ¯ Rb
⇒ a ¯ f 0 ¯ dx ≤ a v0 dx = v ( b) − v (a) = Vab f < ∞ .说明 f 0 ∈ L1
10.2. 有界变差函数空间 187
Rb
然而和单调函数一样,对于有界变差函数,不能保证 f (b) − f (a) = a f 0 dx
的成立。反之,若要使等式成立,f 必须 ∈ BV [a, b],事实上,我们有下面的定理.
Rx
定理 10.6. g ∈ L1 [a, b], f ( x) = g ( t) dt,则
R xa¯ 0 ¯ R
¯
1. f ∈ C [a, b] ∩ BV [a, b], a f ¯ dt ≤ Vax f ≤ ax | g| dt
2. f ≡ 0 ⇔ g = 0,a.e.(比较书 271,例)
Rx R x¯ 0 ¯
3. f 0 = g,a.e., f ( x) = f 0 ( t) dt,Vax f = ¯ f ( t)¯ dt
a a
¯ ¯
Proof. xn → x,则 g1[a,xn ] → g1[a,x] ,¯ g1[a,xn ] ¯ ≤ | g| ∈ L1
R xn
Rx
由控制收敛定理 g ( t) dt → a g ( t) dt ⇒ f ∈ C [a, b],
a
Rx Rx
g = g+ − g− , f ( x) = a g+ dt − a g− dt 为两个增函数之差 ⇒ f ∈ BV [a, b],
R x ¯ 0¯ ¡R R ¢
¯ f ¯ dt ≤ V x f 前面已证,V x f = V x x g+ dt − x g− dt
a a a a a a
µZ x ¶ µZ x ¶ Z x Z x Z x
+ − + −
≤ Vax g dt + Vax g dt = g + g = | g| dt
a a a a a
Rx Rd
f ≡ 0 ⇔ ∀ x ∈ [a, b], a g ( t) dt = 0 ⇔ c g ( t) = 0,∀ ( c, d ) ⊂ [a, b]
R R
⇔ ∀ 开集 U ⊂ [a, b], U g ( t) dt = 0 ⇔ ∀G δ 集合 G ⊂ [a, b], G g ( t) dt = 0
R
⇔ ∀ 可测集 E ⊂ [a, b], E g ( t) dt = 0⇔ g = 0a, e.
R x+ h
0 d
Rx g(t)dt g(ξ)h
先看 g 连续情形, f = dx a g ( t) dt = lim x h = lim h = g ( x)
h→0
R b ¯ h→0¯
因为 g ∈ L1 ,故 ∀ε > ¯ ¯
0 ,可找到连续函数 φ 使 a g− ¯ φ dt < ε.
R b¯ 0 ¯ R b ¯¯ d R t ¡ ¢ ¯
故 a ¯ f − g¯ dt = a ¯ dt a g ( x) − φ ( x) dx + φ ( t) − g ( t)¯ dt
Z b¯
¯ Z x ¯ Z
¡ ¢ ¯ b¯ ¯
≤ ¯ d ¯
g ( t) − φ ( t) dt¯ dx + ¯ g ( t) − φ ( t)¯ dt
¯ dx
a a a
R b¯ ¯
≤2 ¯ g ( t) − φ ( t)¯ dt [思考,如何推得]
a
R b¯ ¯
< 2ε ⇒ a ¯ f 0 − g¯ dt = 0 ,即 f 0 = g,a.e..

注意. 如何利用定理证明 [a, b] 上的有界变差函数 f 的变差 V ( x) = Vax f 满足


V 0 ( x) = | f 0 ( x)|, a.e.x ∈ [a, b]

由于 f 0 与 g 是几乎处处相等,下面具体说明使其相等的点是怎样的.
先考虑 g = 1E 情形,那么 f 0 在 E 上几乎处处为 1,而在 E c 上几乎处处为
0.
f (x+ h)− f (x) R
1 x+ h m(E ∩[x,x+ h])
若 h 6= 0,则 h = h x 1E dt = h
故我们不难得到下面的定理
188 CHAPTER 10. 有界变差函数

定理 10.7. E 为 R 中可测集,则
(
m (E ∩ [ x − h, x + k]) 1 a.e.E
lim =
h→0; h≥0 h+k 0 a.e.E c
k→0; k≥0

称极限为 1 的点为密集点.

可以推出 m (E ∩ B) ≥ αm (B),B = [ x − h, x + k],0 < α < 1.


比较 2.4 节定理 2.19
这是下面更一般定理的特殊情形,即考察局部可积的 g.

定 义. 可 测 函 数 f :R→ R 称 为 局 部 可 积 的, 如 果 对 任 意 有 界 区 间 [a,b],
Rb
a | f | dt < ∞.
R x0 +h
定义. f 在 x0 的某个邻域 [ x0 − h, x0 + h] 上可积,若 lim h1 x0 | f ( x) − f ( x0 )| dx =
h→0
0,
称 x0 为 f 的 Lebesgue 点.
Rx ¯
定理 10.8. 若 f 在 [a, b] 上可积,x0 是 f 的 Lebesgue 点,则 d
dx a f ( t) dt ¯ x= x0 =
f ( x0 ).
Rx
Proof. 记 F ( x) = a f ( t) dt,则
¯ ¯ Z
¯ F ( x0 + h) − F ( x0 ) ¯ 1 x0 + h
¯ − f ( x )¯ ≤ | f ( x) − f ( x0 )| dx → 0
¯ h
0 ¯
h x0

这就是说 Lebesgue 点上必定有 f 0 = g,但反过来使 f 0 = g 的并不都是


Lebesgue 点的.
P∞ ±P∞ P∞
例. 取收敛的正项级数 1 an 且 an n+1 a k → 0,记 a = 1 a n ,作 [0, a] 上的
函数:
[0, a] = x1 ∪ ( x3 , x2 ] ∪ ( x4 , x3 ] ∪ · · · 其中 x1 = a, xn − xn+1 = a n
 ¡ ¤

 x ∈ xn+1 , 21 ( xn + xn+1 )
 1 ¡ ¤
f ( x) = −1 x ∈ xn +2xn+1 , xn


 0 x=0
使 f 成为 [−a, a] 上的偶函数.
10.3. 绝对连续函数 189

在 x = 0 点,设 h ∈ ( xn+1 , xn ],记 h − xn+1 = η,η ≤ a n .


¯ Z h ¯ ¯ Z h ¯
¯1 ¯ ¯ 1 ¯ η
¯ ¯ ¯
f ( t) dt¯ = ¯ P∞ f ( t) dt¯¯ < P →0
¯h η + n+1 a k xn+1 η + ak
0

同样,左导数为 0⇒0 点导数为 0.


R
1 h
然而 |
h 0 f ( t)| dt = 1,故 0 点不是 Lebegue 点.

下面定理说明可积函数的 Lebesgue 点是很多的.

定理 10.9. f ∈ L1 [a, b],则 [a, b] 上几乎所有点都是 Lebesgue 点

Proof. 设 r 为有理数,则 | f ( x) − r | ∈ L1 [a, b].记 [a, b] 中使下式成立的 x 的全体


为 Er.
d
Rx
|
dx a f ( t) − r | dt = | f ( x) − r | ,由定理可知 m (E r ) = b − a,让 r 取遍一切有理
数.
T P
记 E= r ∈Q E r ,则 m ([a, b] \E ) ≤ r m ([a, b] \E r ) = 0
我们证明 E 中点都为 Lebesgue 点,设 x0 ∈ E ,∀ε > 0,取有理数 r 0 ,使
| f ( x0 ) − r 0 | < 2ε ,
Z x0 + h Z
1 1 x0 +h
| f ( x) − f ( x0 )| dx ≤ | f ( x) − r 0 | dx + | f ( x0 ) − r 0 |
h
x0 h x0
R
令 h → 0 有 lim h1 xx00 +h | f ( x) − f ( x0 )| dx ≤ | f ( x0 ) − r 0 | + | f ( x0 ) − r 0 | < ε
h→0
R
令 ε → 0 就有 lim h1 xx00 +h | f ( x) − f ( x0 )| dx = 0
h→0

练习

1. 根据密集点的定理证明:若 [0, 1] 上的可测集 E ,存在某个 α > 0 使 m (E ∩ I ) ≥


α m ( I ),对任意 [0, 1] 中的区间 I ,那么 m (E ) = 1.
2. 可积函数的连续点都是 Lebesgue 点
3. f : [a, b] → R ,x0 ∈ [a, b].若 [a, b] 中有可测集 E 以 x0 为密集点,且 f ( x) 沿
着 E 在 x0 处连续,称 f 在 x0 为近似连续的。证明若 f 可测,则其在 [a, b]
中几乎处处是近似连续的。(应用 Lusin 定理)
4. F 是 R 中的闭集,则对于 F 的密集点 x, d ( x + y, F ) = o(| y|),即对于任意
ε > 0,存在 η,使得当 | y| ≤ η 时总有 d ( x + y, F ) ≤ ε| y|
190 CHAPTER 10. 有界变差函数

10.3 绝对连续函数
定义. (绝对连续函数) 函数 f : R → R 称为绝对连续的,如果 ∀ε > 0,存在 δ > 0

P ¢
使得任意有限个互不相交的开区间 (a 1 , b1 ),…,(a n , b n ) 上若有 b j − a j < δ,
Pn ¯¯ ¡ ¢ ¡ ¢¯
¯
1
就有 1 f b j − f a j < ε,则称 f 在 R 上绝对连续,如果这些区间都在 [a, b]
上,称 f 在 [a, b] 上绝对连续,记为 AC [a, b].

注:显然 f ∈ AC [a, b] ⇒ f ∈ C [a, b],而且还是一致连续(紧集上的连续函数


,只需在定义中取 n = 1.不过一致连续不一定保证绝对连续。
都是一致连续的)
¯ ¡ ¢ ¡ ¢¯
同 时, 若 f 处 处 可 微 而 f 0 有 界, 则 f ∈ AC [a, b]. 因 ¯ f b j − f a j ¯ ≤
¯ ¯¡ ¢
max ¯ f 0 ¯ b j − a j ,
根据定义,首先可以证明:

定理 10.10. [a, b] 上的绝对连续函数是有界变差的.

Proof. 给定任意分割 a = x0 < · · · < xn = b,由于分割加细之后,变差是增加的.


¯ ¯ P
故不妨设 ¯ x j − x j−1 ¯ < δ,而在 [ x j−1 , x j ] 上,Vx jj−1 f = sup | f ( x i ) − f ( x i−1 )| ≤ ε,
x

⇒ Vab f ≤ ε n < ∞

练习

1. f ∈ AC [a, b],则 Vax f ∈ AC [a, b]


不过有界变差并不能保证绝对连续,我们从下面的定理可以看出:
Rx
f ( t) dt 是绝对连续的, f = F 0 , a.e..反过
定理 10.11. f ∈ L1 [a, b],F ( x) = C + a
R
来,若 F 绝对连续,则 F 0 ∈ L1 [a, b],F ( x) = C + ax F 0 ( t) dt.
R
Proof. 首先,Lebesgue 积分具有绝对连续性,即当 m(D ) < δ 时, D | f | dm <ε(书
R
。令 f n = f 1{x:| f |≤n} + n1{x:| f |>n} ,故存在 n 使得
上定理 4.12) D (| f | − f n ) dm <0.5ε,
ε
R R R
显然 δ = 2n 时, D | f | dm = D (| f | − f n ) dm + D f n dm ≤ 0.5ε + nm(D ) ≤ ε。注意到:
¯Z ¯ Z
Xn ¯ ¡ ¢ ¡ ¢¯ X ¯ bj ¯
¯F b j − F a j ¯ = n ¯ f ¯¯ ≤ |f | ≤ ε
1 1¯
aj ∪[a j ,b j ]

说明 F 是绝对连续的.
反过来,我们先证明如下引理。
10.3. 绝对连续函数 191

引理. 若 F 绝对连续,且在 [a, b] 上导数几乎处处为 0,则 F 在 [a, b] 上为常数

(比较: f 在 [a, b] 上连续,若对任意在 [a, b] 具有一阶连续导数,在两个端


Rb
点处取值为 0 的函数 g 都有 a f gdx = 0,则 f 恒为 0)

引理的证明. 令 E 表示 F 在 (a, c) 上导数为 0 的点的集合,∀ x ∈ E ,∃ h > 0,使


| f (x+ h)− f (x)| ε
[ x, x + h] ⊂ (a, c) 且 h < c−a
显然所有 [ x, x + h] 形成 E 的一个 Vitali 覆盖,故存在有限个区间 {[ x i , x i +
h i ]}ni=1 使得
m(E \∪ni=1 [ x i , x i + h i ]) < δ ,不妨设 x1 < ... < xn ,于是 [a, c]\∪ni=1 [ x i , x i + h i ] 对应
P
的区间总长度小于 δ,表明 | f (a)− f ( x1 )|+ | f ( x i + h i ) − f ( x i+1 )|+| f ( xn + h n )− f ( c)| <
| f (a) − f ( c)|
P P
ε,故:< | f (a) − f ( x1 )| + | f ( x i + h i ) − f ( x i+1 )| + | f ( x i + h i ) − f ( x i )| + | f ( xn + h n ) − f ( c)| ,
Pn
εh i
< ε+ c−a ≤ 2ε
i =1

由 c 与 ε 的任意性,结论得证
Rx
这样,(F ( x) − a F 0 ( t) dt)0 几乎处处为 0,定理得证。

这样,我们就可以得到如下的关于 Lebesgue 积分的基本定理:

定理 10.12 (微积分基本定理). F : [a, b] → R .下列条件等价:


1. F 在 [a, b] 上绝对连续
R
2. F ( x) − F (a) = ax f ( t) dt, f ∈ L1 [a, b]
Rx
3. F 在 [a, b] 上几乎处处可微,F 0 ∈ L1 [a, b],F ( x) − F (a) = a F 0 ( t) dt.

Proof. 1 ⇒ 3 是前面推论的结果,3 ⇒ 2 显然,2 ⇒ 1 也已证.

10.3.1 绝对连续函数 AC [a, b] 的性质


±
1. f 、 g ∈ AC [a, b],则 f ± g, f g ∈ AC [a, b],若 g 6= 0,则 f g ∈ AC [a, b],由
f g 的仍属于 AC [a, b],很容易得到下面的分部积分公式
Z Z b¡
0 0 0
b
0
¢
( f g) = f g + f g ⇒ ( f g) dx = f 0 g + f g0 dx
a a
Rb Rb
即 a f ( x) g0 ( x) dx = f ( b) g ( b) − f (a) g (a) − a f 0 ( x) g ( x) dx
[关于复合函数的微分问题相对更为复杂,参见书 5.6 节]
192 CHAPTER 10. 有界变差函数

2. 我们知道绝对连续函数是连续的,有界变差的,但反之并不一定,所以问题
是加上什么条件连续的有界变差函数会变成绝对连续的.

定理 10.13 (Banach-Zarecki 定理). f : [a, b] → R 是绝对连续的 ⇔ f 连续


有界变差,并且将任意零测集映为零测集,即 ∀ A ⊂ [a, b], m ( A ) = 0 有
m ( f ( A )) = 0.(书 P268 例 9 例 10)
Rx
3. f 是 Lipschitz 的 ⇔ f ( x) = C + a g ( t) dt,常数 C 和 g ( x),∃K, s, t.| g| ≤ K ,
a, e.(在 k·kBV 下 Lipschitz 函数在 AC [a, b] 中稠密)
4. 任一 f ∈ BV [a, b] 可以唯一的表示成 f = g + h,其中 g ∈ AC [a, b], g (a) =
0, h0 ( x) = 0,a.e.. 更 进 一 步, 如 果 F 是 [a,b] 上 递 增 右 连 续 函 数, 那
么 F = Fac + F sg ,其中 Fac 绝对连续且递增,F sg 递增右连续且导数几
乎处处为 0,F sg = F c + F j ,其中 F c 连续递增,而 F j 是递增阶跃函数
Rx
(Fac = a F 0 ( t) dt, F sg = F − Fac )。
5. [a, b] 上连续函数 f 处处可导, f 0 ∈ L1 [a, b],则 f ∈ AC [a, b]。注意处处可
Rb
导函数 F 的导数 F 0 不总是 L1 的,所以不能保证 F (b) − F (a) = a F 0 ( t) dt.
所以有另一种称为 Perron 积分可以使得 F 只要处处可微必有 F (b) − F (a) =
Rb
a F 0 ( t) dt.

10.3.2 函数绝对连续和测度绝对连续
定理 10.14. 若 F 是 [a, b] 上的分布函数 (右连续递增),那么 F 绝对连续
⇔ µF ¿ m(即 ∀E , m(E ) = 0 ⇒ µF (E ) = 0)

Proof. 若 µF << m,若 F 不绝对连续,则存在 ε > 0,使得对任意 n,可找到区


©¡ ¢ª ¢ PN ¡ P ¯ ¡ ¢ ¡ ¢¯ S¡ ¢
间 a j, b j b j − a j < 2−n , 1N ¯F b j − F a j ¯ > ε,记 E n = a j , b j
满足 1
S T
相当于 m (E n ) < 2−n ,µF (E n ) > ε,令 F k = ∞ k
E n ,F = ∞ 1 Fk ,
P∞ − n 1− k
则 m (F k ) < k 2 = 2 ⇒ m (F ) = 0,但 µF (F k ) > ε ⇒ µF (F ) ≥ ε,矛盾.
(注意由 u F (F k ) > ε ⇒ µF (F ) ≥ ε,用到 µF 的有限性)
反过来,若 F 绝对连续,E 为 Borel 集 m (E ) = 0,可找到开集列 U1 ⊃ U2 · · · ⊃
¡ ¢
E ,使 m (U1 ) < δ,µF U j → µF (E ).因为每个 U j 都是可数个不相交的开区间的
并,而
PN P
µF ((a k , b k )) ≤ 1N |F ( b k ) − F (a k )| < ε ,∀ N
1
¡ ¢ ¡ ¢
故 µF U j < ε,所以也有 µF (E ) = lim µF U j ≤ ε ⇒ µF (E ) = 0
10.3. 绝对连续函数 193

定理 10.15 (Lebesgue-Radon-Nikodym 定理). v 为符号测度,µ 是 σ 有限测度,


¡ ¢
则存在符号测度 λ 和 f ∈ L1 µ ,使 λ⊥µ(即 ∀E, F ∈ F ,E ∩ F = ;,E ∪ F = Ω,
R
λ(E ) = 0,µ(F ) = 0)
,v ( E ) = λ ( E ) + E f d µ,∀ 可测集 E ∈ F .(可测空间 (Ω, F ))

注意,满足 F (−∞) = 0 右连续的单调增函数一一对应于某个测度 µF ((−∞, x]) =


F ( x).为简化和我们所学的对应起来,不妨考虑限制于 [a, b] 上的分布函数与测
度,即
F (a) ≥ 0 ,F ( b) = 1,µF ((a, x]) = F ( x) − F (a)
练习表明 µ½ F 最终是可以定义在所有 Lebesgue ¾ 可测集上的,对可测集 E
P
∞ ¡ ¢ ∞¡
S ¢ © ª
µF (E ) = inf µF a j , b j : E ⊂ a j , b j = inf µ (U ) : U ⊃ E,U 开
1 1
0 F ((x,x
1 0 µ + r])
由于 F 递增故 F ∈ L ,而且 F ( x) = limr→0 m((x,x + r]) ,由 L − R − N 定理,有
R
如下分解 µF (E ) = λ (E ) + E f dm,λ⊥ m
存在可测集 A 使 λ ( A ) = 0 = m ( A c ),显然 A 也可以是个 Borel 集.
µF ((x,x+ r]) 0
我们证明 lim
r → 0 m((x,x+ r]) 几乎处处等于 f ( x),这样就有 F = f ,a.e..
R
( x,x+ r ] f dm λ((x,x+ r])
显然 lim m((x,x + r]) = f ( x),a.e..故只需证 lim m((x,x + r])
= 0,a.e.
r →0 ½ r →0 ¾
λ((x,x+ r]) 1
或者说只要 F k = x ∈ A : lim m((x,x + r])
> k 的 m ( F k ) = 0.
r →0
对集合 A ,可找到开集 Uε ⊃ A ,使 µF (Uε \ A ) < ε,则 λ (Uε \ A ) < ε,也有
λ (Uε ) < ε,而在 F k 中每个点,由上极限 > 1k ,可知存在 r x 使 ( x, x + r x ] ⊂ Uε 且
λ (( x, x + r x ]) > 1k r x 。故可找到一列 r xn → 0,同样满足前面两个条件,也就是说
©£ ¤ ª S £ ¤
x, x + r xn , x ∈ F k , n ≥ 1 是 x∈Fk ,n≥1 x, x + r xn 的 Vitali 覆盖,故
µ ¶ ³h i´
S £ ¤ P

m (F k ) ≤ m x, x + r xn = m x j, x j + rxj n
x∈F k ,n≥1 1 j
∞ ¡¡
P ¤¢
≤ λ x j , x j + r x j n j · k ≤ kλ (Uε ) = kε → 0
1

这 就 表 明 若 µF ⊥ m, 则 有 F 0 = 0,a.e. 反 之 也 成 立. 而 µF ¿ m 则 等 价 于
Rx
F ( x) = F ( a) + a F 0 ( t) dt,不过 µF 关于 m 绝对连续是从测度之间的比较进行的,
按理也可按函数角度之间陈述.

练习

1. G 是 [a,b] 上的递增连续函数,G(a)=c,G(b)=d
证明:1) E ⊂ [ c, d ] 且为 Borel 集,则 m(E ) = µG (G −1 (E ))
194 CHAPTER 10. 有界变差函数
Rd Rb
2) f 是 Borel 可测函数,在 [c,d] 上可积,则 c f ( y) d y = a f (G ( x)) dG ( x) =
Rb
a f (G ( x)) d µG
3) 若 G 只是右连续,说明 2) 可能不成立。
2. 设 f 是 [a, b] 上的绝对连续函数,u : [ c, d ] → [a, b] 单调且绝对连续,则复合
函数 f ◦ u 是 [ c, d ] 上的绝对连续函数

You might also like